You are on page 1of 1129
REVISED NINTH EDITION A Textbook of FLUID MECHANICS HYDRAULIC MACHINES Dr. R.K. Bansal A TEXTBOOK OF FLUID MECHANICS AND HYDRAULIC MACHINES WIN A CASH AWARD OF Rs. 500.00 It has come to our notice that some booksellers are fraudulently selling fake/duplicate copies of some of our fast. selling titles. In our sincere efforts to provide you with our genuine books and to protect you against these counterfeit books, Laxmi Publications (LP) has put a Hologram on the cover of some of its fast selling titles. The Hologram displays a unique 3D multi-level, multi-colour effect from different angles. It has the following three levels of flat graphics merged together. The background artwork ‘seems to be ‘under’ or ‘behind! the Hologram and gives the illusion of depth unlike the fake Hologram on the fake/duplicate books. (Preto ‘Channel 1 ‘Channel 2 Background Presently, only some titles have got the Holograms. In this ease, A Text Book of Fluid ‘Mechanics and Hydraulic Machines (2010 edition, priced at Rs. 450.00) has. got the Hologram. If you or any of your friends finds anywhere in India/abroad any book of this Edition without the LP Hologram, he/she is requested to write to us at M/s LAXMI PUBLICATIONS PVT. LTD., 113, Golden House, Daryaganj, New Delhi-110002, giving the name and address of the bookseller from where he/she purchased this book, together with the photocopy of the cover and the 2nd page on which the price of the book and name of the printer are printed. He/She will be sent a cash award of Rs, 500.00. How to decide if the book is genuine or fake ? 1. The above information may or may not_be printed. 2. The counterfeit edition of the book may have no LP Hologram or ifit has, it will be without the illusionary depth as described above. ‘What is the harm in purchasing duplicate books ? — Poor quality of paper and printing which affect your eyes, — No royalty to authors who are scholars and have put their hard labour in writing the book, ‘thus depriving them of their intellectual rights. (Warning : Selling or buying pirated books is an offence. Legal action shall be taken against the| Dookseller(s) and student{s) or whoever found guilty of such an offence in any way. A TEXTBOOK OF FLUID MECHANICS AND HYDRAULIC MACHINES (in $.1. Units) [For Degree, U.P.S.C. (Engg. Services), A.M.LE. (India)] By Dr. R.K. BANSAL B. Sc. Engg. (Mech.), M.Tech. Hons. (I.L.T:, Delhi) Ph. D., M.LE. (India) Formerly Professor, Department of Mechanical Engineering, Dean (U.G. Studies), Delhi College of Engineering, Delhi LAXMI PUBLICATIONS (P) LTD BANGALORE © CHENNAI © COCHIN — + GUWAHATI. + HYDERABAD JALANDHAR * KOLKATA © LUCKNOW + MUMBAI =» RANCHI NEW DELHI Published by LAXMI PUBLICATIONS (P) LTD 113, Golden House, Daryaganj, New Delhi-110002 Phone : 011-43 53 25 00 Fax : 011-43 53 25 28 www laxmipublications.com {info@laxmipublications.com Author : Dr. B.K. Bansal Compiled by : Smt. Nirmal Bansal © All rights reserved with Author and the Publishers. No part of this publication may be reproduced, stored in a retrieval system, or transmitted in any form or by any means, electronic, mechanical, photocopying, recording or otherwise without the prior written permission of the publisher. Price : Rs. 495.00 Only. First Edition : Sept. 1983 Ninth Edition ; 2005 Reprint : 2006, 2007, 2008, 2009 Revised Ninth Edition : 2010 OFFICES © Bangalore 080-26 61 15 61 © Chenn 044-24 34 47 26 © Cochin —_ 0484-237 70 04, 405 13 03 = © Guwahati 0361-254 36 69, 25138 81 © Hyderabad 040-24 65 23 33 © Jalandhar 0181-222 12 72 © Kolkata 033-22 27 43 84 © Lucknow 0522-220 95 78 © Mumbai 022-24 91 54 15, 24 92 78 69 © Ranchi 0651-221 47 64 EFM-0559-495-FLUID MECHANICS & HM-BAN, Cc ‘Typeseted at : Shubham Composer, New Delhi Printed at : Repro India Ltd, Mumbai The louing memory of ny daughter, Cablé PREFACE TO THE NINTH EDITION ‘The popularity of the eighth edition and reprints of the book A Textbook of Fluid Mechanics and Hydraulic Machines amongst the students and the teachers of the various Universities of the country, has prompted the bringing out of the ninth edition of the book so soon, The ninth edition has been thoroughly revised and brought up-to-date. A large number of problems from different B.E. degree examinations of Indian Universities and other examining bodies such as Institution of Engineers and U.P.S.C. upto Summer 2002 examinations have been selected and have been solved at proper places on this edition. Most of these problems have been worked out in S.J. units. All of the text along with existing problems have been converted into S.1. Units In the ninth edition, a new chapter entitled Ideal Flow (or Potential Flow) has been added. Potential flow has been included in most of Indian Universities. This chapter has been written ina simple and easy-to-follow language so that even an average student can grasp the subject matter by self-study. Also a few new topics such as “Liquids in Relative Equilibrium” and “Pipe Network” have been added in this edition. The topic of Pipe Network has been included in the chapter of Flow Through Pipes. The pipe network is mostly used in city water supply system, Laboratory supply system or house hold supply of water and gas, ‘The objective type multiple-choice questions are often asked in the various competitive examinations. Hence a large number of objective type questions with answers have been added in the end of the book. With these additions, it is hoped that the book will be quite useful for the students of different branches of Engineering at various Engineering Institutions. Texpress my sincere thanks to my colleagues, friends, students and the teachers of differ- ent Indian Universities for their valuable suggestions and recommending the book of their students, ‘Suggestions for the improvement of this book are most welcome and would be incorporated in the next edition with a view to make the book more useful. ~ Author PREFACE TO THE FIRST EDITION am glad to present the book entitled, A Textbook of Fluid Mechanics and Hydraulic Machines to the engineering students of mechanical, civil, electrical, aeronautical and chemical and also to the students preparing for the new scheme of Section B of A.M.LE. Examination of Institution of Engineers (India). The course contents have been planned in such a way that the general requirements of all engineering students are fulfilled. During my long experience of teaching this subject to undergraduate and post-graduate engineering students for the past 16 years, I have observed that the students face difficulty in understanding clearly the basic principles, fundamental concepts and theory without adequate solved problems along with the text. To meet this very basic requirement to the students, a large number of the questions taken from the examinations of the various Universities of India and from other professional and competitive examinations (such as Institution of Engineers and U.P.S.C. Engineering Service Examination) have been solved along with the text in M.K.S. and S.1 units. The book is written in a simple and easy-to-follow language, so that even an average stu- dents can grasp the subject by self-study. At the end of each chapter highlights, theoretical questions and many unsolved numerical problems with answer are given for the students to solve them. Tam thankful to my colleagues, friends and students who encouraged me to write this book. I am grateful to Institution of Engineers (India), various Universities of India and those authorities whose work have been consulted and gave me a great help in preparing the book. I express my appreciation and gratefulness to my publisher. Shri R.K. Gupta (as Mechanical Engineer) for his most co-operative, painstaking attitude and untiring efforts for bringing out the book in a short period. Mrs. Nirmal Bansal deserves special credit as she not only provided an ideal atmosphere at home for book writing but also gave inspiration and valuable suggestions. ‘Though every care has been taken in checking the manuscripts and proof reading, yet claiming perfection is very difficult. I shall be very grateful to the readers and users of this book for pointing any mistakes that might have crept in. Suggestions for improvement are most weleome and would be incorporated in the next edition with a view to make the book more useful. - Author Chapter Chapter La 1.2. 13. Ma 1.5. 16, 4a. CONTENTS 1. Properties of Fluids Introduction Properties of Fluids 12.1. Density or Mass Density 122. — Specific Weight or Weight Density 123. — Specific Volume 124. Specific Gravity Solved Problems 1.11.2 Viscosity Units of Viseosity Kinematie Viscosity Newton's Law of Viscosity Variation of Viscosity with Temperature ‘Types of Fluids Solved Problems 1.3—1.19 ‘Thermodynamic Properties 1 Dimension of R 142. — Isothermal Process 14.3. Adiabatie Process 144. — Universal Gas Constant Solved Problems 1.20—1.22 Compressibility and Bulk Modulus Solved Problems 1.23—1.24 Surface Tension and Capillarity 1 Surface Tension on Liquid Droplet 16.2. Surface Tension on a Hollow Bubble L ‘Surface Tension on_a Liquid Jet Solved Problems 1.25—1.27 164, Capillarity Solved Problems 1.28—1.32 Vapour Pressure and Cavitation Highlights Exercise Chapter 2. Pressure and Its Measurement 2a. 2.2. 23. Fluid Pressure at a Point Pascal's Law Pressure Variation in a Fluid at Rest Solved Problems 2.1—2.7 @) Pages 134 24 35-68 35 35 37 ) 2.4. Absolute, Gauge, Atmospheric and Vacuum. Pressures Solved Problem 2.8 2.5. Measurement of Pressure Manometers ‘Mechanical Gauges ple Manometers Piezometer U-tube Manometer Solved Problems 2.9—2.13 2.6.3. Single Column Manometer Solved Problem 2.14 2.7. Differential Manometers 2.7.1, U-tube Differential Manometer Solved Problems 2.15—2.17 2.7.2. Inverted U-tube Differential Manometer Solved Problems 2.18—2.21 2.8. Pressure at a Point in Compressible Fluid 28.1. Isothermal Process Adiabatic Process ‘Temperature at any Point in Compressible Fluid ‘Temperature Lapse-Rate (L) Solved Problems 2.22—2.26 Highlights Exercise Chapter 3. Hydrostatic Forces on Surfaces 3.1, Introduction 8.2, Total Pressure and Centre of Pressure 3.3. Vertical Plane Surface Sub-merged in Liquid Solved Problems 3.1—3.12 8.4, Horizontal Plane Surface Sub-merged in Liquid Solved Problem 3.13, 8.5. Inclined Plane Surface Sub-merged in Liquid Solved Problems 3.14(a)—3.21 8.6. Curved Surface Sub-merged in Liquid Solved Problems 3.22—3.31 8.7. Total Pressure and Centre of Pressure on Lock Gates Solved Problems $.32—3.33 8.8. Pressure Distribution in a Liquid Subjected to Constant Horizontal/Vertical Acceleration 3.8.1. Liquid Containers Subject to Constant, Horizontal Acceleration Solved Problems 3.34—3.36 3.8.2. Liquid Containers Subjected to Constant Vertical Acceleration Solved Problems 3.37—3.38, Highlights Exercise 107 109 2 12 115 120 122 124 125 (ei) Chapter 4. Buoyancy and Floatation 4a. 4.2. 43. 44, 45. 46. Aa. 48. 49. Introduction Buoyancy Centre of Buoyancy Solved Problems 4.14.6 Meta-centre Meta-centrie Height Analytical Method for Meta-Centre Height Solved Problems 4.7—4.11 Conditions of Equilibrium of a Floating and ‘Sub-merged Bodies 4.7.1. Stability of a Sub-merged Body 4.72. Stability of a Floating Body Solved Problems 4.12—4.18 Experimental Method of Determination of ‘Meta-centrie Height Solved Problems 4.19—4.20 Oscillation (Rolling) of a Floating Body Solved Problems 4.21—4.22 Highlights Exercise Chapter 5. Kinematics of Flow and Ideal Flow Bal. 5.2. 5.3. 54. 5.5. 5.6. 5.7. 5.8. A. KINEMATICS OF FLOW Introduction ‘Methods of Deseribing Fluid Motion ‘Types of Fluid Flow 5.3.1, — Steady and Unsteady Flows 5.3.2. Uniform and Non-uniform Flows 5.3.3. Laminar and Turbulent Flows 5. Compressible and Incompressible Flows 5.3.5. Rotational and Irrotational Flows 5.3.6. One, two and Three-Dimensional Flows Rate of Flow or Discharge (Q) Continuity Equation Solved Problems 5.1—5.5 Continuity Equation in Three-Dimensions 5.6.1. Continuity Equation in Cylindrical Polar Co-ordinates Solved Problems 5.54. Velocity and Acceleration 5.7.1. Local Acceleration and Convective ‘Acceleration Solved Problems 5.6—5.9 Velocity Potential Function and Stream Function 5.8.1, Velocity Potential Function 5.82. Stream Funetion 5.8.3. Equipotential Line 5.84. Line of Constant Stream Function 181-162 131 131 131 131 136 136 137 138 43 143 143 44 154 155 156 158 159 160 163-258, 163 163 163 163 164 164 164 165 165 165 165 166 170 a. 173 44 175 175, 181 181 182 183 183 5.9. 5.10, BAL. Bz, 5B. Bad. 5.15. 5.16. BAT. (aii) 5.8.5. Flow Net 5.8.6. Relation between Stream Function and ‘Velocity Potential Function Solved Problems 5.10—5.17 ‘Types of Motion 5.9.1. Linear Translation 5.9.2. Linear Deformation 5.9.3. Angular Deformation or Shear Deformation 5.9.4. Rotation 5.95. Vorticity Solved Problems 6.18—5.19 Vortex Flow 5.10.1. Forced Vortex Flow 5.10.2. Free Vortex Flow 5.10.3. Equation of Motion for Vortex Flow 5.10.4. Equation of Forced Vortex Flow Solved Problems 5.20—5.25 5.10.5. Closed Cylindrical Vessels Solved Problems 5.26—5.31 5.10.6. Equation of Free Vortex Flow Solved Problem 5.32 (B) IDEAL FLOW (POTENTIAL FLOW) Introduction Important Cases of Potential Flow Uniform Flow 5.13.1. Uniform Flow Parallel to x-Axis, 5.13.2. Uniform Potential Flow Parallel to y-Axis| Source Flow Sink Flow Solved Problems 5.33—5.85 Free-Vortex Flow ‘Super-Imposed Flow 5.17.1. Source and Sink Pair Solved Problems 5.36—5.37 5.17.2. Doublet Solved Problem 5.38 5.17.3. A Plane Source in a Uniform Flow (Flow Past a Half-Body) Solved Problems 5.39—5.41 5.17.4. A Source and Sink Pair in a Uniform Flow (Flow Past Rankine Oval Body) Solved Problem 5.42 5.17.5. A Doublet in a Uniform Flow (Flow Past a Circular Cylinder) Solved Problems 5.43—5.44 Highlights Exercise 184 184 184 191 191 191 192 192 192 192 193 193, 194 195 196 197 202 202 209 210 210 au 2 au 213 214 216 216 219 221 221 225 228 231 233 237 241 244 246 250 252 254 (ii Chapter 6. Dynamics of Fluid Flow 61. 6.2. 63. 64. 6.5. 6.6. 6.7. 6.8. 6.9. 6.10. Introduction Equations of Motion Euler's Equation of Motion Bernoulli's Equation from Euler's Equation Assumptions Solved Problems 6.1—6.6 Bernoulli's Equation for Real Fluid Solved Problems 6.7—6.9 Practical Applications of Bernoulli's Equation 67.1, Venturimeter Solved Problems 6.10—6.21 6.7.2. — Orifice Meter or Orifice Plate Solved Problems 6.22 —6.23 6.7.3. — Pitot-tube Solved Problems 6.24—6.28 "The Momentum Equation Solved Problems 6.29—6.35 ‘Moment of Momentum Equation Solved Problems 6.36—6.37 Free Liquid Jets Solved Problems 6.38—641 Highlights Exercise Chapter 7. Orifices and Mouthpieces Ta. 7.2. 73. 14. 15. 7.6. 11. 7.8. Introduction Classifications of Orifices Flow Through an Orifice Hydraulic Co-efficients 1. Covefficient of Velocity (C,) 742. Covefficient of Contraction (C,) 743. Covefficient of Discharge (C,) Solved Problems 7.1—7.2 Experimental Determination of Hydraulic Coefficients Toa Determination of Co-fixion of Dincharg (C7 Determination of Co-efficient of Velocity (C,) 752. Determination of Co-efficient of Contraction (C,) Solved Problems 7.3—7.10 Flow Through Large Orifices 7.6.1. Discharge Through Large Rectangular Orifice Solved Problems 7.11—7.13 Discharge Through Fully Sub-merged Orifice Solved Problems 7.14—7.15 Discharge Through Partially Sub-merged Orifice Solved Problem 7.16 259-316 259 259 260 261 261 261 265 266 268 268 270 281 283 285 286 288 289 298 298 301 303 307 309 317-354 317 317 317 318 318 319 319 319 320 320 321 321 321 327 328 828 330 331 331 382 79. 710, 7a. 72, 713. 74. 715. xiv) ‘Time of Emptying a Tank Through an Orifice at its Bottom Solved Problems 7.17—7.18 ‘Time of Emptying a Hemispherical Tank Solved Problems 7.19—7.21 ‘Time of Emptying a Circular Horizontal Tank Solved Problems 7.22—7.23 Classification of Mouthpieces Flow Through an External Cylindrical Mouthpiece Solved Problems 7.24—7.25 Flow Through a Convergent-Divergent Mouthpiece Solved Problems 7.26—7.28 Flow Through Internal or Re-entrant on Borda's Mouthpiece Solved Problem 7.29 Highlights Exercise Chapter 8. Notches and Weirs 86. 8.7. 89. 8.10. aul. 82, 813. Introduction . Classification of Notches and Weirs . Discharge Over a Rectangular Notch or Weir Solved Problems 8.18.3 . Discharge Over a Triangular Notch or Weir Solved problems 8.48.6 5. Advantages of Triangular Notch or Weir over Rectangular Noteh or Weir Discharge Over a Trapezoidal Notch or Weir Solved Problem 8.7 Discharge Over a Stepped Notch Solved Problem 8.8 Effect on Discharge Over a Notch or Weir Due to Error in the Measurement of Head 88.1. For Rectangular Weir or Notch 882. For Triangular Weir or Notch Solved Problems 8.9—8.11 (a) Time Required to Empty a Reservoir or a ‘Tank with a Rectangular Weir or Notch (b) Time Required to Empty a Reservoir or a ‘Tank with a Triangular Weir or Notch Solved Problems 8.12—8.14 Velocity of Approach Solved Problems 8.15—8.19 Empirical Formulae for Discharge Over Rectangular Weir Solved Problems 8.20—8.22 Cipolletti Weir or Notch Solved Problems 8.23—8.24 Discharge Over a Broad-Crested Weir 332 333 335 338 366 367 368 370 370 374 374 376 377 378 ad. 815. 816. (w) Discharge Over a Narrow-Crested Weir Discharge Over an Ogee Weir Discharge Over Sub-merged or Drowned Weir Solved Problems 8.25—827 Highlights Exercise Chapter 9. Viscous Flow 9.1. 9.2. 9.3. 9.4. 9.5. 9.6. 9.7. 9.8. Introduction Flow of Viscous Fluid Through Circular Pipe Solved Problems 9.19.6 Flow of Viscous Fluid between Two Parallel Plates Solved Problems 9.7—9.12 Kinetic Energy Correction and Momentum, Correction Factors Solved Problem 9.13 Power Absorbed in Viscous Flow 9.5.1. Viscous Resistance of Journal Bearings Solved Problems 9.14—9.18 9.5.2. Viscous Resistance of Foot-step Bearing Solved Problems 9.19—9.20 9.5.3. Viscous Resistance of Collar Bearing Solved Problems 9.21—9.22 Loss of Head Due to Friction in Viscous Flow Solved Problems 9.23—9.24 ‘Movement of Piston in Dash-pot Solved Problem 9.25 ‘Methods of Determination of Co-efficient of Viscosity 98.1. Capillary Tube Method Falling Sphere Resistance Method Rotating Cylinder Method Orifice Type Viscometer Solved Problems 9.26—9.32 Highlights Exercise Chapter 10. Turbulent Flow 10.1. 10.2. 10.3. 10.4. Introduction Reynolds Experiment, Frictional Loss in Pipe Flow 10.3.1. Expression for Loss of Head Due ‘to Frietion in Pipes 10.3.2. Expression for Co-efficient of Friction in Terms of Shear Stress Shear Stress in Turbulent Flow 10.4.1. Reynolds Expression for Turbulent Shear Stress 10.4.2. Prandtl Mixing Length Theory for ‘Turbulent Shear Stress 379 379 379 380 381 383 387-432 387 387 391 397 400 404 404 407 407 408 4 412 412 413 aut 415 417 418 419 419 420 421 422 423 427 429 433-464 433 433 434 434 436 437 437 438 10.5. 10.6. (xvi) Velocity Distribution in Turbulent Flow in Pipes 10.5.1. Hydrodynamically Smooth and Rough Boundaries 10.5.2. Velocity Distribution for Turbulent Flow in Smooth Pipes 10.5.3. Velocity Distribution for Turbulent Flow in Rough Pipes Solved Problems 10.1—10.4 10.5.4. Velocity Distribution for Turbulent Flow in Terms of Average Velocity Solved Problems 10.5—10.6 10.5.5. Velocity Distribution for Turbulent Flow in Smooth Pipes by Power Law Resistance of Smooth and Rough Pipes Solved Problems 10.7—10.13 Highlights Exercise Chapter 11. Flow Through Pipes wa. 112. 1s. 4. 1s. 11.6. 17. 11.8. 19. 11.10. Introduction Loss of Energy in Pipes Loss of Energy (or head) Due to Friction Solved Problems 11.1—11.7 ‘Minor Energy (Head) Losses, 11.4.1. Loss of Head Due to Sudden Enlargement 11.4.2. Loss of Head Due to Sudden Contraction Solved Problems 11.8—11.14 11.4.8. Loss of Head at the Entranee of a Pipe 114.4. Loss of Head at the Exit of Pipe 11.4.5. Loss of Head Due to an Obstruction ina Pipe 11.4.6. Loss of Head Due to Bend in Pipe 11.4.7. Loss of Head in Various Pipe Fittings Solved Problems 11.15—11.21 Hydraulie Gradient and Total Energy Line 115.1. Hydraulic Gradient Line 11.5.2. Total Energy Line Solved Problems 11.22—11.26 Flow Through Syphon Solved Problems 11.27—11.29 Flow Through Pipes in Series or Flow Through Compound Pipes Solved Problems 11.30—11.30A. Equivalent Pipe Solved Problem 11.31 Flow Through Parallel Pipes Solved Problems 11.32—11.41 Flow Through Branched Pipes Solved Problems 11.42—11.44 438 440 441 442 442 446 450 450 453 461 462 465-558 465 465 465 467 an an 473 474 482 482 483 483 491 491 491 498 498 508 524 525 wu 1.12. 11.13. 1.44. (wwii) Power Transmission Through Pipes 1111.1. Condition for Maximum ‘Transmission of Power 1.11.2. Maximum Efficieney of Transmission of Power Solved Problems 11.45—11.47 Flow Through Nozzles 11.2.1. Power Transmitted Through Nozzle 11.122. Condition for Maximum Power ‘Transmitted Through Nozzle 11.12.83. Diameter of Nozale for Maximum ‘Transmission of Power Through Nozzle Solved Problems 1148—11.51 Water Hammer in Pipes 11.13.1, Gradual Closure of Valve 11.132. Sudden Closure of Valve and Pipe is Rigid 11.13.8. Sudden Closure of Valve and Pipe is Elastic 11.134, Time Taken by Pressure Wave to Travel from the Valve to the Tank and from Tank to the Valve Solved Problems 11.52—11.55 Pipe Network 1114.1. Hardy Cross Method Solved Problem 11.56 Highlights Exercise Chapter 12. Dimensional and Model Analysis 12a. 122. 12.3. 124. 12.5. 12.6. 12.7. 12.8, Introduction Secondary or Derived Quantities Solved Problem 12.1 Dimensional Homogeneity ‘Methods of Dimensional Analysis, 12.4.1. Rayleigh's Method Solved Problems 12.2—12.7 12.4.2. Buckingham’s x-Theorem 12.4.3. Method of Selecting Repeating Variables 12.4.4. Procedure for Solving Problems by Buckingham's x-Theorem Solved Problems 12.8—12.14 ‘Model Analysis Similitude-Types of Similarities ‘Types of Forces Acting in Moving Fluid Dimensionless Numbers 12.8.1. Reynold’s Number (R,) 12.82. Froude’s Number (F,) 12.8.3. Euler's Number (£,) 12.8.4. Weber's Number (W,) 12.8.5. Mach’s Number (M) 530 581 537 537 538 539 BAL 542 542 543 545 545 BAT 549 552 554 559-610 559 559 560 561 561 561 562 565 566 566 568 578 579 580 581 581 582 582 582 12.9. 12.10. 1211, Chapter 13.1. 13.2. 13.3, 13.4. 13.5. 13.6. 13.7. (wit) ‘Model Laws or Similarity Laws 12.9.1, Reynold’s Model Law Solved Problems 12.15—12.18 12.9.2. Froude Model Law Solved Problems 12.19—12.27 12.9.3. Euler's Model Law 12.9.4. Weber Model Law 12.9.5. Mach Model Law Solved Problem 12.28 Model Testing of Partially Sub-merged Bodies, Solved Problems 12.29—12.32 Classification of Models 12.1.1. Undistorted Models 12.11.2. Distorted Models 12.11.3. Seale Ratios for Distorted Models Solved Problem 12.33 Highlights Exercise 13. Boundary Layer Flow Introduction Definitions 13.2.1, Laminar Boundary Layer 13.2.2. Turbulent Boundary Layer 13.2.3. Laminar Sub-layer 13.2.4, Boundary Layer Thickness (5) 13.2.5. Displacement Thickness (3*) 13.2.6, Momentum Thickness (8) 13.2.7. Energy Thickness (5**) Solved Problems 13.1—13.2 Drag Force on a Flat Plate Due to Boundary Layer 13.8.1, Local Co-efficient of Drag [Cp*1 13.3.2. Average Co-efficient of Drag [Cp] 13.3.8, Boundary Conditions for the Velocity Profiles Solved Problems 13.3—13.12 Turbulent Boundary Layer on a Flat Plate Solved Problem 13.13 Analysis of Turbulent Boundary Layer Total Drag on a Flat Plate Due to Laminar and ‘Turbulent Boundary Layer Solved Problems 13.14—13.17 Separation of Boundary Layer 13.7.1. Effect of Pressure Gradient on Boundary Layer Separation 13.7.2. Location of Separation Point Solved Problem 13.18 583 583 584 587 590 595 596 596 597 598 600 604 604 605 605 608 606 607 611-656 eu 612 612 613 613 613 613 615 615 616 619 622 622 622 622 638 638 641 641 642 648 648 649 650 (xix) 13.7.3. Methods of Preventing the Separation of Boundary Layer Highlights Exercise Chapter 14. Forces on Sub-merged Bodies 14a. 14.2. 14.3. 14.4. 145. 14.6. 147. 14.8, Introduction Force Exerted by a Flowing Fluid on a Stationary Body M421. Drag 14.22. Lift Expression for Drag and Lift 14.3.1, Dimensional Analysis of Drag and Lift Solved Problems 14.1—14.15 14.3.2. Pressure Drag and Friction Drag 14.3.3. Stream-lined Body 14.34. Bluff Body Drag on a Sphere Solved Problem 14.16 ‘Terminal Velocity of a Body Solved Problems 14.17—14.20 Drag on a Cylinder Development of Lift on a Circular Cylinder 14.7.1. Flow of Ideal Fluid Over Stationary Cylinder 14.7.2. Flow Pattern Around the Cylinder when a Constant Circulation T is Imparted to the Cylinder 14.7.3. Expression for Lift Foree Acting on Rotating Cylinder 14.74. Drag Force Acting on a Rotating Cylinder 14.7.5. Expression for Lift Co-efficient for Rotating Cylinder 14.7.6. Location of Stagnation Points for a Rotating Cylinder in a Uniform Flow-field 14.7.7. Magnus Effect Solved Problems 14.21—14.23, Development of Lift on an Airfoil 14.8.1. Steady-state of a Flying Object, Solved Problems 14.24—14.25 Highlights Exercise Chapter 15. Compressible Flow 15. 1522. Introduction ‘Thermodynamic Relations 15.2.1. Equation of State 15.2.2. Expansion and Compression of Perfect Gas 651 678 680 682 682 15.3. 154. 15.5. 15.6, 15.7. 15.8, 15.9. 15.10. 15.11. x) Basie Equations of Compressible Flow 15.8.1. Continuity Equation 15.3.2. Bernoulli's Equation Solved Problems 15.1—15.3 15.3.3. Momentum Equations Velocity of Sound or Pressure Wave in a Fluid 15.4.1. Expression for Velocity of Sound ‘Wave in a Fluid 15.4.2. Velocity of Sound in Terms of Bulk Modulus 15.4.3. Velocity of Sound for Isothermal Process. 15.4.4. Velocity of Sound for Adiabatic Process Mach Number Solved Problems 15.4—15.7 Propagation of Pressure Waves (or Disturbances ) in a Compressible Fluid 15.6.1. Mach Angle 15.6.2. Zone of Action 15.6.3. Zone of Silence Solved Problems 15.8—16.10 Stagnation Properties 15.7.1. Expression for Stagnation Pressure (p,) 15.7.2. Expression for Stagnation Density (p,) 15.7.3. Expression for Stagnation Temperature (T,) Solved Problems 15.11—15.12 Area Velocity Relationship for Compressible Flow Flow of Compressible Fluid Through Orifices and Nozzles Fitted to a Large Tank 15.9.1. Value of n or 21 for Maximum Value Pa of Mass Rate of Flow 15.9.2. Value of V, for Maximum Rate of Flow of Fluid 15.9.3. Maximum Rate of Flow of Fluid Through Nozzle e 15.9.4. Variation of Mass Rate of Flow of Compressible Fluid with Pressure ratio (24) 15.9.5. Velocity at Outlet of Nozzle for Maximum Rate of Flow is Equal to Sonie Velocity Solved Problems 15-13—15.15 Mass Rate of Flow of Compressible Fluid Through Veaturimeter Solved Problem 15.16 Pitot Static Tube in a Compressible Flow Solved Problem 15.17 Highlights Exercise 695 695 695 697 702 702 702 704 705 705 705 706 708 709 710 710 710 m1 mu 715 75 716 718 79 a1 722 723 723 724 727 728 730 731 731 734 (oxi) Chapter 16. Flow in Open Channels 137-802 16.1. Introduction 737 16.2. Classification of Flow in Channels os 737 16.2.1. Steady Flow and Unsteady Flow : 737 162.2. Uniform Flow and Non-uniform Flow 737 16.2.3. Laminar Flow and Turbulent Flow 738 16.2.4. Sub-eritical, Critical and Super-Critical Flow 738 16.3. Discharge Through Open Channel by Chezy's Formula 739 Solved Problems 16.1—16.7 ” 740 16.4. Empirical Formulae for the Value of Chezy’s Constant 744 Solved Problems 16.8—16.12 os 145 16.5. Most Economical Section of Channels - 749 16.5.1. Most Economical Rectangular Channel 749 Solved Problems 16.13—16.15 ” 750 16.5.2. Most Economical Trapezoidal Channel 752 Solved Problems 16.16—16.22 - 54 16.5.3. Best Side Slope for Most Economical ‘Trapezoidal Section 762 Solved Problems 16.23—16.24 . 763 16.5.4. Flow Through Circular Channel 766 Solved Problems 16.25—16.29 ” 766 16.5.5. Most Economical Circular Section . 77 Solved Problems 16.30—16.32 - 115 16.6. Non-Uniform Flow through Open Channels TT 16.1. Specific Energy and Specific Energy Curve : 17 16.7.1. Critical Depth (A) c 779 16.7.2. Critical Velocity (V,) 5 779 16.7.3. Minimum Specific Energy in Terms of Critical Depth e 780 Solved Problems 16.33—16.35 : 780 16.7.4. Critical Flow 781 16.7.5. Streaming Flow or Sub-critical Flow or ‘Tranquil Flow 782 16.7.6. Super-Critical Flow or Shooting Flow or ‘Torrential Flow oe 782 16.7.7. Alternate Depths : 782 16.7.8. Condition for Maximum Discharge for a Given Value of Specifie Energy 782 Solved Problems 16.6—16.37 - 782 16.8. Hydraulic Jump or Standing Wave 783 16.8.1. Expression for Depth of Hydraulic Jump 784 16.8.2. Expression for Loss of Energy Due to Hydraulic Jump 786 16.8.3. Expression for Depth of Hydraulic Jump in Terms or Upstream Froude Number e 787 16.8.4. (xii) Length of Hydraulic Jump Solved Problems 16.38—16.42 16.9. Gradually Varied Flow (G.V-F.) 16.9.1 16.9.2. 16.9.3. Equation of Gradually Varied Flow Solved Problems 16.43—16.44 Back Water Curve and Affux Expression for the Length of Back Water Curve Solved Problem 16.45 Highlights Exercise Chapter 17. Impact of Jets and Jet Propulsion 17.1. Introduction 17.2. Force Exerted by the Jet on a Stationary Vertical Plate 12.1. 17.2.2. Force Exerted by a Jet on Stationary Inclined Flat Plate Force Exerted by a Jet on Stationary Curved Plate Solved Problems 17.1—17.6 17.8. Force Exerted by a Jet on a Hinged Plate Solved Problems 17.7—17.10 (a) 174. Force Exerted by a Jet on Moving Plates MALL 174.2. 1743. 14a. 1145. 1746. Force on Flat Vertical Plate Moving in the Direction of Jet Force on the Inclined Plate Moving in ‘the Direction of the Jet Solved Problems 17.11—17.13 Force on the Curved Plate when the Plate is Moving in the Direction of Jet Solved Problems 17.14—17.17 Force Exerted by a Jet of Water on an Unsymmetrical Moving Curved Plate when Jet Strikes Tangentially at one of the Tips Solved Problems 17.18—17.23 Force Exerted by a Jet of Water on a Series of Vanes Force Exerted on a Series of Radial Curved Vanes Solved Problems 17.24—17.26 17.5. Jet Propulsion 175.1. 17.5.2. Jet Propulsion of a Tank with an Orifice Solved Problems 17.27—17.28 Jet Propulsion of Ships Solved Problems 17.29—17.33 Highlights Exercise 787 787 790 790 792 793 794 795 796 799 ‘803-852 803 803 804 805 807 809 810 14 815 815 816 818 819 823 826 833 (exit) Chapter 18, Hydraulic Machines—Turbines 18.1. 18.2. 18.3. 184. 18.5. 18.6. 18.7. 18.8. 18.9. 18.10. 18.11. 18.12. 18.13. Introduction Turbines General Layout of a Hydroelectric Power Plant Definitions of Heads and Efficiencies of a Turbine Classification of Hydraulic Turbines Pelton Wheel (or Turbine) 186.1. Velocity Triangles and Work Done for Pelton Wheel 186.2. Points to be Remembered for Pelton Wheel Solved Problems 18.1—18.10 18.6.3. Design of Pelton Wheel Solved Problems 18.11—18.13 Radial Flow Reaction Turbines 18.7.1. Main Parts of a Radial Flow Reaction Turbine 18.7.2. Inward Radial Flow Turbine 18.7.3. Degree of Reaction 18.7.4, Definitions Solved Problems 18.14—18.20 18.7.5. Outward Radial Flow Reaction Turbine Solved Problems 18.21—18.22 Francis Turbine 18.8.1, Important Relations for Francis Turbines Solved Problems 18.23—18.26 Axial Flow Reaction Turbine 18.9.1. Some Important Point for Propeller (Kaplan Turbine) Solved Problems 18.27—18.33 Draft-Tube 18.10.1. ‘Types of Draft Tubes 18.10.2, Draft-Tube Theory 18.103. Efficiency of Draft-Tube Solved Problems 18.33 (a)—18.35, Specific Speed 18.111, Derivation of the Specifie Speed 18.1.2. Significance of Specific Speed Solved Problems 18.36—18.41 Unit Quantities 18.12.1. Unit Speed 18.122. Unit Discharge 18.12.3. Unit Power 18.124. Use of Unit Quantities (N,, Qu, P.) Solved Problems 18.41 (a)—18.45 Characteristic Curves of Hydraulic Turbines 18.13.1. Main Characteristic Curves or Constant Head Curves 18.13.2, Operating Characteristic Curves or Constant Speed Curves ‘853-944 853 853 853 853 856 857 905 905 915 915 916 916 917 920 920 921 921 927 927 928 928 929 933 933 934 18.14. (xxiv) 18.13.3. Constant Efficiency Curves or Muschel Curves or Iso-Efficieney Curves Governing of Turbines Highlights Exercise Chapter 19. Centrifugal Pumps 19.1. 19.2. 19.3. 19.4. 19.5. 19.6. 19.7. 19.8. 19.9. 19.10. 19.11. 19.12. 19.13. 19.14. Introduction Main Parts of a Centrifugal Pump Work Done by the Centrifugal Pump (or by Impfier) on Water Definitions of Heads and Efficiencies of a Centrifugal Pump Solved Problems 19.1—19.12 Minimum Speed for Starting a Centrifugal Pump Solved Problems 19.13—19.15 Multistage Centrifugal Pumps 19.6.1. Multistage Centrifugal Pumps for High Heads 19.6.2. Multistage Centrifugal Pumps for High Discharge Solved Problems 19.16—19.17 Specific Speed of a Centrifugal Pump (N,) 19.7.1. Expression for Specific Speed for a Pump Model Testing of Centrifugal Pumps Solved Problems 19.18—19.22 Priming of a Centrifugal Pamp Characteristic Curves of Centrifugal Pumps 19.10.1, Main Characteristic Curves 1910.2. Operating Characteristic Curves 19.10.3. Constant Efficiency Curves Cavitation 19.11.1. Precaution Against Cavitation 19.112. Effects of Cavitation 19.11.3. Hydraulic Machines Subjected to Cavitation 19.114. Cavitation in Turbines 19.115. Cavitation in Centrifugal Pumps Solved Problem 19.23 Maximum Suction Lift (or Suction Height) Net Positive Suction Head (NPSH) Cavitation in Centrifugal Pump Solved Problem 19.24 Highlights Bxereise Chapter 20. Reciprocating Pumps 20.1. 20.2. 20.3. Introduction Main Parts of a Reciprocating Pump Working of a Reciprocating Pump 935 936 937 939 945-992 945 945 947 948 951 965 966 968 968 969 969 971 971 972 973 978 978 978 979 979 980 980 981 981 981 981 982 983 985 985 986 987 989 993-1040 993 993 994 20.4. 20.5. 20.6. 20.7. 20.8. 20.9. 20.10. oo) 20.3.1. Discharge Through a Reciprocating Pump 20.3.2, Work Done by Reciprocating Pump 20.3.3. Discharge, Work Done and Power Required to Drive a Double-acting Pump Slip of Reciprocating Pump 20.4.1. Negative Slip of the Reciprocating Pump Classification of Reciprocating Pumps Solved Problems 20.1—20.2 Variation of Velocity and Acceleration in the Suction and Delivery Pipes Due to Acceleration of the Piston Effect of Variation of Velocity on Friction in the Suction and Delivery Pipes Solved Problem 20.3 Indicator Diagram 20.8.1. Ideal Indicator Diagram 20.8.2. Bffect of Acceleration in Suction and Delivery Pipes on Indicator Diagram Solved Problems 20.4—20.9 20.8.8. Bffect of Friction in Suction and Delivery Pipes on Indicator Diagram 20.8.4. Biffect of Acceleration and Friction in Suction and Delivery Pipes on Indicator Diagram Solved Problems 20.10—20.12 20.8.5. Maximum Speed of a Reciprocating Pump Solved Problem 20.13 Air Vessels Solved Problems 20.14—20.18 Comparison between Centrifugal Pumps and Reciprocating Pumps Highlights Exercise Chapter 21. Fluid System 21a. 21.2. 21.3. 214. 21.5. Introduction ‘The Hydraulic Press 21.2.1. Mechanical Advantage 212.2. Leverage of the Hydraulic Press 21.2.3. Actual Heavy Hydraulic Press Solved Problems 21.1—21.5 ‘The Hydraulic Accumulator 21.3.1. Capacity of Hydraulic Accumulator Solved Problems 21.6—21.11 21.3.2. Differential Hydraulic Accumulator ‘The Hydraulic Intensifier Solved Problems 21.12—21.18 ‘The Hydraulic Ram Solved Problems 21.14—21.15 994 995 995 996 997 997 997 998 1001 1001, 1003, 1003 1004 1004 1012 1013 1015 1019 1020 1021, 1030 1037 1037 1038 1041-1070 1041 1041 1042 1042 1042 1043, 1045, 1046 1047 1051 1051 1053 1053 1055 (xvi) 21.6. The Hydraulic Lift 21.6.1. Direct Acting Hydraulic Lift 21.6.2. Suspended Hydraulic Lift Solved Problems 21.16—21.17 21.7. The Hydraulic Crane Solved Problems 21.18—21.20 21.8. The Fluid or Hydraulic Coupling 21.9. The Hydraulic Torque Converter 21.10. The Air Lift Pump 21.11. The Gear-Wheel Pump Highlights Exercise Objective Type Questions Appendix Subject Index 1056 1057 1057 1058 1060 1060 1063, 1064 1065 1066 1067 1068 1071-1094 1095-1096 1097-1102 PROPERTIES OF rao tN > 1.1 INTRODUCTION Fluid mechanics is that branch of science which deals with the behaviour of the fluids (liquids or ‘gases) at rest as well as in motion. Thus this branch of science deals with the static, kinematics and dynamic aspects of fluids. The study of fluids at rest is called fluid staties. The study of fluids in motion, where pressure forces are not considered, is called fluid kinematics and if the pressure forces are also considered for the fluids in motion, that branch of science is called fluid dynamics. > 1.2 PROPERTIES OF FLUIDS 1.2.1 Density or Mass Density. Density or mass density of a fluid is defined as the ratio of the ‘mass of a fluid to its volume. Thus mass per unit volume of a fluid is called density. It is denoted by the symbol p (tho). The unit of mass density in SI unit is kg per cubic metre, i, kg/m®. The density of liquids may be considered as constant while that of gases changes with the variation of pressure and temperature. ‘Mathematically, mass density is written as Mass of fluid ‘Volume of fluid © ‘The value of density of water is 1 gm/cm? or 1000 kg/m>, 1.2.2. Specific Weight or Weight Density. Specific weight or weight density of a fluid is the ratio between the weight of a fluid to its volume. Thus weight per unit volume of a fluid is called ‘weight density and itis denoted by the symbol w. Weight of fluid _ (Mass of fluid) x Acceleration due to gravity ‘Thus mathematically, " * Volume of uid Volume of fluid = Mass of fluid x g © Volume of uid cox { Mass of luid__ } ace Volume of fluid wpe (ay [2 Fluid Mechanics ] ‘The value of specific weight or weight density (w) for water is 9.81 x 1000 Newton/m* in SI units. 1.2.3 Specific Volume. Specific volume of a fluid is defined as the volume of a fluid occupied. by a unit mass or volume per unit mass of a fluid is called specific volume. Mathematically, it is expressed as Volume of fluid 1 1 volume Mass of fluid Mass of fluid“ 5 ‘Volume of fluid ‘Thus specific volume is the reciprocal of mass density. It is expressed as m’/kg. It is commonly applied to gases. Speci 1.2.4. Specific Gravity. Specific gravity is defined as the ratio of the weight density (or density) of a fluid to the weight density (or density) of a standard fluid. For liquids, the standard fluid is taken water and for gases, the standard fluid is taken air. Specific gravity is also called relative density. Itis dimensionless quantity and is denoted by the symbol S. Weight density (density) of liquid ‘Weight density (density) of water ‘Weight density (density) of gas Weight density (density) of air Sx Weight density of water X 1000 x 9.81 Nim? Sx Density of water = 5x 1000 kg/m’, (LIA) If the specific gravity of a fluid is known, then the density of the fluid will be equal to specific gravity of fluid multiplied by the density of water. For example, the specific gravity of mercury is 13.6, hence density of mercury = 13.6 x 1000 = 13600 kg/m’. Mathematically, S(for liquids) ‘S(for gases) = ‘Thus weight density of a liqui ‘The density of a liquid Problem 1.1 Calculate the specific weight, density and specific gravity of one litre of a liquid which weighs 7 N. Solution. Given : 1 1 ow 3 Volume = 1 tre = itre= ire = lume = 1 tire= 55m? (' Thte=—A morte = 1000 em) Weight = 7N Weight ___7N i), Specific weight (w a OE: CD) ‘Volume ( 1 a 1000 (i Density (p) = A = me kg/m? = 713.5 kg/m®. Ans. Density of liquid _ 7135 Density of water 1000 (Density of water = 1000 kg/m?} (iii) Specific gravity [ Properties of Fluids 3) Problem 1.2 Calculate the density, specific weight and weight of one litre of petrol of specific gravity = 0.7 Solution. Given: Volume = 1 hie= 1 1000 em® = 02 m*= 0.001 m? Sp. gravity s=07 (0 Density (p) Using equation (1.14), Density (p) = 5 x 1000 kg/m? = 0.7 x 1000 = 700 kg/m*, Ans. (id) Specific weight (w) Using equation (1.1), w= px g= 7009.81 Nim? = 6867 N/m’. Ans. (i) Weight (W) Weight We know that specific weight = a ete Volume or 6867 = ‘0007 0.001 W= 6867 x 0.001 = 6.867 N. Ans. > 1.3 VISCOSITY Viscosity is defined as the property of a fluid which offers resistance to the movement of one layer of fluid over another adjacent layer of the fluid. When two layers of a fluid, a distance “dy” apart, move one over the other at different velocities, say u and u + du as shown in Fig. I.1, the viscosity together with relative velocity causes a shear stress acting between the fluid layers, The top layer causes a shear stress on the adjacent lower ayer wie te lowe layer eases \ a shear stress on the adjacent top layer. This shear a stres is proportional tothe rate of change of ve- t [Lew locity with respect to y. It is denoted by symbol y a ft) A-veoory prone Mathematically, eo or 1.2) ‘where 1 (called mu) is the constant of proportionality and is known as the co-efficient of dynamic viscosity ony viscosity. represent thereof shear stn rate of shea efomatono velo grate du dy Thus viscosity is also defined as the shear stress required to produce unit rate of shear strain. 1.3.1 Units of Viscosity. The units of viscosity is obtained by putting the dimensions of the {quantities in equation (1.3) From equation (1.2), we have jt (1.3) Fig. 11 Velocity variation near a solid boundary. [4 Fluid Mechanics ] Shear stress Foree/Area Change of velocity EG. Tene), Change of distance (Time )* Length Force/(Length)*_ Force x Time 1 (Length) Time In MKS system, force is represented by kgf and length by metre (m), in CGS system, force is represented by dyne and length by cm and in SI system force is represented by Newton (N) and length by metre (m).. 1. MKS unit of viscosity = REESE m dyne-see em In the above expression N/m? is also known as Pascal which is represented by Pa, Hence N/m? = Pascal CGS unit of viscosity Sl unit of viscosity = Ns/m? = Pa s. Sl unit of viscosity Newion-see _ Ne ‘The numerical conversion of the unit of viscosity from MKS unit to CGS unit is given below : 981N-see one kgf-sec (1 kgf= 9.81 Newton) m m But one Newton = one kg (mass) x one ( ) (acceleration) wee (1000 gm) x (100 em) ~ sec 1000 x 100 © = 1000 x 100 dyne , dyne=am x $2} me KBE-SEE 9 81 109000 IEE « 0,81 x 100000 9H m = 100 x 100 x en = 98.1 HME 08.1 poise {: sneree, Poise} ‘Thus for solving numerical problems, if viscosity is given in poise, it must be divided by 98.1 to get its equivalent numerical value in MKS. one kgf-sec _ 981NS one Ns _ 981 LNs 8 poise = 10 poise or One poise = + NS m 981° a Pose 0 m But = 98.1 poise [ Properties of Fluids 5) Alternate Method. One poise = 2%°%S. (teste xo But dyne kg One poise m too 2-1 KS, ke sm 10 sm sm Note, (i) In SI units second is represented by ‘s’ and not by ‘sec’. 4i)_IE viscosity is given in pois, it must be divided by 10 to get its equivalent numerical value in SI unit. Sometimes a unit of viscosity as centipoise is used where 1 1 1 centipoise = 1 poise or 1eP=—- p [cP = Cemtipoise, P = Poise poise = 755 pol mm? & ip 1 ‘The viscosity of water at 20°C is 0.01 poise oF 1.0 centipoise 1.3.2. Kinematic Viscosity. It is defined as the ratio between the dynamic viscosity and density of fluid. It is denoted by the Greek symbol (v) called ‘nu’. Thus, mathematically, Viscosity _ a Density ~ p ‘The units of kinematic viscosity is obtained as Unitsof_ Force x Time ___ Force x Time Units of (Leng? x Mass" “Mass Teng)” — Length Length Mass x xTime = (Time)? b Mass Length (Length)? = Time” In MKS and SI, the unit of kinematic viscosity is metre”/sec or m*/sec while in CGS units it is written as em?/s. In CGS units, kinematic viscosity is also known as stoke. ts ; cette = ue 100 1.3.3. Newton's Law of Viscosity. It states that the shear stress (¢) on a fluid element layer is directly proportional to the rate of shear strain. The constant of proportionality is called the co- efficient of viscosity. Mathematically, itis expressed as given by equation (1.2) or as [6 Fluid Mechanics ] Fluids which obey the above relation are known as Newtonian fluids and the fluids which do not ‘obey the above relation are called Non-Newtonian fluids. 1.3.4 Variation of Viscosity with Temperature. Temperature affects the viscosity. The viscosity of liquids decreases with the increase of temperature while the viscosity of gases increases With the increase of temperature. This is due to reason that the viscous forces in a fluid are due to cohesive forces and molecular momentum transfer. In liquids, the cohesive forces predominates the molecular momentum transfer, due t0 closely packed molecules and with the increase in temperature, the cohesive forces decreases with the result of decreasing viscosity. But in case of gases the cohesive forces are small and molecular momentum transfer predominates. With the increase in temperature, molecular momentum transfer increases and hence viscosity increases. The relation between viscosity and temperature for liquids and gases are: .(— ay Trartpr where t= Viscosity of liquid at °C, in poise ‘iscosity of liquid at 0°C, in poise (0 For liquids, ©, B= Constants forthe liquid For water, Hy = 1.79 x 10° poise, «= 0.03368 and B = 0.000221. Equation (1.4) shows that with the increase of temperature, the viscosity decreases. (ii) For a gas, H= Hy +r - BP (1.48) where for air [ly = 0.000017, 0 = 0.000000056, f = 0.1189 x 10°°. Equation (1.48) shows that with the increase of temperature, the viscosity increases. 1.3.5 Types of Fluids. The fluids may be classified into the following five types : 1. Ideal fluid, 2. Real fluid, 3. Newtonian fluid, 4, Non-Newtonian fluid, and 5. Ideal plastie fluid. 1. Ideal Fluid. A fluid, which is incompressible and is having no viscosity, is known as an ideal fluid. eal fluid is only an imaginary fluid as all the fluids, which exist, have some viscosity 2. Real Fluid. A fluid, which possesses viscosity, is known as real fluid. All the fluids, in actual practice, are real fluids. 3. Newtonian Fluid. A real fluid, in which the shear stress is directly proportional to the rate of shear strain (or velocity gradient, is known as a Newtonian fluid. — vetoorry eraoienr ($3) 4. Non-Newtonian Fluid. A real fluid, in which the ‘yi shear stress is not proportional to the rate of shear strain (or ig. 1.2 Types of fluids. velocity gradient), known as a Non-Newtonian fluid. 5. Ideal Plastic Fluid. A fluid, in which shear stress is more than the yield value and shear stress is proportional to the rate of shear strain (or velocity gradient), is known as ideal plastic fluid. —+ shear srress: Problem 1.3. if the velocity distribution over a plate is given by u = z y =¥ in which w is the velocity in metre per second at a distance y metre above the plate, determine the shear stress at y = Ocand y = 0.15 m. Take dynamic viscosity of fluid as 8.63 poises. [ Properties of Fluids 7) Solution. Given : u ),_.( w 863 Value of [= 8.63 poise SI units = 0.863 N sim? Now shear stress is given by equation (1.2) as t= #. (0 Shear suess at y= 0 is given by cas) setoroenasrmican (i) Shear stress at y= 0.15 mis given by ©, -01s=H (#) 7 088 0.367 = 03167 Nin. A Problem 1.4 A plate 0.025 mm distant from a fixed plate, moves at 60 cm/s and requires a force of 2N per unit area i.e., 2 N/m to maintain this speed. Determine the fluid viscosity between the plates. Solution. Given : 1 Distance between plates, dy = .025 mm B 025 x 107 m Velocity of upper plate, = 60 em/s = 0.6 mis Te N FIXED PLATE. Force on upper plate, e205 9 ‘This is the value of shear stress i.e., Let the fluid viscosity between the plates is 1. du Using the equation (1.2), we have t where 2 OX M2510" gy 495 BE = 8.33 x 10° x 10 poise = “ poise. Ans. Problem 1.5 A flat plate of area 1.5 x 10° mn’ is pulled with a speed of 04 mis relative 10 ‘another plate located ata distance of 0.15 mm from it Find the force and power required to maintain this speed, ifthe fluid separating them is having viscosity as I poise. [8 Fluid Mechanics ] Solution. Given : ‘Area of the plate, A= 15x 10° mm? = 1.5 m* Speed of plate relative to another plate, du = 0.4 m/s, Distance between the plates, dy = 0.15 mm = 0.15 x 10° m ec r a (2) Le dy 10 15x10 > im oO Shear force, F = 7X area = 266.66 x 1.5 = 400 N. Ans. (W Power* required to move the plate at the speed 0.4 m/sec = Fx w= 400 x 0.4 = 160 W. Ans. Problem 1.6 Determine the intensity of shear of an oil having viscosity = 1 poise. The oil is used {for lubricating the clearance between a shaft of diameter 10 em and its journal bearing. The clearance is 1.5 mm and the shaft rotates at 150 r.p.m. Solution. Given : w= 1 poise= + Ss 10 m Dia. of shaft, D=10em=0.1m Distance between shaft and journal bearing, dy = 1.5 mm=15x 10m Speed of shaft, N= 150rp.m, ‘Tangential speed of shaft is given by EDN _ KxX01X150 B= 0.785 mis Using equation (1.2), where d= change of velocity between shaft and bearing = « - 0 =u 10785 10" 15x10 Problem 1.7 Calculate the dynamic viscosity of an oil, which is used for lubrication between a square plate of size 0.8 m x 0.8 m and an inclined plane with angle of inclination 30° as shown in Fig. 1.4. The weight of the square plate is 300 N and it slides down the inclined plane with a uniform velocity of 0.3 m/s. The thickness of oil film is 1.5 mm. Solution. Given : 52.33 Nim*. Ans. Area of plate, A= 0.8% 0.8 = 0.64 m? Angle of plane, @=30° Weight of plate, W=300N Velocity of plate, u=03 mis * Power= PxuNmis= Fx uW(: Nes = Watt) Properties of Fluids 9) Thickness of oil film, t= dy=15mm= 15109 m Let the viscosity of fluid between plate and inclined plane is Component of weight W, along the plane = W cos 60° = 300 cos 60° = 150 N ‘Thus the shear force, F, on the bottom surface of the plate = 150 N and shear stress, te OO Nm ‘Area 068 Now using equation (1.2), we have du dy where du = change of velocity = u—0 = w= 0.3 mis dy= 215% 10% m 150 03 ost" 15x10" p= ROIS AIO 117 N sim? = 1.17 x 10 = 11.7 poise. Ans. 064% 03 Problem 1.8 Two horizontal plates are placed 1.25 em apart, the space between them being filled with oil of viscosity 14 poises. Calculate the shear stress in oil if upper plate is moved with a velocity of 25 mis Solution. Given : Distance between plates, dy = 1.25 em = 0.0125 m 4 FI Viscosity, = 14 poise = 4 N sim? y w= 14 poise = 7 Velocity of upper plate, w= 2.5 m/sec. Shear stress is given by equ: du 12) as,t=4 (12) as, t= 4 where du = Change of velocity between plates = u ~ 2.5 misec. dy = 0.0125 m. 425 ; T= 19% pias = 280. Nim*. Ans. Problem 1.9 The space between two square flat parallel plates is filled with oil. Each side of the late is 60 cm. The thickness of the oil film is 12.5 mm. The upper plate, which moves at 2.5 metre per sec requires a force of 98.1 N to maintain the speed. Determine : (i) the dynamic viscosity of the oil in poise, and (ii) the kinematic viscosity of the oil in stokes ifthe specific gravity of the oil is 0.95. Solution. Given : Each side of a square plate = 60.em = 0.60 m Area, A= 0.6 x 0.6 = 0.36 m? Thickness of oil film, dy = 12.5 mm = 12.5 x 10° m Velocity of upper plate, = 2.5 m/sec [10 Fluid Mechanics ] Change of velocity between plates, du = 2.5 mise = 981.N Force _F__981N Area A 0.36m* (i) Let 1 = Dynamic viscosity of oil Force required on upper plate, Shear stress, du | 981 25 Se ee dy” 036 125x10™ 981, 12.5%10" 036 25 3635 x 10 = 13.635 poise. Ans. Using equation (1.2), = cas 7 NS= 10 poise) (Gi) Sp. gr. of oil, $ = 0.95 Let v = kinematic viscosity of oi Using equation (1.14), Mass density of oil, p= Sx 1000 = 0.95 x 1000 = 950 kg/m? 1 360 *s) Using the relation, v= 4, we get y= ——™/ = 001435 m*ise 930 = 14,35 stokes. Ans. ( em?ls = stoke) Problem 1.10 Find the kinematic viscosity of an oil having density 981 kg/m*. The shear stress at 44 point in oil is 0.2452 N/m? and velocity gradient at that point is 0.2 per second. Solution. Given : Mass density, Shear stress, Velocity gradient, Using the equation (1.2), 02452 “0.200 Kinematic viscosity v is given by we _ 1226 | = 1.226 Nsim? 125 x 10°? m/sec p 98L 0.125 x 10? x 10 em*/s = 0.125 x 10? emis = 12.5 cm/s = 12.5 stoke. Ans. (Cz em*Is = stoke) Problem 1.11 Determine the specific gravity of a fluid having viscosity 0.05 poise and kinematic viscosity 0.035 stokes. Solution. Given : soy = 005 pote = MS Nant [ Properties of Fluids 11) (0.035 stokes 0.035 em*/s = 0.035 x 10°* mis Kinematic viscosity, stoke = em*Is} Using the relation v= !., we get 0.035 x 104 e ° P 905 1 _ 3408.5 kein? 10 * 0.03510 of liquid _ 14285 _ Det of liquid = rece Density of water 1000 = 1.4285 = 1.43. Ans. Problem 1.12 Determine the viscosity of a liquid having kinematic viscosity 6 stokes and specific gravity 19. Solution. Given : Kinematic viscosity V = 6 stokes = 6 em*/ Sp. gt. of liquid 9 Let the viscosity of Hiquid = yt _ Density of the liquid Density of water 6x10 ms Now sp. gr. of a liquid or 1.9 = Density of liquid 1000 kg “. Density of liquid = 1000 x 1.9 = 1900 “8 m :. Using the relation x 10" x 1900 = 1.14 Ns/m? -14 x 10 = 11.40 poise. Ans. Problem 1.13 The velocity distribution for flow over a flat plate is given by u = y—y* inwhich wis the velocity in metre per second at a distance y metre above the plate. Determine the shear stress, at y = 0.15 m. Take dynamic viscosity of fluid as 8.6 poise. Solution. Given : At y=0.15, Viscosity, [12 Fluid Mechanics ] du _ 85 N Hy SONS ge = ORRRS Te Ams. Problem 1.14 The dynamic viscosity of an oil, used for lubrication between a shaft and sleeve is 6 poise. The shaft is of diameter 0.4 m and rotates at 190 r.p.m, Calculate the power lost in the bearing {fora sleeve length of 90 mm. The thickness of the oil film is 1.5 mm. Solution. Given : $0 Viscosity H=6 poise Using equation (1.2), Dia. of shaft, Speed of shaft, Sleeve length, ‘Thickness of oil film, ‘Tangential velocity of shaft, w 60 Using the relation cep where di 98 mis Change of velocity =u — dy = Change of distance x 103m t=10x oe 592 N/m? 15x10° This is shear stress on shaft Shear force on the shaft, F = Shear stress x Area 1592 x RD x L= 1592 x rx 4x90 x 10°? = 180.05 N 04 Tonge onthe shat, T= Foxe x 2= 18005 x 84 =3601 Nm = 2RNT _ 2nx190%36.01 ~— 60 60 Problem 1.18 ifthe velocity profile ofa fluid over a plate is parabolic with the vertex 20 em from the plate, where the velocity is 120 emlsec. Calculate the velocity gradients and shear stresses at a distance of 0, 10 and 20 cm from the plate, ifthe viscosity of the fluid is 8.5 poise. Solution. Given : Distance of vertex from plate = 20 em Velocity at vertex, = 120 emisee “Power lost = 716.48 W. Ans. SSNs 10 m? Viscosity, H=85 poise * Power in $. uni [ Properties of Fluids 13) ‘The velocity profile is given parabolic and equation of velocity profile is, uray tbyte od) where a, b and ¢ are constants. Their values are determined from boundary conditions as : (@) aty=0,u=0 (®) aty = 20 em, u = 120 em/see du (© aty= 20m, = 0. Substituting boundary condition (a) in equation (i), we get Boundary condition (b) on substitution in (i) gives 120 = a(20)* + b(20) = 4000+ 200 Ai) Boundary condition (c) on substitution in equation (i) gives du Grete ii) or 0=2xax204D=400+b Solving equations (i) and (i) for @ and b From equation (iii), b=-40a Substituting this value in equation (i), we get 120 = 4000 + 20 x ( 40a) = 4000 - 8000 2023 =400 *~ 70 . b==40x(-03) = 120 ‘Substituting the values of a, b and ¢ in equation (i), u=—0.3y? + 12y. Ei a -03 Velocity Gradient == 03x 2y+ 12=-0.6y +12 aty = 0.6 x 0+ 12 = 124. Ans. at y= 10cm, 0.6 x 10 + 12=-6 + 12 = 6/s. Ans. at y= 20cm, 0.6 x 204 12 =~ 12+ 12=0. Ans. Shear Stresses Shear stress is given by, [14 Fluid Mechanics ] 0.85 x 12.0 = 10.2 Nim’. (@ Shear stress at (24) =08860=51 wm i) Shear stress at y = 10, w 5) ey (Gil) Shear stress at y= 20, Problem 1.16 A Newtonian fluid is filled in he clearance between a shaft and a concentric sleeve. The sleeve attains a speed of 50 cmd, when a force of 40 N is applied to the sleeve parallel to the shaft. Determine the speed if a force of 200 N is applied. Solution. Given : Speed of sleeve, uy = 50 em/s when force, F,=40N. Let speed of sleeve is te when force, F = 200 N. Using relation where t = Shear stress ‘du = Change of velocity dy = Clearance {¥ A,qand y are constant} Substituting values, we get 42 = 200 50 My 50x20 = 59 x5 50 cm/s. Ans. 40 Problem 1.17 A 15 cm diameter vertical cylinder rotates concentrically inside another eylinder of diameter 15.10 cm, Both cylinders are 25 em high. The space between the cylinders is filled with a liquid whose viscosity is unknown. If a torque of 12.0 Nm is required to rotate the inner cylinder at 100 r-p.m., determine the viscosity of the fluid. Solution. Given : Diameter of cylinder = 15 em = 0.15 m Dia. of outer cylinder = 15.10 em = 0.151 m Length of cylinders, 1 = 250m=0.25 m Torque, T = 12.0 Nm [ Properties of Fluids 15) Speed, Let the viscosity RDN _ rx 015x100 Tangential velocity of eylinder, «= = 0.7854 mis o SENS. 0 Cy Surface area of eylinder, A= RD XL= nx 0.15 x 0.25 = 1178 m? Now using relation ee ay where dus u-0=u= 7854 mis y= O1SI=0150 Fy - 0905 m 2 yx. 7854 [0005 Shear foree, D Torque, TeFxe 120 BME srg tS 12.0%.0005 2 9 s64 N gm? 7854 x 1178 15 = 0.864 x 10 = 8.64 poise. Ans. Problem 1.18 Two large plane surfaces are 2.4 em apart. The space between the surfaces is filled with glycerine. What force is required to drag a very thin plate of surface area 0.5 square metre between the two large plane surfaces at a speed of 0.6 ms, if (i) the thin plate is in the middle of the two plane surfaces, and (ii) the thin plate is at a distance of 0.8 em from one of the plane surfaces ? Take the dynamic viscosity of glycerine = 8.10 x 10"! N s/n? Solution. Given : Distance between two large surfaces = 2.4 em separa ‘Area of thin plate, A=05m? 12cm Velocity of thin plate, = 0.6 mis er Viscosity of glycerine, = 8.10 10" N sim? idem Case I. When the thin plate is in the midale of the two plane surfaces [Refer to Fig. 1.7 (a)] ooh Let F, = Shear force on the upper side of the thin plate Fig. 17 (@) F, = Shear force on the lower side of the thin plate F =Total force required to drag the plate Then FeF\+hy ‘The shear stress (t,) on the upper side of the thin plate is given by equation, [16 Fluid Mechanics ] ‘du dy}, where du = Relative velocity between thin plate and upper large plane surface 0.6 misee dy = Distance between thin plate and upper large plane surface 1.2 em = 0.012 m (plate isa thin one and hence thickness of plate is neglected) as n=si0x10"x( 26) 40.5 Nim? 012 Now shear force, hear stress x Area 1 XA= 40.5 x 0.5 = 20.25 N Similarly shear stress (,) on the lower side of the thin plate is given by 8.10 10 x { 2% 0012, ) = 40.5 im? Shear force, : 5x05 = 20.25N Total force, Fs F, + Fy= 20.25 + 20.25 = 40.5 N. Ans. Case I. When the thin plate is ata distance of 0.8 em from one of the plane surfaces [Refer to Fig. 1.7 (0)] yey ee Let the thin plate is at a distance 0.8 em from the lower plane surface. 166m “Ten distance ofthe plate rom the upper plane sutace atm | =24-0.8= 16cm 16m Va (Nepecting thcknes ofthe pate) ‘The shear force onthe upperside of he thin pate, = Shear stress Area t,x vig 170) ay, ‘The shear force on the lower side of the thin plate, du axAcu |“) xa +(e), 06 087100 ‘Total force required = F, + F, = 15.18 + 30.36 = 45.54 N. Ans. Problem 1.19 A vertical gap 2.2 cm wide of infinite extent contains a fluid of viscosity 2.0 N s/m* and specific gravity 0.9. A metallic plate 1.2 m X 1.2 m X0.2 em is to be lifted up with a constant velocity of 0.15 m/sec, through the gap. If the plate is in the middle of the gap, find the force required. The weight of the plate is 40 N. Solution. Given : Width of gap = 2.2 em, viscosity, y= 2.0 N s/m? Sq. gr. of fluid 09 =n(“*) xa=8.10%107x 22) x05 = 15.18 | 0.016 Fy =810310 (28) x05=20366 [ Properties of Fluids 17) Weight density of fluid 091000500 gt = 9009.8 Nim? (oD kgf = 9.81 N) Volume of plate = 1.2mx1.2mx0.2em 1.2 1.2 x 002 m*= 00288 m? ‘Thickness of plate 0.2em Velocity of plate = 0.15 m/sec Weight of plate 40N. ‘When plate is in the middle of the gap, the distance of the plate from vertical surface of the gap Fig. 18 . (wa of gap~Thickness of pie) . 2 C208) = em= 01m. 2 Now the shear force on the left side of the metallic plate, F, = Shear stress x Area =» (22) xam20s(28)xranian (cr Area = 1.2 x 1.2m?) Similarly, the shear force on the right side of the metallic plate, ‘Total shear force = 1+ Fy = 43.2 443.2 = 864 N, In this case the weight of plate (which is acting vertically downward) and upward thrust is also to be taken into account. ‘The upward thrust = Weight of fluid displaced (Weight density of fluid) x Volume of fluid displaced 81 x 900 x.00288 N (C+ Volume of fluid displaced = Volume of plate = .00288) = 2543 N, ‘The net force acting in the downward direction due to weight of the plate and upward thrust = Weight of plate ~ Upward thrust = 40 ~ 25.43 = 14.57 N Total force required to lift the plate up = Total shear force + 14.5 86.4 + 14.57 = 100.97N. Ans. > 1.4 THERMODYNAMIC PROPERTIES Fluids consist of liquids or gases. But gases are compressible fluids and hence thermodynamic properties play an important role. With the change of pressure and temperature, the gases undergo [18 Fluid Mechanics large variation in density. The relationship between pressure (absolute), specific volume and temperature (absolute) of a gas is given by the equation of state as pV =RTor t RT (1.5) where p = Absolute pressure of a gas in N/m? VW = Specific volume = F Gas constant Absolute temperature in °K. Density of a gas. 1.4.1 Dimension of R. The gas constant, R, depends upon the particular gas. The dimension of R is obtained from equation (1.5) as (© In MKS units For air, Rin MKs = 293 “82m kg °K Rin s1=29.3x9.81 S® = 287 igrk 8 eek 1.4.2 Isothermal Process. If the change in density occurs at constant temperature, then the process is called isothermal and relationship between pressure (p) and density (p) is given by 6) 1.4.3 Adiabatic Process. If the change in density occurs with no heat exchange to and from the 288, the process is called adiabatic. And if no heat is generated within the gas due to friction, the relationship between pressure and density is given by 2 =Constant an where & = Ratio of specific heat of a gas at constant pressure and constant volume. = 1 for air. [ Properties of Fluids 19) 1.4.4. Universal Gas Constant Let ‘m= Mass of a gas in kg V = Volume of gas of mass m solute pressure solute temperature Then, we have p= mRT (1.8) where R = Gas constant. Equation (1.8) can be made universal, Z., applicable to all gases if itis expressed in mole-basis. Let n= Number of moles in volume of a gas V = Volume of the gas __ Mass of the gas molecules * “Mass of a hydrogen atom ‘m= Mass of a gas in kg Tenwepine nate Sting eve fmm qutin (1, ne poems? as) ‘pods Ms ld ergs const and sega 88 8s mo and 8314 J/kg-mole K in SI units. (One kilogram mole is defined as the product of one kilogram mass of the gas and its molecular weight. Problem 1.20 A gas weighs 16 N/m! at 25°C and at an absolute pressure of 0.25 N/mm. Deter- ‘mine the gas constant and density of the gas. Solution. Given : Weight density, w= 16 Nim’ Temperature, 125°C T= 273 4 1= 273 + 25 = 288°K p= 0.25 Nimm? (abs.) = 0.25 x 10° Nim? = 25 x 10 Nim? (0 Using relation w = pg, density is obtained as w_ 16 . JS 21.63 kg/m’. Ans. Pe ost ss (ii) Using equation (1.5), 2 = RT P P= 25x10" 539.55 NM ang pT 163x288 kek Problem 1.21. A cylinder of 0.6 m' in volume contains air at 50°C and 0.3 Ninn? absolute pressure. The air is compressed to 0.3 m*. Find (i) pressure inside the cylinder assuming isothermal process and (i) pressure and temperature assuming adiabatic process. Take k = 1.4 Solution. Given : Initial volume, y > [20 Fluid Mechanics ] ‘Temperature Pressure py = 03 Nimm? = 0.3 x 10° N/m? = 30 x 10* N/m? Final volume 3m? kod (@ Isothermal process + Using equation (1.6), 30x10" x06 0.6 x 10° N/m? = 0.6 N/mm?. Ans. (ii) Adiabatic process : Using equation (1.7), 3, (06) tot (28 nxt (2) = 0.791 x 10° Nim? = 0.791 Némm?, Ans. For temperature, using equation (1.5), we get pY = RT and also p W* = Constant P or rv! or Tw Constant Constant {+ Ris also constant} ) = 323x204 = 426.2°K Ans. Problem 1.22 Calculate the pressure exerted by 5 kg of nitrogen gas at a temperature of 10°C if the volume is 0.4 m’. Molecular weight of nitrogen is 28. Assume, ideal gas laws are applicable. Solution. Given : Mass of nitrogen =5kg ‘Temperature, 10°C o T=273+10= ‘Volume of nitrogen, v=04m? Molecular weight =28 Using equation (1.9), we have pV = nx Mx RT [ Properties of Fluids 21) where Mx R = Universal gas constant = s314 —Nm___ Kg-mole*K and one kg-mole = (kg-mass) x Molecular weight = (kg-mass) x28 for nitrogen = S314 956.9 _N™ 28 ke°K ‘The gas laws for nitrogen isp = mR, where R = Characteristic gs constant or px 0d = 5 x2969 x 283 p= 252969285 _ 1050283.7 Nim? = 1.08 Nim, Ans. 04 > 1.5 COMPRESSIBILITY AND BULK MODULUS Compressibility is the reciprocal of the bulk modulus of — = ‘elasticity, K which is defined as the ratio of compressive stress to volumetric strain, Consider a eylinder fited with a piston as shown in Fig. L9. ° Let V = Volume of a gas enclosed in the cylinder ‘p= Pressure of gas when volume is : Let the pressure is increased to p+ dp, the volume of gas CYLINDER decreases from WV to V~ dV. Fig. 19 ‘Then increase in pressure Ip ketim> Decrease in volume av Volumetric strain paral v ~ ve sign means the volume decreases with increase of pressure. Bulk modulus K = Inotease of pressure, Volumetric strain dp__-4dp 1 nae ay 1.10) ¥ 1 Compressibiit et ay pressibilty 3 Relationship between Bulk Modulus (K) and Pressure (p) for a Gas The relationship between bulk modulus of elasticity (K) and pressure for a gas for two different processes of compression are as () For Isothermal Process. Equation (1.6) gives the relationship between pressure (p) and density (p) of a gas as 2 = Constant Pp [22 Fluid Mechanics ] or pV = Constant Differentiating this equation, we get (p and V both are variables) pav+Vdp=0 or pdv=-Vdp or p Substituting this value in equation (1.10), we get K=p (a2) (ii) For Adiabatic Process. Using equation (1.7) for adiabatic process. - Constant or p V* = Constant Differentiating, we get pa(v*) + V'(dp) or px kx dv 4 VE dp or pa + dp 0 [Cancelling W*! to both sides} or ka =~ dp or pk Hence from equation (1.10), we have K= pk (1.13) = Ratio of specifie heat Problem 1.23 Determine the bulk modulus of elasticity of a liquid, if the pressure of the liquid is increased from 70 Nicm? to 130 Néem?. The volume of the liquid decreases by 0.15 per cent. Solution. Given : Initial pressure = 70 Niem? Final pressure = 130 Nlem? dp = Increase in pressure = 130 - 70 = 60 N/em? Decrease in volume = 0.15% ous where K = Bulk modulus and 100 Bulk modulus, K is given by equation (1.10) as dp _ 60N/em* _ 60100 cdv IS aS v 100 Problem 1.24 Whar is the bulk modulus of elasticity of a liquid which is compressed in a cylinder ‘from a volume of 0.0125 m' at 80 Nem? pressure to a volume of 0.0124 m* at 150 Nien? pressure ? Solution. Given : Initial volume, ¥ = 0.0125 m* Final volume 0.0124 m?* s+ Decrease in volume, dV =.0125 ~.012: K 4x 10Niem?, Ans. [ Properties of Fluids 23) Initial pressure Final pressure Increase in pressure, dp = (150 ~ 80) = 70 N/em® Bulk modulus is given by equation (1.10) as v > 1.6 SURFACE TENSION AND CAPILLARITY Surface tension is defined as the tensile force acting on the surface of a liquid in contact with a gas ‘or on the surface between two immiscible liquids such that the contact surface behaves like a ‘membrane under tension, The magnitude of this force per unit length of the free surface will have the same value as the surface energy per unit area. It is denoted by Greek letter 6 (called sigma). In MKS units, itis expressed as kgf/m while in SI units as N/m. ‘The phenomenon of surface tension is explained by Pee coRrael Fig. 1.10. Consider three molecules A, B, C of a liquid in a mass of liquid. The molecule A is attracted in all directions equally by the surrounding molecules of the liquid. Thus the resultant force acting on the molecule A is zero, But the molecule B, which is situated near the free surface, is acted upon by upward and downward forces which are unbalanced, Thus a net resultant force on molecule B is acting in the downward direction. The molecule C, situated on the free surface of liquid, does experience a resultant downward force. All the molecules on the free surface experience a downward force. Thus the free surface of the liquid acts like a very thin film under tension of the surface of the liquid act as though it is an elastic membrane under tension. Fig. 110 Surface tension. 1.6.1 Surface Tension on Liquid Droplet. Consider a small spherical droplet of a liquid of radius ‘r’. On the entire surface of the droplet, the tensile force due to surface tension will be acting, Let @'= Surface tension of the liquid p = Pressure intensity inside the droplet (in excess of the outside pressure intensity) d= Dia, of droplet. Let the droplet is cut into two halves. The forces acting on one half (say left half) will be (0 tensile force due to surface tension acting around the circumference of the cut portion as shown in Fig. 1.11 (b) and this is equal to = 6 x Circumference oxnd [24 Fluid Mechanics ] Rep ® ii) pressure force on the area ~ d? = p x ~ d® as shown in (i ps ier Fig. 1.11 (0). These two forces will be equal and opposite under equilibrium conditions, ée., MIDEORe I eeteAce rersion x x= Peoxnd ext , co ged 49 aay me 4 (©) PRESSURE FORCES Equation (1.14) shows that with the decrease of diameter Fig. 1.11 Forces on droplet. of the droplet, pressure intensity inside the droplet increases, 1.6.2 Surface Tension on a Hollow Bubble. A hollow bubble like a soap bubble in air has two surfaces in contact with air, one inside and other outside. Thus two surfaces are subjected to surface tension. In such case, we have pxt@ 2x(6x Rd) 2ond _ 86 crab (1.15) 4 1.6.3 Surface Tension on a Liquid Jet. Consider a liquid jet of diameter ‘a’ and length ‘L’ as shown in Fig. 1.12, Let p = Pressure intensity inside the liquid jet above the outside pressure {= Surface tension of the liquid A Consider the equilibrium of the sem jet, we have Z Force due to pressure 1p Xarea of semi jet Z pxLxd L of Force due to surface tension = 0X2. Z Equating the forces, we have Z pXLxd =0x2L j ett ay Fig. 112 Forces on liquid jet. Problem 1.25 The surface tension of water in contact with air at 20°C is 0.0725 Nim. The pressure inside a droplet of water isto be 0.02 N/em? greater than the outside pressure. Calculate the diameter of the droplet of water. Solution. Given : Surface tension, 6 = 0.0725 Nim Pressure intensity, p in excess of outside pressure is oat % 7 Let of the droplet [ Properties of Fluids 25) d= ADO op m= 0085 100 = 148 mm. Ams. 0.02 x(10) Problem 1.26 Find the surface tension in a soap bubble of 40 mm diameter when the inside pressure is 2.5 Nin? above atmospherie pressure. Solution. Given : Dia. of bubble, d= 40 mm = 40x 10m Pressure in excess of outside,p = 2.5 N/m? For a soap bubble, using equation (1.15), we get $o or 25 = SxO 40x10 25%40x10° o 8 Problem 1.27 The pressure outside the droplet of water of diameter 0.04 mm is 10.32 N/ent (at- mospheric pressure). Calculate the pressure within the droplet if surface tension is given as 0.0725 Nim of water. Solution. Given : N/m = 0.0125 Nim. Ans. Dia. of droplet, d= 0.04 mm = .04 x 10 m Pressure outside the droplet = 10.32 N/em? = 10.32 x 10* N/m? Surface tension, = 00725 Nim The pressure inside the droplet, in excess of outside pressure is given by equation (1-14) or p= 49. -4%00725 _ 7959 nim? = 7250N. 0795 Nem? d= 04x10 10% cm? Pressure inside the droplet = p + Pressure outside the droplet 725 + 10.32 = 11.045 Niem?. Ans. 1.6.4 Capillarity. Capillarity is defined as a phenomenon of rise or fall of a liquid surface in a small tube relative to the adjacent general level of liquid when the tube is held vertically in the liquid. The rise of liquid surface is known as capillary rise while the fall of the liquid surface i known as capillary depression. Itis expressed in terms of cm or mm of liquid. Its value depends upon the specific weight of the liquid, diameter of the tube and surface tension of the liquid. Expression for Capillary Rise. Consider a glass tube of small diameter ‘a’ opened at both ends and is inserted in a liquid, say water. ‘The liquid will rise in the tube above the level of the liquid. Let f= height of the liquid in the tube. Under a state of equilibrium, the weight of liquid of height 1 is balanced by the force atthe surface of the liquid in the tube. But the force at the surface of the liquid in the tube is due to surface tension. Fig. 143, Let 6 = Surface tension of liquid = Angle of contact between liquid and glass tube. ‘The weight of liquid of height iin the tube = (Area of tube x h) x p xg Capillary rise. [26 Fluid Mechanics ] Pxhxpxe 0.17) where p = Density of liquid Vertical component of the surface tensile force = (6x Circumference) x cos ® xnd xcos 0 (1.18) For equilibrium, equating (1.17) and (1.18), we get FP xx p xe = Xd x cos 0 _ BX md xcosO _ 4.60080 or j= SARA X CSD _ Ao cos0 (1.19) Faxpxs pxgxd ‘The value of @ between water and clean glass tube is approximately equal to zero and hence cos @ is ‘equal to unity. Then rise of water is given by _ 49 “ pxexd Expression for Capillary Fall. Ifthe glass tube is dipped in mercury, the level of mercury in the tube will be lower than the general level of the outside liquid as shown in Fig. 1.14. Let = Height of depression in tube. ‘Then in equilibrium, two forces are acting on the mercury inside the tube. First one is due to surface tension acting in the downward direction and is equal to Gx rd x cos 0. Second force is due to hydrostatic force acting upward and is equal to intensity of pressure at a depth “I x Area h (1.20) =pxt@ pexhx Si ( p=pshh x 4 Equating the two, we get oxndxcos O=pghx Tf 4G .cos® ae 21) pad Value of @ for mercury and glass tube is 128°. MERCURY Fig. 114 Problem 1.28 Calculate the capillary rise in a glass tube of 2.5 mm diameter when immersed vertically in (a) water and (b) mercury. Take surface tensions = 0.0725 N/m for water and o= 0.52 Nim {for mercury in contact with air. The specific gravity for mercury is given as 13.6 and angle of contact = 130°, Solution. Given : Dia. of tube, a Surface tension, 6 for water 6 for mercury Sp. gr. of mercury Smm=2.5x 10° m 0725 N/m 52. Nim 3.6 [ Properties of Fluids 27) Density = 13.6 x 1000 kg/m’. (a) Capillary rise for water (@ = 0°) 4x 00725 pxgxd 1000x981x 25x10” = 0118 m= 1.18 em, Ans. Using equation (1.20), we get = () For mercury Angle of contact between mercury and glass tube, @ = 130° 4.0088 4 052 x €08 130° pxgxd 136x100 x981x25x10~ 004 m = - 0.4 em. Ans. ‘The negative sign indicates the capillary depression. Using equation (1.21), we get t= Problem 1.29 Calculate the capillary effect in millimetres in a glass tube of 4 mm diameter, when immersed in (i) water, and (ii) mercury. The temperature of the liquid is 20°C and the values of the surface tension of water and mercury at 20°C in contact with air are 0.073575 Nim and 0.51 Nim respectively. The angle of contact for water is zero and that for mercury is 130°, Take density of water ‘at 20°C as equal to 998 kg/m. Solution. Given : Dia. of tube, d=4mm=4x 10m ‘The capillary effect (i.¢., capillary rise or depression) is given by equation (1.20) as p= 45.6088 pxexd where 6 = surface tension in N/m @ = angle of contact, and p = density (i Capillary effect for water 6 = 0.073575 Nim, 8 = 0° p= 998 kg/m? at 20°C = 40073575 x 0080" 998 x981%4 x10 (ii) Capillary effect for mercury 6 =0.51 Nim, @= 130° and 3. gr. x 1000 = 13.6 x 1000 = 13600 kg/m? _ 4051 608 130° "= 13600981 x4 x 10 ‘The negative sign indicates the capillary depression. =7.51 x 10? m=7.51 mm. Ans, .46 x 10° m= = 2.46 mm. Ans. Problem 1.30 The capillary rise in the glass tube is not to exceed 0.2 mm of water. Determine its ‘minimum size, given that surface tension for water in contact with air = 0.0725 N/m. Solution. Given: Capillary rise, h= 0.2 mm = 0.2 x 10° m Surface tension, 6 = 0.0725 Nim [28 Fluid Mechanics ] Let dia. of tube ‘The angle @ for water Density (p) for water = 1000 kg/m? Using equation (1.20), we get 40 3 _ 4x 00725 pxexd "97%! = Too x9Bixd 4x 00725 ‘1000 x 981x 2x10 Thus minimum diameter of the tube should be 14.8 em. = 0.148 m = 14.8 em. Ans. Problem 1.31. Find out the minimum size of glass tube that can be used 10 measure water level if the capillary rise in the tube is to be restricted to 2 mm. Consider surface tension of water in contact with air as 0.073575 N/m. Solution. Given : Capillary rise, f= 2.0 mm =2.0 10 m Surface tension, 6 = 0.073575 Nim Let dia. of tube a ‘The angle @ for water ‘The density for water, p= 1000 kg/m? Using equation (1,20), we get 40 9.9 x 19 = 4% 0.073575 pxgxd 1000 x 981 xd 4x 0073575, 4X 0075979 ___ = 0,015 m= 15 em. Ans. 1000 x 981 x 2.x 10™ ‘Thus minimum diameter of the tube should be 1.5 em. Problem 1.32 An oil of viscosity 5 poise is used for lubrication between a shaft and sleeve. The diameter of the shaft is 0.5 m and it rotates at 200 r-p.m. Calculate the power lost in oil for a sleeve length of 100 mm. The thickness of ol film is 1.0 mm. Solution. Given : Viscosity, I= 5 poise = 0.5 N sim? 10 Dia. of shaft, D=05m Speed of shaft, N= 200 rpm. Sleeve length, L = 100 mm = 100 x 10 m=0.1 m Thickness of oil film, 121.0 mm=1x 10% m RDN _ ©x05x200 Tangential velocity of shaft, u = = BK05%200 _ 5.235 mis es y 7 7 du Using the relation, ne wy [ Properties of Fluids 29) where, du = Change of velocity = u ~ 0 = u = 5.235 mls dy = Change of distance 1x 103m 255235 _ 36175 Nin? et The he sear aes on te a 1 Stew force on esha, P= heat wren x Area = 2617S XXD XL (Ara aD XE) zasinsxx03%01 = 41098 ea or Bw 41095 eae eee! oo 10274 252200 950 W = 2.5 1 Ane > 1.7 VAPOUR PRESSURE AND CAVITATION ‘A change from the liquid state to the gaseous state is known as vaporization. The vaporization (which depends upon the prevailing pressure and temperature condition) occurs because of continuous ‘escaping of the molecules through the free liquid surface. Consider a liquid (say water) which is confined in a closed vessel. Let the temperature of liquid is 20°C and pressure is atmospheric. This liquid will vaporise at 100°C. When vaporization takes place, the molecules escapes from the free surface of the liquid. These vapour molecules get accumulated in the space between the free liquid surface and top of the vessel. These accumulated vapours exert a pressure on the liquid surface. This pressure is known as vapour pressure of the liquid or this is the pressure at which the liquid is converted into vapours. Again consider the same liquid at 20°C at atmospheric pressure in the closed vessel. If the pressure above the liquid surface is reduced by some means, the boiling temperature will also reduce. If the pressure is reduced t0 such an extent that it becomes equal to or less than the vapour pressure, the boiling of the liquid will start, though the temperature of the liquid is 20°C. Thus a liquid may boil ‘even at ordinary temperature, if the pressure above the liquid surface is reduced so as to be equal or less than the vapour pressure of the liquid at that temperature. Now considera flowing liquid in a system. If the pressure at any point in this flowing liquid becomes. ‘equal to of less than the vapour pressure, the vaporization of the liquid starts. The bubbles of these vapours are carried by the flowing liquid into the region of high pressure where they collapse, giving rise to high impact pressure. The pressure developed by the collapsing bubbles is so high that the ‘material from the adjoining boundaries gets eroded and cavities are formed on them. This phenomenon is known as cavitation. Hence the cavitation is the phenomenon of formation of vapour bubbles of a flowing liquid in a region where the pressure of the liquid falls below the vapour pressure and sudden collapsing of these vapour bubbles in a region of higher pressure. When the vapour bubbles collapse, a very high pressure iscreated. The metallic surfaces, above which the liquid is flowing, is subjected to these high pressures, which cause pitting action on the surface. Thus cavities are formed on the metallic surface and hence the name is cavitation, 20NT- ky, THe angular velocity w= 22%. 0,000 0 + Power ncasofSL. Unit Pxcwor 2 at Fluid Mechanics 2 ). Bulk modulus of elasticity is given as K= HIGHLIGHTS ‘The weight density or specific weight of a fluid is equal to weight per unit volume. It is also equal to, w=pxe. 2. Specific volume is the reciprocal of mass density. The shear sess is proportional tthe velocity gradient 1. Mathematically. x= we > ” Kinematic viscosity vis given by v= 5. Poise and stokes are the units of viscosity and kinematic viscosity respectively. 5. To convert the unit of viscosity from poise to MKS units, poise should be divided by 98.1 and to convert poise into ST units, the poise should be divided by 10. SI unit of viscosity is Ns/m” or Pa, where N/m = Pa = Pascal 1. For a perfect gas, the equation of state is = Rr Pp 293 ket ke°K where R= gas constant and for = 287 Ske °K. etre ete le cen tena ee ote eee ae paar w ¥ Compress ith sip of ak dl fey or = Surface esion isexpesed in Nino dynlem, The relation Bete face tension (6 an erence “ Of pressure (p) between the inside and outside of a liquid drop is given as p = 80 For aso buble, p= $8 20 Fen eee eet quid jt, p= 29 Copiary ie ofa of Hai i given by n= 252288 ‘The value of @ for water is taken equal to zero and for mercury equal to 128° EXERCISE (A) THEORETICAL PROBLEMS |. Define the following fluid properties Density, weight density, specific volume and specific gravity of af . Differentiate between : (?) Liquids and gases, (i) Real fluids and ideal fluids, (if) Specific weight and specific volume of a Mui. What is the difference between dynamic viscosity and kinematic viscosity ? State their units of ‘measurements Properties of Fluids 31) 4. Explain the terms : (2 Dynamic viscosity, and (i?) Kinematic viscosity. Give their dimensions, '5. State the Newton's law of viscosity and give examples of its application, 66. Enunciate Newton's law of viscosity. Explain the importance of viscosity in fluid motion, What is the effect of temperature on viscosity of water and that of air? 7. Define Newtonian and Non-Newtonian fluids. 8. What do you understand by terms : (7) Isothermal process, (i) Adiabatie process, and (iit) Universal-gas constant, 9. Define compressibility. Prove that compressibility fora perfect gas undergoing isothermal compression is + white for a perfect gas undergoing isentropic compression is 10, Define surface tension Prove that he lationship Betcen surface tenson and presse inside a droplet of iil nese foie presi given byp= 11 Expat leaner of pili Oi a ees or apy a gi 12 sing acne Ill ls Ep tp of ceil ad aoe (Deine he te: ey, se vole, spe ri aa prt, compete a comes fs (ah Vsmopte, Bhopal $200) 13 tins spn Now's yf iso 14 Comet kpkin aoe sce aoe 18 Wipe von ves hens onperse wile a ld deer wi ina nnn? 16) How oe vaca of i vay wh epr (iw oxmpier ne src nn cs fas ponent. NTU, Arba § 2002) 17. () Denson expen freon fern ogee Pins dpe gd nd te see nn (op de owing Nwtonan and NowNewona Bis vps pes, ad somes (REPL, opt $2001 (B) NUMERICAL PROBLEMS 1, One litre of crude oil weighs 9.6 N. Calculate its specific weight, density and specific gravity, Ams. 9600 Nim’, 978.6 kg/m", 0.978] velocity in metre per second at a distance y metre above the plate. Determine the shear stress at y = 9 em, Assume dynamic viscosity as 8 poise (Nagpur University) (Ans. 0.839 N/m?) A plate 0,025 mm distant from a fixed plate, moves at 50 ems and requires @ force of 1.471 Nim* to ‘maintain this speed. Determine the fluid viscosity between the plates in the poise. (Ans. 7.357 x 10°] 4. Determine the intensity of shear of an oil having viscosity = 1.2 poise and is used for lubrication in the clearance between a 10 cm diameter shaft and its journal bearing. The clearance is 1.0 mm and shaft rotates at 200 rpm. Ans. 125.56 Nim’) ‘Two plates are placed at a distance of 0.15 mm apart. The lower plate is fixed while the upper plate having surface area 1.0 m* is pulled at 0.3 mS. Find the force and power required to maintain this speed, if the fluid separating them is having viscosity 1.5 poise Ans. 300 N, 89.8 W] 66. Anil film of thickness 1.5 mm is used for lubrication between a square plate of size 0.9 m>0.9 m and an inclined plane having an angle of inclination 20°, The weight ofthe squate is 392.4 N and it slides down the plane with a uniform velocity of 0.2 m/s. Find the dynamic viscosity of the oil. [Ans. 12.42 poise] Fluid Mechanics ] 10. 12 1a. 16. 1. wv. aL 2. In a stream of glycerine in motion, ata certain point the velocity gradient is 0.25 metre per see per metre. ‘The mass density of fluid is 1268.4 kg per cubie metre and kinematic viscosity is 6.30 x 10~ square metre per second. Calculate the shear stress at the point. [Ans, 0.2 Nim?) Find the kinematic viscosity of an oil having density 980 kg/m? when ata certain point in the ol, the shear Pennine Nadeniteseieeen ios [amor eesao wis . Determine the specific gravity of a Muid having viscosity 0.07 poise and kinematic viscosity 0.042 stokes. {Ans 1.667) Determine the viscosity ofa lguid having kinematic viscosity 6 stokes and specific gravity 20. TAs. 11.99 poise) It the velocity distribution ofa fuid over a plates given by w= (3/4) yy", where wis the velocity in metre per second ata distance of y metres above the plate, determine the shea sess at y = 0.15 mete. Take dynamic viscosity of the fluid as 8.5 x 10 kg-sec/m’. (Ans, 3.825 x 10° kgffm"] ‘An oil of viscosity 5 poise is used for Ibriation between a shaft and sleeve. The diameter of shaft is 05 m and it oates at 200 rpm. Calculate the power lost inthe il fora sleeve length of 100 mm. The thickness ofthe ol lm is 1.0 mm, {Ans.2.15 kW) distance of y m above the plate, Determine the shear stess at y = 0,0.1 and 0.2 m. Take H = 6 poise. [Ams. 0.4, 0.028 and 0.159 Nim?) 1. In question 13, find the distance in metres above the plate, at which the shear stress is zero, [Ans, 0.333 m) ‘The velocity profile of a visous fd over a plat is parabolic with vertex 20m from the Pate, where the velocity is 120 m/s Calculate the velocity gradient ad sheat sess at distances of, 3 and 15 em fom the plate, given the viscosity of the fluid = 6 poise. [Ans. 12/8, 7.18 Ném?; 9/8, 5.385 Nim? ; 3/8, 1.795 Nim?) ‘The weight of a gas is given as 17.658 N/m’ at 30°C and at an absolute pressure of 29.43 N/m’, Deter- [ase Late, 29550 n] m "kek A cylinder of 0.9 m? in volume contains air at 0°C and 39.24 Niem? absolute pressure. The air is compressed t0 0.45 m'. Find (i) the pressure inside the cylinder assuming isothermal process, (pressure and temperature assuming abate process. Take f= 1 fora (Ans. (i) 78.48 Néem’, (ii) 103.5 N/m’, 140°C] Calculate the pressure exerted by 4 kg mass of nitrogen gas at a temperature of 15°C ifthe volume is 0.35 m?. ‘mine the gas constant and also the density of the gas. Molecular weight of nitrogen is 28. Ans. 97.8 Nem] ‘The pressure ofa liquid is increased from 60 N/cm? to 100 N/em? and volume decreases by 0.2 per cent Determine the bulk modulus of elasticity, [Ans. 2 « 10" Nim’) |. Determine the bulk modulus of elasticity of a fluid which is compressed in a eylinder from a volume of 0.009 mat 70 Nlem? pressure to a volume of 0.0085 m? at 270 Nécm* pressure. (Ans. 3.6 10° Nem] The surface tension of water in contact with air at 20°C is given as 0.0716 Nim. The pressure inside @ droplet of water is to be 0.0147 Nlem? greater than the outside pressure, calculate the diameter of the droplet of water. (Ans. 1.94 min) Find the surface tension in a soap bubble of 30 mm diameter when the inside pressure is 1.962 Nim? above atmosphere [Ans. 0.00735 Nim] ‘The surface tension of water in contact with ai is given as 0.0725 Nim. The pressure outside the droplet of| water. [Ans. 1.77 Neem?) Properties of Fluids 33) 24. Calculate the capillary rise in a glass tube of 3.0 mm diameter when immersed vertically in (a) water, and (b) mercury. Take surface tensions for mercury and water as 0.0725 N/m and 0.52 Nim respectively in contact with ai, Specific gravity for mercury is given as 13.6. [ns. 0.966 em, 0.3275 em} 25, The capillary rise in the glass tube used for measuring water level is not to exceed 0.5 mm, Determine its ‘minimum size, given that surface tension for water in contact with air= 0.07112 Nim. [Ans. 5:8 cm] 26. (SI Units). One litre of crude oil weighs 9.6 N, Calculate its specific weight, density and specific gravity. [Ans. 9600 Nim’; 979.6 kg/m’ ; 0.9786] 27, (SI Units). A piston 796 mm diameter and 200 mm long works in a cylinder of 800 mm diameter. Ifthe annular space is filled with a lubricating oil of viseosity 5 ep (centi-poise), calculate the speed of descent of the piston in vertical position. The weight ofthe piston and axial load are 9.81 N. Ans. 7.84 m/s} 28, (SI Units). Find the capillary rise of water in a tube 0.03 cm diameter. The surface tension of water is 0.0735 Nin, [Ans. 9.99 em] 29. Calculate the specific weight, density and specific gravity of two lites ofa liquid which weight 15 N. [Ans. 7500 Nim’, 764.5 kg/m’, 0.764] 30. A 150 mm diameter vertical cylinder rotates concentrically inside another cylinder of diameter 151 mm. Both the eylinders are of 250 mm height. The space hetween the eylinders i filled witha liquid of viscos- ity 10 poise. Determine the torque required to rotate the inner cylinder at 100 rpm, (Ans. 13.87 Nm} AM. A shaft of diameter 120 mm is rotating inside a journal bearing of diameter 122 mm ata speed of 360 rpm. The space between the shaft and the bearing is filled with a lubricating oll of viscosity 6 poise. Find the power absorbed in oil ifthe length of bearing is 100 mm, [Ans. 115.73 W] A shaft of diameter 100 mm is rotating inside a journal bearing of diameter 102 mm at a space of 360 rpm. The space between the shaft and bearing is filed with a lubricating oil of viscosity 5 poise. The length of the bearing is 200 mm. Find the power absorbed in the lubricating oil. [Ans. 111.58 W] Assuming that the bulk modulus of elasticity of water is 2.07 x 10° kN/m? at standard atmospheric conditions, determine the increase of pressure necessary to produce 1% reduction in volume at the same temperature, (Hint. K z 07 x 10" KNim? Increase in pressure (dp) 34, A square plate of size I m1 m and weighing 350 N slides down an inelined plane with a uniform velocity of 1.5 ml. The inclined plane is laid on a slope of 5 vertical to 12 horizontal and has an oil flm of 1 mm thickness. Calculate the dynamic viscosity of oil UNI, Hyderabad, S 2002] (Hint, A= 11 =1 mW =350N, «= 15més, tan 6= 2 = BC ¢, 12 AB Component of weight along the plane = W x sin @ le 2 AC=fAB+ BC where sin @ = PC= 5 13) s ac" 13 =Vites=13 te-ssan FeWsin@=350% 5 = 134615 Fig. 1.5 Now 1%, where de = uO = 1.5 mv/sand dy= 1 mm-= 1% 102m cy Fy dy 134615, 1x10 Ns or ae = 0.0807 *S 0,897 poise) BT 15 m Ga | Logs) UNL Ie MEASUREMENT > 2.1 FLUID PRESSURE AT A POINT Cone sl ae nema of fi eis stony, he x xe by the surrounding fluid on the area dA will always be perpendicular to the surface dA. Let dF is the force tng nt ae a nth ol ston. Tent ato ot hk ate ney of pressure or simply pressure and this ratio is represented by p. Hence mathematically the pressure at a point in a fluid at rest is ae oe If the force (F) is uniformly distributed over the area (A), then pressure at any point is given by EF _ Fovee a A” Arca’ + Force or pressure force, F'= p XA. The units of pressure are : (i) kgfim? and kgfiem ? in MKS units, (if) Newton/m ? or Nim? and N/mm? in SI units. N/m? is known as Pascal and is represented by Pa. Other commonly used units of pressure are : kPa= kilo pascal = 1000 Nim? bar = 100 kPa = 10° N/m?, » 2.2 PASCAL'S LAW It states that the pressure or intensity of pressure at a point in a static fluid is equal in all directions. This is proved as : ‘The fluid element is of very small dimensions ie., dx, dy and ds. Consider an arbitrary fluid element of wedge shape in a fluid mass at rest as shown in Fig. 2.1. Let the width of the ‘element perpendicular to the plane of paper is unity and p,, Fig. 2.1 Forces om a fluid element. 35 [36 Fluid Mechanics ] Py and p, are the pressures or intensity of pressure acting on the face AB, AC and BC respectively. Let -ZABC = 8. Then the forces acting on the element are : 1, Pressure forces normal to the surfaces, and 2. Weight of element in the vertical direction, ‘The forces on the faces are Force on the face AB =p. Area of face AB x dy x Similarly force on the face AC = p, x dx 1 Force on the face BC x ds x 1 Weight of clement (Mass of clement) x ¢ = (Volume x p) x ¢ = (2x4E x1) xpxe, where p= density of fluid Resolving the forees in x-direction, we have dy 1~ p (ds 1) sin (90° — 8) or p.xdy x1 ~p, ds% 1 608 8 But from Fig. 2.1, ds cos 8 = AB= dy : pexdy t= ppxdyx1 =0 or Pe 21) Similarly, resolving the forces in y-direction, we get gy. aexdy Py x dx = pexds 1 605 (90? - 0) - 2% xt xp xg=0 or Py Xd — pds sin @-AD xp xg =0. But ds sin 0 = dx and also the element is very small and hence weight is negligible. s pyle px de or Ps 22) From equations (2.1) and (2.2), we have panier 2.3) ‘The above equation shows that the pressure at any point in x, y and z directions is equal. Since the choice of fluid element was completely arbitrary, which means the pressure at any point is the same in all directions. > 2.3 PRESSURE VARIATION IN A FLUID AT REST ‘The pressure at any point ina fluid at rest is obtained by the Hydro- static Law which states that the rate of increase of pressure in a ve cally downward direction must be equal to the specific weight of the fluid at that point. This is proved as : Consider a small fluid element as shown in Fig, 2.2 Let A4= Cross-sectional area of element AZ= Height of fluid element p= Pressure on face AB Z= Distance of fluid element from free surface. ‘The forces acting on the fluid element are : FREE SURFACE OF FLUID Fig. 22. Forces on a fluid element. [ Pressure and its Measurement 37) 1. Pressure force on AB = p X AA and acting perpendicular to face AB in the downward direction. direction. 3. Weight of fluid element = Density x g x Volume = p x g x (AA x AZ). 4, Pressure forces on surfaces BC and AD are equal and opposite. For equilibrium of fluid ‘element, we have par (p+ a7] ats pxex (aa xa o aa pas - 2 az or pda paa- 2 azaas pxexadxzZ =0 or ~ Bazan ep xe asaz =0 uw xe * or 2B azaa = 2 =pXxgXxAAAZ of pg [cancelling AAAZ on both sides} az az Bapxgew pxgew) 2.4) where w= Weight density of fluid. Equation (2.4) states that rate of increase of pressure in a vertical direction density of the fluid at that point. This is Hydrostatic Law. By integrating the above equation (2.4) for liquids, we get equal to weight or 25) where p is the pressure above atmospheric pressure and Z is the height of the point from free surfaces. From equation (2.5), we have Z=—P— 2.6) Here Z is called pressure head. Problem 2.1 A hydraulic press has a ram of 30 cm diameter and a plunger of 4.5 em diameter. Find the weight lifted by the hydraulic press when the force applied at the plunger is 500 N. Solution. Given : Dia. of ram, D Dia. of plunger, a Force on plunger, F Find weight lifted x Area of ram, (03) = 0.07068 m? x Area of plunger, = 7 (0.045)? = 00159 m? [38 Fluid Mechanics Pressure intensity due to plunger _ Force on plunger _ F _ 500 ‘Area of plunger @ 00159 Nim’ |-PLUNGER. Due to Pascal’s law, the intensity of pressure will be equally transmitted in all directions. Hence the pressure intensity at the ram. 500 “00159 = 314465.4 Nim? But pressure intensity at ram Creactram =a 7 DIO ae 07068 Weight ‘= 314465.4 x.07068 = 22222 N = 22.222 KN. Ans. Problem 2.2 A hydraulic press has a ram of 20 em diameter and a plunger of 3 cm diameter. Itis used for lifting a weight of 30 KN, Find the force required at the plunger. Solution. Given : Dia. of ram, D=200m= 144654 2m ‘Area of ram, Dia. of plunger ‘Area of plunger, Weight lifted, See Fig. 2.3. 10 KN = 30 x 1000 N = 30000 N. Fore _ F rea a By Pascal's Law, this pressure is transmitted equally in all directions Pressure intensity developed due to plunger = F Hence pressure transmitted at the ram = — Force acting on ram = Pressure intensity x Area of ram Fg Fx0314 a 7.068 x10 But force acting on ram = Weight lifted = 30000 N 30000 = 2 %.0314 7.068 x10 = 30000%7.068 x10" _ Gog 9y Ans, 0314 Problem 2.3 Calculate the pressure due to a column of 0.3 of (a) water, (b) an oil of sp. gr. 0.8, and (c) mercury of sp. gr. 13.6. Take density of water, = 1000 kg/m’. Solution. Given : Height of liquid column, [ Pressure and its Measurement 39) ‘The pressure at any point in a liquid is given by equation (2.5) as eZ 1000 kg/m® 19Z = 1000 x 9.81 x 0.3 = 2943 Nim? (@) For water, (®) For oil of sp. gr. 0.8, From equation (1-14), we know that the density of a fluid is equal to specific gravity of fluid ‘multiplied by density of water. Density of ol, p. gt. of oll x Density of water (p= Density of oil) 8X p= 0.8 x 1000 = 800 kg/m? Now pressure, 0 8 XZ, = 8009.81 03 = 23544 3, = BEN (©) For mercury, sp. gr =136 From equation (1.14) we know that the density of a fluid is equal to specific gravity of fluid ‘multiplied by density of water Density of mercury, P. pecific gravity of mercury x Density of water = 13.6 x 1000 = 13600 kg/m? P=D.XEXZ = 13600 x9. x 03 = 40005 $0025 = an, ns. 10° em’ Problem 2.4 The pressure intensity at a point in a fluid is given 3.924 N/cm?, Find the correspond- ing height of fluid when the fluid is: (a) water, and (b) oil of sp. gr. 0.9. Solution. Given : Pressure intensity, p= 3.924 X= 3.92410, The corresponding height, Z, of the fluid is given by equation (2.6) as gee exe (a) For water, p= 1000 kg/m? p__ 3924 x10" TO ET Amat water Ans. (6) For oil, sp. gr 9 «. Density of oil 0.9 x 1000 = 900 kg/m* z= P= S410" _ 4.44 m of ol. Ans. PoXe 900x981 [40 Fluid Mechanics ] Problem 2.5 An oil of sp. gr. 0.9 is contained in a vessel. At a point the height of oil is 40 m. Find the corresponding height of water at the point. Solution. Given : Sp. gr. of oil, S209 Z,= 40m Po = Sp. gr. of oil x Density of water = N Height of oil, Density of oil, .9 x 1000 = 900 ke/m® Intensity of pressure, P= PoX@X Zy= 900 x 9.81 x 40 p Corresponding height of water = ————?_ Ponding height of water = 7 sity of water Xe 900 x 981% 40 AOKOSTAO _ 0.9 x 40 = 36 m of water. Ans. 1000 x 9.81 ona Problem 2.6 An open tank contains water upto a depth of 2 m and above it an oil of sp. gr. 0.9 for ‘a depth of I m. Find the pressure intensity (i) at the interface of the two liquids, and (ii) at the botiom of the tank. Solution. Given : Height of water, m Height of oil, m Sp. gr. of oll, $= 0.9 Density of water, 1000 kg/m? Density of oil, p. gr. of oll x Density of water = 0.9 x 1000 = 900 kg/m? Pressure intensity at any point is given by P=pXexZ (0 Atimterface, ie., at A 0.8829 N/em?. Ans. (i) At the bottom, ie, at B P= 2 aZz + 9, X XZ, = 900 X 9.81 x 1.0 + 1000 x 9.81 x 2.0 = 8829 + 19620 = 28449 Nim? = Problem 2.7 The diameters of a small piston and a large piston of a hydraulic jack are 3m and 10 em respectively. A force of 80 N is applied on the small piston. Find the load lifted by the large Piston when : (a) the pistons are at the same level. (b) small piston is 40 em above the large piston. The density of the liquid in the jack is given as 1000 kg/m’ Solution. Given : Dia. of small piston, a em Rak 5 ‘Area of small piston, a= 2 @= 2x Gy =7.068em? ist Fe = ExOy [ Pressure and its Measurement 411) Dia of arg piston, D= lem Area of larger piston, A= 2 x (10)? = 78.54 em* Fore on smal piston, Fe s0n Lette loa ie w. re (@) When the pistons are at the same level Pressure intensity on small piston | sanoe ||] suas ow Lace Fe) Nem? a” 7068 ‘This is ansmited equally on the lrg piston Pressure intensity onthe large piston vig is %0 7068 Force onthe large piston = Pressure x Area 0 SO 5.78.54 N = 888.96 N. Ans. gg 7854 N = 988.96 () When the small piston is 40 cm above the large piston Pressure intensity on the small piston a” 708 Pressure intensity a section AA = 4 Pressure intensity due to height of 40 em of liquid, But pressure intensity due © 40 em of quid =p gx k= 1000 x 9.81 x 0.4 Nim? 10° Pressure intensity at Seton A-A ae 80. Os ose24 7.068 1.32 + 0.3924 = 11.71 Niem? Pressure intensity transmit tothe lrg piston = Nlem? = 0.3924 Nlem? 1.71 Nic Force on the large piston = Pressure x Area of the large piston SILTLX A= 1171 X 78.54 = 919.7 N. > 2.4 ABSOLUTE, GAUGE, ATMOSPHERIC AND VACUUM PRESSURES ‘The pressure on a fluid is measured in two different systems. In one system, it is measured above the absolute zero or complete vacuum and it is called the absolute pressure and in other system, pressure is measured above the atmospheric pressure and it is called gauge pressure. Thus : 1. Absolute pressure is defined as the pressure which is measured with reference to absolute ‘vacuum pressure. 2. Gauge pressure is defined as the pressure which is measured with the help of a pressure mea- suring instrument, in which the atmospheric pressure is taken as datum. The atmospheric pressure on the scale is marked as zero. (42. Fluid Mechanics ] (GAUGE PRESSURE ATMOSPHERIC PRESSURE iesoure meson J . or Pas= Par + Peage ‘ABSOLUTE ZERO PRESSURE (ii) Vacuum pressure = Atmospheric pressure ~ Absolute pressure Note. () The atmospheric pressure at sea level at 15°C is 101.3 kN/m? or 10.13 Néem* in SI unit In case of MKS units it 8 equal to 1.033 kgtlem’ (Gi) The atmospheric pressure head is 760 mm of mercury oF 10.33 m of water Problem 2.8 What are the gauge pressure and absolute pressure at a point 3 m below the free surface of a liquid having a density of 1.53 x 10° kg/m if the atmospheric pressure is equivalent 10 3. Vacuum pressure is defined as the pres- sure below the atmospheric pressure. ‘The relationship between the absolute pressure, ‘gauge pressure and vacuum pressure are shown in Fig. 2.7. Mathematically : (0 Absolute pressure = Atmospheric pressure + Gauge pressure ——+ pressure Fig. 2.7 Relationship between pressures. 750 mm of mercury ? The specific gravity of mercury is 13.6 and density of water = 1000 kg/m’. Solution. Given : Depth of liquid, 223m Density of liquid, 53 x 10° ke/m? Atmospheric pressure head, 50 mm of Hg 750 = RR = 075 m of He Atmospheric pressure, Pam = Py % 8 X Zp where py = Density of Hg = Sp. gr. of mercury x Density of water = 13.6 x 1000 kg/m? and Zy= Pressure head in terms of mercury. Pam = (13.6 X 1000) x 9.81. 0.75 N/m? %= 0.73) = 100062 Nim? Pressure at a point, which is at a depth of 3 m from the free surface of the liquid is given by, P=PLXEXZ 1.53 x 1000) x 9.81 x 3 = 45028 Nim* 15028 Nim?. Ans. Gauge pressure + Atmospheric pressure 15028 + 100062 = 145090 N/m?. Ans. > 2.5 MEASUREMENT OF PRESSURE Gauge pressure, Now absolute pressure The pressure of a fluid is measured by the following devices : 1. Manometers 2. Mechanical Gauges. 2.5.1 Manometers. Manometers are defined as the devices used for measuring the pressure at 4 point ina fluid by balancing the column of fluid by the same or another column of the fluid. They are classified as : (@) Simple Manometers, (0) Differential Manometers. [ Pressure and its Measurement 43 2.5.2. Mechanical Gauges. Mechanical gauges are defined as the devices used for measuring the pressure by balancing the fluid column by the spring or dead weight. The commonly used mechani- cal pressure gauges are : (a) Diaphragm pressure gauge, (0) Bourdon tube pressure gauge, (©) Dead-weight pressure gauge, and (@) Bellows pressure gauge. » 2.6 SIMPLE MANOMETERS A simple manometer consists of a glass tube having one of its ends connected to a point where pressure is to be measured and other end remains open to atmosphere. Common types of simple ma- nometers are: 1. Piezometer, 2. U-tube Manometer, and 3. Single Column Manometer. 2.6.1, Piezometer. Its the simplest form of manometer used for measuring gauge pressures. One end of this manometer is connected to the point where pressure i to be measured and other end is open to the atmosphere as shown in Fig. 2.8. The rise of liquid gives the pressure head at that point. If at a point A, the height of liquid say water is Jin Piezometer tube, then pressure at A N pxgxh m ig. 28 Piezometer. 2.6.2 U-tube Manometer. It consists of glass tube bent in U-shape, one end of which is connected to a point at which pressure is to be measured and other end remains open to the atmosphere as shown in Fig. 2.9. The tube generally contains mercury or any other liquid whose specific gravity is greater than the 2pocific gravity of the liquid whose pressure is to be measured. (2) For gauge pressure (©) For vacuum pressure Fig. 2.9 U-tube Manometer. (a) For Gauge Pressure. Let B is the point at which pressure is to be measured, whose value is p. ‘The datum line is A-A. Let Jy, = Height of light liquid above the datum line ‘hy = Height of heavy liquid above the datum line S) = Sp. gr. of light liquid 1, = Density of light liquid = 1000 x 5, Sy = Sp. gr. of heavy liquid = Density of heavy liquid = 1000 x 5, [44 Fluid Mechanics ] [As the pressure isthe same for the horizontal surface. Hence pressure above the horizontal datum line 4-A in the let column and in the right column of U-tube manometer should be same. Pressure above A-A in the left column D+ pix eX, Pressure above A-A in the right column 2X8 Xp Hence equating the two pressures p+ pth = pushy 5 P= ath ~ Py X 8 ¥hy)- (2.7) (6) For Vacuum Pressure. For measuring vacuum pressure, the level of the heavy liquid in the ‘manometer will be as shown in Fig. 2.9 (b). Then Pressure above A-A in the left column = pass + pyeh, +p Pressure head in the right column above A-A = 0 Pash, + pygh + p=0 5 =~ (Pash + pigh). 2.8) Problem 2.9. The right limb of a simple U-tube manometer containing mercury is open to the atmosphere while the left limb is connected to a pipe in which a fluid of sp. gr. 0.9 is flowing. The centre of the pipe is 12 cm below the level of mercury in the right limb. Find the pressure of fluid in the pipe if the difference of mercury level in the two limbs is 20 cm. Solution. Given : Sp. of Mui, s,=09 Density of fluid, P, = S, x 1000 = 0.9 x 1000 = 900 ke/m* eae eae 7 Density of mercury, 3.6 x 1000 kgim* 200m Difference of meeury vel, y= 20.em = 02 m | Height of fluid from A-A, hy —12= 8 cm = 0.08 m Let p = Pressure of fluid in pipe Equating the pressure above A-A, we get P+ Pishy = Pgh, or +900 x 9.81 x 0,08 = 13.6 x 1000 x 9.81 x.2 p= 13.6 x 1000 x 9.81 x .2 ~ 900 x 9.81 x 0.08 16683 - 706 = 25977 Nim? = 2.597 Niem*. Ans. Problem 2.10 A simple U-tube manometer containing mercury is connected to a pipe in which a Aluid of sp. gr. 0.8 and having vacuum pressure is flowing, The other end of the manometer is open to ‘atmosphere. Find the vacuum pressure in pipe, ifthe difference of mercury level in the two limbs is 40 cm and the height of fluid in the left from the centre of pipe is 15 em below Solution. Given : Sp. gr. of fluid, Fig. 2.10 0.8 Sp. gr. of mercury, 136 Density of fluid, 800 Density of mercury, 13.6 x 1000 Difference of mercury level, y= 40 em = 0.4 m. Height of liquid in left limb, /, A = 15 cm = 0.15 m. Let the pressure in pipe line A-A, we get Fig. 211 Pash, + pyehy + p= 0 [ Pressure and its Measurement 45] [pres + pishil 113.6 x 1000 x 9.81 x 0.4 + 800 x 9.81 x 0.15] [53366.4 + 1177.2] = ~ $4543.6 N/m? = ~ 5.484 N/em®. Ans. Problem 2.11 A U-Tube manometer is used to measure the pressure of water in a pipe line, which is in excess of atmospheric pressure. The right limb of the manometer contains mercury and is open to atmosphere. The contact between water and mercury is in the left limb. Determine the pressure of water in the main line, if the difference in level of mercury in the limbs of U-tube is 10 em and the free surface of mercury isin level with the centre of the pipe. Ifthe pressure of water in pipe line is reduced to 9810 N/m’, calculate the new difference in the level of mercury. Sketch the arrangements in both cases. Solution. Given : Difference of mercury = 10 em = 0.1 m The arrangement is shown in Fig. 2.11 (a) Ist Part Let py = (pressure of water in pipe line (ce, at point A) ‘The points B and C lic on the same horizontal line. Hence pressure at B should be equal to pressure at C. But pressure at B = Pressure at A + Pressure due to 10 em (or 0.1 m) of water =patpxexh where p = 1000 kg/m? and fr = 0.1 m = pg + 1000 x 9.81 x 0.1 = py + 981 Nim? amo) Pressure at C = Pressure at D+ Pressure due to 10 em of mercury = 04 pyX8% hy where pp for mercury = 13.6 x 1000 kg/m* and fig = 10 em = 0.1 m Pressure at C = 0+ (13.6 1000) x 9.81 0.1 = 133416 N ii) But pressure at B is equal to pressure at C. Hence equating the equa- tions (and (i), we get Pa + 981 = 133416 Fig. 2.11 (@ 4 = 133416 - 981 = 123606 IInd Part Given, py = 9810 Nim? Find new difference of mercury level. The arrangement is shown in Fig. 2.11 (b). In this case the pressure at A is 9810 Nim? which is less than the 12360.6 Nim?. Hence mercury in left limb will rise. ‘The rise of mercury in left limb will be equal to the fall of mercury in right limb as the total volume of ‘mercury remains same, Let x= Rise of mercury in left limb in em ‘Then fall of mercury in right limb = x cm ‘The points B, C and D show the initial conditions whereas points B*, C+ and D* show the final conditions. [46 Fluid Mechanics ] ‘The pressure at BY = Pressure at C or Pressure at A + Pressure due to (10 ~ x) cm of water = Pressure at D* + Pressure due 1 (10 ~ 24) em of mercury OF Pg +P x8, = Pot + PX hy 10-x (=) ise of =0 + (136 x 1000) x981 x (2=24) * Dividing by 9.81, we get or 1000 + 100 ~ 10x = 1360 ~ 272 or 272x~ 10x= 1360 ~ 1100 or 262x = 260 x= 200. 9.992 em 262 New difference of mercury = 10 - 2x em =10 ~ 2 x 0.992 Fig. 211 (6) 8.016 em. Ans. Problem 2.12 Fig. 2.12 shows a conical vessel having its outlet at A to which a U-tube manometer is connected. The reading of the manometer given in the figure shows when the vessel is empty. Find the reading of the manometer when the vessel is completely filed with water. Solution. Vessel is empty. Given Difference of mercury level fy = 206m Let ft, = Height of water above XX Sp. gr. of mercury, = 6 Sp. er. of water, S,= 10 Density of mercury, py = 13.6 x 1000 Density of water, p, = 1000 Equating the pressure above datum line X-X, we have Pa Xe Xs =p) XB xy or 13.6 x 1000 x 9.81 x 0.2 = 1000 x 9.81 fy hy = 2.72 m of water. Vessel is full of water. When vessel is full of water, the Fig. 2:2 pressure in the right limb will increase and mercury level in the right limb will go down, Let the distance through which mercury goes down in the Fight limb be, y cm as shown in Fig. 2.13. The mercury will rise in the left by a distance of y em. Now the datum line is 7-Z. Equating the pressure above the datum line Z-Z. Pressure in left limb = Pressure in right limb 13.6 x 1000 x9.81 (0.2 + 29/100) = 1000 x 9.81 x (3 + hy + y/100) [ Pressure and its Measurement 47) or 13.6 x (0.2 + 29/100) = (3 +2.72 + y/100) (2 fy = 272 em) or 2.12 + 27.2y/100 = 3 + 2.72 + y/100 or (27.2y - y100 = 3.0 or 262y = 3 x 100 = 300 300 300 = 145 em 362 71145 The difference of mercury level in two limbs = (20 + 2y) em of mercury = 2042x1145 = 20+22.90 = 42.90 em of mercury + Reading of manometer = 42.90 em. Ans, Problem 2.13 A pressure gauge consists of wo cylindrical bulbs B and C each of 10 sq. cm cross- sectional area, which are connected by a U-tube with vertical limbs each of 0.25 sq. cm cross-sectional area. A red liquid of specific gravity 0.9 is filled into C and clear water is filled into B, the surface of separation being in the limb attached to C. Find the displacement of the surface of separation when the Pressure on the surface in C is greater than that in B by an amount equal to 1 cm head of water. Solution. Given : Area of each bulb Band C, A= 10 em? ‘Area of each vertical limb, Sp. gr. of red liquid Let XX ie= Height of red liquid above X-X ig = Height of water above X-X Pressure above X-X in the left limb = 1000 x 9.81 X hip Pressure above X-X in the right limb = 900 x 9.81 x lic Equating the two pressure, we get 1000 X 9.81 x hip = 900 X 9.81 X hic p= 09 he @) Its density = 900 kg/m* initial separation level When the pressure head over the surface in C is increased by 1 cm of water, let the separation level falls by an amount equal to Z. Then Y-Y becomes the final separation level. Now fall in surface level of C multiplied by cross- sectional area of bulb C must be equal to the fall in separation level multiplied by cross-sectional area of limb. FINAL SEPARATION LEVEL. SEPARATED LEVEL Fall in surface level of C = Fall in separation level xa A [48 Fluid Mechanics Zxa _Zx025_ Z A 10 40 Also fall in surface level of C ise in surface level of B ze 40 ‘The pressure of 1 cm (or 0.01 m) of water = pgh = 1000 x 9.81 x 0.01= 98.1 Nim? Consider final separation level ¥-¥ “ Psa sve FY nie b= 100981 (2h +2) ton 091 (24422) = (22h Z) 00981 «84 ° e40. 40. " 100 (2h + 2) =a (24-2) 210 = = Dig yt. wet Zhe Z 209 (24-2) +01 40 40, But from equation i), ig =0.9 hic 2409 net & = 09409 he+ 0.01 0-40 or 42 3B x 974 01 40 ~ 40 or 2(H-2*2) 01 or (4 40 40 7= R208) «9.0678 m = 6.78 cm. Ans. 2.6.3 Single Column Manometer. Single column manometer is a modified form of a U-tube ‘manometer in which a reservoir, having a large cross-sectional area (about 100 times) as compared to the area of the tube is connected to one of the limbs (say left limb) of the manometer as shown in Fig. 2.15. Due to large cross-sectional area of the reservoir, for any variation in pressure, the change in the liquid level in the reservoir will be very small which may be neglected and hence the pressure is given by the height of liquid in the other limb. The other limb may be vertical or inclined, Thus there are two types Of single column manometer as : 1. Vertical Single Column Manometer. 2, Inelined Single Column Manometer. 1. Vertis -al Single Column Manometer Fig. 2.15 shows the vertical single column manometer. Let X-X be the datum line in the reservoir and in the right limb of the manometer, when it is not connected to the pipe. When the manometer is [ Pressure and its Measurement 49] connected to the pipe, due to high pressure at A, the heavy liquid in the reservoir will be pushed downward and will rise in the right limb. Let Ah = Fall of heavy liquid in reservoir ise of heavy liquid in right limb Height of centre of pipe above X-X Pq = Pressure at A, which is to be measured A= Cross-sectional area of the reservoir Cross-sectional area of the right limb 5S, = Sp. gr. of liquid in pipe 5p. gf of heavy liquid in reservoir and right limb Density of liquid in pipe Fig. 245. Vertical single column manometer. Density of liquid in reservoir Fall of heavy liquid in reservoir wil cause arise of heavy liquid level in the right limb. AX Ah = aX, A Now consider the datum line ¥-¥ as shovin in Fig. 2.15. Then pressure in the right limb above Y-¥. = 2x x (Ah + hy) 12 x (Ah + hy) + Pg Ah i) Pressure in the left limb above ¥-Y Equating these pressures, we have 2X gx (Ah + hy) = 18 x (Ah + hy) + Pa or a= Pa (Ah + hy) — pg(Ah+ hy) = Ahlp 2g - pig] + hyPog — hype. But from equation (i), ahs Oh axhy Pa= 2 [Pag — Pig + MnP 28 — hiPis (2.9) AS the area A is very large as compared to a, hence ratio “ becomes very small and can be neglected. Then P, = h2P2g — Iypis. (2.10) From equation (2.10), it is clear that as hy is known, and hence by knowing rz or rise of heavy liquid in the right limb, the pressure at A can be calculated. 2. Inclined Single Column Manometer Fig. 2.16 shows the inclined single column manom- eter. This manometer is more sensitive. Due to inclina- tion the distance moved by the heavy liquid in the right ‘Fig. 2.16 Inclined single column limb will be more. ‘manometer. (50. Fluid Mechanics Let = Length of heavy liquid moved in right limb from X-X inclination of right limb with horizontal Vertical rise of heavy liquid in right limb from X-X = Lx sin @ From equation (2.10), the pressure at A is Pa = hoPoe — IPs Substituting the value of ftp, we get Pa = sin 8X p29 ~ hypyg- 2.41) Problem 2.14 A single column manometer is connected to a pipe containing a liquid of sp. gr. 0.9 as shown in Fig. 2.17. Find the pressure in the pipe if the area of the reservoir is 100 times the area of the tube for the manometer reading shown in Fig. 2.17. The specific gravity of mercury is 13.6. Solution. Given : Sp. gr. of liquid in pipe, Density Sp. gt of heavy liquid, Density, p= 13.6 1000 Area of reservoir _ A ‘Area ofrightlimb 7 a= \° Height of tiquid, fy Rise of mercury in righ im, Fig. 247 fy =40.m=04m Let y= Pressure in pipe Using equation (2.9), we get a= Malas ~ Pus + Pas — MP = Gp * L136 x 1000 % 9.81 900 x 981] + 04% 13.6 x 1000 x 981 ~ 0.2 x 900 x 9.81 = {Ee (133416 ~ 8829] + 538604 - 1765.8 33.664 + 5336.4 ~ 1765.8 Nim? = 52134 Nim? = 5.21 Néem?, Ans. > 2.7. DIFFERENTIAL MANOMETERS Differential manometers are the devices used for measuring the difference of pressures between ‘wo points in a pipe or in two different pipes. A differential manometer consists of a U-tube, contain- ing a heavy liquid, whose two ends are connected to the points, whose difference of pressure is to be measured. Most commonly types of differential manometers are : 1. U-tube differential manometer and 2. Inverted U-tube differential manometer. 2.7.1 U-tube Differential Manometer. Fig. 2.18 shows the differential manometers of Utube type. [ Pressure and its Measurement 51) y i : trvepoaanncions biked Bev etineen ind Fig. 218 Usubedifferential manometer. In Fig. 2.18 (a), the two points A and B are at different level and also contains liquids of different sp. gr. These points are connected to the U-tube differential manometer. Let the pressure at A and B are py and Pp Let t= Difference of mercury level in the U-tube. y= Distance of the centre of B, from the mercury level in the right limb, x= Distance of the centre of A, from the mercury level in the right limb. py = Density of liquid at A. 2 = Density of liquid at B. = Density of heavy liquid or mercury. ‘Taking datum line at X-X. Pressure above X-X in the left limb = where py = pressure at A. Pressure above X-X in the right limb = p, x gx I+ p> X@XY+ Pp where pp = Pressure at B. Equating the two pressure, we have Pigthh +2) + Pq =PyX 8X N+ Posy + Pp 1X 8 I+ Pogy — pyslh + x) % s(P_~ 1) + P28) ~ Pik (2.12) Difference of pressure at A and B= hx g(p,~ Pi) + P2&y’ ~ Pree In Fig. 2.18 (6), the two points A and B are at the same level and contains the same liquid of density Py. Then Pressure above X-X in right limb = p,Xg xh+p,X@xx+Dp Pressure above X-X in left limb =p, xg x (+2) + Py Equating the two pressure P_X 4 XN PRE Pye PLX H+ + Py E Pa-Pa=PeX 8 N+ Pig — pial +) = 8x MP, pp. (2.13) Problem 2.15 A pipe contains an oil of sp. gr. 0.9. A differential manometer connected at the 1wo points A and B shows a difference in mercury level as 15 cm. Find the difference of pressure at the 10 points. gh #2) + Dy Pa-Pa [52 Fluid Mechanics ] Solution. Given : Sp. gr. of oil, 192. Density, py = 0.9 x 1000 = 900 kg/m? Difference in mercury level, 5 em = 0.15 m Sp. gf of mercury, 5,= 13.6 +. Density, py = 13.6 x 1000 kg/m® ‘The difference of pressure is given by equation (2.13) or Pa~Pp= 8X h(,~ Py) 81 x 0.15 (13600 - 900) = 18688 N/m? Ans. Problem 2.16 A differential manometer is connected at the two points A and B of two pipes as shown in Fig, 2.19. The pipe A contains a liquid of sp. gr. = 1.5 while pipe B contains a liquid of sp. gr. = 0.9. The pressures at A and B are 1 kgffcm® and 1.80 keffom? respectively. Find the difference in mercury level in the differential manometer. Space 15 ‘Solution, Pa =1 kat font Sp. gr. of liquid at A, $= 1.5 =. p, = 1500 Sp. gt. of liquid at B, $= 0.9 +. _p,= 900 im SPOS Pressure at A, py = I kgtfem? = 1 x 10* kgfim® 10*x 9.81 Nim? (1 kgf = 9.81 N) Pressure at B, py = 1.8 kgtfem? ae 1.8 x 10* kgfim? = 18 x 10° 981 Nim? (- Density of mercury = 13.6 x 1000 kg/m? Taking X-X as datum line, Pressure above X-X in the left limb = 13.6 x 1000 x9.81 x h + 1500 x 9.81 x 2 +3) +P, 13.6 x 1000 x 9.81 x h + 7500 x 9.81 + 9.81 x 10 Pressure above X-X in the right limb = 900 x 9.81 X(t + 2) + Pp = 900 x 9.81 x (i+ 2) + 1.8 x 108 981 Equating the two pressure, we get 136 x 1000 x 9.814 + 7500 x 9.81 + 9.81 x 10" = 900 x 9.81 x (i+ 2) + 18x 10*% 981 Dividing by 1000 x 9.81, we get T kgf = 981 N) * Fig. 2.19 136k +7.5 + 10= (h + 2.0) x9 + 18 or 13.6h + 17.5 = 0.9h + 18+ 18 =0.9h + 19.8 or (136 ~ 0.9)h= 19.8 ~ 17.5 oF 12.7 = 2.3 23. _ 9.181 m= 18.1 em. Ans. 27 Problem 2.17 A differential manometer is connected at the two points A and B as shown in Fig. 2.20. At B air pressure is 9.81 N/cnt (abs), find the absolute pressure at A. Solution. Given : Ait pressure at B= 9.81 Niem™ or 81x 10* Nim? Pressure and its Measurement 53) Density of oil = 0.9 x 1000 = 900 kg/m? Density of mercury 3.6 x 1000 ke/m* Let the pressure at A is py Taking datum tine at X-X Pressure above X-X in the right limb 1000 x 9.81 x 0.6 + Dp 1886 + 98100 = 103986 Pressure above X-X in the left limb x 1000 x 9.81 x0.1 +900 X9.81 0.2 + py = 133416 + 1765.8 + D4 MERCURY Equating the two pressure heads uRy 103986 = 13341.6 + 1765.8 + p, Spge= 136 Px = 103986 ~ 15107.4 = 8876.8 Fig. 2.20 (88876 8N_ N *{0000em? ~ Absolute pressure at A = 8.887 Nlem?, Ans. 2.7.2 Inverted U-tube Differential Manometer. It consists of an inverted U-tube, containing a light liquid. The two ends of the tube are connected 10 the points whose difference of pressure is to be measured. It is used for measuring difference of low pressures. Fig. 2.21 shows an inverted U-tube differential manometer connected to the two points A and B. Let the pressure at A is more than the pressure at B. Let ‘h, = Height of liquid in left limb below the datum line X-X Jk, = Height of liquid in right timb Difference of light liquid Density of liquid at A 2 = Density of liquid at B p, = Density of light liquid a= Pressure at A Pp = Pressure at B. ‘Taking X-X as datum line. Then pressure in the left limb below X-X Pa~PLX 8X ty Fig. 221 Pressure in the right limb below X-X = PaX 8X P,X Xt Equating the two pressure ~ 1X 8X hy =Pp~ PrX8XIig~P,XBXA or Pa~Pe=PiX 8X ly PaX EX lig—P,X Exh 2.14) Problem 2.18 Water is flowing through two different pipes to which an inverted differential ‘manometer having an oil of sp. gr. 0.8 is connected. The pressure head in the pipe A is 2 m of water, {find the pressure in the pipe B for the manometer readings as shown in Fig. 2.22. Solution. Given : aaa Pe Py =PX eX 2= 1000 9.81 x 2= 19620 Nim? Fig. 2.22 shows the arrangement, Taking X-Xas datum in, Pressure below X-X in the left imb = p,~ p, x ¢ x hy Pressure head at m of water [54_ Fluid Mechanics ] = 19620 ~ 1000 x 9.81 x 0.3 = 16677 Nim’. Pressure below X-X in the right limb ~ 1000 x 9.81 0.1 ~ 800 x 9.81 x 0.12 Pp ~ 981 ~ 941.76 = py ~ 1922.76 Equating the two pressure, we get 16677 = py ~ 1922.76 or Pq = 16677 + 1922.76 = 18599.76 Nim? or Py = 1.8599 Nem? Ans. Problem 2.19 in Fig. 2.23, an inverted differential manometer is connected to two pipes A and B which convey water. The fluid in manometer is oil of sp. gr. 0.8. For the manometer readings shown in the figure, find the pressure difference between A and B. Solution. Given : Sp. gr. of oil =0.8 + p,= 800 kg/m* Difference of oil in the two limbs = 30 + 20) ~ 30 = 20 em Taking datum line at X-X Pressure in the left limb below X-X = pa 1000 x 9.81 x0 = py — 2943 Pressure in the right limb below X-X Pp 1000 x 9.81 x 0.3 ~ 800 x 9.81 x 0.2 = py — 2943 — 1569.6 = py 4512.6 Eaguating the evo presse p,2983 = py — 4512.6 Pp ~ Pq = 4512.6 ~ 2943 = 1569.6 Nim’. Ans. Problem 2.20 Find out she differential reading ‘h’ of an inverted U-tube manometer containing oil of specific gravity 0.7 as the manometric fluid when connected across pipes A and B as shown in Fig. 2.24 below, conveying liquids of specific gravities 1.2 and 1.0 and immiscible with manometric fluid. Pipes A and B are located at the same level and assume the pressures at A and B 10 be equat. Solution. Given : Fig. 2.24 shows the arrangement. Taking X-X as datum line. Let Density of liquid in pipe A Density of liquid in pipe B Density of oil [ Pressure and its Measurement 55) Now pressure below X-X in the left limb 24 ~ 1200 x 9.81 x 0.3 ~ 700 x 9.81 x h Pressure below X-X in the right limb = Py — 1000 x 9.81 x (r+ 0.3) Equating the two pressure, we get P4— 1200 x 9.81 x03 ~ 700 x 9.81 x h But Pa ~ 1200 x 9.81 x 0.3 ~ 700 x 9.81 x h Dividing by 1000 x 9.81 1g ~ 1000 x 9.81 (hr + 0.3) 9 (given) 1000 x 9.81 (h + 0.3) = 1.203 -0.7h=- (n+ 03) or 03 x 1.24 0.Th= + 0.3 oF 0.36 - 0.3 = h-0.Th = 0. h = O06, 030” 030 £100 20cm, Problem 2.21 An inverted U-tube manometer is connected 10 1wo horizontal pipes A and B through which water is flowing. The vertical distance between the axes of these pipes is 30 em, When an oil of specific gravity 0.8 is used as a gauge fluid, the vertical heights of water columns in the two limbs of the inverted manometer (when measured from the respective centre lines of the pipes) are {found 10 be same and equal to 35 cm. Determine the difference of pressure between the pipes. Solution. Given : Specific gravity of measuring liquid = 0.8 The arrangement is shown in Fig. 2.24 (a) > Let py pressure at A Pp pressure at B 306m The points Cand D lie on the same horizontal line. Hence pressure at C should be equal to pressure at D. : E But pressure at C= p4~pek = p4~ 1000 x 9.81 x (0.35) WATER —a90m Oo” ‘And pressure at D = py ~ pyehy ~ posh “| | = pp 1000 x 9.81 x (0.35) ~ 800 x 9.81 0:3 But pressure at C= pressure at D aN WATER: Px ~ 1000 x 9.81 x 35 Fig. 2.24 (a) = py ~ 1000 x 9.81 x 0.35 ~ 800 x 9.81 x 0.3 or 800 9.81 x 0.3 = pp—p, or Ppa = 800 X 9.81 x 0.3 = 2384.4 [56 Fluid Mechanics ] > 2.8 PRESSURE AT A POINT IN COMPRESSIBLE FLUID For compressible fluids, density (p) changes with the change of pressure and temperature. Such problems are encountered in aeronautics, oceanography and meteorology where we are concerned ‘with atmospheric® air where density, pressure and temperature changes with elevation. Thus for fluids with variable density, equation (2.4) cannot be integrated, unless the relationship between p and p is, known. For gases the equation of state is Perr 2.15) P or ° dp Now equation (2.4) is cee 1, the lapse-rate is negative which means temperature decreases with the increase in height In atmosphere, the value of & varies with height and hence the value of temperature lapse-rate also varies. From the sea-level upto an elevation of about 11000 m (or 11 km), the temperature of air decreases uniformly at the rate of 0.0065°C/m. from 11000 m to 32000 m, the temperature remains constant at ~ 56.5°C and hence in this range lapse-rate is zero. Temperature rises again after 32000 m in air. [60 Fluid Mechanics ] Problem 2.22 (SI Units) Ifthe atmosphere pressure at sea level is 10.143 N/ent, determine the pressure at a height of 2500 m assuming the pressure variation follows (i) Hydrostatic law, and (i) isothermal law. The density of air is given as 1.208 kg/m’. Solution. Given : Pressure at sea-level, Po = 10.143 Nlem? = 10.143 x 10* Nim? Height, Z= 2500 m Density of air, Pp = 1.208 kg/m? (0 Pressure by hydrostatte law. For hydrostatic aw, p is assumed constant and hence p is given dp by equation 2? =~ y equation 57 =~ pg Integrating, we get or For datum line at sea-level, : P&Z or p= po~ Paz 10.143 x 10" ~ 1.208 x 981 x 2500 [> p= py = TSO fem? 10 208] 101430 - 29626 = 71804 or im = 7.18 Nlem?. Ans. (id Pressure by Isothermal Law. Pressure at any height Z by isothermal law is given by equation (2.18) as, p= pent Po. Po ar sincere [p Be eradgyg=ny _toxs 0.143 x 108 e = 10,143 x 108 ¢ © 2500% 1208 981)10.145 «104 1 (3391 75743 Nim? = 101430 x e° = 101430 x 5 7.874 Niem?, Ans. Problem 2.23 The barometric pressure at sea level is 760 mm of mercury while that on a mountain top is 735 mm. If the density of air is assumed constant at 1.2 kg/m’, what is the elevation of the ‘mountain top? Solution. Given : Pressure® at sea, ‘Po = 760 mm of Hg, 760 2 2 <2 x 13.6 x 1000 x 9.81 N/m? = 101396 N/m 1000 * Here pressure head (Z) is given as 760 mm of Hg. Hence (p/pg) = 760 mm of Hg. The density (p) for mercury = 136% 00h He psn wll geo 4% 2 136100981 x 2 Pressure and its Measurement 61 Pressure at mountain, p= 735 mm of Hg 235 2 SS 13.6 x 1000 x 9.81 = 98060 Nim’ (000 Density of air, p= 1.2 kgim? Let ft = Height of the mountain from sea-level. ‘We know that as the elevation above the sea-level increases, the atmospheric pressure decreases. Here the density of air is given constant, hence the pressure at any height ‘h’ above the sea-level is, given by the equation, P=Po-PX8xh PoP _ 101396 - 98060 px 12x981 Problem 2.24 Calculate the pressure at a height of 7500 m above sea level if the atmospheric pressure is 10.143 Nem’ and temperature is 15°C at the sea-level, assuming (i) air is incompressible, (ii) pressure variation follows isothermal law, and (ii) pressure variation follows adiabatic law. Take the density of air at the sea-level as equal to 1.285 kg/m’. Neglect variation of g with altitude. Solution. Given : or h 283.33 m. Ans. Height above sea-level, Z= 7500 m Pressure at sea-level, Po = 10.143 Nie? = 10.143 x 10* Nim? ‘Temperature at sea-level, t= 15°C s Ty = 273 + 15 = 288°K Density of air, = Po = 1.285 kg/m* ( Pressure when air is incompressible : dp oe palZ— Zo) [lap = fete oe pn or P=Po-peZ (= datum line = 0} = 10,143 x 10"~ 1.285 9.81 x 7500 101430 ~ 94543 = 6887 Nini? = 0688S, Ans. (ii) Pressure variation follows isothermal law : ‘Using equation (2.18), we have P= poe °T pete Poser. Peat “ee { Po Po wl = 101430 e890 = 101430 @° 780 1288 x981/01430 101430 e~ °°” = 101430 x 39376 199.39 N/m? or 3.993 N/em?. Ans. (iii) Pressure variation follows adiabatic law : [k= 14] oD wz 2] where p= 1.285 kg/m? : Dstpssuion 219, wetne —p=py[t [62 Fluid Mechanics ] p= 101430 | 1— 14=10) 9g (75001285) 9 ns 101430 = 101430 [1 ~ 2662]! = 101430 x (.7337)" N = 34310 Nim? or 3.4315. Ans. om Problem 2.25 Calculate the pressure and density of air ata height f 4000 m from sea-level where pressure and temperature of the air are 10.143 N/em® and 15°C respectively. The temperature lapse rate is given as 0.0065°C/m, Take density of air at sea-level equal to 1.285 kg/m". Solution. Given : Height, Z= 4000 m Pressure at sea-level, Po = 10.143 N/em? = 10.143 x 10 = 101430 Temperature at sea-level, 5°C ‘Temperature lapse-rate, Using equation (2.22), we have L az or — 0.0065 PRIEST), wnere Ps 101430 _ 274.09 RCE Poly 1285x288 = 0.0065 28 (#1) 27409 “Ck 0.0065 274,09 90065 % 27409 _ 9,181 981 uo ‘ ‘ALL .1815] i TW 1815 8188 ‘This means that the value of power index k = 1.222. (i Pressure at 4000 m height is given by equation (2.19) as onl! 9200) where k= 1.222 and py = 1.285 Po a2 1222-10 4000 x L285 p = 101430|1-(1222=19) 91 oe torso [t-( Tear) x98 = 101430 [1 0.09} = 101430 x 595 = 60350 Nim? = 6.035 ,., Ans. [ Pressure and its Measurement 63) (i Density. Using equation of state, we get where p= Pressure at 4000 m height 1p = Density at 4000 m height T= Temperature at 4000 m height Now Tis calculated from temperature lapse-rate as rats m 2+ x00» 15-068 4000» 13-2651 T=273+1= 273-11 = 262K P 60350 RT 274,09 x262 Problem 2.26 An aeroplane is flying at an altitude of 5000 m, Calculate the pressure around the aeroplane, given the lapse-rate in the atmosphere as 0.0065°K/m. Neglect variation of g with altitude. Take pressure and temperature at ground level as 10.143 N/em? and 15°C and density of air as 1.285 kg/em', Solution. Given : Height, Density is given by Pp kg/m? = 0.84 kg/m*. Ans. Lapse-rate, Pressure at ground level, Density, Temperature a 5000 m height = Tp + E. Height = 288 ~ 0065 x 5000 = 288 ~ 32.5 = 255.5°K. First find the value of power index k as From equation (2.22), we have = 2 -4(E) wR or = 0065 = Po___ 101430 where R= a Poly 1.285% 288 = 274.09 ~.0065 . k= 1.222 The pressure is given by equation (2.19) as ry 64 Fluid Mechanics Lm 1222-10 5000 x 1285 8-10 = 10140 1 -[ a 98 22002288 = 101430 [1-222 x 9,81 x 5000 1285 | Bt 101430 101430 [1 ~ 0.11288]°*? = 101430 x 0.5175, 5.249 Niem?, Ans. HIGHLIGHTS ‘The pressure at any point in a fluid is defined as the force per unit are. ‘The Pascal law states that intensity of pressure for a fluid at rest is equal in all directions Pressure variation ata point in a fluid at rest is given by the hydrostatic law which states thatthe rate of Increase of pressure in the vertically downward direction is equal to the specific weight of the fluid, 1 2 B -wapxs 4, The pressure at any point in a incompressible fluid (liquid) is equal tothe product of density of fluid at that point, acceleration due to gravity and vertical height from free surface of fluid, P=pxexZ 5. Absolute pressure isthe pressure in which absolute vacuum pressure is taken as datum while gauge pressure i the pressure in which the atmospherie pressure is taken as datum, Pate = Pa * Posse 6. Manometer isa device used for measuring pressure ata point in a fluid 7. Manometers are classified as (a) Simple manometers and (b) Differential manometer 8. Simple manometers are used for measuring pressure ta point while diferential manometers are used for measuring th difference of pressures between the two points in a pipe, or two different pipes 9. A single column manometer (or micrometer) is used for measuring small pressures, where aecuracy is required, 10. ‘The pressure ata point in static compressible fluid is obtained by combining two equations, 2, equation ‘of state for a gas and equation given by hydrostatic law. IL, The pressure at a height Z in a static compressible fluid (gas) under going isothermal compression b=) where py = Absolute pressure at sea-level or at ground level Z= Height from sea or ground level R= Gas constant = peat T= Absolute temperature 12, The pressure and temperature at a height Z in a static compressible fuid (gas) undergoing adiabatic ‘compression (p/p Pressure and its Measurement 65] 1B and temper, renii- where p, Ty are pressure and temperature at sea-level k = 1.4 fo ai. ‘The rate at which the temperature changes with elevation is known as Temperature Lapse-Rate, It is siven by es () RE if) & = 1, temperature is zero. Gi) > 1, temperature decreases with the increase of height. EXERCISE (A) THEORETICAL PROBLEMS «Define pressure. Obtain an expression for the pressure intensity ata point in a fluid, . State and prove the Pascal's law. ‘What do you understand by Hydrostatic Law ? |. Differentiate between : (2) Absolute and gauge pressure, (ji) Simple manometer and differential manom- eter, and (if) Piezometer and pressure gauges. . What do you mean by vacuum pressure ? What is @ manometer ? How are they classified ? What do you mean by single column manometers ? How are they used for the measurement of pressure ? What is the difference between U-tube differential manometers and inverted U-tube differential ‘manometers ? Where are they used ? ). Distinguish between manometers and mechanical gauges. What are the different types of mechanical Pressure gauges ? Derive an expression forthe pressure at a height Z from sea-level fora static air when the compression of the air is assumed isothermal. The pressure and temperature at sea-levels are and T, respectively Prove that the pressure and temperature for an adiabatic process ata height Z from sea-level fora static air * RT az. th se cea where pp and Ty are the pressure and temperature at sea-level What do you understand by the term, ‘Temperature Lapse-Rate'? Obtain an expression for the temperature Lapse-Rate. What is hydrostatic pressure distribution? Give one example where pressure distribution is non-hydrostatic, Explain briefly the working principle of Bourdon Pressure Gauge with a neat sketch (LNTU,, Hyderabad, $ 2002) ZI “and (B) NUMERICAL PROBLEMS A hrydraulic press has a ram of 30 cm diameter and a plunger of 5 em diameter. Find the weight lifted by the hydraulic press when the force applied atthe plunger is 400 N. [Ans. 14.4 KN] ‘A hydraulic press has @ ram of 20 em diameter and a plunger of 4 em diameter, It is used for lifting a ‘weight of 20 KN. Find the force required at the plunger. [Ans. 800 N] uid Mechanics ] 10, 2 13. Ma Calculate the pressure due to a column of 0.4 m of (a) water, (b) an ol of sp. gr 0.9, and (c) mercury of sp. g. 136. Take densiy of water, p= 100048. (Ans. (a) 03924 Niem?, () 0383 Nlem™ (e) 5.38 Nem] ‘The pressure intensity at a point in a fluid is given 4.9 N/em!. Find the corresponding height of fluid when it is: (a) water, and (b) an oil of sp. gr. 0.8. Ans. (a) 5 m of water, (b) 6.25 m of oil] ‘An oil of sp. gr. 0.8 is contained in a vessel. At a point the height of oi is 20 m. Find the corresponding height of water at that point (Ans. 16 m} ‘An open tank contains water upto a depth of 1.5 m and above it an oil of sp. gr. 0.8 fora depth of 2m. Find the pressure intensity: () at the interface of the two liquids, and (i) atthe bottom of the tank. Ans. () 1.57 Néem*, (i) 3.04 Nlem?] ‘The diameters of a small piston and a large piston of a hydraulic jack are 2 em and 10 em respectively. A force of 60 N is applied onthe small piston. Find the load lifted by the large piston, when : (a) the pistons are atthe same level, and (b) small piston is 20 em above the large piston. The density of the liquid in the jack is siven as 1000 *E., [Ans. (a) 1500 N, (6) 1520.5 N] Determine the gage and absolute pressure at a point whichis 2.0 m below the fee surface of water, Take atmospheric pressure as 10.1043 Niem™ [Ans. 1.962 Nem’ (gauge), 12.066 Nem’ (abs) ‘A simple manometer is use to measure the pressure of oil (Sp. a= 08) flowing ina pipeline. Hs sight Jimb is open tothe atmosphere and let limi is connected to the pipe. Te centre of the pipe is 9 em below the level of mercury (sp. gr 13.6) in the right Limb. I the difference of mercury level inthe two limbs is 15 cm, determine the absolute pressure of the il in the pipe in Neem Ans. 12088 Neem] ‘A simple manometer (U-tube) containing mercury is connected toa pipe in which an ol of sp. gO. is, flowing, The pressure inthe pipe is vacuum, The other end of the manometer is open to the atmosphere. Find the vacuum, pressure in pipe, if the dference of mercury level inthe two limbs is 20 em and height, o el in the let limb from the centre ofthe pip is 15 em below. [Ans. 27.86 Neem") ‘Asingle column vertial manometer (Le, mietometer) is connected wa pipe containing el of sp. a. 09. ‘The area ofthe reservoir is 80 times the area of the manometer tube. The reservoir contains mercury of sp. 136. The level of meroury in the reservoir i ata height of 30 em below the centre ofthe pipe and difference of mercury levels inthe reservoir and right limb is $0 em. Find the pressure in the pipe Ans. 6474 Neem?) {Apipe contains an il of sp. gr 08. A differential manometer connected atthe two points A and B of the Pipe shows a difference in mercury level as 20 em. Find the difference of pressure atthe two points {Ans.25113.6Nin) [A Usube differential manometer connects (wo pressure pipes A and B. Pipe A contains carbon tetrachloride having a specific gravity 1.594 under a pressure of 11.772 Nem and pipe B contains oil of Sp. gr. 08 under a pressure of 11.772 Nim. The pipe A lies 2.5 m above pipe B. Find the diference of pressure measured by mercury as fluid filling U-tube. Ans. 31.36 em of mercury] A Gifferential manometer is connected at the two points A and B as shown in Fig, 2.25, At B air pressure is 7.848 Niem? (ahs.), find the absolute pressure at A, [Ans. 6.91 Nem] Fig. 2.26 Pressure and its Measurement 67) 15. 16. 1. v. 20. a. 2. 24, 26. ‘An inverted differential manometer containing an oll of sp. gr. 0.9 is connected to find the difference of Pressures at two points ofa pipe containing wate. If te manometer reading is 40 cm, find the diference Of pressures. {Ans 392.4 Nin] In above Fig. 226 shows an inverted differential manometer connected to two pipes A and B containing ‘water. The fluid in manometer i oil of sp. gr. 08. For the manometer readings shown inthe figure, find the difference of pressure head between A and B. {Ans. 0.26 m of water] It the atmospheric pressure at sea-level is 10.143 Nem, determine the pressure ata height of 2000 m assuming thatthe pressure variation follows: () Hydrostatie law, and (i) Isothermal law. The density of air is given as 1.208 ke/m’ (Ans. (i) 7.77 Nlem*, (i) 8.03 fem") Caleulate the pressure a a height of 8000 m above sea-level if the atmosphere pressure is 101.3 kN/n® and temperature is 15°C atthe sea-level assuming (air is incompressible, i) pressure variation follows adiabatic law, and (1) pressure variation follows isothermal law. Take the density of ai atthe sea-level as equal to 1.285 kgln®. Neglect variation of g with altitude {Ans (607.5 Nin? i) 31.5 KNin? (i) 37.45 KN?) Caleutate the pressure and density of air ata height of 3000 m above sea-level where pressure and tem- perature ofthe air are 10.143 Nem’ and 15°C respectively. The temperature lpse-ate is given as 0.0068" Kim. Take density of air at seacevel equal to 1.285 kgf’ {Ans 6896 Nlem’ 0.937 kgf’) ‘An aeroplane is fying at an altitude of 4000 m. Caleulate the pressure around the aeroplane, given the lapse-rate in the atmosphere as 0.0068°K/n. Neglect variation of g with altitude. Take pressure and temperature at ground level as 10.143 Néem? and 15°C respectively. The density of air at ground level is, sven as 1.285 kgf’ {Ans. 6.038 Neem’) ‘The atmospheric pressure atthe sea-level is 101.3 kN/n? and the temperature is 15°C. Caleulate the pressure 800 m above ses-level, assuming ) aris incompressible, (1 isothermal variation of pressure and density, and (i) adiabatie variation of pressure and density Assume density of air at sea-level as 1.285 kg/m’, Neglect variation of *g° with altitude {Ans. i) 501.3 Nin, (i) 37.45 kN/n?, i) 31.5 KN/m} An oil of sp gr. is 08 undera pressure of 137.2 kNim? (i) Whats the pressure head expressed in metre of water? (i) What isthe pressure head expressed in metre of oil? (Ans. (?) 14m, (i) 175m) \. The atmospheric pressure atthe sea-level is 101.3 kN/m? and temperature is 15°C, Calculate the pressure '8000 m above sea-level, assuming : (#) isothermal variation of pressure and density, and (i) adiabatic variation of pressure and density. Assume density of air at sea-level as 1.285 kg/m. Neglect variation of “with altude, Derive the formula that you may use. [Ans, (() 37.45 kNin?, (i) 31.5 kNém] What are the gauge pressure and absolute pressure at a point 4 m below the free surface of a liquid of specific gravity 1.53, if atmospheric pressure is equivalent to 750 mm of mercury. [Ans. 60037 Nin? and 160099 Nim] 5. Find the gauge pressure and absolute pressure in Nim at a point 4 m below the free surface of a liquid of sp. . 1.2, ifthe atmospheric pressure is equivalent to 750 mm of mercury. (Ans. 47088 Nim ; 147150 Nim] ‘A tank contains a liguid of specific gravity 0.8. Find the absolute pressure and gauge pressure at a point, ‘Which is 2 m below the free surface ofthe liquid. The atmospheric pressure head is equivalent fo 760 mm of mercury. [Ans. 117092 Nim? ; 15696 Nim?) i: HYDROSTATIC FORCES, Or ucianm eos > 3.1 INTRODUCTION This chapter deals with the fluids (i.c., liquids and gases) at rest. This means that there will be no relative motion between adjacent or neighbouring fluid layers. The velocity gradient, which is equal to the change of velocity between two adjacent fluid layers divided by the distance between the layers, will be zero or MH The shear stress which sequal to jt. il also be zero. Then the frees acting ‘on the fluid particles will be : 1, due to pressure of fluid normal to the surface, 2. due to gravity (or self-weight of fluid particles). > 3.2. TOTAL PRESSURE AND CENTRE OF PRESSURE Total pressure is defined as the force exerted by a static fluid on a surface either plane or curved when the fluid comes in contact with the surfaces. This force always acts normal to the surface. Centre of pressure is defined as the point of application of the total pressure on the surface. There are four cases of submerged surfaces on which the total pressure force and centre of pressure is to be determined. The submerged surfaces may be : 1. Vertical plane surface, 2. Horizontal plane surface, 3. Inclined plane surface, and 4, Curved surface. > 3.3 VERTICAL PLANE SURFACE SUBMERGED IN LIQUID Consider a plane vertical surface of arbitrary shape immersed in a liquid as shown in Fig. 3.1. Let A= Total area of the surface Distance of C.G. of the area from free surface of liquid ‘entre of gravity of plane surface Gs ‘entre of pressure ‘h® = Distance of centre of pressure from free surface of liquid. 69 [70 Fluid Mechanics (@) Total Pressure (F). The total pressure on the surface _ FREE SURFACE OF LIQUID. may be determined by dividing the entire surface into a number of small parallel strips. The force on small strip is then caleu- lated and the total pressure force on the whole area is calculated by integrating the force on small strip. Consider a strip of thickness dh and width b at a depth of h from free surface of liquid as shown in Fig. 3.1 Pressure intensity on the strip, p= pg (See equation 2.5) ‘Area of the strip, dA=bxdh Total pressure force on strip, dF Total pressure force on the whole surface, F=far= foghxoxan 5 foxnra bt Joxnah = firs Moment of suri are toute fe surface of id = Are of sufce x Dsoeof C6. trom te sate xh . F=pgah GB.) For water the value of p = 1000 kg/m? and g = 9.81 mv/s?. The force will be in Newton, (b) Centre of Pressure (h*). Centre of pressure is calculated by using the “Principle of Moments”, Which states that the moment ofthe resultant force about an axis is equal to the sum of moments of the ‘components about the same axis. The resultant force F is acting at P, at a distance h* from free surface of the liquid as shown in Fig. 3.1. Hence moment of the force F about free surface of the liquid = F x h* 32) ‘Moment of force dF, acting on a strip about free surface of liquid xh = pghx bx dhxh Sum of moments of all such forces about free surface of liquid AF = pgh xb x dh} = Joahsbxaich= pg fox hichts = ps fo at Joran = fonran pg je dA (es bdh = dA) loment of Inertia ofthe surface about free surface of liquid Sum of moments about free surface = ely 63) [ Hydrostatic Forces on Surfaces_71| Equating (3.2) and (3.3), we get xi" = poly But F=pgah peal x H* = ply or ne = Pele, Gay peal” An By the theorem of parallel axis, we have helgt Axe where fg = Moment of Inna of area about an axis pasing through the C.G.of the area and parallel tothe fre surface of liquid Substituting Jpn equation (3.4), we get let AP ty yy os Ah Ah ) In equation (3.5), fis the distance of C.G. of the area of the vertical surface from free surface of the liquid. Hence from equation (3.5), itis clear that (i) Centre of pressure (iH) lies below the centre of gravity of the vertical surface, (id. Thedstance of centre of pressure fom free surface of liquid is independent ofthe density ofthe vig ie ‘Table 3.1 The moments of inertia and other geometric properties of some important plane surfaces ‘Moment of inertia | Moment of ‘about an axis passing | inertia about Plane surface CG. fromthe | Area | through CG.and | — base (1p) base parallel 10 base (Io) 1. Rectangle oa a we oe 2 36 2 Contd... 72_ Fluid Mechanics ‘Moment of inertia | Moment of Plane surface CG fromthe | Area | about an axis passing | inertia about base through C.G.and— | base (Ip) parallel to base (Ic) Problem 3.1 A rectangular plane surface is 2 m wide and 3 m deep. It lies in vertical plane in water. Determine the total pressure and position of centre of pressure on the plane surface when its upper edge is horizontal and (a) coincides with water surface, (b) 2.5 m below the free water surface. Solution. Given : Width of plane surface, = 2m Depth of plane surface, d=3m (a) Upper edge coincides with water surface (Fig. 3.2). Total pressure is given by equation (3.1) as F=pgAh {Fr a Fe DOWNSTREAM HINGE Fig 39 F, = 1450 x 9.81 x 24 x 2.1 = 71691 N Similarly, Fy= pat Alia where 3 = 1,000 kg/m* fia = Depth of C.G. of gate from free surface of water 1 x12=06m 2 “ F, = 1000 x 9.81 x 24 x 0.6 = 14126 N (i Resultant force on the gate = F, — F, = 71691 ~ 14126 = $7565 N. Ans. (i) Position of centre of pressure of resultant force. The force F, will be acting at a depth of 1i,* from free surface of liquid, given by the relation nee ais A bd _ 2x12" 2 2 where Ig 88 * 2ax2 Distance of F, from hinge = (15 + 1.2) ~ fy? = 2.7 2.1571 = 0.5429 m The force F, will be acting at a depth of h,* from free surface of water and is given by +21 = 0.0571 + 21 hy = 2.1571 m [78 Fluid Mechanics ] 1288 mF Distance of F; from hinge = 1.2- 0.8 ‘The resultant force 57565 N will be acting at a distance given by _ 1691 x 54291412604 ~ 57565 = 3821-56504 shove hinge 57565 = 0.578 m above the hinge. Ans. (iid) Force at the top of gate which is capable of opening the gate. Let F is the force required ‘on the top of the gate to open it as shown in Fig. 3.9. Taking the moments of F, F, and F, about the hinge, we get Fx 124 F, x04 =F, x.5429 FXx.5429- F x04 or 12 _ 11691 x.5429 -14126 x04 _ 38921 ~ 56504 12 12 = 277255 N. Ans. Problem 3.8 A caisson for closing the entrance to a dry dock is of trapezoidal form 16 m wide at the top and 10 m wide at the bottom and 6 m deep. Find the total pressure and centre of pressure on the caisson if the water on the outside is just level with the top and dock is empty. Solution. Given : Width at top =16m WATER SURFACE, —— te Wh Bom 10m ar Der, arom Arne tapeoia ABCD, mee 42 BOXAD) 2 8 (10+16) 2 b—tom—4 = 09 608 ER Depth of C6 of aerial aes ABCD from ive sarass of wt, 10x6x3+ 19-19) 165 1x6 i 2 *0%3 78 = 80235 «769m rom water sate, ( Total Pressure (F). Total pressure, F is given by [ Hydrostatic Forces on Surfaces 79 F = pgAh = 1000 x 9.81 x 78 x 2.769 N = 2118783 N = 2.118783 MN. Ans. (ii) Centre of Pressure (h#). Centre of pressure is given by equation (3.5) as a wer Ig= MOI. of waperolda ABCD shoot C3 tet ig = MOL. of rectangle FECE shout CG. fa = MOLL of 90 As ABF and ECD aout ts C Ten y= OnE «a0 at To a g, is the M.O1L of the rectangle about the axis passing through Gy. ‘M.OLL of the rectangle about the axis passing through the C. f the trapezoidal /,, + Area of rectangle x x)? where x; is distance between the C.G, of rectangle and C.G. of trapezoidal = 3.0 - 2.769) = 0.231 m M.OLL of FBCE passing through C.G. of trapezoidal = 180 + 10 x 6 x (0.231)? = 180 + 3.20 = 183.20 m* ba® Now 1g, =M.O1L. of ABD in Fig. 3.11 about G, = SF _ (16-10) x6? ~ 36 The distance between the C.G. of triangle and C.G. of wapezoidal = (2.769 — 2.0) = 0.769 M.OL of the two As about an axis passing through C.G. of trapezoidal = Ig, + Atea of triangles x (.769)? 6 m* A Re > = 36.04 9%6 x (769)? A = 36.0-+ 10.64 = 46.64 rt 1g.= MOLL. of trapezoidal about its CG. MOLL. of rectangle about the C.G. of trapezoidal om +M.O.L of triangles about the C.G. of the trapezoidal = 183.20 + 46.64 = 229.84 m* { Bc Fig. 3.11 where A= 78, h = 2.769 229.84 * Fax2.769 + 2.769 = 1.064 + 2.769 = 3.833 m. Ans. Alternate Method ‘The distance of the C.G. of the trapezoidal channel from surface AD is given by (refer to Table 3.1 ‘on page 71) [80 Fluid Mechanics ] (a+b) h (a+b) °3 _ 210416) 6 ““(10+16) 3 a= 10,6 = 16 and h=6) = 3x 2=2.769 m This is also equal (0 the distance of the C.G. of the trapezoidal from free surface of water. i= 2.769 m Total pressure, F = peak ( 78) = 1000 x 9.81 78 x 2.769 N = 2118783 N. Ans. Centre of Pressure, (h*)= 2.4% ~ vi Now fg from Table 3.1 is given by, (@+4ar+6%) 9, (IOP +4x10x16+164) jg = yp = OO 6 36(a+0) 36(10 +16) (100 +640 + 256) 4 = M004 610 256) 946 = 299.846 m 3ox26 — *716= 220-846 = 22S 9 769 A=T8m) 78x 2.769 = 3.833 m. Ans. Problem 3.9 A trapezoidal channel 2 m wide at the bottom and I m deep has side slopes I : 1. Determine (i) the total pressure, and (ii) the centre of pressure on the vertical gate closing the channel when it is full of water. Solution. Given : Width at bottom Depth, m /}-——— 4m + Side slopes a ‘A WATER SURFACE, D Top width, +1+l=4m = F e 4 ‘Area of rectangle FBEC, x1=2m? + 2m Area of trapezoidal ABCD, A=A,\+A,=2+1=3m? Depth of C.G. of rectangle FBEC from water surface, Fig. 3.12 0.5 m [ Hydrostatic Forces on Surfaces 81] Depth of C.G. of two triangles ABF and ECD from water surface, = Depth of C.G. of trapezoidal ABCD from free surface of water Asti Ay fia _ 20541033333 yay (A+ Ay) +) (0 Total Pressure (F). Total pressure F is given by F=pgAh = 1000 x 9.81 x 3.0 x 0.44444 = 13079.9 N. Ans. (ii) Centre of Pressure (h*). M.O.L of rectangle FBCE about its C.G., abd _2KP 1 oD 12 6 MOLL. of FBCE about an axis passing through the C.G. of trapezoidal or Ig, =o, +A, X [Distance between C.G. of rectangle and C.G. of trapezoidal]? = ba ax [hi aP = t +2. [0.5 ~ 4444]? = .1666 + .006182 = 0.1727 MOLL. of the two triangles ABF and ECD about their C.G., od? _ (ex 2 O° 36° 360~=~SO6 CB M.OLL. ofthe two triangles about the C.G. of trapezoidal, Ig," = lo, # Ap x [Distance between C.G. of triangles and C.G. of trapezoidal]? a} = he i[aas 1 sagt 0555 + (1111)? 1 A =t sc ig DY 55 + 0.01234 = 0.06789 m+ ‘M.OLL of the trapezoidal about its C.G, Ig= lo, *+ 1g," 1727 + 06789 = 0.24059 m+ Then cee of esr) onthe veal pel tw eg = HO ai 3d 2 0425 Ams. + A444 = 0.18046 + 4444 = 0.6248 [82 Fluid Mechanics ] Alternate Method ‘The distance of the C.G. of the trapezoidal channel from surface AD is given by (refer to Table 3.1 ‘on page 71). a+b) 0x24) 1, (a+b) “3 @¥4) “3 0.444 m 0.444 m = peli = 1000 x 9.81 3.0 x 44 » A=30) = 13079 N. Ans. =4 and h= 1) i Total pressure, F Centre of pressure, We = Ssh where Ig, from Table 3.1 is given by pal stab?) op Bedeaxde#) 52 4 = j6a+6) e+) <= 36x6 = ne = 02407444 = 0,625 m. Ans. 30x44 Problem 3.10 A square aperture in the vertical side of a tank has one diagonal vertical and is completely covered by a plane plate hinged along one of the upper sides of the aperture. The diagonals of the aperture are 2 m long and the tank contains a liquid of specific gravity 1.15. The centre of aperture is 1.5 m below the free surface. Calculate the thrust exerted on the plate by the liquid and position ofits centre of pressure. Solution. Given : Diagonals of aperture, AC = BD = 2m «+ Area of square aperture, A = Area of AACB + Area of AACD = ACKBO , ACXOD _ 2x1 a) 2 2 Sp. gr of liquid 1s Density of liquid, p= 1.15 x 1000 = 1150 kg/m? Depth of centre of aperture from free surface, hatsm 42 Peery 2 8 SUUAKE Wy c VY #2 Fig. 3.13 [ Hydrostatic Forces on Surfaces 83] (O The thrust on the plate is given by F = peAh = 1150%9.81 x2 x 1.5 = 338445, Ans. (4 Centre of pressure (H") is given by we ateah ah MOL. of ABCD about diagonal AC MOL. of wiangle ABC about AC+ M.OL of triangle ACD about AC AcxOBT , ACxOD" ++ M.O.Lof atriangleaboutits base = 2 2 2 2xP 2 2x? 12 1 We = 345 = — 1) 415 = 1.611 m. Ans. 2x15 3x2x15 Problem 3.11. A tank contains water upto a height of 0.5 m above the base. An immiscible liquid of sp. gr. 0.8 is filled on the top of water upto 1 m height. Calculate : (i) total pressure on one side of the tank, (ii) the position of centre of pressure for one side of the tank, which is 2 m wide. Solution. Given : Depth of water Depth of liquid Sp. gr of liquid ; Density of liquid, 1p, = 0.8 x 1000 = 800 kg/m? Density of water, 1000 kg/m* Width of tank m () Total pressure on one side is calculated by drawing pressure diagram, which is shown in Fig. 3.14. Intensity of pressure on top, p20 Intensity of pressure on D (or DE), pp = igh 100 x 9.81 x 1.0 = 7848 Nim? a T ets = 1 I } “a J, = : 8 7048 * 4005, Fig. 3.14 Pushy + pag 0.5 848 + 1000 x 9.81 x 0.5 = 7848 + 4905 = 12753 Nim? Now force F, = Atea of AADE x Width of tank Intensity of pressure on base (or BO).p; Lxapxpe nao bi x7sn20=7848N [84 Fluid Mechanics ] Force F, = Area of rectangle DBFE x Width of tank 0.5 x 7848 x 2 = 7848. N Fy = Area of AEFC x Width of tank 1 = Ex EF x PCK20= Fx 0.5 4905 2.0 = 2452.5 N ++ Total pressure, FF,+Fy+Fy = 7848 + 7848 + 2452.5 = 18148.5 N, Ans. (ii) Centre of Pressure (h*). Taking the moments of all force about A, we get Pentti 2ap eras} aps FyaDs 2 2) sis ie 708x278 (1048) 24825 (10423) 18312 8840) rm op As Problem 3.12 A cubical tank has sides of 1.5 m. It contains water for the lower 0.6 m depth. The upper remaining partis filled with oil of specific gravity 0.9. Calculate for one vertical side of the tank: (a) total pressure, and (b) position of centre of pressure. Solution. Given : Cubical tank of sides 1.5 m means the dimensions of the tank are 1.5. m x 1.5 mx 1.5 m. Depth of water 0.6 m Depth of liquid 0.6 =0.9 m Sp. gt. of liquid 09 Density of liquid, Py = 0.9 x 1000 = 900 kg/m* Density of water, 2 = 1000 kg/m? (@) Total pressure on one vertical side is calculated by drawing pressure diagram, which is shown in Fig. 3.15. | Aon osm) oborsran-oa | wf . : Hydrostatic Forces on Surfaces 85] Intensity of pressure at A, p, = 0 Intensity of pressure at D,pp = pyg X h = 900 x 9.81 x 0.9 = 7946.1 N/m” Intensity of pressure at B, py= p,gh, + pagh, = 900 x 9.81 x 0.9 + 1000 x 9.81 x 0.6 = 7946.1 + 5886 = 13832.1 Nim? Hence in pressure diagram : DE = 7946.1 Nim?, BC = 13832.1 Nim?, FC = 5886 N/m? ‘The pressure diagram is split into triangle ADE, rectangle BDEF and triangle EFC. The total pres- sure force consists of the following components : (@ Force F, = Area of triangle ADE x Width of tank = (fx ADx DE) x 1.5 Width = 1.5 m) = (40.9% 79461) x 15 N 5363.6 N (ii) Force F = Area of rectangle BDEF x Width of tank ps peyets= 6x Ra) x= ISLS ‘hiss wilh acing the C6. of ese RDEP ie atx dss f 096 SS «12m teow A Ci ae Fs Ae of inal EFC Wh ak =( "haf ils tig the C6. tg EF dans 0.92 x06 X EF x FC) x 1.5 = (4% 06x 5886) x 1.5 = 2688.7 N below 4 Total pressure force on one vertical face of the tank, FoF, +F,+Py 5363.6 + 7151.5 + 2648: (6) Position of centre of pressure Let the total force F is acting ata depth of h* from the free surface of liquid, ie. from A. Taking the moments of all forces about A, we get Fxht = Fx 064 Fx 12+ F,x 13 _ A x06+ F x12+ 6 x13 - F __ 53636 06+ TISLS x 12+ 2648713 151638 = 1.005 m from A. Ans. 151638 N. Ans. or he » 3.4 HORIZONTAL PLANE SURFACE SUBMERGED IN LIQUID Consider a plane horizontal surface immersed in a static fluid, As every point of the surface is at the ‘same depth from the free surface of the liquid, the pressure intensity will be equal on the entire surface and equal to, p = pgh, where h is depth of surface. [86 Fluid Mechanics ] Let A= Total area of surface FREE SURFACE Then total force, F, on the surface =p x Area= pg xh XA = pgAh h Depth of centre of pressure from free surface = h. Problem 3.13 Fig. 3.17 shows a tank full of water. Find : Total pressure on the bottom of tank (ii) Weight of water in the tank. (iii) Hydrostatic paradox between the results of (i) and (ii). Width of tank is 2 m. Solution. Given : pam Depth of water on bottom of tank hy =3+06=3.6m Width of tank 2m Length of tank at bottom = =4m ‘Arca at the bottom, A =4x2=8m? (i Total pressure F, on the bottom is F=pgAh = 1000x9.81x8x36 0 [> amo]? 282528 N. Ans. Fig. 347 »@ x Volume of tank = 1000 x9.81 x [3 x0.4x244x.6 x2] = 1000 x 9.81 [2.4 + 4.8] = 70632.N. Ans. (iii) From the results of (i) and (i), itis observed that the total weight of water in the tank is much Jess than the total pressure atthe bottom of the tank. This is known as Hydrostatic paradox. Fig. 3.16 (i) Weight of water in tank > 3.5 INCLINED PLANE SURFACE SUBMERGED IN LIQUID Consider a plane surface of arbitrary shape immersed in a liquid in such a way that the plane of the surface makes an angle @ with the free surface of the liquid as shown in Fig, 3.18. FREE LIQUID SURFACE; Fig. 318 Inclined immersed surface. Let A = Total area of inclined surface T; = Depth of C.G. of inclined area from free surface int = Distance of centre of pressure from free surface of liquid 1 = Angle made by the plane of the surface with fee liquid surface. [ Hydrostatic Forces on Surfaces_87| Let the plane of the surface, if produced meet the free liquid surface at 0. Then 0-0 is the axis perpendicular to the plane of the surface. Let J = distance of the C.G. ofthe inclined surface from 0-0 _y® = distance of the centre of pressure from 0-0. Consider a small strip of area dA at a depth ‘h from free surface and at a distance y from the axis 0-0 as shown in Fig. 3.18. Pressure intensity on the strip, P=pah Pressure force, dF, on the strip, dF = p x Area of sttip = pgh x dA Total pressure force onthe whole, F=f dF= pga puctomrie sis, HAA cing hh=ysin P= Jonexyxsind ‘Water surface Dia. of plate, 4=30m Area, A= © G.0)? = 7.0685 m? zm 4 a Distance, DC=1m,BE=2m AB_BE-AE_ BE~DC_20-1.0_1 In aasc, sin @= AB. BEWAE _ BE- DC. C7 Be RC 3g. 3a ‘The centre of gravity ofthe plate i atthe middle of BC, i. ata distance 1.5 m from C. The distance of centre of gravity from the free surface of the water is given by F=cD+CGsino=10+15%4 =15m. (0 Total pressure on the front face ofthe plate is given by F=peAh = 1000 9.81 x 7.0685 x 1.5 = 104013 N. Ans. (i Let the distance of the centre of pressure from the free surface ofthe water be h*. Then using equation 3.10), we have ‘sin 8 = Je. sin?@ = fosinle, a i Al [92 Fluid Mechanics ] n plt eT w = eZ eas aaz = here Ig FO A= Fa = 15 mand sin O= 4 Substituting the values, we get x (2 Zatx(3) : nee 3S 4524 ais Rees 16 "9x15 4 3 + 152.0416 415 => 5416 m. Ans. 16x9x15 Problem 3.17 A rectangular gate 5 m x2 mis hinged at its base and inclined at 60° to the horicon- tal as shown in Fig. 3.22. To keep the gate in a stable position, a counter weight of 5000 kefis attached at the upper end of the gate as shown in figure. Find the depth of water at which the gate begins to fall. Neglect the weight of the gate and friction at the hinge and pulley. Solution. Given : Length of gate =5m Width of gate =2m a= 60" Weight, W= 5000 ket 5000 x 9.81 N = 49050 N (co L kgf = 9.81 N) |Asthe pully isretonless the force acting at B= 49050 N. First find the total force F acting on the getc AB for a given depth of wate. AE _h__S 2h sind ~ sino” 372 VE Area of gate immersed in water, A = AD x Width x 3H <2 = From figure, AD= Also depth of the C.G. ofthe immersed area A Total force F is given by F = pgAh = 1000 x9.81 x 44 8 ix “The centre of pressure of the immersed surface, ht i given by Igsin?0 = hee +h Ah where Jg = M.O.L of the immersed area _ bx (ADy 2x “1 12 WB. _ to ae “12X3xV3 9x3 [ Hydrostatic Forces on Surfaces 93] Now inte Scho, c= he=2%, ceoie= HL in 60° ror) AC=AD-CD a ee BONS RB ON” Taking the moments about hinge, we get 19620 49050 x 5.0 = Fx At 39240 245250 “ 3x3 3 _ 9% 245250 _ 1 = OES - 56.5 n= (56.25)"® = 3.83 m. Ans. Problem 3.18 An inclined rectangular sluice gate AB, 1.2 m by 5 m size as shown in Fig. 3.23 is installed to control the discharge of water. The end A is hinged. Determine the force normal to the gate applied at B 10 open it Solution. Given : A= Area of gate = 1.2 x 5.0 = 6.0 m? Depth of C.G. of the gate from free surface of the water = ft = DG= BC - BE FREE WATER SURFACE O cD = 5.0- BG sin 45° 1 =5.0- 06x — = 476m “The total pressure force (F) acting on the gat, F= pedi = 1000 x 9.81 x 6.0 x 4.576 = 269343 N This force is acting at H, where the depth of Hf from Fig. 323 free surface is piven by Io sin?@ = + [94 Fluid Mechanics ] where [= MOL of gate = 24 = 50X12" 2 2 72m AS 4 4.576 = 013 + 4.576 = 4.589 m Depth of centre of pressure h* = 6x4576 a But from Fig. 3.23 (a), * asin ase ig. (a), OH sin Distance, 589 x JF = 6489 m Distance, Bo=—>_ = 5x J2=70T1m ana Distance, BH = BO ~ OH = 7.071 ~ 6489 = 0.582 m Distance 2 - 0.582 = 0.618 m Taking the moments aout the hinge A P x AB = F x (AH) where Ps the fore nomal tthe gate applied at B Px 12 209383 x 0518 2683430618 ~ 12 Problem 3.19 A gate supporting water is shown n Fig. 3.24 Find th eight ofthe water 50 that the pte tps about ihe hinge, Tak he wlth o the gate ani Solution. Given: 0 =60° hh * sin 60° V3. P = 138708 N. Ans. Distance, where h = Depth of water. The gate will stat tipping about hinge B if the resultant pressure force acts at B. If the resultant pressure force passes through a point which i lying from B to C anywhere on the gate, the gate will ip ‘over the hinge. Hence limiting ease is when the resultant force passes through B. But the resultant force passes through the centre of pressure. Hence for the given position, point B becomes the centre of pressure. Hence depth of centre of pressure, hv = (h—3)m FREE WATER SURFACE = losin? @ 5 Ah HINGE But h* is also given by ‘Taking width of gate unity. Then Area, AzACxI Fig. 3.24 bd? _1xAC? 82h? 2 12 12 12x33 9x3 [ Hydrostatic Forces on Surfaces 95] 3 2 2h x in? 60° he 4 Equating the two values of h*, 7 he ++ Height of water for tipping the gate = 9m. Ans. Problem 3.20 A rectangular sluice gate AB, 2 m wide and 3 m long is hinged at A as shown in Fig. 3.25. Itis kept closed by a weight fixed to the gate. The total weight of the gate and weight fixed to the gate is 343350 N. Find the height of the water ‘h’ which will just cause the gate to open. The centre of gravity of the weight and gate is at G. Solution. Given : Width of gate, b= 2m; Length of gate L= 3m Area, x3=6m? x3=9m Weight of gate and Let / is the required height of water. Depth of C.G. of the gate and weight From Fig. 3.25 (a), ~ ED = h~(AD~ AE) ~ (AB sin @~ EG tan 8) = h~G sin 45° - 0.6 tan 45°) ~ 2.121 ~ 0.6) = (h~ 1.521) m ‘The total pressure force, F is given by F = pgAh = 1000 x9.81 x6 x (h 1.521) = 58860 (lr ~ 1.521) N. ‘The total force F is acting at the centre of pressure as shown in Fig. 3.25 (b) at H. The depth of H from free surface is given by h* which is equal to 45 xsin? 45° 0375 “ —— ——— + (h- 1.521 6x(h— 1521) * isa *4 X [96 Fluid Mechanics ] HINGE ons" @ Fig. 3.25 Now taking moments about hinge A, we get 343350 x EG = F x AH AK or 343350 x 0. sin 45° [Fm AAKH, Fig.325 (b) AK = AH sin @= AH sin 45°. A ae 58860 (ht ~ 1.521) x AK ‘sin 45° 343350 x0.6 xsin 45°_ 03535 x7 AK fi 38860 (1521) (1521) 2 37s But Ak =h -ac= — 375 _ 4 (41.521) -AC wolf 1320 * 7 oy But AC = CD ~ AD = h~AB sin 45° = h-3 xin 45° = h-2.121 Substituting this value in (i), we get 315 $= 1521) h- 2.121 nisi * ~« , 315 315 +2421 - 1.521 +06 Ait 521 uu) Equating the two values of AK from (i) and (ii) L521 [ Hydrostatic Forces on Surfaces 97| 0.3535x7 _ _ 0.375 h=152 = 1521 or 375 + 0.6 (h~ 1.521) = 0.375 + 0.6 h~ 0.6 x 1.521 or . ATAS — 375 + 0.6 x 1.521 = 2.0995 + 0.9126 = 3.0121 +06 m. Ans. Problem 3.21 Find the total pressure and position of centre of pressure on a triangular plate of base 2 mand height 3 m which is immersed in water in such a way that the plane of the plate makes an angle of 60° with the free surface of the water. The base ofthe plate is parallel to water surface and at 4@ depth of 2.5 m from water surface. SURFACE, Solution. Given : Base of plate, m Height of plate, h=3m Area, Ae PRL DS 3 yt 2 2 Inclination, 6 = 60° Depth of centre of gravity from free surface of water, hi =2.5 + AG sin 60° Fig. 3.26 1 BB 1 sebeaxtl —— {sacLortiatuia} = 2.5 + 866 m= 3.366 m (0 Total pressure force (F) F = peAl = 1000 x 9.81 x 3 x3.366 = 99061.38 N. Ans. (Go Centre of pressure (h). Depth of centre of pressure from free surface of water i given by 29 where I,= ME = 2%3 3 3% 6D 1S xsin? 60° 3X 2366 + 3.366 = O.111 + 3.366 = 3.477 m. Ans. > 3.6 CURVED SURFACE SUB-MERGED IN LIQUID Consider a curved surface AB, sub-merged in a static fluid as shown in Fig. 3.27. Let dA is the arca of a stall strip at a depth of fr from water surface. ‘Then pressure intensity on the area dA is = pgh and pressure force, dF = p x Arca = pghx dA GAD This force dF acts normal to the surface. Hence total pressure force on the curved surface should be F: J pghaa (3.12) [98 Fluid Mechanics WATER n SURFACE _C |5 7 3 iar io ancoso E ——— ‘AREA dA 8 0) (a) a Fig. 327 But here as the direction of the forces on the small areas are not in the same direction, but varies from point to point. Hence integration of equation (3.11) for curved surface is impossible. The problem can, however, be solved by resolving the force dF in two components dF, and dF, in the x and y directions respectively. The total force in the x and y directions, i.e., F, and F, are obtained by integrating dF, and dF, Then total force on the curved surface is +R B.13) FE and inclination of resultant with horizontal is tan @ = — B14) Resolving the force dF given by equation (3.11) in x and y directions : dF, = dF sin = pghdA sin 8 : dF = pghdA} and DahdA cos 8 “Total orcs in th x andy dition ae Fy J dP. J pati sin 0 pe f ht sin 0 19 and Fy= | aFy= J penta cos 0= pe tas oor 1) Fig. 3.27 (b) shows the enlarged area dA. From this figure, ie., AEFG, ‘Ths inequation (3.15). sin 9 PE Verte pojectinf the rca dt nd hene the exprsin oe J isn ® repens th press fore on te pote asa ofthe cued suc on he vere plane, Ths F, = Total pressure force on the projected area of the curved surface on vertical plane. (3.17) Also dA cos 6 horizontal projection of dA and hence MA cos @ isthe volume of the liquid contin nthe elena ae dA upto fee suace ofthe Lig Ths [A cos is the ol volume conte betwen the cured surface extend ut five src, Hence pf td cos is th ttl wight uppity the carved surface. Thus Fy=pe [nas cos 0 = weight of lid supported bythe ved surface upto fie surface of ii ous) [ Hydrostatic Forces on Surfaces 99 In Fig. 3.28, the curved surface AB is not supporting any fluid. In such cases, F, is equal to the weight of the imaginary liquid supported by AB upto free surface of liquid. The direction of F, will be taken in upward direction, Problem 3.22 Compute the horizontal and vertical components of the total force acting on a curved surface AB, which is in the form of a quadrant of a circle of radius 2 m as shown in Fig. 3.29. Take the width of the gate as unity. Solution. Given : Width of gate Radius of the gate Distance ‘equation (3.17) as, F, = Total pressure force on the projected area of curved surface AB on vertical plane rotal pressure force on OB {projected area of curved surface on vertical plane = OB x 1) = ped wo 981 x21 (+2) WATER SURFACE Fig. 3.28 {Area of OB = A= BOx1=2x1=2, Ti = Depth of C.G. of OB from free surface = 1.5+ 3) Fy 81 x 2000 x 2.5 = 49050 N. Ans. +2 2x25 75 = 0.1333 + 2.5 = 2.633 m from free surface. Vertical force, F,, exerted by water is given by equation (3.18) F, = Weight of water supported by AB upto free surface Weight of portion DABOC Weight of DAOC + Weight of water AOB 1g [Volume of DAOC + Volume of AOB] = 1000 x 9.81 [apxaoxi+ = (aoy x1] [100 Fluid Mechanics ] “| = 1000 x 9.81 [3.0 + RIN = 60249.1 N. Ans. Problem 3.23 Fig. 3.30 shows a gate having a quadrant shape of radius 2 m. Find the resultant force due to water per metre length of the gate. Find also the angle at which the total force will act. Solution. Given : a Radius of gate Width of gate Horizontal Force = 1000 x 9.81 [1sx20x1+ 2x Force on the projected area of the curved surface on vertical plane = Force on BO = peal 1 where A= Areaof BO=2x1=2m’,h astm F, = 1000 x 9.81 x2. 1 = 19620 N Thi wile at dn of 22 = m fom ssi i Vertical Foree, F, F, = Weight of water (imagined) supported by AB 1g x Area of AOB x 1.0 = 1000 9.81 x E (2)4x 1.0 = 30819 N ‘This will act ata distance of 28 = 4%20 anon Resultant force, Fis given by Fs (haP = 196205 30BT9 = [5859445004 DABBIOTOT = 368344 N. Ans. ‘The angle made by the resultant with horizontal is given by F, _ 30819 F, 19620 . @ = tan"! 1.5708 = $7° 31’. Ans. Problem 3.24 Find the magnitude and direction of the resultant force due to water acting on a roller gate of cylindrical form of 4.0 m diameter, when the gate is placed on the dam in such a way that water is just going t0 spill. Take the length of the gate as 8 m. Solution. Given : Dia. of gate Radius, Length of gate, = 0.848 m from OB. tan 5708 [ Hydrostatic Forces on Surfaces 101) Horizontal force, F, acting on the gate is WATER, SURFACE F, = peAli = Force on projected area of curved surface ‘ACB on vertical plane = Force on vertical area AOB where A = Area of AOB = 4.0 x 8.0 = 32.0 m? I; = Depth of CG. of AOB = 4/2 = 2.0m F, = 1000 9.81% 32.0 2.0 627840 N aa Vertical force, F, is given by F, = Weight of water enclosed or supported (actually or imaginary) by the curved surface ACB = pg x Volume of portion ACB 1g Area of ACB XI = 1000 x 9.81 x F(R) x8.0 = 9810 x 52)" x 81 It will be acting in the upward direction. Resultant force, = «[F2 + Fe = 6270+ AVSION = 798328 N. Ans. F, _ 493104 Direction of resultant force is given by tan = 0.785 ia wlan L a F, 627840 ee 0=31°8' Ans. Problem 3.25 Find the horizontal and vertical component of water pressure acting on the face of a tainter gate of 90° sector of radius 4 m as shown in Fig. 3.32. Take width of gate unity. Solution. Given : Radius of gate, R=4m Horizontal component of force acting on the gate is F, = Force on area of gate projected on vertical plane = Force on area ADB mesure = pea where A= ABx Width of gate =2KADx1 (cs AB = 2AD) Fig. 3.32 x 4x sin 45° = 8 x .707 = 5.656 m? (1 AD = 4 sin 45°} AB 25.856 <9 998m 2. 2 F, = 1000 x 9.81 x 5.656 x 2.828 N = 186911 N. Ans. Vertical component F, = Weight of water supported or enclosed by the curved surface Weight of water in portion ACBDA = pg x Area of ACBDA x Width of gate 1000 x 9.81 x [Area of sector ACBOA ~ Area of ABO] x 1 [102 Fluid Mechanics ] AOx BO 2 : AAOB is a right angled] Kp? = 9810 x| ER? s10%|% = 9810 [4° 224) «44796 N. Ans ao Problem 3.26 Calculate the horizontal and vertical components of the water pressure exerted on a tainter gate of radius 8 m as shown in Fig. 3.33. Take width of gate unity. Solution. The horizontal component of water pressure is given by Ah “orce on the vertical area of BD where A= BD x Width of gate = 4.0 1 = 4.0 m ‘orce on the area projected on vertical plane WATER SURFACE ¢ = 1000 x 9.81 x 4.0 x 2.0 = 78480 N. Ans. Fig. 333 Vertical component of the water pressure is given by F, = Weight of water supported or enclosed (imaginary) by curved surface CB = Weight of water in the portion CBDC 1g x [Area of portion CBDC] x Width of gate = pg x [Area of sector CBO ~ Area of the triangle BOD] x 1 30. BDx be = 010981 xan = 360 2 2 (+ DO = BO cos 30° = 8 x cos 30°) 9810 x [16.755 — 13.856] = 28439 N. Ans. Problem 3.27 A cylindrical gate of 4 m diameter 2 m long has water on its both sides as shown in Fig. 3.34. Determine the magnitude, location and direction of the resultant force exerted by the water ‘on the gate. Find also the least weight of the cylinder so that it may not be lifted away from the floor. Solution. Given : WATER SURFACE A Dia. of gate Radius m (@ The forces acting on the left side of the cylinder are = ‘The horizontal component, F,, where F,, = Force of water on area projected on vertical plane = Force on area AOC Hydrostatic Forces on Surfaces 103) F,, = weight of water enclose by ABCOA = 1000 9.81 x[ER| 200 = 9810 x © x 2? x20 = 123276 N. Right Side of the Cylinder = ptA;fz = Force on verical area CO = 1000 9.81 «2%23{ Ay=COx1=2«1=2 mh: 9240 N Weight of water enclosed by DOCD = pax [Ee] x Width of gate = 1000 x 9.81 x © x 2?x2 = 61638 N Resultant force in the direction of x, F,= F,,~ Fe, = 156960 ~ 39240 = 117720 N Resultant force in the direction of y, F, =F, + Fy, = 123276 + 61638 = 184914 N (H Resultant force, F is given as, F= JF +F = ¥(117720)' +(184914)° = 219206 N. Ans. (id Direction of resultant force is given by F, _tsdo1a an 5707 an O= 117720 0 =57° 31" Ans. (iii) Location of the resultant force 24 — 2.67 m from the top surface of water on left side, while F,, 33 m from free surface on the right side of the cylinder. The resultant Force, F,, acts ata distance of 2 scsaradisaceot2 2 force F, in the direction of x will act at a distance of y from the bottom as F,xy= F, [4-267] ~ Fy, [2-133] or 117720 x y = 156960 x 1.33 ~ 39240 x .67 = 208756.8 - 26290.8 = 182466 12466 _ 155 m from the bottom. 11720 ae) 8488 m from AOC towards Force Facts ata distance 28 from AOC ora distance " an leat 0c Aso a8 ata distance #2 088 m om AOC toast ip of AOC. Te eat force F, will act ata distance x from AOC which is given by [104 Fluid Mechanics ] Fy xx= Fy, X 8488 — Fx 8488 or 184914 x x= 123276 x 8488 - 61638 x .8488 = 8488 [123276 - 61638] = 52318.4 318.4 Teaoqa 702829 m from AOC. (iv) Least weight of cylinder. The resultant force in the upward direction is F, = 184914 N ‘Thus the weight of cylinder should not be less than the upward force F,. Hence least weight of cylinder should be at least. 184914 N. Ans. Problem 3.28 Fig. 3.35 shows the cross-section of a tank full of water under pressure. The length of the tank is 2 m. An empty cylinder lies along the length of the tank on one of its corner as shown. Find the horizontal and vertical components of the force acting on the curved surface ABC of the oylinder. Solution. Radius, R= 1m o2 katen? Length of tank, 1=2m Pressure, 2 keflem’ {962 Nlem? = 1.962 x 10* Nh _p 196210" pg 1000x981 Free surface of water will be at a height of 2 m from the top of the tank. Fig. 3.36 shows the equivalent free surface of water. Pressure head, (® Horizontal Component of Force F,= peak where A= Area projected on vertical plane 1.5 2.0 = 3.0 m? Ls 5 1000 x 9.81 x 3.0 x 2.75 = 80932.5 N. Ans. (ii) Vertical Component of Force ‘eight of water enclosed or supported actually or imaginary by curved surface ABC ‘eight of water in the portion CODE ABC ‘Weight of water in CODFBC ~ Weight of water in AEFB But weight of water in CODFBC ‘= Weight of water in [COB + ODFBO] = os 7 + nox 00] 2 = 1000%9.81 Fat st0x25] x2 4458.5 N 9g [Area of AEFB] x 2.0 Weight of water in AEFB Hydrostatic Forces on Surfaces 105 = 1000 x 9.81 [Area of (AEFG + AGBH ~ AHB)] x 2.0 In AAHO, sing = AH _ 05 10 ‘AO BH @= 30° (0 ~ HO = 1.0 - AO cos @ = 1.0 ~ 1x 00s 30° = 0. ABH = Area ABO ~ Area AHO = ni 3 AHXHO _ AR? 05% 866 _ 5455 30020 Weight of water in AEF 810 x [AE x AG + AG x AH — 0.0453] x 2.0 = 9810 x [2.0 x .134 + .134 x .5 ~ 0453] x 2.0 18.10 x [.268 + .067 — .0453] x 2.0 = 5684 N : F, = 6458.5 ~ 5684 = 587745 N, Ans. Problem 3.29 Find the magnitude and direction of the resltant water pressure acting on a curved Area, face of adam which is shaped according to the elation y = as shown in Fig. 3.37. The height ofthe water retained by the dam is 10 m. Consider the width of the dam as unity. Solution, Equation of curve AB is Height of water, Width, ‘The horizontal component, F, is given by Fig. 3.37 ressure due to water on the curved area projected on vertical plane ressure on area BC seh where A=BCx1=10x1m,h=4+x10=5m F, = 1000 «9.81 x 10 x5 = 490500 N Vertical component, F, is given by F, = Weight of water supported by the curve AB ‘eight of water in the portion ABC = palArea of ABC] x Width of dam [[xe]1.0 {arora ay asaase [0] ar) 000 x 9.81 x [3yF ay = 2043022)" 2pep -m00 5] 20402 fy"? = 19620 110% 19620 x 31.622 = 620439 N [106 Fluid Mechanics ] Resultant water pressure on dam [F? + F} = y(490500)° +(620439)" = 790907 N = 790.907 KN. Ans. tiven by F, _ 620439 F, 490500 = 51° 40", Ans. F Direction of the resultant tan @ = 1.265 Problem 3.30 A dam has a parabolic shape y (z) as shown in Fig, 3.38 blow having x =6m and yo = 9m. The fluid is water with density = 1000 kg/m’. Compute the horizontal, vertical and the resultant thrust exerted by water per metre length of the dam. Solution. Given : Equation of the curve OA is yi Width of dam, _ ( Horizontal thrust exerted by water F, = Force exerted by water on vertical surface OB, ie., the surface obtained by projecting ont [53h 6 m the curved surface on vertical plane = pet Fig. 3.38 = 1000981 (9 x 1) x5 (ii) Vertical thrust exerted by water F, = Weight of water supported by curved surface OA upto free surface of water ‘eight of water in the portion ABO 9g x Area of OAB x Width of dam = 10009381 x( fx dy] x10 1 ) = 1909.81 [faa] 10 vey ap A = 19620 x|2—_| = x= 9] sox] 9620 x 5 I 19620227 = 8160, As [ Hydrostatic Forces on Surfaces 107) (iii) Resultant thrust exerted by water F= JF) +82 = (3973054 353160 = 531574 N. Ans. Direction of resultant is given by 353160 _ 5 agg F, 397305 O= tan”! 0.888 = 41.63". Ans. Problem 3.31 A cylinder 3 m in diameter and 4 m long retains water on one side. The cylinder is supported as shown in Fig. 3.39. Determine the horizontal reaction at A and the vertical reaction at B The cylinder weighs 196.2 kN. Ignore friction. WATER SURFACE Solution. Given : Dia. of cylinder Length of cylinder Weight of cylinder, W= 196.2 KN = 196200 N Horizontal force exerted by water F,,= Force on vertical area BOC = pedi eal tx 2 F, = 1000 x 9.81 x 12 x 1.5 = 176580 N ‘The vertical force exerted by water F, = Weight of water enclosed in BDCOB where A=BOCxI=3x4= 12m’, = pe (Ea?) x1 10003 9.81«% x 4= 138684 N Force Fis acting in the upward direction, For the equilibrium of eylinder Horizontal reaction at A "76580 N = Weight of cylinder ~ F, 196200 ~ 138684 = S7516 N. Ans. Vertical reaction at B > 3.7. TOTAL PRESSURE AND CENTRE OF PRESSURE ON LOCK GATES Lock gates are the devices used for changing the water level in a canal or a river for navigation. Fig. 3.40 shows plan and elevation of a pair of lock gates. Let AB and BC be the two lock gates. Each gate is supported on two hinges fixed on their top and bottom at the ends A and C. In the closed position, the gates meet at B. Let F = Resultant force due to water on the gate AB or BC acting are right angles to the gate R = Reaction at the lower and upper hinge P = Reaction at the common contact surface of the two gates and acting perpendicular to the contact surface. Let the force P and F meet at O. Then the reaction R must pass through O as the gate AB is in the ‘equilibrium under the action of three forces. Let @ is the inclination of the lock gate with the normal to the side of the lock. [108 Fluid Mechanics ] In ZABO, ZOAB = ZABO = 8. Resolving all forces along the gate AB and putting equal to zero, we get R cos 0 P cos = OorR=P (3.19) WATER SURFACE UPSTREAM SIDE DOWNSTREAM SIDE Fig. 3.40 Resolving forces normal to the gate AB Rsin 0+ P sin @- F=0 o F=Rsin 8+ P sin 9=2P sin 0 (eRe?) F 6.20 Zaind ae To calculate P and R In equation (3.20), P can be calculated if F and @ are known. The value of 8 is calculated from the angle between the lock gates. The angle between the two lock gate is equal to 180° — 28. Hence @ can be calculated. The value of F is calculated as : me hot water onthe apt sie igh wa om dw ie aren onthe pon ota ie rps nh on Sora ie of ep i wisn oe Now 8A i 1 x Hy x 1x a 2 i ea oat : simu, Fess = pact i al 1H? _pgltt? | — Fy Pal pale Substituting the value of @ and F in equation (3.20, the value of P and R can be calculated. Reactions at the top and bottom hinges Let R, = Reaction of the top hinge Resultant force F= [ Hydrostatic Forces on Surfaces 109) R, = Reaction of the bottom hinge Then R=R, +R, ‘The resultant water pressure F acts normal to the gate. Half of the value of F is resisted by the hinges of one lock gates and other half will be resisted by the hinges of other lock gate. Also Fy acts at A Hy a distance of 5 from bottom while F, ats at a distance of “> from botiom. ‘Taking moments about the lower hinge R,xsinOxH 0 where H/= Distance between two hinges Resolving forees horizontally R,sin 0 +R, sin = LB alii) 2 From equations (i) and (i), we can find R, and Ry. Problem 3.32 Each gate of a lock is 6 m high and is supported by two hinges placed on the top and bottom of the gate. When the gates are closed, they make an angle of 120°. The width of lock is 'm. Ifthe water levels are 4 m and 2 mon the upstream and downstream sides respectively, determine the magnitude of the forces on the hinges due to water pressure Solution. Given : HINGE Ry, Height of lock Width of lock Width of each lock gate Angle between gates “ 6 2 = 30° Height of water on upstream side Ay and " Fig. 3.41 Total water pressure on upstream side F, = pgAyh where Ay = Hy x1= 40 «2.887 m? = 1000 x 9.81 x 4 x 2.887 x 2.0 = 226571 N Foxe Fw beating at adsanee ot 133m om oom Similarly, total water pressure on the downstream side F = pgAzhia, where A= Hz x= 2% 2.887 m? = 1000 x 9.81 x 2x 2.887 x 1.0 [110 Fluid Mechanics ] = 56643 N Hy 2 FF, will act ata distance of S*= > = 0.67 m from bottom, Resultant water pressure on each gate F = Fy ~ F, = 26571 ~ 56643 = 169928 N. Let xis height of F from the bottom, then taking moments of F,, F and F about the bottom, we have Fxx= F,x133- Fx 0.67 or 169928 x x = 226571 x 1.33 ~ 56643 x 0.67 226571 x 133 — 56643 x 0.67 _ 30139-37950 _ | 551, 169928 169928 F__ 169928 From equation (3.20), P 169928 N 2sind 2sin30 From equation (3.19), R= P = 169928 N, If Ry and Rp are the reactions atthe top and bottom hinges, then Ry + Ry= R= 169928 N. ‘Taking movements of hinge reactions R, Ry and R about the bottom hinges, we have Ry X60 + RyX0= RX 155 ~ Ry = 169928 ~ 43898 = 126030 N. Ans. Problem 3.88 The end gates ABC of a lock are 9 m high and when closed include an angle of 120°. The width of the lock is 10 m. Each gate is supported by two hinges located at I m and 6 m above the bottom of the lock. The depths of water on the two sides are 8 m and 4 m respectively. Find: (i) Resultant water force on each gate, (ii) Reaction between the gates AB and BC, and (iii) Force on each hinge, considering the reaction of the gate acting in the same horizontal plane as resultant water pressure. Solution. Given : Height of gate Inclination of gate PUN ) ELEVATION Fig. 3.42 [ ‘Hydrostatic Forces on Surfaces 111] Width of lock =10m 5 Width of each lock = S30 8 I= 577m Depth of water on upstream side, Hy Depth of water on downstream side, Hy (0 Water pressure on upstream side F,= pea where Ay = DH, = 5.773 x 8 = 46.184 m, hy = BE $=40m F, = 1000 x 9.81 x 46.184 x 4,0 = 1812260 N = 1812.26 kN Water pressure on downstream side, Fre peAshs where Ay = 1% Hy 773 x4 = 23.092 m, Tn = 4 = 20 1000 x 9.81 x 23.092 x 2.0 = 453065 N = 453.065 kN Resultant water pressure 1 ~ Fy = 1812.26 ~ 453,065 = 1359.195 kN (i Reaction between the gates AB and BC. The reaction (P) between the gates AB and BC is given by equation (3.20) as pe P= 1389195 = sin 2xsin 30° = 1359.195 kN. Ans. (ii) Force on each hinge. If Ry and Ry are the reactions at the top and bottom hinges then Ry + Ry But from equation (3.19), R = . Ry + Ry 359.195 359.195 epee 3 ‘The resultant force F will act at a distance x from bottom is given by Fx 2.67 - F, x 133 or 1812.26%2.67-453.065%133 “7359195 Hence R is also acting at a distance 3.11 m from bottom, ‘Taking moments of Ry and R about the bottom hinge Ry X [6.0 ~ 1.0] = Rx (x ~ 1.0) Rx(e=L0)_ 1359195211 r 50 50 Ry= R~ Ry = 1359.195 ~ 573.58 85.615 kN. Ans. 67m om toma Ft Hn 1.3m fom tom. [112 Fluid Mechanics ] > 3.8 PRESSURE DISTRIBUTION IN A LIQUID SUBJECTED TO CONSTANT HORIZONTAL/VERTICAL ACCELERATION In chapters 2 and 3, the containers which contains liquids, are assumed to be at rest. Hence the liquids are also at rest. They are in static equilibrium with respect to containers. But if the container ‘containing a liquid is made to move with a constant acceleration, the liquid particles initially will move relative to each other and after some time, there will not be any relative motion between the liquid particles and boundaries of the container. The liquid will take up a new position under the effect of acceleration imparted to its container. The liquid will come to rest in this new position relative to the container. The entire fluid mass moves as a single unit, Since the liquid after attaining a new position is in static condition relative to the container, the laws of hydrostatic can be applied to determine the liquid pressure. As there is no relative motion between the liquid particles, hence the shear stresses and shear forces between liquid particles will be zero. The pressure will be normal (0 the surface in contact with the liquid. ‘The following are the important cases under consideration : (Liquid containers subject to constant horizontal acceleration, (i) Liquid containers subject to constant vertical acceleration, 3.8.1 Liquid Containers Subject to Constant Horizontal Acceleration. Fig. 3.43 (a) shows a tank containing a liquid upto a certain depth. The tank is stationary and free surface of liquid ishorizontal, Let this tank is moving with a constant acceleration ‘a’ in the horizontal direction towards right as shown in Fig. 3.43 (b). The initial free surface of liquid which was horizontal, now takes the shape as shown in Fig. 3.43 (b). Now AB represents the new free surface of the liquid. Thus the free surface of liquid due to horizontal acceleration will become a downward sloping inclined plane, with the liquid rising at the back end, the liquid falling at the front end. The equation for the free liquid surface can be derived by considering the equilibrium of a fluid element C lying on the free surface. ‘The forces acting on the element C are : Free surface of y que Origa igi surface Free surface of fue |e . — Tank (staionay) @ ) ‘Tank moving Fig. 3.43 (D the pressure force P exerted by the surrounding fluid on the element C. This force is normal to the free surface. (id, the weight of the fluid element i.e., mx g acting vertically downward. (iii) accelerating force ie., m x a acting in horizontal direction. [ Hydrostatic Forces on Surfaces 113) Resolving the forces horizontally, we get Psin@+mxa=0 or P sin @=~ ma oli) Resolving the forces vertically, we get P cos @~ mg =0 or P cos O=mxg ii) Dividing (0) by (i), we get tn o=- 4 (onan) 11.204) ee ‘The above equation, gives the slope ofthe free surface ofthe liquid which i contained in a tank which is subjected to horizontal constant acceleration. The term (a/g) isa constant and henee tan 8 will be constant The -ve sign shows thatthe fee surface of ligud is sloping downwards. Hence the tree surface isa stright plane inlined down at an angle @ along the direction of acceleration. Now let us find the expression forthe pressure at any point D in the liquid mass subjected to horizontal acceleration. Let the point Dis ata depth of “Wr fiom the fee surface. Consider an elementary pris DE of height "F"and eros-secional area dA as shown in ig. 3-44 [Unes of constant pressure Consider the equilibrium of the elementary prism DE. The forces acting on this prism DE in the vertical direction are ()_ the atmospheric pressure force (pp x dA) at the top end of the prism acting downwards, (ii) the weight of the element (p x g x hx dA) at the C.G. of the element acting in the downward direction, and (ii) the pressure force (p x dA) at the bottom end of the prism acting upwards. Since there is no vertical acceleration given to the tank, hence net force acting vertically should be ze10. px dA ~ px dA~pgh dA =0 or P~Po~peh=0 or p=po+psh or P~Po=pah fof gauge pressure at point D is given by P=pgh or pressure head at point D, 2 =h. Pa [114 Fluid Mechanics ] From the above equation, it is clear that pressure head at any point in a liquid subjected to a ‘constant horizontal acceleration is equal to the height of the liquid column above that point, Therefore the pressure distribution in a liquid subjected to a constant horizontal acceleration is same as hydrostatic pressure distribution. The planes of constant pressure are therefore, parallel to the inclined surface as shown in Fig. 3.44, This figure also shows the variation of pressure on the rear and front ‘end of the tank. If hy = Depth of liquid at the rear end of the tank ‘hy = Depth of liquid at the front end of the tank. F = Total pressure force exerted by liquid on the rear side of the tank F,,= Total pressure force exerted by liquid on the front side of the tank, then Fy = (Area of triangle AML) x Width ps = (FX LM XAM xb) = 4X phy ity x6 2 and = (Area of triangle BNO) x Width fx iyxpahy b= beh See ee eee ae The vaio fF an ofan Wo obmia a (Retro Fig 88 (oh 4 x BN x NO) F, - hk pa XA, x Ta, where Ay = hy xb and fs = AL hod (hy xb) x tL = 4 pg. b Px gx xb)x = > ps pg XA,% fn, where Ay Px Ex (hy xb) x oh pg bichd Fig, 344(@) It can also be proved that the difference of these two forces (i.e., F, ~ F) is equal to the force required to accelerate the mass of the liquid contained in the tank i.e, F\-F,=Mxa where M = Total mass of the liquid contained in the tank lorizontal constant acceleration, Note : () Ifa tank completely filled with liguid and open at the top is subjected to a constant horizontal acceleration, then some of the liqhid will spill out from the tank and new free Surface with its slope given by equation tan =~ © will be developed z (i a tank party fled with liqud and open atthe top is subjected toa constant horizontal acceleration, spilling ofthe quid may take place depending upon ie nage ofthe acceleration. (i) tatnk completely filed with gud and closed atthe tp is subjected to a constant horizontal acceler tion, then the guid would not spill ou from the tank and also there wil be no adjustment in he surface clevation ofthe gui. But the equation tan 0 =~ * is applicable for this case as. z (iv) The example for a airplane during take off. k with liquid subjected to a constant horizontal acceleration, is a fuel tank on an [ Hydrostatic Forces on Surfaces 115) Problem 3.34 A rectangular tank is moving horizontally in the direction of its length with a constant acceleration of 2.4 m/s. The length, width and depth of the tank are 6 m, 2.5 m and 2m respectively. If the depth of water in the tank is I'm and tank is open atthe top then calculate : (i) the angle of the water surface to the horizontal, (i) the maximum and minimum pressure intensities at the bottom, (iii) the total force due to water acting on each end of the tank Solution. Given : Grttied Constant acceleration, Length = 6 m ; Wid Depth of water in tank, (i. The angle of the water surface to the horizontal Let @ = the angle of water surface to the horizontal Using equation (3.20), we get Original too = 24 ms?, ooo 2.5 mand depth = 2m. 2") Im | 2.24 onus Fig. 3.45 3 98l (the —ve sign shows that the free surface of water is sloping downward as shown in z tan @ = 0.2446 (slope downward) ‘ @= tan! 0.2446 = 13.7446" or 13° 44.6". Ans. (i The maximum and minimum pressure intensities at the bottom of the tank From the Fig. 3.45, Depth of water at the front end, ig. 3.45) hy = 13 tan @= 1 ~3 x 0.2446 = 0.2662 m Depth of water at the rear end, hy = 143 tan @ = 1 +3 x 0.2446 = 1.7338 m ‘The pressure intensity will be maximum at the bottom, where depth of water is maximum. Now the maximum pressure intensity at the bottom will be at point A and it is given by, xXx hy 1000 x 9.81 x 1.7338 N/m? = 1708.5 N/m?. Ans. ‘The minimum pressure intensity at the bottom will be at point B and it is given by Pain = PX 8X hy = 1000 x 9.81 x 0.2662 = 2611.4 Ném?. Ans. (iii) The total force due to water acting on each end of the tank Prax Let otal force acting on the front side (ie., on face BD) tal force acting on the tear side (ie., on face AC) Then where Ay = BD x width of tank = hy x2.5 = 0.2662 x 2.5 and 9266? = 0.1331 m = 1000 x 9.81 x (0.2662 x 2.5) x 0.1331 168.95 N. Ans. [116 Fluid Mechanics ] and F= pga, where Ay = AB x width of tank = hy x 2.5 = 1.7338 x 2.5 AB 1338 = AB oe 17338 «9.5669 m 2 2 1000 x 9.81 x (1.7338 x 2.5) x 0.8669 368618 N. Ans. Resultant force = F, ~ Fy 36861.8 N - 868.95 ¢ 35992.88 N Note. The difference of the forces acting on the two ends of the tank is equal to the force necessary to accelerate the liquid mass. This ean be proved as shown below : Consider the control volume of the liquid i., control vol- ume is ACDBA as shown in Fig. 346, The net force acting on the control volume in the horizontal direction must be equal 10 the produet of mass of the liquid in control volume and accel- eration ofthe liquid, Liquid (water) (F-F)=Mxa (p X volume of control volume) x a = (1000 x Area of ABDCE x width) x 2.4 x[ns (22280) canewis pas [area eaperiom ) xoxa.sxa4 (5 AC = hi = 1.7338 m, BD = hy = 0.2662 m, and AB = 6 m, width = 2.5m) The above force is nearly the same as the difference of the forces acting on the two ends of the tank. (i. 35902.88 = 36000), Problem 3.35 The rectangular tank of the above problem contains water to a depth of 1.5 m. Find the horizontal acceleration which may be imparted to the tank in the direction ofits length so that (i) the spilling of water from the tank is just on the verge of taking place, (ii) the front bottom corner of the tank is just exposed, ii) the bottom of the tank is exposed upto its mid-point. Also calculate the total forces exerted by the water on each end of the tank in each case. Also prove that the difference between these forces is equal to the force necessary to accelerate the mass of water tan. Solution. Given : Dimensions of the tank from previous problem, L=6 m, width (b) = 2.5 m and depth = 2 m [ Hydrostatic Forces on Surfaces 117) Depth of water in tank, = 1.5 m vnoname Horizontal acceleration imparted to the tank oe! surface) (0 (@) When the spiling of wate from the tank is nat just on the verge of taking place ama tle __ ‘When the spilling of water from the tank is just on the [14 verge of taking place, the water would rise upto the rear | top comer of the tank as shown in Fig. 3.47 (a) _AC_(2-13) “A073 os +n + 1667 3 tan @ Fig. 347 (a) Spilling of water is just on ie the verge of taking place. But from equation (3.20) tan @ = (Numerically) a= gX tan 0 = 9.81 X0.1667 = 1.635 mvs?. Ans, (b) Total forces exerted by water on each end of the tank ‘The force exerted by water on the end CE of the tank is F, =pgAyhi, where A, = CE x width of the tank = 2x 2.5 jee u2 i= Ge2- 22 1000 x 9.81 x (2X 2.5) x I = 49050 N. Ans. ‘The force exerted by water on the end FD of the tank is Im Fy As x Tin, where Ay = FD % width= 12.5 (se AC=BD=0.5m, 2. FD=BF-BD=1: FD_1 a7 0.5m = 1000x981 x(1x2.5)x0.5 he = 1262.5 N. Ans. (0) Difference of the forces is equal to the force necessary to accelerate the mass of water in the tank Difference of the forces = Fy - F, = 49050 ~ 12262.5 = 36787.5 N ‘Volume of water in the tank before acceleration is imparted to it = Lx b x depth of water, = 625% 15=22.5 m ‘The force necessary to accelerate the mass of water in the tank Mass of water in tank x Acceleration 1.635 mis?) = 1000 x 22.5 x 1.635 [There is no spilling of water and volume of water = 22.5 m*] = (px volume of water) x 1.635 = 36787.5 N [118 Fluid Mechanics ] Hence the difference between the forces on the two ends of the tank is equal to the force necessary to accelerate the mass of water in the tank. Vola faerie cad svotme = (EPP) er wh Retro Fig. 3.47 (a)] Froe suriaco | Original free = (222) } for this case is given by equation as fF eantt+ 33a Fig. 351 Po = gh (+4) z = 1000 «9.81 x0.5 (1+ 243) «613125 nin? 381 This pressure is represented by line BC. Now the force on the side AB = Area of triangle ABC x Width of tank = (40.5% 6131.25) x2 (BC = 3065.6 N. Ans. (O Force on the side of the tank, when tank is moving vertically downward. The pressure variation is shown in Fig. 3.52. For this case, the pressure at the bottom of the tank (ie, at point B) is given by equation (3.23) as 131.25 and b = 2m) Pe= Pah (: = 1000981 «0.5 3) ‘= 3678.75 Nim? ‘This pressure is represented by line BC. sé S New te foosontte ae AB Arete ABCA |. (Lx ABx BC) xb Fig. 3.52 = ($05 x367875) x2 (+ BC = 3678.75, b = 2) = 1839.37 N. Ans. [ Hydrostatic Forces on Surfaces 123) (id Force on the side of the tank, when tank is stationary. ‘The pressure at point B is given by, Po = pgh = 1000 x 9.81 x 0.5 = 4905 Nim? This pressure is represented by line BD in Fig. 3.52 Force on the side AB = Area of triangle ABD x Width = ($x ABx BD) xb = (£05 x 4905) x 2 BD = 4905) 2452.5 N. Ans. For this case, the force on AB can also be obtained as Fag = pach where A= AB x Width = 0.5 x2= 1m? AB 293 0.25 m= 1000 x 9.81 x 1x 0.25 2452.5 N. Ans. Problem 3.38 A tank contains water upto a depth of 1.5 m. The length and width of the tank are 4 m and 2 m respectively. The tank is moving up an inclined plane with a constant acceleration of 4 mis. The inclination of the plane with the horizontal is 30° as shown in Fig. 3.53. Find, (i) the angle made by the free surface of water with the horizontal. (ii) the pressure at the bottom of the tank at the front and rear ends Solution. Given : Depth of water, f= 1.5 m ; Length, Width, 6 = 2m Constant acceleration along the inclined plane, a=4ms? Amand Inclination of plane, = 30° Let @ = Angle made by the free surface of water after the acceleration is imparted to the tank Pa = Pressure at the bottom of the tank at the front end and pp = Pressure at the bottom of the tank at the rear nd, Fig. 3.53 This problem can be done by resolving the given acceleration along the horizontal direction and vertical direction. Then each of these cases may be separately analysed according to the set procedure. Horizontal and vertical components of the acceleration are : 400s 30° = 3.464 mis? asin «= 4 sin 30° = 2 m/s? ‘When the tank is stationary on the inclined plane, free surface of liquid will be along EF as shown in Fig. 3.53. But when the tank is moving upward along the inclined plane the free surface of liquid will be along BC. When the tank containing a liquid is moving up an inclined plane with a constant acceleration, the angle made by the free surface of the liquid with the horizontal is given by 124. Fluid Mechanics o @= tan"! 0.2933 = 16.346 or 16° 20.8’. Ans. Now let us first find the depth of liquid at the front and rear end of the tank. Depth of liquid at front end = h, = AB Depth of liquid at rear end = fy = CD From Fig. 3.53, in triangle COE, tan @ = g or CE = £0 tan 8 = 2x 0.2933 (BO = 2m, tan 0 = 0.2933) 5866 m :D + CE = 1.5 + 0.5866 = 2.0866 m AB = AF ~ BF 5 — 0.5866, (Ce AP = LS, BF = CE = 0.5866) Similarly hy = 0.9134 m ‘The pressure at the bottom of tank at the rear end is given by, 4 Po= Pah |1+—— @ 2 eso xsat aon 2 ero ont xa (16 2) ‘The pressure at the bottom of tank at the front end is given by 4 Pa= Pah (+4) 8 2 on 980914 (1+ 2.) «oer NA -24642.7 Nim?. Ans Hydrostatic Forces on Surfaces 125) ‘The centre of pressure for a plane vertical surface lies at a depth of two-third the height of the lorizontal force on curved surface and is equal to total pressure force on the projected area of the eurved surface on the vertical plane, eA and F, = Vertical force on sub-merged curved surface and is equal to the weight of liquid actually for imaginary supported by the curved surface. ‘The total force on a curved surface is given by where Fy ‘The inclination of the resultant force on curved surface with horizontal, tan 8 ‘The resultant force on a sluice gate, F = Fy ~ Fy where F; F,, = Pressure force on the downstream side of the sluice gate. messure force on the upstream side of the sluice gate and Fora lock pate, the reation between the two gates is equal tothe reaction atthe hinge, = P. Also the reaction between the two gates, P a Zain where F = Resultant water pressure on the lock gate “Fy and @= Inclination ofthe gate with the normal fo the side of the lock. EXERCISE (A) THEORETICAL PROBLEMS |. What do you understand by “Total Pressure’ and ‘Centre of Pressure’ ? Derive an expression for the force exerted on a sub-merged vertical plane surface by the static liquid and locate the position of centre of pressure. Prove that the centre of pressure of a completely sub-merged plane surface is always below the centre of gravity of the sub-merged surface or at most eoineide with the centre of gravity when the plane surface is horizontal Prove that the total pressure exerted by a stati liquid on an inclined plane sub-merged surface Is the same as the force exerted on a vertical plane surface as long as the depth of the centre of gravity of the surface is unaltered. Derive an expression for the depth of centre of pressure from free surface of liquid of an inclined plane surface sub-merged in the liquid. (@) How would you determine the horizontal and vertical components of the resultant pressure on a sub- merged curved surface ? (&) Explain the procedure of finding hydrostatic forces on curved surfaces, (Delhi University, Dec. 2002) Explain how you would find the resultant pressure on a curved surface immersed in a liquid. Why the resultant pressure on a curved sub-merged surface is determined by first finding horizontal and vertical forces on the curved surface ? Why is the same method not adopted for a plane inclined surface sub-merged in a liquid ? {126 Fluid Mechanics ] 9. 10, a 2 13. 16. Describe briefly with sketches the various methods used for measuring pressure exerted by fluids Prove that the vertical component of the resultant pressure on a sub-merged curved surface is equal to the weight of the liquid supported by the curved surface. What is the difference between sluice gate and lock gate ? Prove that the reaction between the gates of a lock is equal to the reaction atthe hinge Derive an expression for the reaction between the gates as P= sin inclination of the gate with normal to the side of the Tock. where F = Resultant water pressure on lock gate, |. When will centre of pressure and centre of gravity of an immersed plane surface coincide ? Find an expression forthe force exerted and centre of pressure fora completely sub-merged inclined plane surface. Can the same method be applied for finding the resultant foree on a curved surface immersed in the liguid ? IF not, why ? ‘What do you understand by the hydrostatic equation ? With the help of this equation derive the expressions for the (otal thrust on a sub-merged plane area and the buoyant force acting on a sub-merged body. (8) NUMERICAL PROBLEMS Determine the total pressure and depth of centre of pressure on a plane rectangular surface of 1 m wide and 3 m deep when its upper edge is horizontal and (a) coincides with water surface (b) 2_m below the free water surface. {[Ans.(a) 44145 N, 2.0m, () 103005 N,3.714 m] Determine the total pressure on a circular plate of diameter 1.5 m which is placed vertically in wat such @ way that centre of plate is 2 m below the free surface of water, Find the position of centre of pressure also [Ans, 4668.54 N, 2.07 m] A rectangular sluice gate is situated on the vertical wall of a lock. The vertical side of the sluice is 6 m in length and depth of centroid of area is 8 m below the water sueface. Prove thatthe depth of centre of pressure is given by 8.475 m, A circular opening, 3 m diameter, n a vertical sie of a tank is closed by a dise of 3 m diameter which can rotate about a horizontal diameter. Calculate : (i) the force on the dise, and (i) the toque required to ‘maintain the disc in equilibrium in the vertical position when the head of water above the horizontal diameter is 6 m. [Ans () 416.05 KN, (ii) 39005 Nm} ‘The pressure atthe centre of a pipe of diameter 3 m is 29.43 Nlem®. The pipe contains oil of sp. gr. 0.87 and is filled with a gate valve. Find the force exerted by the oil on the gate and position of centre of pressure. [Ans. 2.08 MN, .016 m below centre of pipe} Determine the total pressure and centre of pressure on an isosceles triangular plate of base $m and altitude 5 m when the plate is immersed vertically in an oil of sp. gr. 0.8. The base of the plate is 1 m below the free surface of water [Ans.261927N, 3.19 m] ‘The opening in a dam is 3 m wide and 2.m high. A vertical sluice gate is used to cover the opening. On the upstream of the gate, the liquid of sp. gr. 1.5, ies upto a height of 2.0 m above the top of the gate, whereas on the downstream side, the water is available upto a height of the top of the gate. Find the resultant force acting on the gate and position of centre of pressure. Assume that the gate is higher at the bottom. [Ams. 206010 N, 0.964 m above the hinge} Hydrostatic Forces on Surfaces 127) 8. A caisson for closing the entrance to a dry dock is of trapezoidal form 16 m wide atthe top and 12 m wide at the botiom and 8 m deep. Find the total pressure and centre of pressure on the ealsson if the ‘water on the outside is 1 m below the top level of the caisson and dock is empty. [Ans. 3.164 MN, 4.56 m below water surface] A sliding gate 2.m wide and 1.5 m high lies in a vertical plane and has a co-efficient of friction of 0.2 between itself and guides. Ifthe gate weighs one tonne, find the vertial force required to raise the gate if its upper edge is at a depth of 4 m from free surface of water. Ans. 37768.5 NI 10. A tank contains water upto height of 1 m above the base. An immiscible liquid of sp. gr. 0.8 is filled on the top of water upto 1.5 m height. Calculate: (i) total pressure on one side of the tank, (i) the position of centre of pressure for one side of the tank, which is 3 m wide. [Ans. 76518 N, 1.686 m from top] ‘A rectangular tank 4 m long, 1-5 m wide contains water upto a height of 2 m. Calculate the force due to ‘water pressure on the base of the tank. Find also the depth of centre of pressure from free surface. Ans. 117720 N, 2 m from free surface] 12, A rectangular plane surface 1 m wide and 3 m deep lies in water in such a way that its plane makes an angle ‘of 30° with the free surface of water, Determine the total pressure and position of centre of pressure when the upper edge of the plate is 2 m below the free water surface. [Ans, 80932.5 N, 2.318 m] A citcular plate 3.0 m diameter is immersed in water in such a way thatthe plane of the plate makes an angle of 60° with the fee surface of water. Determine the total pressure and positon of centre of pressure ‘when the upper edge of the plate is 2 m below the free water surface. [Ans. 228.69 kN, 3.427 m from free surface] 14, A rectangular gate 6 m x2 m is hinged at its base and inclined at 60° to the horizontal as shown in Fig. 3.54 To keep the gate in a stable position, a counter weight of 29430 N is atached atthe upper end of the gate. Find the depth of water at which the gate begins to fall. Neglect the weight ofthe gate and also friction at the hinge and pulley. Ans, 3.43 m] WATER SURFACE. 13. Fig. 3.54 Fig. 3.55 15. An inclined rectangular gate of width S m and depth 1.5 m is installed to control the discharge of water as shown in Fig. 3.55. The end A is hinged. Determine the force normal to the gate applied at B to open it. [Ans.97435.8N] 16. A gate supporting water is shown in Fig. 3.56. Find the height FREE WATER SURFACE, “h’ of the water so that the gate begins to lip about the hinge. Take the width of the gate as unity (Ans. 3% /3 m] 17. Find the total pressure and depth of centre of pressure on a Uwiangular plate of base 3 m and height 3 m which is immersed in ‘water in such a way that plane of the plate makes an angle of 60° Fig. 3.56 withthe free surface, The base of the plate is parallel to water surface and ata depth of 2 m from water surface Ans, 126.52 KN, 2.996] [128 Fluid Mechanics ] 18. Find the horizontal and vertical components of the total force acting on a curved surface AB, which is in the form of a quadrant of a circle of radius 2 m as shown in Fig, 3.57, Take the width ofthe gate 2m. (Ams. F,= 117.72 kN, F, = 140.114 KN] WATER SURFACE WAIER SURFACE Opie Fig. 3.57 Fig. 3.58 19. Fig. 3.58 shows a gate having a quadrant shape of radius of 3 m. Find the resultant force due to water per ‘metre length ofthe gate. Find also the angle at which the total force will act. (Ans. 82.201 KN, 0 = 57° 31'] 20. A roller gate is shown in Fig. 3.59, Iti cylindrical form of 6.0 m diameter. It i placed on the dam. Find the magnitude and direction of the resultant force due to water acting on the gate when the Water is just going to spill. The length of the gate is given 10 m, [Ans 2.245 MN, 6= 38° 8) ROLLER GATE. WATER SURFACE 60m pam Fig. 3.59 Fig. 3.60 21. Find the horizontal and vertical components of the water pressure exerted on a tainter gate of radius 102.44 N} 4m as shown in Fig. 3.60. Consider width of the gate unity. [Ans.F,=19.62KN, F, 22. Find the magnitude and direction of the resultant water pressure acting on a curved face of a dam which is shaped as shown in Fig. 361. The 7 is according to the relation y = height of water retained by the dam is 12 m. Take the width ‘of dam as unity. (Ans, 970.74 KN, @= 43° 1] 23. Bach gate of a Jock is $ m high and is supported by two hinges placed on the top and bottom of the gate. When the gates are closed, they make an angle of 120°. The width ofthe lock is 4m, I the depths of water on the two sides ofthe gates are 4 m and 3 m respectively, determine : (the magnitude of resultant pressure on each sate, and (jf) magnitude of the hinge reactions. {Ans.(°) 79.279 KN, i) Rp=27.924 KN, Ry=S1.385KN] 24. The end gates ABC of a lock are 8 m high and when closed make an angle of 120°, The width of lock 10 m. Each gate is supported by two hinges located at | m and 5 m above the bottom of the lock. The ‘depth of water on the upstream and dovinsieam sides ofthe lock are 6 m and 4 m respectively. Find (H, Resultant water force on each gate Hydrostatic Forces on Surfaces 129) (ii) Reaction between the gates AB and BC, and (iii) Force on each hinge, considering the reaction of the gate acting in the same horizontal plane as resultant water pressure. [Ans. 566.33 KN, (i) 566.33 kN, and (it) Ry= 173.64 kN, Ry = 392.69 kN] 2, A hollow circular plate of 2 m external and I m internal diameter is immersed vertically in water such that the centre of plate is 4 m deep from water surface. Find the total pressure and depth of centre of pressure. [Ans 92.508 KN, 4.078 m] 126. A rectangular opening 2.m wide and 1 m deep in the vertical side ofa tank is closed by a sluice gate of the same Size. The gate can turn about the horizontal cengoidal axis. Determine : () the total pressure on the sluice gate and (ip the torque on the sluice gate, The head of water above the upper edge of the gate is 1.5 m. [Ans (7 39.24 KN, (i) 1635 Nm} 27. Determine the total force and location of centre of pressure on one face Of FREE SURFACE OF LIQUID. the plate shown in Fig. 3.62 immersed in a liquid of specific gravity 0.9. + [Ans.62.4 KN, 3.04 m} 28. A circular opening, 3 m diameter in the vertical side of water tank is closed by a dise of 3 m diameter which can rotate about a horizontal diameter ? Calculate: (9 the foree on the disc, and (i) the torque required to maintain the dise in equilibrium in the vertical position when the head of water above the horizontal diameter is 4 m, [Ans (i) 270 KN, and (i) 38 KN m) 29. A penstock made up by a pipe of 2 m diameter contains a circular dist of same diameter {0 act as a valve which controls the discharge passing through it I can rotate about a horizontal diameter. Ifthe head of water above its centre is 20 m, find the total force acting on the dise and the Fig. 3.62 torque required to maintain it in the vertical position, 30. A circular drum 1.8 m diameter and 1.2 m height is submerged with its axis vertical and its upper end at ‘depth of 1.8 m below water level. Determine {(@ total pressure on top, botiom and curved surfaces of the drum, (Gi) resultant pressure on the whole surface, and iid) depth of centre of pressure on curved surface. 31. A circular plate of diameter 3 m is immersed in water in such a way tha its least and greatest depth from the free surface of water are 1_m and 3 m respectively. For the front side ofthe plat, find (i total force exerted by water and (ii) the position of centre of pressure Ams. (138684 N 5 (i) 2.125 m] 32. A tank contains water upto a height of 10 m, One of the sides of the tank is inclined. The angle between free surface of water and inclined side is 60°. The width of the tank is Sm. Find : () the force exerted by ‘water on inclined side and (i) position of centre of pressure. [Ans. (i) 283.1901 KN, (4) 6.67 m] 33. A circular plate of 3 m diameter is under water with its plane making an angle of 30° with the water surface. If the top edge of the plate is 1 m below the water surface, find the force on one side of the plate and its location. (LNT.U,, Hyderabad § 2002) 30" (Hint, d= 3 m, 8= 30°, height of op edge = 1 m, h = 1+ 1,5 xsin 30° =175 F = pgAh = 1000 x 9.81 x xo) ascaanasin EG! oso, 5. BO) "2 sas eoone1 750188) ae 8 cas BUOYANCY AND FROATATION > 4.1 INTRODUCTION In this chapter, the equilibrium of the floating and sub-merged bodies will be considered. Thus the chapter will include : 1. Buoyancy, 2. Centre of buoyancy, 3. Metacentre, 4. Metacentric height, 5. Analytical method for determining metacentric height, 6. Conditions of equilibrium of a floating and sub-merged body, and 7. Experimental method for metacentric height. » 4.2 BUOYANCY When a body is immersed in a fluid, an upward force is exerted by the fluid on the body. This upward force is equal to the weight of the fluid displaced by the body and is called the force of buoyancy or simply buoyancy. > 4.3 CENTRE OF BUOYANCY It is defined as the point, through which the force of buoyancy is supposed to act. As the force of buoyancy is a vertical force and is equal to the weight of the fluid displaced by the body, the centre of buoyancy will be the centre of gravity of the fluid displaced. Problem 4.1 Find the volume of the water displaced and position of centre of buoyancy for a wooden block of width 2.5 m and of depth 1.5 m, when it floats horizontally in water. The density of wooden block is 650 kg/m and its length 6.0 m. Solution. Given : width WATER Depth SURFACE, w Length a Ins. Volume of the block = 2.5 x 1.5 x6.0 = 22.50 m* hy Density of wood, p= 650 kg/m? . Weight of block = px @x Volume 25m 50 x 9.81 x 22.50 N = 143471 N Fig. 4 131 [132 Fluid Mechanics For equilibrium the weight of water displaced = Weight of wooden block = 143471 N Volume of water displaced _ Weight of water displaced __143471 © Weight density of water 1000x981 (Ce Weight density of water = 1000 x 9.81 Nim?) Position of Centre of Buoyancy. Volume of wooden block in water = Volume of water displaced = 14.625 m’. Ans. or 2.5 x hx 6.0 = 14.625 m°, where /h is depth of wooden block in water “ os ).975 m 33x60 Cente of Buoyancy = 2275 = 48 m rom bse Ans Problem 4.2 A wooden log of 0.6 m diameter and 5 m length is floating in river water. Find the depth of the wooden log in water when the sp. gravity of the log is 0.7. Solution. Given : of log Length, Sp. er. Density of log = 0.7 x 1000 = 700 kg/m* Weight density of log, w=pxg = 700 x 9.81 Nim? Find depth of immersion or h Weight of wooden log = Weight density x Volume of log = 700 x 9.81 x 5 (D)*xL = 100 x9.81 x © (6)° x5 N = 989.6981 N For equilibrium, Weight of wooden log = Weight of water displaced = Weight density of water x Volume of water displaced Volume of water displaced = 2895*98! _ 9.9396 m? 1000 9.81 ‘Weight density of water = 1000 x 9.81 N/m?) Let h is the depth of immersion Volume of log inside water = Area of ADCA x Length = Area of ADCA 5.0 But volume of log inside water = Volume of water displaced = 0.9896 m* [ Buoyancy and Floatation 133] 0.9896 = Area of ADCA x 5.0 s. Atea of ADCA = 9286 — 0.1979 m? 50 But area of ADCA = Area of curved surface ADCOA + Area of AAOC aa [= 38] 1 B = wr | Say | + 5 08 x 2r sin @ 5 e).> avin] +Pawsane >f,_2 : 0.1979 = n(3) + (3)? 00s sin @ (3 [ al 3 0.1979 = 2827 ~ 00157 8 + 0.9 cos @ sin 8 or 00157 6.09 cos 8 sin @= 2827 ~ .1979 = 0.0848 08 0848 “pors7 ©8988 = -o9rs7 or 6 = 57.32 00s @ sin 8 = 54.01 or @~ 57.32 cos @ sin 8 ~ 54.01 =0 For 8 60 ~ 57.32 x 0.5 x .866 ~ 54.01 = 60 ~ 24.81 ~ $4.01 = ~ 18.82 For @ 10 ~ 51.32 x 342 x 0.9396 - $4.01 = 70 - 18.4 ~ 54.01 =~ 2.41 For 8 = 72°, 72 ~ 57.32 x 309 x 951 - $4.01 = 72 - 16.84 - 54.01 = + 1.14 For 8= 71°, 71 ~ 51.32 x 325 x..9455 — 54.01 = 71 - 17.61 - 54.01 = - 0376 0= 715%, TLS ~ $7.32 x 3173 x 948 — 54.01 = 715 ~ 17.24 ~ 54.01 = + 248 Then r+ cos 715° = 0.3 +03 x 0.3173 = 0.395 m. Ans. Problem 4.3 A stone weighs 392.4 N in air and 196.2 N in water. Compute the volume of stone and its specific gravity. Solution. Given : Weight of stone in air 92.4 N Weight of stone in water = 196.2.N For equilibrium, ‘Weight in air ~ Weight of stone in water = Weight of water displaced or 392.4 ~ 196.2 = 196.2 = 1000 x 9.81 x Volume of water displaced Volume of water displaced 1962 Ng = 1 10% cm = 2.x 10 em’, Ans. 1000x981 50” ~ 50 = Volume of stone x10 em’, Ans. Volume of stone [134 Fluid Mechanics ] Specific Gravity of Stone Weight in air _ 3924 Mass of tone B24 ong ¢ 88i Density of tone Massinaie . ADO KE «49 x 59 = 2000 KE ‘Voume 1s m 50 Sp. gt. of stone Density of stone _ 2000 _ 3.0, ans. Density of water — 1000 Problem 4.4 A body of dimensions 1.5 m x 1.0 m x2 m, weighs 1962 N in water. Find its weight in air. What will be its specific gravity ? Solution. Given : Volume of body = 1.50 x 1.0 x 2.0 = 34 Weight of body in water 1962. N Volume of the water displaced = Volume of the body = 3.0 m? Weight of water displaced = 1000 x 9.81 x 3.0 = 29430 N For the equilibrium of the body Weight of body in air ~ Weight of water displaced = Weight in water Wa, — 29430 = 1962 19430 + 1962 = 31392 N ert Weight inait _ 31392 _ 5509 py g 981 Density of the body Mass_ _ 3200 _ 1066.67 Volume 3.0 1066.67 vity of the bod! .067. Ans. Sp. gravity of the body Too = 1067. Ans Problem 4.5 Find the density of a metallic body which floats at the interface of mercury of sp. gr. 13.6 and water such that 40% of its volume is sub-merged in mercury and 60% in water. Solution. Let the volume of the body = V m> ‘Then volume of body sub-merged in mercury 40 100, Volume of body sub-merged in water =04 Vm? 60. 3 = ye v=06Vm oo XV 08 For the equilibrium of the body Total buoyant force (upward force) = Weight of the body But total buoyant force = Force of buoyancy due to water + Force of buoyancy due to mercury Force of buoyancy due to water = Weight of water displaced by body Density of water x g x Volume of water displaced = 1000 x g x Volume of body in water [ Buoyancy and Floatation 135] = 1000 g x 0.6.x VN and Force of buoyancy due to mercury = Weight of mercury displaced by body x Density of mercury x Volume of mereury displaced 4X 13.6 x 1000 x Volume of body in mercury 8 13.6 x 1000 x0.4 VN ‘Weight of the body = Density x g x Volume of body = px gx V where pis the density of the body For equilibrium, we have Total buoyant force = Weight of the body 1000 x g x 0.6 x V+ 13.6 x 1000x ¢x 4V=pxgxV or p= 600 + 13600 x 4 = 600 + 54400 = 6040.00 kg/m? Density of the body = 6040.00 kg/m’. Ans. Problem 4.6 4 float valve regulates the flow of oil of sp. gr. 0.8 into a cistern. The spherical float is 15 em in diameter. AOB is a weightless link carrying the float at one end, and a valve at the other end which closes the pipe through which oil flows into the cistern. The link is mounted in a frictionless hinge at O and the angle AOB is 135°. The length of OA is 20.cm, and the distance between the centre of the float and the hinge is 50 cm. When the flow is stopped AO will be vertical. The valve is 10 be pressed on to the seat with a force of 9.81 N to completely stop the flow of oil into the cistern. It was observed that the flow of oil is stopped when the free surface of oil in the cistern is 35 cm below the hinge. Determine the weight of the float. Solution. Given = ort, Sp. gr. of ol sure, a, ~. Density of oil, oo on. Se ‘SURFACE Dia. of float, Force, Find the weight of the float. Let it is equal to W. ‘When the flow of oil is stopped, the centre of float is shown in Fig. 4.4 wom an00, singer = OB = OEHD , or = 35 = 35.355 ~ 35 = 0.355 cm = 00355 m. Wa ‘The weight of float is acting through B, but the upward buoyant force is acting through the centre of weight of oil displaced. Volume of oil displaced Zar snxne [136 Fluid Mechanics ] + x mx (.075)° + .00355 x m x (.075)? = 0.000945 m* :. Buoyant force = Weight of oil displaced 0 g x Volume of oil = 800 x 9.81 x .000945 = 7.416 N ‘The buoyant force and weight of the float passes through the same vertical line, passing through B. Let the weight of float is W. Then net vertical force on float = Buoyant force ~ Weight of float = (7.416 ~ W) ‘Taking moments about the hinge O, we get Px 20 = (7.416 ~ W) x BD = (7.416 ~ W) x 50 x c0s 45° or 9.81 x 20 = (7.416 — W) x 35.355, W=7416— aoeaet 7.416 ~ 5.55 = 1.866 N. Ans. 35355 > 4.4 META-CENTRE Itis defined as the point about which a body starts oscillating when the body is tilted by a small angle. The meta-centre may also be defined as the point at which the line of action of the force of buoyancy will meet the normal axis of the body when the body is given a small angular displacement. Consider a body floating in a liquid as shown in Fig. 4.5 (a). Let the body is in equilibrium and Gis the centre of gravity and B the centre of buoyancy. For equilibrium, both the points lie on the normal axis, which is vertical. NORMAL Axis ANGULAR fy DISPLACEMENT NORMAL AXIS @ ©) Fig. 4.5 Meta-centre Let the body is given a small angular displacement in the clockwise direction as shown in Fig. 4.5 (b). ‘The centre of buoyancy, which is the centre of gravity of the displaced liquid or centre of gravity of the portion of the body sub-merged in liquid, will now be shifted towards right from the normal axis. Let it is at B, as shown in Fig. 4.5 (b). The line of action of the force of buoyancy in this new position, will intersect the normal axis of the body at some point say M. This point M is called Meta-centre. > 4.5 META-CENTRIC HEIGHT ‘The distance MG, i.e., the distance between the meta-centre of a floating body and the centre of gravity of the body is called meta-centric height. [ Buoyancy and Floatation 137] >» 4.6 ANALYTICAL METHOD FOR META-CENTRE HEIGHT Fig. 4.6 (a) shows the position of a floating body in equilibrium. The location of centre of gravity and centre of buoyancy in this position is at G and B. The floating body is given a small angular displacement in the clockwise direction. This is shown in Fig. 4.6 (b). The new centre of buoyancy is at By. The vertical line through B, cuts the normal axis at M. Hence M is the meta-centre and GM is, meta-centric height. “ANGULAR ISPLACEMENT L | | (@ PUN OF B00" AT WATER UNE & Fig. 4.6 Metacenire beight of floating body. The angular displacement of the body in the clockwise direction causes the wedge-shaped prism. BOB’ on the right ofthe axis to go inside the water while the identical wedge-shaped prism represented by AOA’ emerges out of the water on the left of the axis. These wedges represent a gain in buoyant force on the right side and a corresponding loss of buoyant force on the left side. The gain is represented by a vertical force dF, acting through the C.G. of the prism BOB’ while the loss is represented by an equal and opposite force dF acting vertically downward through the centroid of AOA’. The couple due to these buoyant forces dy tends to rotate the ship in the counterclockwise direction. Also the moment caused by the displacement of the centre of buoyancy from B to By is also in the counterclockwise direction, Thus these two couples must be equal. Couple Due to Wedges. Consider towards the right of the axis a small strip of thickness dx at a distance x from 0 as shown in Fig. 4.5 (b). The height of strip xx ZBOB’ = xx 8. (> £BOB’ = ZAQA’ = BMB; ‘Area of strip = Height x Thickness = xx 0x de If Lis the length of the floating body, then Volume of strip = Arca x L = xx Ox Lx dr Weight of strip = pg x Volume = pgx OL dx Similarly, if a small stip of thickness dx at a distance x from Q towards the left of the axis is considered, the weight of strip will be pgx® L dx. The two weights are acting in the opposite direction and hence constitute a couple. [138 Fluid Mechanics ] Moment of this couple = Weight of each strip x Distance between these two weights pax OL dx [x +x] = pax OL de x 2x = 2pex? OL de ‘Moment of the couple for the whole wedge = J 2pgx? OL dx 4.) ‘Moment of couple due to shifting of centre of buoyancy from B 0 B, Fy x BB, = Fx BMx0 + BB, = BM x if 0 is very small} = Wx BMx® {iv Fp = W) 4.2) But these two couples are the same. Hence equating equations (4.1) and (4.2), we get Wx BM x 0 =| 2pgx” @ Lax Wx BM x 0 = 2p¢8 J e°Ldr Wx BM = 2p¢ [7Ldx Now Ldx = Elemental area on the wate line shown in Fig. 4.6 (c) and = dA Wx BM = 2p¢ [x44 But from Fig. 4.5 (c) its clear that 2 J x° da is the second moment of area of the plan of the body at water surface about the axis ¥-¥. Therefore Wx BM {whore 1= 2 J? dA} But W= Weight of the body = Weight of the fluid displaced by the body = pg x Volume of the fluid displaced by the body = pg x Volume of the body sub-merged in water ex am = PBT 1 (43) pax om= BM - 86 = - BG Meteonic tet = Ole “sy Problem 4.7 A rectangular pontoon is 5 m long, 3 m wide and 1.20 m high. The depth of immersion of the pontoon is 0.80 m in sea water. If the centre of gravity is 0.6 m above the bottom of the pontoon, determine the meta-centric height. The density for sea water = 1025 kg/m". Solution. Given : Dimension of pontoon Smx3mx 120m Depth of immersion 0.8 m [ Buoyancy and Floatation 139] Distance AG = 0.6 m 3m a} Distance AB = 4 x Depth of immersion 24 x8=04m Sitten Density for sea water = 1025 kg/m? Abe Meta-centre height GM, given by equation (4.4) is af ou= 1-86 | were 1 MO. neta ofthe pl ofthe pontoon aout axis = xsx3'mt= 5 mt ' 2 4 \7= Volume of he boy submerged in water 1 PuaT Waren sricE 0.2m Fig. 4.7 1 ope 8 x ch a22 48 020937502 = 07578 m. Ans 9°?" 35 Problem 4.8 A uniform body of size 3m long x2 m wide x I m deep floats in water. What is the weight of the body if depth of immersion is 0.8 m ? Determine the meta-centric height also. Solution. Given : 39m Dimension of body =3x2x1 Depth of immersion =08m Find (i)Weight of body, W _ (ii) Meta-centric height, GM (i). Weight of Body, W = Weight of water displaced 1g x Volume of water displaced = 1000 x 9.81 x Volume of body in water 1000 x 9.81 x 3x2 x 0.8 N = 47088 N. Ans. ELEVATION (ii) Meta-centric Height, GM ae Using equation (4.4), we get om = 586 where M.0.1 about ¥-¥ axis of the plan of the body 1 3. 3x2? ‘ A x3xes I 2a SEE = 20m Y= Volume of body in water =3x2x08=48m> Ba= AG ~AB=*9-°8 -o5_04=0. 0.1 = 0.4167 ~ 0.1 = 0.3167 m. Ans. [140 Fluid Mechanics ] Problem 4.9 A block of wood of specific gravity 0.7 floats in water. Determine the meta-centric height of the block if its size is 2 m XI m X08 m y Solution. Given : Let depth of immersion = hm i 20m Sp. eof wood =07 oe | Weight of wooden piece = Weight density of wood x Volume ir 07 x 1000 x9.81 x2 1x08N_ _ Weight of water displaced = Weight density of water =) ot Joeme x Volume of the wood sub-merged in water iis = 1000 x 9.81 x2x1xhN le 10+! For equilibrium, Fig. 49 Weight of wooden piece = Weight of water displaced 700 x 9.81 X2x 1x 0.8 = 1000X9.81x2x1Xh 700x 9.812% 1x08 1000 x9.81x2x1 Distance of centre of Buoyancy from bottom, ‘e., 0.7 x0.8 = 0.56 m and AG = 0.8/2.0 = 0.4 m o BG = AG ~ AB = 0.4 -0.28 = 0.12 m ‘The meta-centric height is given by equation (4.4) or -BG 1 q where 2 x 2x1. 12 y Y= Volume of wood in water =2K1xh=2x1x.56= 1.12 Gat = 2 FE = 0.12 = 0.1488 0.12 = 0.0288 m. Ans. Problem 4.10 A solid cylinder of diameter 4.0 m has a height of 3 metres. Find the meta-centric height of the cylinder when it is floating in water with its axis vertical. The sp. gr. of the cylinder = 06, Solution. Given : Dia. of cylinder, D=40m Height of cylinder, h=3.0m + Weight density of wood 1px #, where p = density of wood =0.7 x 1000 = 700 kg/m’, Hence w for wood = 700 x 9.81 Nim’. [ Buoyancy and Floatation 141] Sp. gr. of cylinder =06 bam Depth of immersion of cylinder = 0.6 3.0 = 1.8m pun and ¢. 8 15-09 =06 m Now the meta-centrie height Gi is given by equation (4.4) ie 1 om= 3G L2G But MOLL about Y-Y axis of the plan of the body = pt=% x (4.0)* At J xo and Y= Volume of cylinder in water z x Depth of immersion = FOP x 18m ® 40) = x (40) GM = 4 —___ 06 (4.0) x18 Ba ety A og- 4 _ 16" 18 18 ~ ve sign means that meta-centre, (M) is below the centre of gravity (G). Problem 4.11 A body has the cylindrical upper portion of 3 m diameter and 1.8 m deep. The lower portion is a curved one, which displaces a volume of 0.6 m' of water. The centre of buoyancy of the curved portion is at a distance of 1.95 m below the top of the cylinder. The centre of gravity of the whole body is 1.20 m below the top of the cylinder. The total displacement of water is 3.9 tonnes. Find the meta-centric height of the body. Solution. Given : Dia. of body =3.0m Depth of body 18m ‘Volume displaced by curved portion = 0.6 m’ of water. Let By is the centre of buoyancy of the curved surface and G is the centre of gravity of the whole body. 0. .55 ~ 0.6 = - 0.05 m. Ans. [142 Fluid Mechanics ] ‘Then CB, = 1.95 m 30m— C0= 120 m lie! Total weight of water displaced by body = 3.9 tonnes 9 x 1000 = 3900 kef 3900 x 9.81 N = 38259 N Find meta-centrie height of the body. Let the height of the body above the water surface x m, Total weight of water displaced by body = Weight density of water x [Volume of water displaced] = 1000 x 9.81 x [Volume of the body in water] = 9810 [Volume of cylindrical part in water + Volume of curved portion] ELEVATION Fig. 4.11 = 9810 [Exo x Depth of cyindical part in water + Volume displaced by curved portion or 38259 = 9810 [Ror xas-1) +06] ® ot xag—y +062 3829 7 OY x89 + 06= ‘ 2x3 x (18-1) =3.9- S Tx x (18-9 = 39-06 33x4 Rx3x3 * x= 18 ~ 4668 = 1.33 m Let By is the centre of buoyancy of cylindrical part and B is the centre of buoyancy of the whole body. ‘Then depth of cylindrical part in water = 1.8 ~ x= 0.467 m 467 cB, =x4 907 2 or 18-x 4668 1,33 + 2335 = 1.5635 m. ‘The distance of the centre of buoyancy of the whole body from the top of the cylindrical part is. given as CB = (Volume of curved portion x CB, + Volume of cylindrical part in water x CB,) + (Total volume of water displaced) = 06 X195 +33 15635 _ L17+5.159 (06 +33) Then BG = CB~ CG = 1.623 ~ 1.20 = 423 m. Meta-centric height, GM, is given by = 1.623 m. L Ma L_ GM = 5 - BG [ Buoyancy and Floatation 143] where = M.O.L. of the plan of the body at water surface about ¥-Y axis Byte Ex atmt == xpte 2 x3 “4 ot V = Volume of the body in water = 3.9 m* = ES 423 = 1,019 423 = 0.596 m. Ans. oa “39 > 4.7 CONDITIONS OF EQUILIBRIUM OF A FLOATING AND SUB-MERGED BODIES ‘A sub-merged or a floating body is said to be stable if it comes back to its original position after a slight disturbance. The relative position of the centre of gravity (G) and centre of buoyancy (B,) of a body determines the stability of a sub-merged body. 4.1.1 Stability of a Sub-merged Body. The position of centre of gravity and centre of buoy- ancy in case of a completely sub-merged body are fixed. Consider a balloon, which is completely sub- merged in air. Let the lower portion of the balloon contains heavier material, so that its centre of ‘gravity is lower than its centre of buoyancy as shown in Fig. 4.12 (a). Let the weight of the balloon is W. The weight Wis acting through G, vertically in the downward direction, while the buoyant force Fp is acting vertically up, through B. For the equilibrium of the balloon W= Fy. If the balloon is given an angular displacement in the clockwise direction as shown in Fig. 4.12 (a), then W and F, constitute a ‘couple acting in the anti-clockwise direction and brings the balloon in the original position. Thus the balloon in the position, shown by Fig. 4.12 (a) is in stable equilibrium. @ o) © STABLE EQUILIBRIUM © UNSTABLE EQUILIBRIUM NEUTRAL EQUILIBRIUM Fig. 4.12 Stabilities of sub-merged bodies. (@) Stable Equilibrium. When W = F, and point B is above G, the body is said to be in stable equilibrium. () Unstable Equilibrium. If W= F,, but the centre of buoyancy (B) is below centre of gravity (G), the body is in unstable equilibrium as shown in Fig. 4.12 (b). A slight displacement to the body, in the clockwise direction, gives the couple due to W and F, also in the clockwise direction. Thus the body does not return to its original position and hence the body is in unstable equilibrium. (c) Neutral Equilibrium. If F, = W and B and G are at the same point, as shown in Fig. 4.12 (c), the body is said to be in neutral equilibrium, 4.7.2. Stability of Floating Body. The stability of a floating body is determined from the posi tion of Meta-centre (M). In case of floating body, the weight of the body is equal to the weight of liquid displaced [144 Fluid Mechanics ] (@) Stable Equilibrium. If the point M is above G, the floating body will be in stable equilibrium as shown in Fig. 4.13 (a), Ifa slight angular displacement is given to the floating body in the clockwise direction, the centre of buoyancy shifts from B to B, such that the vertical fine through B, cuts at M. ‘Then the buoyant force F through B, and weight W through G constitute a couple acting in the anti clockwise direction and thus bringing the floating body in the original position. DisTUREING | =~ couPLe wy | lw er a Fo i b, w (ii) ' fe (@ Stabe equilibrium Mis above G (0) Unstable equim is below 6. Fig. 4.13 Stability of lating bodies. (&) Unstable Equilibrium. Ifthe point M is below G, the floating body will be in unstable equilib- rium as shown in Fig. 4.13 (6). The disturbing couple is acting in the clockwise direction. The couple due to buoyant force Fy and W is also acting in the clockwise direction and thus overturning the floating body. () Neutral Equilibrium. Ifthe point Ms atthe centre of gravity of the body, the floating body will be in neutral equilibrium. Problem 4.12 A solid cylinder of diameter 4.0 m has a height of 4.0 m. Find the meta-centric height of the cylinder ifthe specific gravity ofthe material of cylinder = 0.6 and itis floating in water with its axis vertical. State whether the equilibrium is stable or unstable. Solution. Given : D=4m rat Height, h=dm Sp. er 16 Depth of cylinder in water = Sp. gr. xh =06x40=2.4m pute Distance of centre of buoyancy (B) from A re ; S Distance of centre of aevity (@) fom A jsinam I ace Ao fs 202 Fig. 4.14 BG =AG -aB=24 Now the meta-centric height GM is given by Buoyancy and Floatation 145] where = M.OLL. of the plan of the body about ¥-Y axis 5 yt © p= ® x40 a OO Y= Volume of cylinder in water . x DP x Depth of cylinder in water = © x 4? x 2.4m? 40 oe, “a “ F gs tog ae pa 0.4167 m Vo Eyaxaa 1624 24 I GM = 5 ~ BG = 0.4167 ~ 0.8 = ~ 0.3833 m. Ans. ve sign means that the meta-centre (M) is below the centre of gravity (G). Thus the eylinder is in Lunstable equilibrium. Ans. Problem 4.13 A solid cylinder of 10.cm diameter and 40 cm long, consists of two parts made of different materials. The first part at the base is 1.0.¢m long and of specific gravity = 6.0. The other part of the cylinder is made of the material having specific gravity 0.6. State, if it can float vertically in water. Solution, Given: D=10em Length, L=40em Length of Ist part, 1,=1.00m Sp. Br 5,=60 Density of Ist part, py = 6 x 1000 = 6000 kg/m* Length of 2nd part, 1,=40~1.0=39.00m Sp. gr S,=06 i The cylinder will loat vertically in water if its metacentric height GMis 4.) 1 | positive, To find metacense Height ind the location of cente of gravity P" (G) and cenire of buoyancy (B) of the combined solid eylinder. The distance of the centre of gravity of the solid cylinder from A is given as AG = [(Weight of Ist part x Distance of C.G. of Ist part from A) + (Weight of 2nd part of cylinder x Distance of C.G. of 2nd part from A)] + [Weight of Ist part + weight of 2nd part] (J 0? x10x60x05)+(% 0" x390x06x(10%39/2) (Eo? x10x60+* b? x39x06] 4 4 x 6.0 x 0.5 + 39.0 x.6 x (20.5) 10x60 + 39.006 30+479.7 _ 482.7 60+234 ~ 294 642. Cancel $ D* inthe Numerator and Denominato [146 Fluid Mechanics ] To find the centre of buoyancy of the combined two parts or of the cylinder, determine the depth ‘of immersion of the cylinder. Let the depth of immersion of the cylinder is h. Then ‘Weight of the cylinder = Weight of water displaced 39.0, 10 Ray Rap Excaix Bo 981+ Ere 1 9. XCD? x Ty 600 x 9.81 + F (IP TO 6000 x98 Ry F (ay?x* x 1000 x9.81 FD? x ap * 1000 «9.8 [e hisin em) >, 1000 x 9.81 * 100 39.0 x06 +1.0x6.0=h of h=234+60= 294 The distance of the centre of the buoyancy B, of the eylinder from A is AB = nia = 24 2 x or cancelling © (.1 throughout, we get By OD nughout, we g¢ 47 a BG = AG ~ AB = 16.42 ~ 14.70 = 1.72 em. Meta-centric height GM is given by 1 om= 1 - BG v where J = M.O.L of plan of the body about ¥-Y axis ® x = pt= © (10% em* ga Py ly V = Volume of cylinder in water =Epx 4 E ao? . © (0)? x 29.4 m' 7 (lor x 1% aos SQ /F . GM = 0.212 1.72 = ~ 1.508 em As GM is ~ vo, it means that the Meta-centre M is below the centre of gravity (G). Thus the cylinder isin unstable equilibrium and so it cannot float vertically in water. Ans. Problem 4.14 A rectangular pontoon 10.0 m long, 7 m broad and 2.5 m deep weighs 686.7 KN. It carries on its upper deck an empty boiler of 5.0 m diameter weighing 588.6 kN. The centre of gravity ofthe boiler and the pontoon are at their respective centres along a vertical lin. Find the meta-centric height, Weight density of sea water is 10.104 kN/m’. 100 (10)? x 294 = xd = 0.212 Solution. Given : Dimension of pontoon = 10x 7 x 2.5 Weight of pontoon, Dia. of boiler, Weight of boiler, w for sea water = 10.104 kN/m* To find the meta-centric height, first determine the common cen- tue of gravity G and common centre of buoyancy B of the boiler and pontoon. Let G, and G, are the centre of gravities of pontoon and boiler respectively. Then [ Buoyancy and Floatation 147] Iv Ao225+ 822254252500 The distance of common centre of gravity G from A is given as W XAG, +0 x AC wee, (686.7 1.25 + 5886 x5.0 * (686.7 + 588.6) Let his the depth of immersion, Then AG= = 2.98 m, 1 100m Total weight of pontoon and boiler or (686.7 + 588.6) = w x Volume of the pontoon in water = 10.104 x L x b x Depth of immersion 1275.3 = 10.104 x 10x7 xh 12753 al 0x7 x 10104 ‘The distance of the common centre of buoyancy B from A is hh _ 1803 p= b= 1883 < 9915 m 2 2 y BG = AG - AB= Meta-contic heights given by GM = 1 — BG where [= M.O1L. of the plan of the body at the water level along ¥-Y 1 1Ox49x7 4 Fy 100 x P= SAT oy Y= Volume of the body in water bx b= 10.0 x 7 x 1.857 10x49x7 49. 2x10 71857 | 12 x 1857 I om=1_pG= 9.198 ~ 2.078 = 0.12 m, Meta-centric height of both the pontoon and boiler 0.12 m. Ans. wi e v ‘eight of sea water displaced aa Fig. 4.17 Plan of the body at waterline .98 9015 = 2.0785 m = 2.078 m = 2.198 m Problem 4.15 A wooden cylinder of sp. gr. = 0.6 and circular in cross-section is required to float in oil (sp. gr. = 0.90). Find the L/D ratio for the cylinder to float with its in oil, where L is the height of cylinder and D is its diameter. Solution. Given : Dia, of cylinder Height of cylinder Sp. gr. of cylinder, longitudinal axis vertical [148 Fluid Mechanics ] Let the depth of cylinder immersed in oil = ht u FDP XL x06 x 1000 x 9.81= © D*x hx 0.9 x 1000 x 9.81 8 ® 09 3 ° ae ‘The distance of centre of gravity G from A, AG The distance of centre of buoyancy B from A, ap=t=1 [21] 2° 213 B= AG-aB= © ‘The meta-centric height GM is given by ou=4-3c v where T= pt and V = Volume of cylinder in oil = = D? x h 64 4 1 _(® ps /E py 1D De 3D { 2 } go(Eo'/t on) = = 3h nel v & “a ) oh yz, RE 3 3 L Me 36 For stable equilibrium, GM should be +ve or GM>0 or 3D? Lk or a) RL” 6 LAB 9 or o= 9S = 0.494 m. 5 5220 And the distance of centre of gravity G, from A is AG = —> = 1.0m BG = AG AB= .0 = 494 = 506m, now meen ei GM sgnenby cit= 2-6 = xaytmt where ot . GM = .063 ~ .506 = ~ 0.443 m. Ans. AS the meta-centric height is ~ve, the point M lies below G and hence the cylinder will be in unstable equilibrium and hence eylinder will not float vertically. Il [150 Fluid Mechanics Part |. Let the force applied in a vertical chain attached atthe centre of the base of the buoy is T to keep the buoy vertical. Now find the combined positon of centre of gravity (G’) and centre of buoyancy (B’). For the combined W centre of buoyancy, lepth of immersion when the force Tis applied. Then Total downward fore let Weight of water displaced or (7848 + 7) = Density of water x g x Volume of cylinder in water = 1009812 0/4 [wh = eh Fo raiser 7848+ 101043x™ xD? 101043 x= x1 4 4 Ke 1 fasser) 784847 2 2( 79359 |” 158718 from A is The combined centre of gravity (G’) due to weight of eylinder and due to tension T in the chain 101043+7. 79359) AB’ IW1. of cylinder x Distance of C.G. of cylinder from A + Tx Distance of C.G. of T from A] + [Weight of cylinder + T] = (raex2.-7%0) «meen Gap = 1348 (784847) ‘The meta-centric height GM is given by GM = z -BG 7848 — em 7848+T (7848+ T) 158718 BG Exptek ot Fg ty, 184847) 4 7935.9 4 7848+T 79355 a 1, 79359 x (1848+) 16 © (7848+ 7) 479359 r93s9__[_7848__ (784847) T67e48+7) [TRB 7) STIS For stable equilibrium GM shouldbe positive Vv Gm>o or 79359) 7e48__(7848+7)] 4 16(7848+T) | (7848+T) 158718 [ Buoyancy and Floatation 151] 79359 7848 7848+ or 8859 __ SB >0 16(7848+T) (7848+7) 158718 79359 ~ 16x 7848 (7848+ 7) or eee ten eo 16(7848+T) "158718 =117632__, (7848 + T) or aN 0 16(7848+7) "158718 on (78487) | 117632 158718” 16(7848+7) or 734s + 7 > 47832 ise71.8 > 1166894735 > (10802.3)" ‘7848 + T > 10802.3, T > 108023 ~ 7848 > 29843 N. Ans. s+ ‘The force in the chain must be at least 2954.3 N so that the cylindrical buoy can be kept in vertical position. Ans. Problem 4.17 A solid cone floats in water with its apex downwards. Determine the least apex angle of cone for stable equilibrium. The specific gravity of the material of the cone is given 0.8. Solution. Given : Sp. gr of cone =08 Density of cone, p= 0.8 x 1000 = 800 kg/m* Let D=Dia. ofthe cone = Dia. of cone at water level anor 20 = Apex angle of cone Gone A H = Height of cone ~— ot h= Depth of cone in water -— a fF G= Centre of gravity of the cone B = Centre of buoyancy of the cone For the cone, the distance of centre of gravity from the apex A is AC= also AB= height of cone = 44 ‘Volume of water displaced Volume of cone Fig. 4.21 . Weight of cone = 800 x gx 4 RR? H Now from AAEF, — tan= SER FA" H R=Htand Similarly, r=htan@ [152 Fluid Mechanics Weight of cone = 800 gx 4 x mx (H tan @)?x H= 5 Weight of water displaced = 1000 x ¢ x $ x7? h 1000 x gx nx A? tan? 6 = 1000 x g x $ x m(h tan 0 x gx $x r(h tan 8) Ba For equilibrium Weight of cone = eight of water displaced or SOX gXRX A? tan? _ 1000 X98" x tan? 0 30 30 or 800 x HP = H _(1000)"* voit afi n ~ 800 For stable equilibrium, Meta-centric height GM should be positive. But GM is given by -BG where I= M.O.1. of cone at water-line =~ a 4 v= Volume of cone in water and BG = AG~ AB = }H- jh=3(H-h) s GM = 3 htan?@- 3(H- For stable equilibrium GM should be positive or fh tan? @- 3 (H-h)>0 or htan’@-(H-h)>0 or htan?®>(H-h) or htan’O+h>H or tan? @-+ 1] > or 1+tan?@>Hih or sect > t H _(1000)"" But H(2P) e107 h (a) sec? @ > 1.077 or cos? > 9285 1077 cos 8 > 0.9635 @> 15°30’ or 26>31° ‘Apex angle (28) should be at least 31°. Ans. 800 xe xxx H? tan? @ [ Buoyancy and Floatation 153] Problem 4.18 A cone of specific gravity S, is floating in water with its apex downwards. It has a Solution. Given : he gt Dia. of cone =D Height of cone =H cn Sp. gr. of cone = CONE AT Let G-= Centre of gravity of cone ee B = Centre of buoyancy 20 = Apex angle A= Apex of the cone he= Depth of immersion = Dia, of cone at water surface —p—+! ‘Then ‘Also weight of cone =Weight of water displaced. 1000 $x 4 nk? XH = 1000ex Lm? xh oF SRH= 7h But R= Han ,r=htan 0 (H tan 8)’ x H (irtan 0) SxH? xtan?@xH _ SH? a SH?) = S"° A ol) \G— AB h or h Distance, 3 a 3 4 Also .O. Inertia of the plan of body at water surface za of Y= Volume of cone in water Rp 2 Hs! xExa?x @ HS" 7 tas"? [154 Fluid Mechanics ] 1. at VoL ns" ar Now Met-entic height Gis given as ome tpg SH 99 yo" Tens a GM shouldbe 4 for stable eqiibium or GM > 0 or Bas) >0 16.4.5" 4 3d? 3H 113) re Ms Maas 6 16.H.S"? 4 c y cy ‘so we know R= H tan @ and r= han @ RLH_D rh od Dh oD a 2 x Hs! = ps"® H Substituting the value of d in equation (3), we get 3(ps'*) 3 18) Tons 4 o> a pig or ws o P< 4s) > 4.8 EXPERIMENTAL METHOD OF DETERMINATION OF META-CENTRIC HEIGHT ‘The meta-centrie height of a floating vessel can be determined, provided we know the centre of ‘gravity of the floating vessel. Let w is a known weight placed over the centre of the vessel as shown in Fig. 4.23 (a) and the vessel is floating, (a) Floating body (6) Tited body Fig.4.23 Metacentric height. [ Buoyancy and Floatation 155] Let W= Weight of vessel including w, G= Centre of gravity of the vessel B = Centre of buoyaney of the vessel ‘The weight w, is moved across the vessel towards right through a distance x as shown in Fig. 4.23 (b).. ‘The vessel will be tilted. The angle of heel @ is measured by means of a plumbline and a protractor attached on the vessel. The new centre of gravity of the vessel will shift to G, as the weight w, has been moved towards the right. Also the centre of buoyancy will change to B, as the vessel has tilted. Under equilibrium, the moment caused by the movement of the load w, through a distance must be equal to the moment caused by the shift of the centre of gravity from G to Gy. Thus ‘The moment due to change of GG, x W= Wx GM tan ‘The moment due to movement of Ww, =, 70x 10 = 75 x 7 X70 x 10" = 4375 and = Volume of ship in water = Weight of ship___ 19620 _ 1944.74 my? ‘Weight density of water ~ 10.104 f= M5 23053 m Vv ~ Toai74 GM = 2.253 ~ BG or 999 = 253 - BG BG = 2.253 ~ 999 = 1.254 m. From Fig. 4.25, it is clear that the distance of G from free surface of the water = distance of B from water surface ~ BG = 2.25 ~ 1.254 = 0.996 m. Ans. Problem 4.20 4 pontoon of 15696 kN displacement is floating in water. A weight of 245.25 KN is moved through a distance of 8 m across the deck of pontoon, which tilts the pontoon through an angle 4°, Find meta-centric height of the pontoon. Solution. Given : Weight of pontoon = Displacement or W = 15696 kN Movable weight, wy = 245.25 kN Distance moved by weight wy, x= 8m Angle of heel, o=4° The meta-centsic height, GM is given by equation (4.5) on Gara MS = 24525 KN x8 Wtan@~ 15696 kN tan 4° — 1,788 m. Ans. 15696 x 0.0699 > 4.9 OSCILLATION (ROLLING) OF A FLOATING BODY Consider a floating body, which is tilted through an angle by an overturning couple as shown in Fig, 4.26. Let the overturning couple is suddenly removed. The body will start oscillating. Thus, the Buoyancy and Floatation 157] body will be in a state of oscillation as if suspended at the meta-centre M. This is similar to the case of a pendulum. The only force acting on the body is due to the restoring couple due to the weight W of the body force of buoyancy Fy. Fig. 4.26 x Distance GA = Wx GM sin ® This couple tries to decrease the angle Restoring couple re Angular acceleration ofthe body, c= ~ >. iF Zo i) ve sign has been introduced as the restoring couple tries to decrease the angle @. Torque due to inertia a0 wton(-£9) Wa But 1 K ny" = Moment of Inertia about ¥-Y x Angular acceleration where W = Weight of body, K = Radius of gyration about ¥-Y Inertia torque Equating (i) and (i), we get ; wow sino= 2 £8 5 Peer eT Kd’ om x9=-<_ 29 e de 20 GMxgx® Dividing by ——, we get LP, OM *ExO _ a 8 G at * ‘K ) We ar @ 18 : i) ‘The above equation is a differential equation of second degree. The solution is . cms (GMgxt . 8 = C,sin | x14 Coos | Ai) [158 Fluid Mechanics ] where C, and C, are constants of integration, ‘The values of C, and C; are obtained from boundary conditions which are @ ar=0,0=0 i) aw =0 2 ‘where Tis the time period of one complete oscillation. Substituting the Ist boundary condition in (iii), we get 0=C,x040,x1 ©=0 Substituting 2nd boundary conditions in (iii), we get v0 2 But C, cannot be equal to zero and so the other alternative is (Mg T ~- sin @= 0, cos @= 1} 0= O=sin x sin x = 0} (4.6) ‘Time period of oscillation is given by equation (4.6). Problem 4.21. The least radius of gyration of a ship is 8 m and meta-centric height 70 cm, Caleu- late the time period of oscillation of the ship. Solution. Given Least radius of gyration, K = 8 m ‘Meta-centric height, GM = 70 em = 0.70 m ‘The time period of oscillation is given by equation (4.6). T= 2m |X oon | 88 _ - 19.18 sec. Ans. Gug 7 Yor x 981 Problem 4.22 The time period of rolling of a ship of weight 29430 kN in sea water is 10 seconds. The centre of buoyancy of the ship is 1.5 m below the centre of gravity. Find the radius of gyration of the ship if the moment of inertia of the ship at the water line about fore and aft axis is 1000 mi. Take specific weight of sea water as = 10100 Nin’. Solution. Given : Time period, T= 10 see Distance between centre of buoyancy and centre of gravity, BG = 1.5 m ‘Moment of Inertia, T= 10000 m* Weight, W= 20430 KN = 29430 x 1000 N Let the radius of gyration = First calculate the meta-centric height GM, which is given as GM = BM -aG= + -8G Vv Buoyancy and Floatation 159 where [= M.O. Inertia and = Volume of water displaced Weight of ship _ 29430 x 1000 2 = —Weiahtof ship___ 294301000 _ 291.6 ‘Sp. weight of sea water T0104 ™ 10000 = 20000 1.5 = 3433-15 = 1933 29126 ™ Using equation (4.6), we get T= 2n |X 8 ea 16), we Bs = Ge e or 10 2nk * Yi933x981 ~ Y1933 981 10x JI9339RT or = SN = 6.93 m, Ans. HIGHLIGHTS 1. The upward foree exerted by a liquid on a body when the body is immersed in the liquid Is known as buoyancy or force of buoyancy. 2. The point through which force af buoyancy is supposed to act is called centre of buoyancy. 3. The point about which a body starts oscillating when the body is tilted is known as meta-centre. 4. The distance between the meta-centre and centre of gravity is known as meta-centric height. 1 ng yo Be where = Moment of Inertia ofthe floating body (in plan) at water surface about the axis Y-Y V = Volume of the body sub-merged in water BG = Distance between centre of gravity and centre of buoyancy. ‘6. Conditions of equilibrium of a floating and sub-merged body are ‘5. The meta-centric height (GM) is given by Gi Equilibrium Floating Body ‘Sub-merged Body {0 Stable Equilibrium ‘Mis above G Bis above G (i) Unstable Equilibrium Mis below G Bis below G (ii) Neutral Equilibrium ‘Mand G coincide Band G coincide 7, The value of meta-centric height GM, experimentally is given as GM = —1*— a pe iy is gi ao x= Distance through which w, is moved W-= Weight of the ship oF floating body including w, (= Angle through the ship oF floating body is tilted due to the movement of w K '8. The time period of oscillation or rolling of a floating body is given by T= 2x where K= Radius of gyration, GM = Meta-centrie height Tr. me of one complete oscillation {160 Fluid Mechanics ] a. R ). Define the terms : meta-centre, centre of buoyane; EXERCISE (A) THEORETICAL PROBLEMS Define the terms ‘buoyancy’ and “centre of buoyancy’ Explain the ferms ‘meta-centre" and ‘meta-centrie height’ Derive an expression for the meta-centric height of a floating body. Show that he distance Between the meen and centre of buoyancy even by B= where = Moment of inertia of the plan of the floating body at water surface about longitudinal axis. YW = Volume of the body submerged in liquid. What are the conditions of equilibrium of a floating body and a sub-merged body ? How will you determine the meta-centric height of a floating body experimentally ? Explain with neat sketch. . Select the correct statement (@) The buoyant force fora floating body passes through the (8 centre of gravity of the body (Gi) centroid of volume of the body (iti) meta-centre of the body (iv) centre of gravity of the subsmerged part of the body (») centroid of the displaced volume. () A body sub-merged in liquid is in equilibrium when (its meta-centre is above the centre of gravity (Gi) its meta-centre is above the centre of buoyancy (i) its centre of gravity is above the centre of buoyancy iv) its centee of buoyancy is above the centre of gravity (©) none of these, Ans. 7 (a) (0). (6) (9) . Derive an expression forthe time period of the oscillation ofa floating body in terms of radius of gyration and meta-centric height of the floating body. rmeta-centric height, gauge pressure and absolute pressure, ‘What do you understand by the hydrostatic equation ? With the help of this equation, derive the expression for the buoyant force acting on a sub-merged body. With neat sketches, explain the conditions of equilibrium for floating and sub-merged bodies. Differentiate between (0 Dynamic viscosity and kinematic viscosity, (i) Absolute and gauge pressure (ii) Simple and ifferential manometers (i) Centre of gravity and centre of buoyancy, (Dethi University, Dec, 2002) (B) NUMERICAL PROBLEMS ‘A wooden block of width 2 m, depth 1.5 m and length 4 m floats horizontally in water. Find the volume of water displaced and position of centre of buoyancy. The specific gravity of the wooden block is 0.7. {Ans. 84 m?, 0.525 m from the base] Buoyancy and Floatation 161] 2. A wooden log of 0.8 m diameter and 6 m length is floating in river water. Find the depth of wooden log in water when the sp. gr. of the wooden log is 0.7. [Ans.0.54 m] 3. A stone weighs 490.5 N in air and 196.2 N in water. Determine the volume of stone and its specific gravity [Ans, 0.03 mi" of 3x 10* em’, 1.67] 4. A body of dimensions 2.0 m x 1.0 m x 3.0 m weighs 3924 N in water. Find its weight in air. What will be its specific gravity ? [Ans, 62784 N, 1.0667] 'S. A metallic body floats atthe interface of mercury of sp. gr. 13.6 and water in such a way that 30% ofits volume is sub-merged in mercury and 70% in water. Find the density of the metallic body. [Ans. 4780 kg/m’) 66. A body of dimensions 0.5 m x 0.5 m x 1.0 m and of sp. gr. 3.0 is immersed in water. Determine the least force required to lift the body. [Ans. 4905 N) 17. A rectangular pontoon is 4 m long, 3 m wide and 1.40 m high. The depth of immersion of the pontoon is 1.0m in sea-water. Ifthe centre of gravity is 0.70 m above the bottom ofthe pontoon, determine the meta- centric height. Take the density of sea-water as 1030 kg/m’. [Ans 0.45 m) 8. Anitorm body of size 4 m long x 2m wide x 1 m deep floats in water. What isthe weight of the body if depth of immersion is 0.6 m ? Determine the meta-centrc height also. [Ans, 47088 N, 0.355 m] 9. A block of wood of specific gravity 0.8 floats in water. Determine the mete-centri height ofthe block if fis size 3 mx 2.x Lm [Ans. 0.316 m] 10. A soli cylinder of diameter 3.0 m has a height of 2 m. Find the meta-centric height of the cylinder when itis loating in water with its ais vertical. The sp. gr of the eylinder is 0.7. [Ans. 0.1017 m) 11, A body has the cylindrical upper portion of 4 m diameter and 2 m deep. The lower portion is a curved one, Which displaces a volume of 0.9 m’ of water. The centre of buoyancy of the curved portion is ata distance of 2.10 m below the top of the eylinder. The centre of gravity of the whole body is 1.50 m below the top of the cylinder. The total displacement of water is 4.5 tonnes. Find the meta-centric height of the body. (Ans. 2,387 m} 12. A solid cylinder of diameter 5.0 m has & height of 5.0 m, Find the meta-centrie height of the cylinder if the specific gravity of the material of cylinder is 0.7 and itis floating in water with its axis vertical. State whether the equilibrium is stable or unstable, {Ans. ~ 0.304 m, Unstable Equilibrium) 13. A solid eylinder of 15 em diameter and 60 em long, consists of two parts made of different materials. The first part atthe base is 1.20 em long and of specific gravity = 5.0. The other parts ofthe cylinder is made of the material having specific gravity 0.6. Stat, if it can float vertically in water. [Ans. GM =~ 5.26, Unstable, Equilibrium] 14. A rectangular pontoon 8.0 m long, 7 m broad and 3.0 m deep weighs $88.6 KN. It carries on its upper deck tan empty boiler of 4.0 m diameter weighing 392.4 KN. The centee of gravity ofthe boiler and the pontoon are at their respective centres along a vertical line. Find the meta-centric height, Weight density of sea- water is 10104 Nim’ [Ans. 0.325 m] 15. A wooden cylinder of sp. gr 0.6 and circular in cross-section is required to float in oil (sp. gr. 0.8), Find the L/D ratio for the eylinder to float with its longitudinal axis vertical in oil where L is the height of cylinder and D is its diameter. [Ans. (L/D) <0.8164] 16. Show that a eylindrical buoy of 1.5 m diameter and 3 m long weighing 2.5 tonnes will not float vertically in sea-water of density 1030 kp/m’, Find the force necessary ina vertical chain attached at the centre ofthe base ofthe buoy that will keep it vertical [Ans. 10609.5 NI 17. A solid cone floats in water its apex downwards, Determine the least apex angle of cone for stable equilld- rium, The specific gravity of the material of the cone is given 0.7. Ans, 39° 71) 18. A ship 60 m long and 12 m broad has a displacement of 19620 KN. A weight of 294.3 KN is moved across the deck through a distance of 6.5 m. The ship is tilted through 5°. The moment of inertia of the ship at [162 vy. 20. Fluid Mechanics ] ‘waterline about is force and aft axis is 75% of moment of inertia the cirumseribng rectangle. The cent of buoyancy is 2.75 m below waterline, Find the meta-centic height and postion of cente of gravity of ship. Take specific weight of sea water = 10104 Nén’. (Ans. 1.1145 m, 0.53 m below water surface] ‘A pontoon of 1500 tonnes displacement is floating in water. A weight of 20 tonnes is moved through 1 distance of 6 m across the deck of pontoon, which tilts the pontoon through an angle of S°. Find rmeta-centric height of the pontoon. [Ans.0.9145 m] Find the time period of rolling of a solid circular eylinder of radius 2.5 m and 5.0 m long. The specific aravity of the cylinder is 0.9 and is floating in water with its axis vertical, [Ans. 0.35 sec] ‘A, KINEMATICS OF FLOW > 5.1 INTRODUCTION Kinematics is defined as that branch of science which deals with motion of particles without ‘considering the forces causing the motion. The velocity at any point in a flow field at any time is studied in this branch of fluid mechanics. Once the velocity is known, then the pressure distribution and hence forces acting on the fluid can be determined. In this chapter, the methods of determining. velocity and acceleration are discussed. > 5.2. METHODS OF DESCRIBING FLUID MOTION ‘The fluid motion is described by two methods. They are —(#) Lagrangian Method, and (ii) Eulerian Method. In the Lagrangian method, a single fluid particle is followed during its motion and its velocity, acceleration, density, etc., are described. In case of Eulerian method, the velocity, accelera- tion, pressure, density etc., are described at a point in flow field. The Eulerian method is commonly used in fluid mechanics. > 5.3 TYPES OF FLUID FLOW ‘The fluid flow is classified as : (@) Steady and unsteady flows ; (i) Uniform and non-uniform flows ; iii) Laminar and turbulent flows ; (iv) Compressible and incompressible flows ; (») Rotational and irrotational flows ; and (vi) One, two and three-dimensional flows. 5.3.1 Steady and Unsteady Flows. Stcady flow is defined as that type of flow in which the fluid characteristics like velocity, pressure, density, etc., at a point do not change with time. Thus for steady flow, mathematically, we have 163, 164 Fluid Mechanics av =0,(%) = () -0(%)_ 7° whore (32 ted pln fi Hl Unsteady flow is that type of flow, in which the velocity, pressure or density at a point changes with respect to time. Thus, mathematically, for unsteady flow v (Fo (2) soe 5.3.2. Uniform and Non-uniform Flows. Uniform flow is defined as that type of flow in ‘which the velocity at any given time does not change with respect to space (i.e., length of direction of the flow). Mathematically, for uniform flow S.-° where V= Change of velocity {as = Length of flow in the direction 5. Non-uniform flow is that type of flow in which the velocity at any given time changes with respect to space. Thus, mathematically, for non-uniform flow Sw 5.3.3. Laminar and Turbulent Flows. Laminar flow is defined as that type of flow in which the fluid particles move along well-defined paths or stream line and all the stream-lines are straight and parallel. Thus the particles move in laminas or layers gliding smoothly over the adjacent layer. This type of flow is also called stream-line flow or viscous flow. Turbulent flow is that type of flow in which the fluid particles move in a zig-cag way. Due to the ‘movement of fluid particles in a zig-zag way, the eddies formation takes place which are responsible for high energy loss. For a pipe flow, the type of flow is determined by a non-dimensional number called the Reynold number, where D = Diameter of pipe V = Mean velocity of flow in pipe and v= Kinematic viscosity of fluid. If the Reynold number is less than 2000, the flow is called laminar. If the Reynold number is more than 4000, it is called turbulent flow. If the Reynold number lies between 2000 and 4000, the flow may be laminar or turbulent. 5.3.4 Compressible and Incompressible Flows. Compressible flow is that type of flow in which the density of the fluid changes from point to point or in other words the density (p) is not ‘constant for the fluid. Thus, mathematically, for compressible flow p # Constant Incompressible flow is that type of flow in which the density is constant for the fluid flow. Liquids are generally incompressible while gases are compressible. Mathematically, for incompressible flow p= Constant. [ Kinematics of Flow and Ideal Flow 165 5.3.5 Rotational and Irrotational Flows. Rotational flow is that type of flow in which the fluid particles while flowing along stream-lines, also rotate about their own axis. And if the fluid particles while flowing along stream-lines, do not rotate about their own axis then that type of flow is called irrotational flow. 5.3.6 One-, Two- and Three-Dimensional Flows. One-dimensional flow is that type of flow in which the flow parameter such as velocity isa function of time and one space co-ordinate only, say x. For a steady one-dimensional flow, the velocity isa function of one-space-co-ordinate only. The variation of velocities in other two mutually perpendicular directions is assumed negligible, Hence ‘mathematically, for one-dimensional flow w= fs), v= and w=0 where u, v and w are velocity components in x, y and z directions respectively. ‘Two-dimensional flow is that type of flow in which the velocity is a function of time and two rectangular space co-ordinates say x and y. For a steady two-dimensional flow the velocity isa function of two space co-ordinates only. The variation of velocity in the third direction is negligible. Thus, mathematically for two-dimensional flow N= fC), ¥=fylasy) and w= 0. ‘Three-dimensional flow is that type of flow in which the velocity is a function of time and three ‘mutually perpendicular directions. But for a steady three-dimensional flow the fluid parameters are functions of three space co-ordinates (x, y and z) only. Thus, mathematically, for three-dimensional flow 0 YZ ¥ ffx. Yo 2) and w= fyl% Ys 2) > 5.4 RATE OF FLOW OR DISCHARGE (Q) Itis defined as the quantity of a fluid flowing per second through a section ofa pipe or a channel. For an incompressible fluid (or liquid) the rate of flow or discharge is expressed as the volume of fluid flowing across the section per second. For compressible fluids, the rate of flow is usually expressed as the weight of fluid flowing across the section. Thus () For liquids the units of Q are m’/s or litres/s (ii) For gases the units of Q is kgfis or Newton/s Consider aliquid flowing through a pipe in which A= Cross-seetional area of pipe V = Average velocity of fluid across the section Then discharge Q= AX. 6.1) > 5.5 CONTINUITY EQUATION ‘The equation based on the principle of conservation of mass is called continuity equation. Thus for a fluid flowing through the pipe at all the cross-section, the quantity of fluid per second is constant Consider two cross-sections of a pipe as shown in Fig. 5.1 Let_V; = Average velocity at cross-section 1-1 p, = Density at section 1-1 ‘Ay = Arca of pipe at section 1-1 166 Fluid Mechanics and V2, Py, Ay are corresponding values at section, 2-2, ‘Then rate of flow at section 1-1 = pyA\V, a i Rate of flow at section 2-2 = p,A,V> DIRECTION ‘According 10 law of conservation of mass oF FLOW Rate of flow at section 1-1 = Rate of flow at section 2-2 or PLAY: = PoAVe (52) Equation (5.2) is applicable to the compressible as well as incom- Fig. 51 Fluid flowing through pressible luids and is called Continuity Equation. Ifthe fluid is in- @ pipe. compressible, then p, = p and continuity equation (5.2) reduces to AW, = AV 63) Problem 5.1. The diameters ofa pipe atthe sections I and 2 are 10.cm and 5 em respectively. Find the discharge through the pipe if the velocity of water flowing through the pipe at section 1 is 5 m/s. Determine also the velocity at section 2. Solution. Given : Atsection 1, D,=10em=0.1 m Rope ay Aya DP) =F mis. At section 2, Sem = 0.15 m Ys Smee (15)? = 0.01767 m? Fig. 5.2 (® Discharge through pipe is given by equation (5.1) or O=A,xV, = 0.007854 x 5 = 0.03927 m’ss. Ans. Using equation (5.3), we have AV, = 4,V; AY, _ 0.007854 ‘A, 0.001767 Problem 5.2 A 30 cm diameter pipe, conveying water, branches into two pipes of diameters 20 cm and 15 cm respectively. If the average velocity in the 30 cm diameter pipe is 2.5 mls, find the discharge in this pipe. Also determine the velocity in 15 cm pipe if the average velocity in 20 cm diameter pipe is 2 m/s. Solution, Given : (i) V X 5.0 = 2.22 m/s. Ans. Oe 17 ore aatamieg y= 30cm ® a i Fig. 5.3 Kinematics of Flow and Ideal Flow 167 D, = 30m = 0.30 m Ex se im? 5 x 3= 0.07068 5m = 0.15 m (sy =F x 0.225 = 0.01767 m? Find (i) Discharge in pipe 1 or Q, (i, Velocity in pipe of dia, 15 em or Vs Let Qj, Qs and Q, are discharges in pipe 1, 2 and 3 respectively Then according to continuity equation = 02+ Os a (i The discharge Q, in pipe 1 is given by Q, = AiV; = 0.07068 x 2.5 mvs = 0.1767 m'ss. Ans. (i Value of V5 = AV) = 0.0314 x 2.0 = 0.0628 m/s Substituting the values of Q, and Q, in equation (1) 0.1767 = 0.0628 + 0, z Q = 0.1767 ~ 0.0628 = 0.1139 m*/s But Qs = Ay X Vy = 0.01767 x Vy or 0.1139 = 0.01767 x V5 0.1139 : 6.44 mis. Ans. 0.01767 Problem 5.3 Water flows through a pipe AB 1.2 m diameter at 3 m/s and then passes through a pipe BC 1.5 m diameter. At C, the pipe branches. Branch CD is 0.8 m in diameter and carries one- third of the flow in AB. The flow velocity in branch CE is 2.5 m/s. Find the volume rate of flow in AB, the velocity in BC, the velocity in CD and the diameter of CE. Solution. Given Diameter of pipe AB, Du Velocity of flow through AB, Vi Dia. of pipe BC, Dac Dia. of branched pipe CD, Dep Velocity of flow in pipe CE, Veg Let the flow rate in pipe AB Velocity of flow in pipe BC. Velocity of flow in pipe o 168 Flui ‘Mechanics Diameter of pipe Then flow rate through and flow rate through (® Now volume flow rate through AB = Q = Vay x Area of AB =3.0x i (Dyg)? = 3.0 x i (1.2)? = 3.393 m/s. Ans. (ii) Applying continuity equation to pipe AB and pipe BC, Van Atea of pipe AB = Vac x Area of pipe BC Rip? X ,) 3.0 x2 Dyp)? = Voc X= (pc) or 7 Dany x5 Dror : AO." Vex 57 [ovis 2] or 1.92 mis. Ans. we TS (iii) The flow rate through pipe c= 9,2 2 = 383-131 ms 3°03 21= Ven x Area of pipe CD x (Dep)? or 1.131 = Vey og x 0.8? = 0.5026 Ven Veo = M3 = 2.25 mvs. Ans. 0.5026 (iv) Flow rate through CE, Q,= Q~ Q, = 3.393 ~ 1.131 = 2.262 mts 0s = Veg Aro pipe CE = Veg™ (De) a 2a =25 x2 x De? -VLIS2 = 1.0735 m Kinematics of Flow and Ideal Flow 169 Problem 5.4 4 25 cm diameter pipe carries oil of sp. gr. 0.9 at a velocity of 3 m/s. At another section the diameter is 20 cm. Find the velocity at this section and also mass rate of flow of oil. Solution. Given at section 1, at section 2, Mass rate of flow of oil Applying continuity equation at sections 1 and 2, AW = AV or 0.049 x 3.0 = 0.0314 x Vy V, = 20830 4.68 mus. Ans. 0.0314 Mass rate of flow of oil = Mass density x Q = p x A, x V, Density of oil Sp. ar. of oil eens of cl aC Density of water Density of o sp. gf of oll x Density of water = 0.9 x 1000 kg/m? Ns Mass rate of flow = 900 x 0.049 x 3.0 kis = 132.23 kg/s. Ans. Problem 5.5 A jet of water from a 25 mm diameter nozzle is directed vertically upwards. Assuming that the jet remains circular and neglecting any loss of energy, that will be the diameter at a point 4.5 m above the nozzle, ifthe velocity with which the jet leaves the nozzle is 12 m/s. Solution. Dia, of nozzle, Velocity of jet at nozzle, Height of point A, ero | [asm Let the velocity of the jet at a height 4.5 m= V» ware Consider the vertical motion ofthe et from the out ofthe | | nozzle to the point A (neglecting any loss of energy). D1A=25 mn Initial velocity, u=V,=12m/s ae Final velocity, Value of Using, 170 ‘Fluid Mechanics 46 mis V, =Yi2? -2x981x45 = J144— 8829 Now applying continuity equation to the outlet of nozzle and at point A, we get A\V, = AV> or AM = 2% (0.025) X12 _ 9.990796 4x 746 Let Dy = Diameter of jet at point A. Then Ay =F D," oF 0,0007896 =F x D,* (0007896 x4 © > 5.6 CONTINUITY EQUATION IN THREE-DIMENSIONS D; 0.0317 m = 1.7 mm. Ans. Consider a fluid element of lengths dr, dy and dz in the direction of x,y and z. Let u, v and w are the inlet velocity components in x, y and z directions respectively. Mass of fluid entering the face ABCD per second X Velocity in x-direction x Area of ABCD x wx (dy x de) Then miso al ing tie ZFH pe cot ut 2p Gain of mass in x-direction = Mass through ABCD ~ Mass through EFGH per second pw apc pudte- 2 pint a Sp (PH doe) de 2 podett (desma Similarly, the net gain of mass in y-direction a =- 2 (pn drdyde ay (OP a a dow) drayas a PME and in z-direction = Net gain of masses = Since the mass is neither created nor destroyed in the fluid element, the net increase of mass per unit time in the fluid element must be equal to the rate of increase of mass of fluid in the element, But mass Kinematics of Flow and Ideal Flow 171 ap Fedde Equating the two expressions, a a a ap or [ae 0+ G5 (Ov)+ 5-(0w)| dee = SP. dee 58 (ou) +? (pr) +2 (ow ancelling dx.dy.dz from both si or +5 ON) +5, (09) 45-0 = 0 [Cancelling dx.dy.dz from both sides) ...(5.34) ar Equation (5.34) is the continuity equation in cartesian co-ordinates in its most general form. This equation is applicable to = (@) Steady and unsteady flow, (ii) Uniform and non-uniform flow, and (ii) Compressible and incompressible fluids. ap For steady flow, 2? or and hence equation (5.34) becomes as a a a By 4) #35 (PP) + 5-(w) =0 (5.3B) If the fluid is incompressible, then p is constant and the above equation becomes as au, ox dy az Equation (5.4) is the continuity equation in three-dimensions, For a two-dimensional flow, the com- ae ax dy 5.6.1 Continuity Equation in Cylindrical Polar Co-ordinates. The continuity equation in Solna provers (etr hz svetinie) in devo ty prose snes Clon, Considers two dimeaional innpene low field The ‘two-dimensional polar co-ordinates are r and 8. Consider a fluid wy eae, y+ Bear element ABCD between the radii r and r + dr as shown in Cy . Fie 3.7 Theale abendooy tesla che soa ea TH conponcns of te slot Varin the ea eon and apne ange ecton The ie of he lene st faving th ape SiEAB = 8, B= dr, DC=( +47) d8, AD =r i pair same Gia cs ea om tear see Maso i atg tte AB por ani me p» Velo in rection x Area (5A) (5.5) 172. Fluid Mechanics Xu, X (AB x 1) ‘» Area = AB x Thickness = rd@ x 1) Xu, x (rd0X 1) =p + up. rd Mass of fluid leaving the face CD per unit time = px Velocity x Area = x(a) «(ed (Area = €D x1) : (a, + &. a) x (r+ dryd0 + CD = (r+ dr) d8] =p afi xr tude Se tre (ae? | a0 =o xrtu, xcdre rte ar] 40 [The term containing (dr? is very small and has been neglected] Gain of mass in r-ieetion per unit time = (Mass through AB ~ Mass through CD) per unit time = pou, 0 ole upd re | «0 pu, rd0- pu, 1 older ar] 6 2 aa " [This is written in this form because 2%] rodrod9 (7. d8. dr. 1) is equal to volume of or element] o marr Now consider the flow in @-direction Gain in mass in @-direction per unit time = (Mass through BC ~ Mass through AD) per unit time = [p x Velocity through BC x Area — p x Velocity through AD x Area] 4) arx1| + lp tro (4 + Are irx 1) [Multiplying and dividing by 7] ‘Total gain in fluid mass per u ole (3.54) Kinematics of Flow and Ideal Flow 173 But mass of fluid element = px Volume of fluid element px (rd0 x dr x1] =p xrd8.dr Rate of increase of fluid mass in the element with time a = 5, P rd8-an ( rdQ dr. Lis the volume of element and is a constant quantity) ‘Since the mass is neither created nor destroyed in the fluid element, hence net gain of mass per unit time in the fluid element must be equal to the rate of increase of mass of fluid in the element. Hence equating the wo expressions given by equations (5.5 A) and (5.5 B), we get ap SP. nto ar (5.5B) or {Cancelling rdr . d9 from both sides} or By 5.50) ale ar 690 Equation (5.5 C) is the continuity equation in polar co-ordinates for two-dimensional flow. For steady flow 2°. = 0 and hence equation (5.5 C) reduces to or a a au, or Ze +d w= o [: 2em) Bo4,] Equation (5.5 D) represents the continuity equation in polar co-ordinates for two-dimensional steady incompressible flow. Problem 5.5A Examine whether the following velocity components represent a physically possible flow ? 4, = 1 sin 8, y= 2r cos @. Solution. Given : u, = rsin @ and up = 2r cos @ For verily pase flow, the continuity equation, 2 (ru) 2 9 (to) = 0 should be satisfied. Now u,=rsin® Multiplying the above equation by r, we get ru, =P sin 174 Fluid Mechanics Differentiating the preceding equation w.r.t.r, we get a ao y (ru) 3 (F sin 8) =2rsin 8 (2 sin 0s constant watt.) Now 1g = 2r cos @ Differentiating the above equation wart. 8, we get a a 3g (te) = 5g Or eos 8) = 2r(- sin @) (eo 2r is constant w.r.t. @) 2rsin 8 a a 2 (ru) +2 (uy) = 2r sin 8 2r sin 3p ltr) 3g ) = 2r sin 8 2rsin = 0 Hence the continuity equation is satisfied, Hence the given velocity components represent a physi- cally possible flow. » 5.7 VELOCITY AND ACCELERATION Let Vis the resultant velocity at any point in a fluid flow. Let u, v and w are its component in x, y and z directions. The velocity components are functions of space-co-ordinates and time. Mathematically, the velocity components are given as, WE ACY. 2D and Resultant velocity, Let a,, a, and a, are the total acceleration in x, y and z directions respectively. Then by the chain rule of differentiation, we have dtu _u dx , au dy, Ou dz, Ou at ar But au ar Similarly, = (5.6) ae ay” a a a Kinematics of Flow and Ideal Flow 175 or ar gy By OH a, (5.1) a oe ay” a Acceleration vector (5.8) 5.7.1, Local Acceleration and Convective Accéleration. Local acceleration is defined as the rate of increase of vaey ‘with respect to time at a given point in a flow field. In the equation given as convective acceleration. Problem 5.6 The velocity vector in a fluid flow is given V = 4x'i— 10s¢yj + 2th Find the velocity and acceleration of a fluid particle at (2, 1, 3) at time t= 1. Solution. The velocity components u, v and w are w= 43°, v= ~ 10x? y, w = 2¢ For the point 2, 1, 3), we have x= 2, y= and z= 3at time t= 1. Hence velocity components at (2, 1, 3) are 4x 2)* = 32 units = 10(2)°(1) = ~ 40 units 2 units Velocity vector V at 2, 1, 3) = 321 40j + 2k s 24 or Resultant velocity = fu? +97 +? = (32? + (- 40)" + 2? = (1024+ 160044 = 51.26 units. Ans. Acceleration is given by equation (5.6) Qu au Ou, au Ret at” ta ay OF 176 Fluid Mechanics Now from velocity components, we have au _ 52, ou Mee, ar ay av oe aw ar Substituting the values, the acceleration components at (2, 1, 3) at time ¢= 1 are bo (2x2) + (— 10x"y) (0) + 2x) +0 H8x° = 48 x (2)° = 48 x 32 = 1536 units bx (— 20xy) + (= 10x°y) (— 10x”) + 2 (0) +0 = 80x4y + 100x4y = 80 (2)* (1) + 100 (2)* x 1 =~ 1280 + 1600 = 320 units. 2° (0) + C 10x*y) (0) + (2) 0) + 2.1 = 2.0 units Acceleration is Azayi+ aj + a= 15361 + 320) + 2k. Ans. or Resultant A =y(1536)° + (320) +(2)° units = 25592964 1024004 = 1568.9 units. Ans. Problem 5.7 The following cases represent the two velocity components, determine the third com- ponent of velocity such that they satisfy the continuity equation @) u=¥ey ee nye +ay (ii) v = 2y', w = aye Solution, "The continuity equation for incompressible fluid is given by equation (5.4) as, ou FS av, ow ax dy oz Case I. wereye? ayy tay Suisiting the values of and = in continuity equation. eM et dry Pere + Day rhe aw, = 3x- Day + 2 az a Kinematics of Flow and Ideal Flow 177 Integration of both sides gives [d= (3x - 2ay + 22) de (3 where constant of integration cannot be a function of z. But it can be a function of x and y that is f(x,y). ( un 2xyn+ z] +10, y). Ans, 2 av Case aye, » ay ow w= aye, ieee Substituting the values of 2 and 2 in continuity equation, we get ay a © ayer =0 ea o 4y ~ 2ay or du = 4y~ 2x9) dr Integrating, we get w= day 29> + FO.) = dy =3°y + flyy 2) Ams, Problem 5.8 4 fluid flow field is given by Vexyit yy Qye+yc)k Prove that it is a case of possible steady incompressible fluid flow. Calculate the velocity and accel- eration at the point (2, 1, 3). Solution. For the given fluid flow field « = y e 2ay ay & oo 3 = Day = yz Daye - ye? For a case of possible steady incompressible fluid flow, the continuity equation (5.4) should be satisfied. oo ou 9 Ow ~ ay a au av aw Substituting the vatues of 2, 2 ana 2, we get * ie” ay ae NB y+ Dye — Day 2ye = 0 178 Fluid Mechanics Hence the velocity field V= xvi + y*yj - Qayz + yz") kis a possible case of fluid flow. Ans. Velocity at (2, 1, 3) Substituting the values 1 and z= 3 in velocity field, we get yi + yf ~ Qaye + 92") Bx lit Px 3j-(2x2x1K341KIVK 143) 21k. Ans. and Resultant velocity = 4? +3? + C21) = J16+9 + 441 = 466 = 21.587 units. Ans. Acceleration at (2,1, 3) The acceleration components a, a, and a, for steady flow are = Dey - ye, Substituting these values in acceleration components, we get acceleration at (2, 1, 3) a= xy Qay) + y°e ( - Qayz +92") (0) 28 yay: =2QPP4PxPx3=2x8412 16 + 12.= 28 units ay = xy (0) + y"¢ Qyz)- Qrye +32) ") 292 — dry'e— ye xx 2x2 1x3 19x3? = 18- 12-9 =~3 units Py (— Dye) + ye © Bae ~ 24) - aye + ye") 2ry - 292) = eye — VP-P 4 by'y%e + Wy*? + dy? + 297] =-2x2x 1x 3-2x2xPxF- 1x3 FLAP x PB 422% 1x4 dD PT DH PD — 24 - 36 - 27 + [48 + 36 + 72 + 54] =~ 24-36-27 448 + 36+ 72 + 54 = 123 Acceleration ai + a, + a,k = 281-3) + 123k, Ans. Kinematics of Flow and Ideal Flow 179 or Resultant acceleration = 28? + (3) +123* = J784+9 + 15129 = 15022 = 126.18 units. Ans. Problem 5.9. Find the convective acceleration at the middle of a pipe which converges uniformly Srom 04 m diameter to 0.2 m diameter over 2 m length, The rate of flow is 20 lis. Ifthe rate of flow changes uniformly from 20 Us to 40 Ws in 30 seconds, find the total acceleration at the middle of the Pipe at 15th second. Solution. Given : Diameter at section 1, D, = 0.4m; Dy= 0.2 m, L= 2m, Q = 20 Ils = 0.02 m'/s as one litre 001m = 1000 em? Find (i Convective acceleration at middle ée., at A when Q= 20 lis. (ii) Total acceleration at A when Q changes from 20 U/s to 40 l/s in 30 seconds. Case I. In this case, the rate of flow is constant and equal to 0.02 m°/s. The velocity of flow is in «direction only. Hence this is one-dimensional flow and velocity components in y and z directions are zero or v= 0,2=0. au Convective aocleration = 15" only wi) Let us find the value of «and 2 a distance «from inet ex ‘The diameter (D,) ata distance x from inlet of at section X-X is given by, T rs 4 tm p,=04-" ral = (04-019 m L Leg ‘The area oferosssection (A,) at section X-X is given by, ered ton X's ra> Avs "p22" 4-019? 5 n= ED? = Ed " Fig. 58 Velocity (u) at the section X-X in terms of Q (ie in terms of rate of flow) nowt oe Area” A, i Gee = 1.273. (04-01 97? mi TE ee 1273.0 (04 - 04127? mis (i) au To find 5, we must differentiate equation (i) with respect t0 x au a . 2 namg@s-o1) 1.273 0-2) (04-01. x 0.1) [Here Q is constant] .2546 0 (0.4 - 0.1 x)! (iti) Sut Convective acceleration = [1.273 Q (0.4 ~ 0.1 2) 7] x (0.2546 Q (0.4 - 0.1 x7"] .273 x 0.2546 x Q? x (0.4 ~ 0.1 x) 180 Fluid Mechanics = 1.273 x 0.2546 x (0.02)? x 0.4 - 0.1.9 [+ Q=0.02 ms} -Convective acceleration at the middle (where x = I'm) = 1.273 x 0.2546 x (0.02)° x (0.4 = 0.1 x 1) mis? 1.273 x 0.2546 x (0.02)? x (0.3) mis? .0048 m/s. Ans. Case IL. When Q changes from 0.02 m°/s to 0.04 m°Vs in 30 seconds, find the total acceleration at x= I mand f= 15 seconds. ‘Total acceleration = Convective acceleration + Local acceleration at ¢= 15 seconds. ‘The rate of flow at r= 15 seconds is given by .04 m/s and Q, = 0.02 mls = 0) +222 515 where Q,= 0= 0 +2 TE x15 where , X15= 0.03 mis ‘au Ree eee (eee eee ec :. Convective acceleration [1.273 0 (040.1 x)71 x (0.2546 0 (0.4 ~ 0.1.39") .273 x 0.2546 Q? x (0.4 ~ 0.1 x 1)* ++ Convective acceleration (when Q = 0.03 m’/s and.x= I m) 1.273 x 0.2546 x (0.03)° x (0.4 - 0.1 1)? 273 x 0.2546 x (0.03)* x (0.3)* m/s* .0108 mis? liv) Bes ar 11.273 90.40.17] [+s 4 from equation (i) is w= 1.273 Q (0.4 -0.1.x)] a Local aceeleration = 57 sia xco4 ony 22 [ +» Local acceleration is at a point where x is constant but Q is changing] Local acceleration (at x = 1 m) 1273 x (04-04 x 12x 22 ar Vice tot-o12 om xox at-00t 22) = 0.00943 m/s” (vy) Hence adding equations (iv) and (1), we get total acceleration. <- Total acceleration = Convective acceleration + Local acceleration 10108 + 0.00943 = 0.02023 mis’, Ans. Kinematics of Flow and Ideal Flow 181. > 5.8 VELOCITY POTENTIAL FUNCTION AND STREAM FUNCTION 5.8.1 Velocity Potential Function. It is defined as a scalar function of space and time such that its negative derivative with respect to any direction gives the fluid velocity in that direction. Itis defined by 6 (Phi). Mathematically, the velocity, potential is defined as @ = f(x,y, 2) for steady flow such that (5.9) where u, v and w are the components of velocity in x, y and z directions respectively, ‘The velocity components in cylindrical polar co-ordinates in terms of velocity potential function are given by 2 (594) Where u,= velocity component in radial direction (i.e. in r direction) and ug= velocity component in tangential direction (i.e. in @ direction) du | av | ow “The continuity equation for an incompressible steady flow is 4 4 9 4 OY ax ay ae Substituting the values of u, v and w from equation (5.9), we get 2 (_a), 2 (_a 2 *) 2/-2).2 +2(-2) <0 x 3) xl ay)” aa ao , a aierneie) 5.10 or te (5.10) Equation (5.10) is a Laplace equation. Ho, (5.11) For two-dimension case, equation (5.10) reduces to SF +5 ¢ If any value of 6 that satisfies the Laplace equation, will correspond to some case of fluid flow. Properties of the Potential Function. The rotational components* are given by G3) * Please, refer to equation (5.17) on page 192. 182 Fluid Mechanics 1% =) Tae ae 1 (aa ay a Substituting the values, of u, v and w from equation (5.9) in the above rotational components, we get 26 2 2) af oo , ae dy) ay ar. 2| axdy dyax ( 2) Ft ox. Oxdz and 2 *) ae * 2Lay ae ded 114 18 a cominvous function, hen 2-# = 2 ay When rotational components are zero, the flow is called irrotational. Hence the properties of the potential function are : 1. If velocity potential (6 exists, the flow should be irrotational 2. If velocity potential (6) satisfies the Laplace equation, it represents the possible steady incom- pressible irotationa flow 5.8.2 Stream Function. It is defined asthe scalar function of space and time, such that its partial derivative with respect to any direction gives the velocity component aright angles to that direction. It is denoted by y (Psi) and defined only for two-dimensional flow. Mathematically, for steady flow itis defined as y= f(t 9) such that ow ox (5.12) and sou ay The velocity components in cylindrical polar co-ordinates in terms of stream function are given as Lay ay 2M and uy = - 2 5.124 ra ar came where u,= radial velocity and ug = tangential velocity Kinematics of Flow and Ideal Flow 183 Substituting the values of w and v from equation (5.12), we get a (_2v), 2/2) og Sv, Hy xl ) 5 (B)-0% Didy * dxdy ay Hence existence of y means a possible case of fluid flow. The flow may be rotational or irrotational. 1(av_aw ‘The rotational component «, is given by @, = —| 1 is given by ©, 1(% x) Substituting the values of w and y from equation (5.12) in the above rotational component, we get For irrotational flow, @, = 0. Hence above equation becomes as2¥ which is Laplace equation for y. The properties of stream function (y) are = 1. If stream function (y) exists, it is a possible case of fluid flow which may be rotational or irrotational. 2. If stream function (y) satisfies the Laplace equation, itis a possible case ofan irotational flow. 5.8.3 Equipotential Line. A line along which the velocity potential @ is constant, is called cequipotential line ‘ dy = ace ay do = ae dx + ay dy 20 (ude + vdy). For equipotential line, or ~ (uds + vy) aw ae dy av ine of Constant Stream Function = Constant day or udx + vdy = 0 (5.13) But Slope of equipotential line. But d+ ay = 4 vde—udy ay 184. Fluid Mechanics For a line of constant stream function = dy = 0 or vdx~udy = 0 ay acu or (5.14) bur soe ot tam ne, From equations (5.13) and (5.14) itis clear that the product of the slope of the equipotential line and the slope of the stream line at the point of intersection is equal to ~ 1. Thus the equipotential lines are ‘othogonal to the stream lines at all points of intersection. 5.8.5 Flow Net. A grid obtained by drawing a series of equipotential lines and stam lines iscalled flow net. The flow net is an important tool in analysing two-dimensional rotational flow problems. 5.8.6 Relation between Stream Function and Velocity Potential Function From equation (5.9), we have “ From equation (5.12), we have u oy Thus, we have u x ar 9% Hence a ar 5.15) and 3 Problem 5.10 The velocity potential function (6) is given by an expression woe 3 3 (i), Find the velocity components in x and y direction. (ii) Show that 6 represents a possible case of flow. we Pry Solution. Given : =a Ae Gi oe AD ay The partial derivatives of @ wart. x and y are +3 ay and. +2y 2) Kinematics of Flow and Ideal Flow 185 (® The velocity components u and v are given by equation (5.9) From equations (1) and (2), we have ci Now By and ca) ay ar 24 day Day +2 Ans. YP - 204% wien yr 4 2y 24 2y) + 2y+2)=0 Laplace equation is satisfied and hence @ represent a possible case of flow. Ans. Problem 5.11 The velocity potential function is given by 6 = 5 (x? y*). Calculate the velocity components at the point (4, 5). Solution. 6250-97) But velocity components u and v are given by equation (5.9) as oa 26 10x © ly) = toy ‘The velocity components at the point (4, 5), ie., atx=4, y= 5) ~10x4 10x 40 units. Ans. 30 units. Ans. 186 Fluid Mechanics Problem 5.12 A stream function is given by y = 5x —6y. Calculate the velocity components and also magnitude and direction of the resultant velocity at any point Solution. lu? +? = 6 +5? = f36+25 =V6T = 7.81 univisee Resultant velocity vs Direction is given by, tan @ j= 0.833 Ei = tan! 833 = 39° 48’. Ans. Problem 5.13 If for a mwo-dimensional potential flow, the velocity potential is given by ey) determine the velocity at the point P (4, 5). Determine also the value of stream function W at the point P. 9=xQy-1) Solution. Given : (@ The velocity components in the direction of x and y are 9 units/sec = 8 units/sec Velocity at P 9i Resultant velocity at P= 9? +8° = JBI+64 = 12.04 units/sec = 12.04 units/sec. (id) Value of Stream Funetion at P A) Atthe point P (4,5), ie., atx Ans. We know that ii) and. y's we get Integrating equation () w.r. Jdy = J (2y-1)dy or y= Constant of integration. Kinematics of Flow and Ideal Flow 187 ‘The constant of integration is not a function of y but it can be a function of x. Let the value of ‘constant of integration is k. Then way -ytk iii) Differentiating the above equation wars. ‘x, we get oy ax av But from equation (i), SY =~ 2x t from equation (i), SY Integrating this equation, we get k= J ~ 2d Substituting this value of k in equation (i), we get y= y?=y =x. Ams. ++ Stream function y at P (4, 5) = 5-5-4? = 25 - 5-16 = 4 units. Ans. Problem 5.14 The stream function for a two-dimensional flow is given by y = 2xy, calculate the velocity atthe point P (2, 3). Find the velocity potential function 6. Solution. Given > wey ‘The velocity component and vin terms of y are aya 22 @ ay ay ay_a v= 22 ayy 22, or aS ry) = 2y, Atte point P (2,3), we get w= 22 = —4 unitsee ye 2x3 = Ounitlsee Resultant velocity at P= Yu? +v? = Ja? +6 = f16436 = VSD = 7.21 unitsisec. Velocity Potential Funetion We know u=-( 2228 @ -2y ii) Integrating equation (9, we get Jodo =) 2xax 2x 2 = pcevec or o=acete where C is a constant which is independent of x but can be a function of y. Differentiating equation (i wrt. y', we get 22 = 9 a ay 188 Fluid Mechanics But from (ii), Integrating this equation, we get C= [- 2y dy ‘Substituting this value of C in equation (ii), we get @ = x?—y7, Ans. Problem 5.15. Sketch the stream lines represented by Y Also find out the velocity and its direction at point (1, 2). Solution. Given yer? ‘The velocity components 1 and v are 2y 2 m~ 4 UNITS /SEC At the point (1, 2), the velocity components are 2x 1=2 units/sec Resultant velocity = fi ty? = (4) +2? ey =V20 = 4.47 units/see 1 and, tan =~ u ud 2 x @= tan 5 = 26° 34" Resultant velocity makes an angle of 26° 34” with x-axis, Fig. 5.10 = 2 2=-4 unitssec i Sketch of Stream Lines versy? Let = 1, 2,3 and so on. ‘Then we have l=e4y? eter 3erey and so on, Each equation is a equation of a circle. Thus we shall get concentric circles of different diameters as shown in Fig. 5.10. Problem 5.16 The velocity components in a two-dimensional flow field for an incompressible fluid are as follows : we 2 gre eyandve ny? 29-28 obtain an expression for the stream function Y. Kinematics of Flow and Ideal Flow 189 Solution. Given : ua y+ 2e-ey veo? 2y- 88. The velocity components in terms of stream function are ov M eye nt 2-08 0) OY aya y9-204%y Aid) ay Integrating (f) wart. x, we get y= J (9? = 2y 18) de where is a constant of integration which is independent of x but can be a function of y. Differentiating equation (iii) w.t. y, we get But from (ii), Comparing the value of 2¥., we get x oy ak ay YB 244.2 Ineraing, we st k= fe Pay Substituting this value in (ii), we get Problem 5.17 ina two-dimensional incompressible flow, the fluid velocity components are given by u=x-—dyandy =-y-4n. Show that velocity potential exists and determine its form. Find also the stream function. Solution. Given : wex-4y and au ox au, ox ay Hence flow is continuous and velocity potential exists. Let = Velocity potential. 190 Fluid Mechanics Let velocity components in terms of velocity potential is given by ws —(x-4y) x+dy Ai) and. ve-Gy-4nay44e ii) 2 Integrating equation (9), we get 9 =~ + day-+ eoliii) where C is a constant of integration, which is independent of x. This constant can be a function of y. Differentiating the above equation, i.e., equation (iii) with respect to ‘y’, we get co ay ac oars a : 2% Equating the two values of 2° , we get ay ye t4ay +2. Ans. aa) Value of Stream functions Let y = Stream function The velocity components in terms of stream function are ~y-4e Gv) and ow = 4y) 2-4 4y ol) wr 4) iy “ Integrating equation (7) wats. x, we get 42 2 ak (oi veo «wv, where k is a constant of integration which is independent of x but can be a function of y. Kinematics of Flow and Ideal Flow 191 Differentiating equation (vt) was. y, we get 2 =— 2-04 2k ay oy oy Bat from equation (»), we have aw it -quation (v), ay oy ak Bquating the two vatuesof 2%, we get — 242% ay ay 2 Integrating the above equation, we get ae ay? Substituting the value of k in equation (vi), we get yenyx 2x? + 2y7, Ans. > 5.9 TYPES OF MOTION A fluid particle while moving may undergo anyone or combination of following four types of displacements : (®) Linear Translation or Pure Translation, (ii) Linear Deformation, (iii) Angular Deformation, and (iv) Rotation. 5.9.1 Linear Translation. It is defined as the movement of a fluid element in such a way that it moves bodily from one position to another position and the two axes ab and cd represented in new positions by a’b and c’d” are parallel as shown in Fig. 5.11 (a) 5.9.2 Linear Deformation. It is defined as the deformation of a fluid element in linear direction when the element moves. The axes of the clement in the deformed position and un-deformed position are parallel, but their lengths change as shown in Fig. 5.11 (2). y « fe ; «Poy Se 4 Tle sy op = x x (0) LUNEAR TRANELATION = Bes x % (@) ANGULAR DEFORMATION (@) PURE ROTATION Fig. 5.11, Displacement of a fluid element. 192. Fluid Mechanics 5.9.3 Angular Deformation or Shear Deformation. It is defined as the average change in the angle contained by two adjacent sides. Let A@, and AQ, is the change in angle between two adjacent sides of a fluid element as shown in Fig. 5.11 (c), then angular deformation or shear strain rate 1 = 0, + 28, ze + A8,) Now 6, =H 5, At ay Ax ax ‘Angular deformation = + [A8, + A8,] 1fav. aw Shear strain rate = 4/2" (5.16) ce serait a2 3] 5.9.4 Rotation. It is defined as the movement of a fluid element in such a way that both of its axes (horizontal as well as vertical) rotate in the same direction as shown in Fig. 5.11 (d). It is equal 1 (2 ou ax ay (S17) 5.9.5 Vorticity. It is defined as the value twice of the rotation and hence it is given as 20 Problem 5.18 A fluid flow is given by V = 8xi~ 10x". Find the shear strain rate and state whether the flow is rotational or irrotational.. Solution. Given : Vs 8x'i- 10x°yj au ou u = 2407, Br, ro im, yy 2 ov and v= = 1Or'y, 5 = = 2 (@) Shear strain rate is given by equation (5.16) as (& [ Kinematics of Flow and Ideal Flow _193 | (i) Rotation in x - y plane is given by equation (5.17) or (2 *) * 2029-0) == 1039 2 lax ay) 2 As rotation ©, # 0. Hence flow is rotational. Ans. Problem 5.19 The velocity components in a two-dimensional flow are w= + 2x —27y and v = xy? ~2y- 0B. Show that these components represent a possible case of an irrotational flow. Solution. Given : us yB+2x- xy au ar (®)_ For a two-dimensional flow, continuity equation is 2 au Substituting the value of “ and 2, we get ituting Fe am Sr me get au , av BH 8 2 ay 4 2-2-0 ax ay ceded tis a possible case of fluid flow. 1fdv du ii) Rotation, @, is given by @, = +{ 2-2 (i) Rotation, @, is given by ©, (2 = Rotation is zero, which means it is case of irrotational flow. Ans. =1w-_2-G-2= ) 7 WO? -7)- 07-1 =0 > 5.10 VORTEX FLOW ‘Vortex flow is defined as the flow of a fluid along a curved path or the flow of a rotating mass of fluid is known a ‘Vortex Flow’. The vortex flow is of two types namely : 1, Forced vortex flow, and 2. Free vortex flow. 5.10.1 Forced Vortex Flow. Forced vortex flow is defined as that type of vortex flow, in which some external torque is required to rotate the fluid mass. The fluid mass in this type of flow, rotates at constant angular velocity, «. The tangential velocity of any fluid particle is given by veoxr (5.18) [194 Fluid Mechanics ] where r= Radius of fluid particle from the axis of rotation. | CENTRAL AXIS Leu [i fom (8) CYLINDER IS STATIONARY (b) CYLINDER IS ROTATING VERTICAL-"} CYLINDER, Fig.5.12. Forced vortex flow. Hence angular velocity @ is given by © Constant. (5.19) Examples of forced vortex are : 1. A vertical cylinder containing liquid which is rotated about its central axis with a constant angular velocity (as shown in Fig. 5.12. 2. Flow of liquid inside the impeller of a centrifugal pump. 3. Flow of water through the runner of a turbine. 5.10.2 Free Vortex Flow. When no external torque is required to rotate the fluid mass, that type of flow is called free vortex flow. Thus the liquid in case of free vortex is rotating due to the rotation which is imparted to the fluid previously. Examples of the free vortex flow are : 1. Flow of liquid through a hole provided at the bottom of a container. 2. Flow of liquid around a citcular bend in a pipe. 3. A whirlpool in a river. 4, Flow of fluid in a centrifugal pump casing. ‘The relation between velocity and radius, in free vortex is obtained by putting the value of external torque equal to zero, or, the time rate of change of angular momentum, i.e., moment of momentum must be zero. Consider a fluid particle of mass ‘m’ at a radial distance r from the axis of rotation, hhaving a tangential velocity v. Then ‘Angular momentum = Mass x Velocity =m xv Moment of momentum Momentum x r= m xv xr a ‘Time rate of change of angular momentum = o (mvr) a For free vortex 2 (myn ar Constant Integrating, we get mr = Constant or vr = S49" = Constant (5.20) [ Kinematics of Flow and Ideal Flow _ 195] 5.10.3 Equation of Motion for Vortex Flow. Consider a fluid element ABCD (shown shaded) in Fig. 5.13 rotating at a uniform velocity in a horizontal plane about an axis perpendicular to the plane of paper and passing through 0. Let r= Radius of the element from 0. A@ = Angle subtended by the element at 0. ‘Ar = Radial thickness of the element. AA = Arca of cross-section of element. ‘The forces acting on the element are : (@) Pressure force, pAA, on the face AB. (iid) Centrifugal force,"""— acting in the direction away from the centre, 0. Now, the mass of the element: Mass density x Volume x AA x Ar Centrifugal force = pAddr Equating the forces in the radial direction, we get ar (rt) a4 pat ptr : 2 are patar’ necting Ar AA from bah sides, we get = 9 (5.21) Equation (5.21) gives the pressure variation along the radial direction for a forced or free vortex flow na horizontal plane. The expression 2 is called pressure gradient inthe rail direction, AsSE is positive, hence pressure increases with the increase of radius *r. The pressure variation inthe vertical plane is given by the hydrostatic law, i, ap a In equation (5.22), z is measured vertically in the upward direction. ‘The pressure, p varies with respect tor and zor p is a function of r and z and hence total derivative ofpis Pe (5.22) pe Bare, Substituting the values of 2 from equation (5.21) and 22 from equation (5.22), we get ar a2 [196 Fluid Mechanics ] dp = p~— dr~ pad (5.23) Equation (5.23) gives the variation of pressure of a rotating fluid in any plane, 5.10.4 Equation of Forced Vortex Flow. For the forced vortex flow, from equation (5.18), we have veoxr where © = Angular velocity = Constant. Substituting the value of v in equation (5.23), we get oF dp =p x2 dr— pg de. Consider two points 1 and 2 in the fluid having forced vortex flow as shown in Fig. 5.14. Integrating the above equation for points 1 and 2, we get fin = f ouvir Fonte or @-P)= [ro =] - pe [cht or @2~ p= PF tr? = nl 98 f= 21 § long — 0r)'1- pg lesa) 2 son = 2 be —w?l- pg tesa { } If the points 1 and 2 lie on the free surface of the liquid, then p) hence above equation becomes 0 or ps [z~ 211 ~2)= = f2-», or Fan al= 55 ba - nih If the point I lies on the axis of rotation, then v, = @ x r, = @ x 0= 0. The above ‘equation becomes as nk Let act (5.24) [ Kinematics of Flow and Ideal Flow _197| ‘Thus Z varies with the square of r. Hence equation (5.24) is an equation of parabola. This means the free surface of the liquid is a paraboloid, Problem 5.20 Prove that in case of forced vortex, the rise of liquid level at the ends is equal to the fall of liquid level at the axis of rotation. Solution. Let R= radius of the cylinder. (0-0 = Initial level of liquid in cylinder when the cylinder is not rotating. - Initial height of liquid hen) <. Volume of liquid in cylinder = mR? x Height of liquid AR? x (+3) i) Let the cylinder is rotated at constant angular velocity (. The liquid will rise at the ends and will all at the centre. Let Rise of liquid at the ends from 0-0 xr Fall of liquid at the centre from 0-0. Then volume of liquid = [Volume of cylinder upto level B-B] = [Volume of paraboloid] = [RR? x Height of liquid upto level B-B] RR ~ |=» Height of paraboloid eax (he x+y) ee xray) ikon wexneaecsy xen : 2 mR? 2 ety Ai’) Equating (i) and (ii), we get BP th x)= wR EE xe y) or or Fall of liquid at centre = Rise of liquid at the ends. Problem 5.21 An open circular tank of 20 cm diameter and 100 cm long contains water upto a height of 60 cm. The tank is rotated about its vertical axis at 300 r.p.m., find the depth of parabola formed at the free surface of water. Solution, Given : Diameter of cylinder = 20m 20 Radius, R= = 10cm 7710 [198 Fluid Mechanics ] Height of liquid, Speed, ‘Angular velocity, = 2XEX300 «5141 radiecc, 0 Let the depth of parabola Using equation (5.24), = (oy 50.28 cm. Ans. 2g 2x 981 Problem 5.22 An open circular cylinder of 15 cm diameter and 100 cm long contains water upto 4@ height of 80 cm, Find the maximum speed at which the cylinder is to be rotated about its vertical axis $0 that no water spills. Solution. Given : Diameter of cylinder = 15m + Radius, Length of cylinder, Initial height of water Let the cylinder is rotated at an angular speed of « rad/sec, when the water is about to spill. Then using, Rise of liquid at ends Fall of liquid at centre But rise of liquid at ends cength ~ Initial height = 100 ~ 80 = 20 em Fall of liquid at centre = 20 em Height of parabola 20 + 20 = 40 em Z=40 om oR Using the relation, Zz we get 49 = 279) v raid 2981 402X981 _ 1595 75x75 = 13952 = 37.35 rad/s 2aN «. Speed, Nis given by @ = “2. Ps given by a5 or 60x@ _ 603735 2n (xR Problem 5.23 A cylindrical vessel 12 cm in diameter and 30 cm deep is filled with water upto the top. The vessel is open at the top. Find the quantity of liquid left in the vessel, when itis rotated about its vertical axis with a speed of (a) 3000 r.p.m., and (b) 600 r.p.m. Solution. Given : Diameter of cylinder = em Radius, R=60m Initial height of water = 30cm [ Kinematics of Flow and Ideal Flow _199 | Initial volume of water = Area x Initial height of water = Fi x 12? x 30 cm? = 3392.9 em’ (a) Speed, @?R? _ (3141)? x6? 2g 2x981 Height of parabola is given by Z= = 18:10 em, As vessel is initially full of water, water will be spilled if itis rotated. Volume of water spilled is ‘equal to the volume of paraboloid. But volume of paraboloid = [Area of cross-section x Height of parabola] + 2 DP xZ =F x 12? x!810 | 1023.53 cm? 2°4 2 Volume of water left tial volume ~ Volume of water spilled 3392.9 - 1023.53 = 2369.37 em’. Ans. (b) Speed, N= 600 rpm. BAN _ 28600 _ 69.89 rads 0 0 Height of parabola, 720. C8) x6 a 40m, 2g 2x981 [As the height of parabola is more than the height of cylinder the shape of imaginary parabola will be as shown in Fig. 5.17. Let r= Radius of the parabola at the bottom of the vessel. Height of imaginary parabola = 72.40 ~ 30 = 42.40 em. Volume of water left in the vessel ‘olume of water in portions ABC and DEF = Initial volume of water = Volume of paraboloid AOF + Volume of paraboloid COD. Now volume of paraboloid AOF = © x D? x Height of parabola PA" = 4094.12 em? macitiary " maciary 2 CYLINDER’ PARABOLA For the imaginary parabola (COD), « = 62.82 rad/see Fig. 5:17 Z= 424 om Radius at the bottom of vessel [200 Fluid Mechanics ] oF 62.82? xr? ising the relation Zz we get 424 = 9282 xr eee 2g | Ne Bt 2981 ; 9814240 _ 91 79 62.82 x 62.82 r= Y2L079 = 4.59 em ‘Volume of paraboloid COD = J Area athe top ofthe imaginary parabola x Height of parabola 1 ex 4.59? x 42.4 = 1403.89 cm? adn x ard 2 2 Volume of water left = 3392.9 ~ 4094.12 + 1403.89 = 702.67 em*. Ans. Problem 5.24 An open circular cylinder of 15 cm diameter and 100 cm long contains water upto a height of 70 cm. Find the speed at which the cylinder is to be rotated about its vertical axis, so that the axial depth becomes zero. Solution, Given : Diameter of cylinder Radius, Length of cylinder Initial height of water = 70 em. ‘When axial depth is zero, the depth of paraboloid = 100 em. be 18cm —1 Fig. 5.18, Using the relation, 2nH - Speed, Nis given by @ = == ps given by 7 or N= DEO _ 10%5905 «563.98 rpm, Ans. 2 2n Problem 5.25 For the problem (5.24), find the difference in total pressure force (i) at the bottom of cylinder, and (ii) at the sides of the cylinder due to rotation. [ Kinematics of Flow and Ideal Flow _201| Solution. (i) The data is given in Problem 5.24. The difference in total pressure force at the bottom of cylinder is obtained by finding total hydrostatic force at the bottom before rotation and after rotation. Before rotation, force = pg x (0.15)? m?, hi = 70 em = 0.70 m where p= 1000 kg/m’, A = Area of bottom => D’ a Force = 1000 x 9.81 x5 x (0.15)* 0.7 N = 121.35 N After rotation, the depth of water at the bottom is not constant and hence pressure force due to the height of water, will not be constant. Consider a circular ring of radius r and width dr as shown in Fig. 5.19. Let the height of water from the bottom of the tank upto free surface of water at a radius Hydrostatic force on ring at the bottom, AP = pg x Area of ring x Z = 1000 x 9.81 x 2nrdr x © or? 28 9810 x 2.x Rr x xdr Total pressure force at the bottom wy afar = [f 9810x2285 ons =f waco tr a Fig. 519 From Problem $24, = 59.05 rads R75 0m= 075 m. Substituting these values, we get ftal pressure foree __ 196201 x (59.057 [<} 2x981 «| 4 |, 19620 x x x(59.05)* (075) _ 6.69 2x981 4 Difference in pressure forces at the bottom 121.35 ~ 86.62 = 34,73 N. Ans. (i) Forces on the sides of the cylinder Before rotation peal where A'= Surface area ofthe sides ofthe cylinder upto height of water RD x Height of water = m x .15 x 0.70 m? = 0.33 m? [202 Fluid Mechanics ] Ji = CG. of the wetted area of the sides 1 0.70 5 x height of water = > 3 * helaht of water = “> 135m Force on the sides before rotation = 1000 x 9.81 x 0.33 x 0.35 = 1133 N After rotation, the water is upto the top of the cylinder and hence force on the sides 1000981 «Wot ae of te sis High of wa B10 x Dx 10% Ex 1.0= 9810 mx 1S x 5 = 231143 N Difference in pressure on the sides 2311.43 ~ 1133 = 1178.43 N. Ans. 5.10.5 Closed Cylindrical Vessels. If a cylindrical vessel is closed at the top, which contains some liquid, the shape of paraboloid formed due to rotation of the vessel will be as shown in Fig. 5.20 for different speed of rotations. Fig. 5.20 (a) shows the initial stage of the cylinder, when it is not rotated. Fig. 5.20 (b) shows the shape of the paraboloid formed when the speed of rotation is (. Ifthe speed is increased further say , the shape of paraboloid formed will be as shown in Fig. 5.20 (c). In this case the radius of the parabola at the top of the vessel is unknown. Also the height of the paraboloid formed corresponding to angular speed @ is unknown. Thus to solve the two unknown, we should have two equations. One ‘equation is 28 ‘The second equation is obtained from the fact that for closed vessel, volume of air before rotation is equal to the volume of air after rotation, ‘Volume of air before rotation = Volume of closed vessel ~ Volume of liquid in vessel Zz me xZ 2 J T I ‘Volume of air after rotation = Volume of paraboloid formed 1 »? I i @ © ° Fig. 5.20, Problem 5.26 A vessel, cylindrical in shape and closed at the top and bottom, contains water upto 4 height of 80 cm. The diameter of the vessel is 20 cm and length of vessel is 120 cm. The vessel is rotated at a speed of 400 r.p.m. about its vertical axis. Find the height of paraboloid formed. [ Kinematics of Flow and Ideal Flow _203 | Solution. Given : Initial height of water = 80 em Diameter of vessel = 20cm v. Radius, R= 10cm Length of vessel = 120 em 7 Speed, N= 400 rpm. mou 2AM 28400 - 41.88 reais «60 When the vessel is rotated, let Z Height of paraboloid formed no 1r= Radius of paraboloid at the top of the vessel This is the case of closed vessel Fig. 521 ‘Volume of air before rotation = Volume of air after rotation or 2 ptxi-" ptxso=nPxZ 4 4 2 where Z = Height of paraboloid, r= Radius of parabola. or © px 120- © p?x s0= nex Z 4 4 2 or xD? (120 ~ 80) = FD? x40 Rag? Zz or = x 20? x 40 = 4000 x w= mr x = F 20° x 40 = 4000 x x= 7 x S Pxz = ROXRX? _ s009 ld) ® a 4188 x7? _ 4188 x? - Using relation z we get z= SBS 0° MAB 7" _ 9.504 1 * 2 8 28 2xo81 7 OS z be 0804 Substituting this value of rin (), we get 2 x 7= 3000 O08 B = 8000 x 0.894 = 7152 Zea =tasoomam aw | Hind Method 0 Let Z, = Height of paraboloid, ifthe vessel would not have been | ‘closed at the top, corresponding to speed, N= 400 rpm, or = 41.88 rad/s PR? _ 4188? x10? 89.34 cm. 26 2x981 Then zy [204 Fluid Mechanics ] Half of Z, will be below the inital height of water in the vessel 2 _ 3934 A a But height of paraboloid for closed vessel = CO = CA + AO = (120 ~ 80) + 44.67 em 40 + 44.67 = 84.67 cm. Ans. Problem 5.27 For the data given in Problem 5.26, find the speed of rotation of the vessel, when axial depth of water is zero. Solution, Gen ry ie Ao = 44.67 om Diameter of vessel 20 em +. Radius, R= 10cm Initial height of water 80 cm T ‘so Length of vessel = 120m 120 Let ais the angular speed, when axial depth is zero ao When axial depth is zero, the height of paraboloid is 120 em and +. Using the relation, 129= 2% %980 oP =2 x 980 x 120 = 235200 of) ‘Volume of air before rotation = Volume of air after paraboloid mR? x (120 ~ 180) = Volume of paraboloid Baz enexZ 2 or x10? «40 = =x 120 x10? x40x2_ 8000 or Pe RXNO X40? _ 2000 «65.67 x 120 120 Substituting the value of ? in equation (i), we get @? x 66.67 = 235200 59.4 rads 6667 2nN 2 Speed Nis given by @ = 2. P aiven by @= or N= BO OO%504 _ 567.22 ppm. Ans, 2 aR Problem 5.28 The cylindrical vessel of the problem 5.26 is rotated at 700 r.p.m. about its vertical axis. Find the area uncovered at the bottom of the tank: Solution. Given Initial height of water = 80 cm Diameter of vessel 20 em e Radius, 10 em Length of vessel = 120 em [ Kinematics of Flow and Ideal Flow _205 | Speed, N= 700 rpm. PRN _ 228X700 _ 73.30 rads, 0 0 If the tank is not closed at the top and also is very long, then the height of parabola corresponding to @= 733 will be oP xR? _ 7337 x10? 2xg 2980 From Fig. 5.24, 4) + 120 +x, = 274.12 or xy + y= 274.12 ~ 120 = 154.12 em ...() From the parabola, KOM, we have oe _ B3? x4 120 +x) = 2 = eli cs » 2g 2x 980 Se For the parabola, LON, we have Lal * 208, BR xe , ef 7 2g 2x 980 . Fig. 5.24 Now, volume of air before rotation = Volume of air after rotation Volume of air before rotation = nR* x (120 ~ 80) = x x 10° x 40 = 1256.3 em* =i) Volume of air after rotation = Volume of paraboloid KOM ~ volume of paraboloid LON cnr (BEA) gee AL © Equating (iv) and (0), we get m2 (104%) _ efx mit a) _ alc Substituting the value of r,? from (ii) in (vi), we get (120+ x,) «2980 7 1256.3 = =i) (204%) _ nex 2 2980 x (120 + x (733) 12566.3 = x x or 12566.3 = 0.573 (120 + x)? Substituting the value of x from (iff) in the above equation 33? xr J 2 “2980 12566.3 = 0.573 | 120+ 2980 0.573 (120 + 2.74 133-43 x rx ry? = 0.573 [120° + 2.74? r+ 2 120 x 2.74 re] 43 ry [206 Fluid Mechanics = 0.573 [14400 + 7.506 r,! + 657.6 73°] — 4.3 1 125663 21930 = 14400 + 7.506 r,* + 657.6 r;° 4.3 734 0573, or 13! (7.506 ~ 4.3) + 657.6 r,° + 14400 - 21930 = 0 or 3.206 rf + 657.6 r,? ~ 7530 =0 + 657.6" —4 x(~7530) x (3206) > _ 657. 23206 657.6 + [432A37.16 + 96504.72 6a = 9516S 2732 _ _ 915.98 of 10.87 o4l2 Negative value is not possible 2 rp = 10.87 em? +. Area uncovered at the base = mr,? = x 10.87 = 34.149 em®, Ans. Problem 5.29 A closed cylindrical vessel of diameter 30 cm and height 100 cm contains water upto a depth of 80 cm. The air above the water surface is at a pressure of 5.886 N/cm?. The vessel is rotated at a speed of 250 r.p.m. about its vertical axis. Find the pressure head at the bottom of the vessel : (a) at the centre, and (b) at the edge. Solution, Given : Diameter of vessel = 30cm v. Radius, R= 150m Initial height of water, H/= 80 em Length of cylinder, L= 100em Pressure of air above water = 5.886 Niem? or p= 5.886 x 10'S Head due to pressure, h= ppg = 5886%10" 6 m of water 1000x9381 Speed, N= 250 rpm. 2nN _ 2nx250 =e 26.18 rads oO 60 Height of paraboloid formed, if the vessel is assumed open at the top and it is very long. y°R? _ 26.18" x15 2g 2x 981 Let r; is the radius of the actual parabola of height x2 Then we have a = 78.60 cm oo) [ Kinematics of Flow and Ideal Flow 207 | o'r? _ 2618? x4 Then x 35, di 23g 7 a qog1 O57 @ ‘The volume of sir before rotation RR? (100 ~ 80) = wx 15? x 20 = 14137 em? Volume of air after rotation = Volume of paraboloid EOF 1 =p xan But volume of air before and after rotation is same. : 14137 = bx are But from (i), 43 £ 0.35 7)? 14137 = 4 x nr? 035 1? pte RAMS Sosa #x035 r= 25714)" = 12.66 cm Substituting the value of r, in (i), we get 2) = 0.35 x 12.66? = 56.1 em Pressure head at the bottom of the vessel (2) At the centre. The pressure head at the centre, -e., at H = Pressure head due to air + OH .0 + (HL~ LO) (OH = LH- Lo} + HL=1000m=1m = 60 + (1.0 0.561 soeda-assy fore Scien sétal (b) At the edge, i.e., at G = Pressure head due to air + height of water above G soracesos (GM Ht) =60 + G03) om it~ o00s6 { risemceasn | = 6.0 + 0.439 + 0.786 = 7.225 m of water, Ans. Problem 5.30 A closed cylinder of radius R and height H is completely filled with water. It is rotated about its vertical axis with a speed of @ radians/s. Determine the total pressure exerted by water on the top and bottom of the cylinder. Solution. Given Radius of cylinder =R Height of eylinder =H ‘Angular speed =o [208 Fluid Mechanics ] AS the cylinder is closed and completely filled with water, the rise of water level at the ends and depression of water at the centre due to rotation of the vessel, will be prevented. Thus the water will exert force on the complete top of the vessel. Also the pressure will be exerted at the bottom of the cylinder. te Rf ‘Total Pressure exerted on the top of cylinder. The top of cylinder is in contact with water and is in horizontal plane. The pressure variation at any radius in horizontal plane is given by equation (5.21) D or ae Integrating, we get Jap =Spotrdr or p= Consider an elementary circular ring of radius r and width dr on the top of the cylinder as shown in Fig. 5.26. ‘Area of circular ring = 2nrdr Force on the elementary ring = Intensity of pressure x Area of ring Fig. 5.26 = px 2nrdr x 2nrdr. ® wr} & a'r? x amr { e Total force on the top of the cylinder is obtained by integrating the above equation between the limits 0 and R. Toa fore or Fy= [ 207s? xanntr= Be? x2K f" Par 6.25) Total pressure force on the bottom of cylinder, Fy = Weight of water in cylinder + total force on the top of cylinder 1g x MR x H+ Fy (3.26) pgx nex H+ ob x er p= Density of water. Problem 5.31 A closed cylinder of diameter 200 mm and height 150 mm is completely flled with water. Calculate the total pressure force exerted by water on the top and bottom of the cylinder, if it is rotated about its vertical axis at 200 r-p.m. Solution. Given Dia. of cylinder = 200 mm = 0.20 m Radius, R=01m Height of cylinder, H= 150 mm = 0.15 m Speed, N= 200 rp.m, 2nN _ 2nx200 Angular speed, ea) ues oo 60 [ Kinematics of Flow and Ideal Flow 209 | Total pressure force on the top of the cylinder is given by equation (5.25) y= Bat cca « 120 52908 52% 0." SAME N An 4 Now total pressure force on the bottom of the cylinder is given by equation (5.26) as F y= pg x mk? x H+ Fy 1000 x 9.81 x mx (OI)? x 0.15 + 34.44 = 46.22 + 34.44 = 80.66 N. Ans. 5.10.6 Equation of Free Vortex Flow. For the free vortex, from equation (5.20), we have vx r= Constant = say ¢ Substituting the value of v in equation (5.23), we get ap = pdr ~ pg de = px dr~pgde=px 5 dr~pg dz Pxr Consider two points 1 and 2 in the fluid having radius r, and r, from the central axis respectively as shown in Fig. 5.27. The heights of the points from bottom of the vessel is z, and z. Integrating the above equation for the points 1 and 2, we get fio or PoP far fog [ Pearmpe fae [Ef mete 3-vi]-psle-a] v? v3] ps [22-2] (5.21) Equation (5.27) is Bernoulli's equation, Hence in case of free vortex flow, Bernoulli's equation is applicable. Fig. 5.27 [210 Fluid Mechanics ] Problem 5.32 In a free cylindrical vortex flow, at a point in the fluid at a radius of 200 mm and at a height of 100 mm, the velocity and pressures are 10 m/s and 117.72 kN/m? absolute. Find the pressure at a radius of 400 mm and at a height of 200 mm. The fluid is air having density equal to 1.24 kg/m’. Solution. At Point 1 : Given Radius, = 200 mm = 0.20 m Height, z= 100 mm = 0.10 m Velocity, 0 mis Pressure, 17.72. kNN/m? = 117.72 x 10° Nim? At Point 2: 400 mm = 0.4 m 2 = 200 mm = 0.2 m Py = pressure at point 2 p= 1.24 kg/m? For the free vortex from equation (5.20), we have vx r= constant oF 7 10x02 ofa mis But p= 1.24 kg/m’ Uu772x10" 10g my aot 124x981" 2x981 pe 2x981 Py MII, 10? yy) 5? _ 9g pe 124x981 2x98” 2x93 9677.4 + 5.096 + 0.1 ~ 1.274 ~ 0.2 = 9676.22 1676.22 x py = 9676.22 x 1.24 x 9.81 17705 N/m? = 117.705 x 10° Nim? 17.705 kN/m? (abs.) = 117.705 kN/m?, Ans. (B) IDEAL FLOW (POTENTIAL FLOW) > 5.11 INTRODUCTION [Wal fluid is a fluid which is incompressible and inviscid. Incompressible fluid is a fluid for which density (p)remains constant. Inviscid fluid is a fluid for which viscosity (1) is zero. Hence a fluid for which density is constant and viscosity is zero, is known as an ideal fluid. ‘The shearstons is given by, t= 1 SE, Hence for idea id the shear ston wll e210 a8 = 0 for ideal fluid. Also the shear force (which is equal to shear stress multiplied by area) will be zero in [ Kinematics of Flow and Ideal Flow 211 | case of ideal or potential flow. The ideal fluids will be moving with uniform velocity. All the fluid particles will be moving with the same velocity. ‘The concept of ideal fluid simplifies the typical mathematical analysis. Fluids such as water and air have low viscosity. Also when the speed of air is appreciably lower than that of sound in it, the compressibility is so low that air is assumed to be incompressible, Hence under certain conditions, certain real fluids such as water and air may be treated like ideal fluids. > 5.12 IMPORTANT CASES OF POTENTIAL FLOW ‘The following are the important cases of potential flow : (i) Uniform flow, (ii) Source flow, (iii) Sink flow, (iv) Free-vortex flow, (v) Superimposed flow. > 5.13 UNIFORM FLOW In a uniform flow, the velocity remains constant. All the fluid particles are moving with the same velocity. The uniform flow may be : (i) Parallel to x-axis Gi) Parallel to y-axis. 5.13.1. Uniform Flow Parallel to x-Axis. Fig. 5.27 (a) shows the uniform flow parallel to a-axis. In a uniform flow, the velocity remains constant. All the fluid particles are moving with the same velocity. Fig. 5.27 (a) Le U = Velocity which is uniform or constant along x-axis uw and y = Components of uniform velocity U along x and y-axis. For the uniform flow, parallel t0 x-axis, the velocity components u and v are given as u=Uandy=0 (5.28) But the velocity uw in terms of stream function is given by, ay and in terms of velocity potential the velocity u is given by, ay _ a ua VL 5.29) a (5.29) 5 av _% Similarly, it can be shown that y = ~ 2¥ = 2% 5.294 ae te oe ay oe) But w= U from equation (5.28). Substituting u = U in equation (5.29), we have [212 Fluid Mechanics ] Le 5.3 u (5.30) & sand also u = 2 or = and atso u = First part gives dy = U dy whereas second part gives d@ = U dx. Integration of these parts gives as We Uy+C, and = Uxt Cy where C, and Cy are constant of integration, Now let us plot the stream lines and potential lines for uniform flow parallel to x-axis. Plotting of Stream lines. For stream lines, the equation is waUxy+G Let y= 0, where y = 0. Substituting these values in the above equation, we get 0=Ux04C, or C,=0 Hence the equation of stream lines becomes as y weU.y (5.31) ‘The stream lines are straight lines parallel to x-axis and at a distance y from the x-axis as shown in Fig. 5.28. In equation (5.31), U. y represents the volume flow rate (i. m/s) be- tween x-axis and that stream line at a distance y. Note. The thickness of the fluid stream perpendicular to the plane is assumed to be unity. Then y x I or y represents the area of flow. And Uy represents the produet of velocity and area. Hence U. y represents the volume flow rate. Stream lines ° Plotting of potential lines. For potential lines, the equation is O=U.x+C, Let @=0, where x= 0. Substituting these values in the above equation, we get C, Hence equation of potential lines becomes as oe Ux ‘The above equation shows that potential lines are straight lines parallel to y-axis and at a distance of x from y-axis as shown in Fig. 5.29. Fig. 5.30 shows the plot of stream lines and potential lines for uniform flow parallel to x-axis. The stream lines and potential lines intersect each other at right angles. be y cle Potental ines esis Potanal sys ines J rts fe 4 tC ers * We vs chee ease wo Fig. 5.29 Fig. 5.30 [ Kinematics of Flow and Ideal Flow 213] 5.13.2 Uniform Potential Flow Parallel to y-Axis. Fig. 5.31 shows the uniform potential flow parallel to y-axis in which U is the uniform velocity along y-axis. Fig. 531 The velocity components 1, v along x-axis and yraxs are given by u=0andv=U (533) ‘These velocity components in tems of steam function (y) and velocity potential function (9) are piven as av _ 2% = A534 dye ae ay _ a penta (5.35) - ee 635) But from equation (5.33), v = U. Substituting v = U in equation (5.35), we get oy _ 4% oy 29 ya oe ya and aso = % a ay By Firs part gives dy = — U ce whereas second part gives p= U dy Integration ofthese pars gives as, W=-U.x+C, and 0=U.y+Cy (5.36) where C, and C, are constant of integration, Let us now plot the stream lines and potential lines. Plotting of Stream lines. For stream lines, the equation is y= U.x + C, Let w= 0, where x= 0. Then C, = 0. Hence the equation of stream lines becomes as y = ~ U.x (3.37) ‘The above equation shows that stream lines are straight lines parallel to y-axis and at adistance of x from. the y-axis as shown in Fig. 5.32. The ve sign shows that the str:am lines are in the downward direction. stream lines I 5 +7 Pojentatinos Ys al ‘ [214 Fluid Mechanics ] Plotting of Potential lines. For potential lines, the equation is 6 = U.y + C, Let = 0, where y = 0. Then C; = 0. Hence equation of potential lines becomes as 6 = U.y (5.38) ‘The above equation shows that potential lines are straight lines parallel to x-axis and at a distance of 1y from the x-axis as shown in Fig. 5.32. > 5.14 SOURCE FLOW ‘The source flow is the flow coming from a point (source) and, moving out radially in all directions of a plane at uniform rate. Fig. 5.33 shows a source flow in which the point 0 is the source from which the fluid moves radially outward. The strength of a ol source is defined as the volume flow rate per unit depth, The unit 3 ‘ of strength of source is m/s. It is represented by 4. oer Let u, = radial velocity of flow at a radius r from the source O “(sy volume flow rate per unit depth dius ‘The radial velocity u, at any radius r is given by, Fig. 5.33 Source low (Flow atway (5.39) from source) ‘The above equation shows that with the increase of r, the radial velocity decreases. And at a large distance away from the source, the velocity will be approximately equal to zero, The flow is in radial direction, hence the tangential velocity ug = 0. Let us now find the equation of stream function and velocity potential function for the source flow. As in this case, up = 0, the equation of stream function and velocity potential function will be obtained from u, Equation of Stream Function By definition, the radial velocity and tangential velocity components in terms of stream function are given by [See equation (5.124)] But [See equation (5.39)] p24 or A w= hal nr? Qn a Integrating the above equation wat. 8, we gt w= dh x0 6 Cy, whee C, i constant of isp Let y=0, when 0 =0, then C, =0. Hence the equation of stream function becomes as Kinematics of Flow and Ideal Flow 215 4 at 4 vege (5.40) In the above equation, q is constant. ‘The above equation shows that stream function is a function of 8. For a given value of , the stream function will be constant, And this will be a radial line. The stream lines ean be plotted by having different values of @. Here 0 is taken in radians. Plotting of stream lines sa When @=0,y=0 Stream ines 0= 45° = = radians, y=. = =4 units ‘are radal 4 m4 ok q m4 vo 0= 90° = radians, y=. =4 units 4 2 me oe 24 a Equation of Potent wai By definition, the radial and tangential components in Fig. 5.34 Stream line for terms of velocity function are given by source flow. [See equation (5.94)] ar But from equation (5.39), u,= nr Equating the two values of u, we get og 4 2 or aye ar ar wr Or Integrating the above equation, we get ja-[fa Potent ines 2nr mere afl 4 or Ap dar fis Lisaconstan tem ante | ae 4 A top,r St Qn Be (S41) In the above equation, q is constant. ‘The above equation shows, that the velocity potential function is ‘a function of r. For a given value of r, the velocity function 6 will be constant. Hence it will be a circle with origin at the source, The velocity potential lines will be circles with origin at the source as shown in Fig. 5.35. Let us now find an expression for the pressure in terms of radius. Fig. 5.35. Potential lines for source [216 Fluid Mechanics ] Pressure istribution in a plane source flow ‘The pressure distribution in a plane source flow can be obtained with the help of Bernoulli's equa- tion. Let us assume that the plane of the flow is horizontal. In that case the datum head will be same for two points of flow. Let p= pressure at a point 1 which is at a radius r from the source at point 1 1, = velocity at point 1 Po = Pressure at point 2, which is at a large distance away from the source. The velocity will be zero at point 2. [Refer to equation (5.39)] Applying Bernoulli's equation, we get Pt Po gy gq (PAB) ps 28 pg Pg or (pp) =-= Bat fom enusion(S.3), 4, = 52 (6) 2) \ ar, In the above equation, p and q are constants. ‘The above equation shows that the pressure is inversely proportional to the square of the radius from the source. (5.42) > 5.15 SINK FLOW The sink flow is the flow in which fluid moves radially inwards towards a point where it disappears at a constant rate. This flow is just opposite t0 the source flow. Fig. 5.36 shows 4 sink flow in which the fluid moves radially inwards towards Point O, where it disappears at a constant rate. The pattern of stream lines and equipotential lines of a sink flow is the same as that of a source flow. All the equations derived for a source Sink ra flow shall hold to good for sink flow also except that in sink flow equations, q is to be replaced by (~ q). Problem 5.33 Plot the stream lines for a uniform flow of (5 m/s parallel to the positive direction of the x-axis and Fig.5.36 Sink flow (ii) 10 ms parallel to the positive direction of the y-axis. (Flow toward centre) Solution. (i) The stream function for a uniform flow parallel to the positive direction of the avaxis is given by equation (5.31) as w=Uxy The above equation shows that stream lines are straight lines parallel to the x-axis at a distance y from the x-axis. Here U = 5 mis and hence above equation becomes as Kinematics of Flow and Ideal Flow 217 w= sy For y=0, stream function y= 0 For y= 02, stream function y= 5 %0.2=1 unit For y= 044, stream function y= 5 x0.4 = 2 unit The other values of stream function can be obtained by substituting the different values of y. The stream lines are horizontal as shown in Fig. 5.36 (a). ¥ 0g; + y=08 06 ee y=08 04, + y=0a 02} + y=02 ° yo Fig. 8.36 (a) (Gi The stream function fora uniform flow parallel tothe positive direction ofthe y-axis is given by equation (5.37) as, wo-Uxx The above equation shows that stream lines are straight lines parallel to the y-axis at a distance x from the y-axis. Here U = 10 mvs and hence the above equation becomes as ye-l0xx ‘The negative sign shows that the stream lines are in the downward direction. For x=0, — the stream function y= 0 For x= 0.1, the stream function y =~ 10 x 0.1 =~ 1.0 unit For x= 0.2, the stream function y =~ 10x 0.2 =~ 2.0 unit For x= 0.3, the stream function y =~ 10 x 0.3 =~ 3.0 unit ‘The other values of stream function can be obtained by substituting the different values of x. The stream lines are vertical as shown in Fig. 5.36 (b).. 0 01 02 03 Fig. 5.36 (b) [218 Fluid Mechanics ] Problem 5.34 Determine the velocity of flow at radii of 0.2 m, 0.4 m and 0.8 m, when the water is flowing radially outward in a horizontal plane from a source at a strength of 12 m/s. Solution. Given : Strength of source, q = 12 m’is ‘The radial velocity wat any radius r is given by equation (5.39) as, sz nr When r= 0.2 m, Z 9.55 m/s. Ans. In x02 When r= 04 m, = 4.77 mis. Ans. Inxoa When r= 0.8 m, a 2.38 m/s. Ans. 2m x08 Problem §.35 Two discs are placed in a horizontal plane, one over the other. The water enters at the centre of the lower disc and flows radially outward from a source of strength 0.628 m*/s. The pressure, at a radius 50 mm, is 200 kN/m*, Find : (i) pressure in kN/m’ at a radius of 500 mm and (ii) stream function at angles of 30° and 60° if y= 0 at 8 = 0°. Solution. Given : Source strength, q= 0.628 m/s Pressure at radius 50 mm, p, = 200 kN/m’ (@ Pressure at a radius $00 mm Let p= pressure at radius 500 mm (u,), = velocity at radius 50 mm (u,), = velocity at radius 500 mm The radial velocity at any radius r is given by equation (5.39) as 100 x 10° Nim? When r= 50mm =0.05m, (u,), = 288 = 1.998 mis = 2 mis 2m x005 When r= 500mm =0.5m, (u,),= 28 = 0.2 mis 2x05 Applying Bernoulli's equation at radius 0.05 m and at radius 0.5 m, or Kinematics of Flow and Ideal Flow 219 or or +002 or = 202 - 0.02 = 201.98 1000 P:= 201.98 x 1000 Nim? = 201.98 kNim?. Ans. (Gd Stream functions at @= 30° and @= 60° For the source flow, the equation of stream function is given by equation (5.40) as w= £0, where 0 is in radians 2n 0.628 30x 30xn ) When @ = 30°, oe x radians Ve oe 180 80 = 0.0523 m?is. Ans. 0.628 60m 2 Se gp = 01046 ms, Ans. When @ = 60°, v > 5.16 FREE-VORTEX FLOW Free-vortex flow is a circulatory flow of a fluid such that its stream lines are concentric circles. For a free-vortex flow, ug x r = constant (say C) Also, circulation around a stream line of an irrotation vortex is T= 2nr x ug=2nxC where ug = tangential velocity at any radius r from the centre, et O° ar ‘The circulation T’is taken positive if the free vortex is anticlockwise. For a free-vortex flow, the velocity components are 1rXup= ©) Equation of Stream Function By definition, the stream function is given by ay ar In case of free-vortex flow, the radial velocity (1, is zero. Hence equation of stream function will be obtained from tangential velocity, ug, The value of uy is given by r nr Equating the two values of ug, we get Ug = and [See equation (5.124)] up [220 Fluid Mechanics ] a or 2nr 2nr Integrating the above equation, we get or (+ Eisacamanr em) 65) B ‘The above equation shows that stream function is a function of rn Foragionslocof ects icon constant Hone the stream lines are concentric circles as shown in Fig. 5.37. ‘Stream yea af poco foto By dees th poi me tucton Ben nae EB ant 238 Sc quien 3:94) Beet aloe ana eee oterecients: poeatl ete? 7 Qnr ie 5 function wil be obtained from up. TEquating the two value of up We get 13 _ 7 r r +O “ Ie On Integrating the above equation, we get r r = fla or o=Efa Jo= Sas 2 ‘The above equation shows that velocity potential function isa function of @. For a given value of @, potential function is a constant. Hence equipotential lines are radial as shown in Fig. 5.38. Velocty potential tines 6.44) Fig. 5.38 Potential lines are radial. Kinematics of Flow and Ideal Flow 221 > 5.17 SUPER-IMPOSED FLOW ‘The flow patterns due to uniform flow, a source flow, a sink flow and a free vortex flow can be super-imposed in any linear combination to get a resultant flow which closely resembles the flow around bodies. The resultant flow will still be potential and ideal. The following are the important super-imposed flow : (® Source and sink pair (ii) Doublet (special case of source and sink combination) (ii), A plane source in a uniform flow (flow past a half body) iv) A source and sink pair in a uniform flow (©) A doublet in a uniform flow. 5.17.1 Source and Sink Pair. Fig. 5.39 shows a source and a sink of strength q and (~ q) placed at A and B respectively at equal distance from the point O on the x-axis. Thus the source and sink are placed symmetrically on the x-axis. The source of strength q is placed at A and sink of strength (— q) is placed at B. The combination of the source and the sink would result in a flownet where stream lines will be circular ares starting from point A and ending at point B as shown in Fig. 540. o ‘ 3 7 (; Miss ca i. _. es fete a os = - Fig. 5A Stream lines fr sourcesink pair. Equation of stream function and potential function Let P be any point in the resultant flownet of source and sink as shown in Fig. 5.41 [222 Fluid Mechanics Fig. 5.41 Let, @ = Cylindrical co-ordinates of point P with respect to origin O x.) = Corresponding co-ordinates of point P 171.8; = Position of point P with respect to source placed at A ‘ry, 8, = Position of point P with respect to sink placed at B = Angle subtended at P by the join of source and sink i.e., angle APB. Let us find the equation for the resultant stream function and velocity potential function. The ‘equation for stream function due to source is given by equation (5.40) as y, = 4-8 whereas due to sink itis given by these two stream function. = 24a [a= 6,—6).Iniriangle ABP, 0, + 0 + (180° 8) an =180° 3. @=0,- 6) a 5 5.45 £ (5.45) ‘The equation for potential function due to source is given by equation (5.41) as 6, pal log -r, and $e tog.ry. The equation for resultant potential function (@) will be the sum of these two potential function. OR hitb due to sink it is given as 6, 4 =a Lvog.r + ($4) 02.72 an ee ) me Kinematics of Flow and Ideal Flow 223 = E toss, lorerd= os, (8) (5.46) 2n b ‘To prove that resultant stream lines will be circular are passing through source and sink ‘The resultant stream function is given by equation (5.45) as 4.0 2n v 4 <4 @,-6) = 0, - 8, Ze OO) a ) For a given stream line y = constant. In the above equation the term rs is also constant. This ‘means that (Q) ~ 64) of angle « will also be constant for various positions of P in the plane. To satisfy this, the locus of P must be a circle with AB as chord, having its centre on y-axis, as, shown in Fig. 5.40. ‘Consider the equation (5.45) again as, (a= 6-0) or Taking tangent to both sides, we get tan (@, ~ 03) = tan (=) “ 4 Bu tan 8, = (S46) rea Substituting the values of tan 8, and tan @, in equation (), y (ea) y Gre =a) 1+ oe v(e~a)= v(eta) or or ) Fig. 5.41 (a) [224 Fluid Mechanics ] or Poa tay cot (24) a0 a a 24 y+ Day cot =) - q or ay tay ct (284) «cor (2 a a e+|reeca 224) [2cose(224)/ san The above is the equation ofa cicle* with centre on y-axis ata distance of + a cot (=) from q the origin. The ais Fie ce wil be a soe (28), q Similarly, it can be shown that the potential lines for the source-sink pair will be eccentric non- intersecting circles with their centres on the x-axis as shown in Fig. 5.41 (6). ¥ Potential ins Fig. 541 6) Potential lines for source sink pair (Potential lines are eccentric noncintersecting circles with their centres on x-axis) "The equation x7 + y? = a? is the equation of a circle with centre at origin and of radius ‘a Kinematics of Flow and Ideal Flow 225 Problem 5.36 A source and a sink of strength 4 m°/s and 8 m*/s are located at (~ 1, 0) and (1, 0) respectively. Determine the velocity and stream function at a point P (1, 1) which is lying on the ‘lownet of the resultant stream line. Solution, Given : Source strength, q=4 mis Sink strength, q= 8 ms Distance of the source and sink from origin, a = 1 unit ‘The position of the source, sink and point P in the flow field is shown in Fig. 5.42. From Fig. 5.42, itis clear that angle 0, will be 90° and angle 8, can be calculated from right angled triangle ABP. ‘The equation for stream function due to source is given by equation (5.40) as y, = 1 x = 4 x8, rd whereas due to sink it is given by yp ‘The resultant stream function w is given as voy + ve eli) Let us find the values of 8, and 8, in radians. From the geometry, itis clear that the traingle ABP is a right angled tiangle with angle 0, = 90° = 20. xm = radians 180 *"" 2 BP LA ‘Also an, = FES or Substituting these values in equation (i), a. a = 4 x 0463 - 2x v Qn Qn 2 [226 Fluid Mechanics ] m’Js, q> = 8 m’/s) k 8 == xo.463-& an 048 3E* 9 = 0.294 ~ 2 106 m/s. Ans. To find the velocity at the point P, let us first find the stream function in terms of x and y co- ordinates. The stream function in terms of ®, and @, is given by equation (i) above as = 1X _ bx On Oe The values of @, and @, in terms of x, y and a are given by equation (5.464) as tan 8, or a The velocity component 2m (x= a) +y ‘At the point P(1, 1), the component 1 is obtained by subs equation. The value of a is also equal to one. the above ituting x = 1 and y (-1) Kinematics of Flow and Ideal Flow 227 Now a (xtay 2m (x+a) ty oh» Qn (xtay + At the point P(L, 1). ye yt ___ ——— Qn (i+ly +P 2e (=I) +P Myla) 2n5 201 ai 4.1 8 2 fxt-Bet xl-F 20072-1272 =~ 1.14 Pox SEA go dg = 01272 ~ 1272 = ~ 1.145 mis ‘The resultant velocity, V= fu? +v* = 0.2544? +(-1145)" = 1.174 mis. Ans. Problem 8.37 For the above problem, determine the pressure at P(1, 1) ifthe pressure at infinity is zero and density of fluid is 1000 kg/m’ Solution, Pressure at infinity, pp Density of fluid, = 1000 kg/m® The velocity* of fluid at infinity will be zero. If Vp = velocity a infinity, then Vo = 0. The resultant velocity of fluid at PCL, 1) = 1.174 ins (calculated above) or Ve 1174 ms. Let p = pressure at P(1, 1) Applying Bernoulli's theorem at point at infinity and at point P, we get Fo,Mo PV pg 2g pe or o+o-2 or V= 1.174 mis) * From equation (5.39), the velocity ata distance ‘from source or sink is given by u,= 4. At infinity, ris very very large hence velocity is zero. [228 Fluid Mechanics __ 1174? 1000 ~ 2 5.17.2 Doublet. It isa special case of a source and sink pair (both of them are of equal strength) ‘when the two approach each other in such a way that the distance 2a between them approaches zero and the product 2a . q remains constant. This product 2a . q is known as doublet strength and is denoted by 1. Doublet strength, w= 2a.g (5.48) Let q and (~ q) may be the strength of the source and the sink respectively as shown in Fig. 5.43. Let 2a be the distance betweem them and P be any point in the combined field of source and sink. or 689.14 Nim’, Ans. Fig. 543, Let @ is the angle made by P at A whereas (@ + 66) is the angle at B. Now the stream function at P, 4 fe (3.49) From B, draw BC Jon AP. Let AC = 8r, CP = rand AP = r+ Br. Also angle BPC = 88. The angle {80 is very small. The distance BC can be taken equal to r x 88. In triangle ABC, angle BCA = 90° and hence distance BC is also equal to 2a . sin @, Equating the two values of BC, we get x 80 = 2a. sin @ 2a.sin® 0 Substituting the value of 69 in equation (5.49), we get _ 4, 2asin® nr A on In Fig. 5.43, when 2a — 0, the angle 88 subtended by point P with A and B becomes very small. Also dr — 0 and AP becomes equal to r. Then v [ie 2a. q = from equation (5.48)] ..(5.50) Subst ting the value of sin @ in equation (5.50), we get Kinematics of Flow and Ideal Flow 229 hy Pavey) referer) 6 » (5.504) or The above equation can be written as Fever gie(h) (a) <0 [sine an sabre (2) any * (any, any or e+(+4) (4) (5.51) H The above is the equation of a circle with centre {0,=4—) and radius 4. The centre of the au es ce (0 i) aati Pte ee Circle lies on y-axis ata distance of —— from x-axis. As the radius ofthe circle is also equal to —#—, any 4m hence the citcle will be tangent to the x-axis. Hence stream lines of the doublet will be the family of circles tangent to the x-axis as shown in Fig. 5.44, Stream ines are circles ‘tangent to x-axis with conte on y-axis ‘Stream ines y toes Fig. 5.44 Stream lines for a doublet. Potential function at P Refer to Fig. 5.43. The potential function at P is given by o= Log r+8n+ (- 4 og.r [Refer to equation (5.41)] on 2x [230 Fluid Mechanics = Litog, (r+ 6)- Log r= tog, (78) = to ( a = Ltos.(r+ an £ toe. to, (: =e. 1b : o£ |&+(%) xb... =£.% _[neSEisa sma qumiy.ten( 5) tems ns] Dot in gS. om wine 4, we et = cs ‘. br = 2a cos @ Substituting the value of Br, we get a Hy 24.0088 Qn 038 x nr In Fig. 5.43, when 2a — 0, the angle 68 becomes very small. Also r — 0 and AP becomes equal to r. Then [2a x q = from equation (i) ...5.52) on = 42 Also AP* = AD? + PD? ot ? = 1°+ y? Substituting the value of cos @ in equation (5.52), we get vey o 0 ‘The above equation can be writen as Fl . ets (4) (8 [ase and svat (5) ] 2n (amp) ~ (ano, ‘and, a ro) aye (LY 5.53) (a) +¥=(5) (3.53) ‘The above is the equation of a circle with centre (40) and radius {The centre of the ano ano u circle lies on x-axis at a distance of 4 from y-axis. As the radius of the circle is equal to the distance of the centre of the circle from the y-axis, hence the circle will be tangent to the y-axis. * Expansion of log, (1 +x)=x+ = ‘pa gL ta) xt Kinematics of Flow and Ideal Flow 231 Hence the potential lines of a doublet will be a family of circles tangent to the y-axis with their centres oon the x-axis as shown in Fig. 5.45, Porn! nes oF ines ae circles vith centre on axis buttangont to y-axis Potential ines Fig. 5.45 Potential lines for a doublet. Problem 5.38 A point P(0.5, 1) is situated in the flow field of a doublet of strength 5 m/s. Calculate the velocity at this point and also the value of the stream function. Solution. Given : Point P(0.5, 1). This means x = 0.5 and ‘Strength of doublet, B= Sms (®) Velocity at point P ‘The velocity at the given point can be obtained if we know the stream function (y). But stream function is given by equation (5.504) a8 H. y 2 (ey) ‘The velocity components w and v are obtained from the stream function as 10 we- and [232 Fluid Mechanics ] “keel eleoy ‘Substituting the values of p= 5 m7/s, x = 0.5 and y = 1.0, wet), (os? +17) Resultant velocity, V= yu? +? = y(-0382)' +(-0.509)" = 0.636 mvs. Ans. (i Value of stream function at point P , we get the velocity components as and 2n Solution in polar co: The above question can also be done in r, 0 (ie, polar) co-ordinates. The stream function in r, 8 co-ordinates is given by equation (5.50) as eli) [+ isa contanttxm and also sconstant v.40] 2 1 =r cos 0 ii) zal Eee and. = arl on 8 isa constant wor.t.r (iti) Kinematics of Flow and Ideal Flow 233 Now rex? ty? = 0s? +P =Vi25 : sin @= 2=—1_ = 0,894 and cos 6 =~ ris r Substituting the values ofr, sin @ and eos 6 in above equations (i, (i) and (ii), we get ht sin@_ 5 0894 - x TEE = - 0.636 mis. Ans. Woon rom Vis x05 @=- x1 x 0,447 = - 0.2845 mis 2n ” (125) and. Resultant velocity, 0569)? = 0.636 m/s. Ans. 5.17.3 A Plane Source in a Uniform Flow (Flow Past a Half-Body). Fig. 5.46 (a) shows uniform flow of velocity U and Fig. 5.46 (b) shows a source flow of strength q. When this uniform flow is flowing over the source flow, a resultant flow will be obtained as shown in Fig. 5.46. This resultant flow is also known as the flow past a half-body. Let the source is placed on the origin O. Consider a point P(x, y) lying in the resultant flow field with polar co-ordinates and @ as shown in Fig. 5.46. y @) (2) Unitorm fow (©) Source fow Hatfbody Point Px. y) Fig. 5.46 Flow pattern resulting from the combination of a uniform flow and a source. [234 Fluid Mechanics ] ‘The stream function (y) and potential function (6) for the resultant flow are obtained as given below : \y = Stream function due to uniform flow + stream funetion due to source su.y+ Lo (5.54) 2n =U. rsind+ £0 y= sin @) (5.54) 2x and = Velocity potential function due to uniform flow + Velocity potential function due to source 4 5s. =U.xt A ogy =U .rcos 0+ 4 tog,r 5.548) 2 Be 2m ee (S.54B) ‘The following are the important points for the resultant flow pattern : (® Stagnation point. On the left side of the source, at the point lying on the x-axis, the velocity of uniform flow and that due to source are equal and opposite to each other. Hence the net velocity of the combined flow field is zero. This point is known as stagnation point and is denoted by S. The polar co- ordinates of the stagnation point S are rs and , where rs is radial distance of point S from 0. ‘The net velocity (or resultant velocity) is zero at the stagnation point S. 1 a1? fore 2°) [ =U.rsino+ £6] Para (Urano 2n = {[urcosoe £]=v.cos 0+ r 2 ar ‘At the stagnation point, ® = m radians (180°) and r ‘sand net velocity is zero. This means u, = 0 and vp = 0. Substituting these values in the above equation, we get 0=U.. cos 180° + 4 [> u,=0, 8 = 180° and r= re} 2s =-U+— on er 4 or neo (5.55) Inv cane From the above equation it is clear that position of stagnation point depends upon the free stream velocity U and source strength q. At the stagnation point, the value of stream function is obtained from equation (5.544) as 4 =U.rsinos 1.0 V Qn For the stagnation point, the above equation becomes as u.r,sin 190° + 1 x8 2n [At stagnation point, @ = x radians = 180° and r = rs] so+te4 2° 2 ‘The above relation gives the equation of stream line passing through stagnation point. We know that no fluid mass crosses a stream line. Hence a stream line is a virtual solid surface. (5.56) Kinematics of Flow and Ideal Flow 235 (i Shape of resultant flow. At the stagnation point S, the net velocity is zero. The fluid particles that issue from the source cannot proceed further to the left of stagnation point. They are carried along the contour BSB’ that separates the source flow from uniform flow. The curve BSB’ can be regarded as the solid boundary of a round nosed body such as a bridge pier around which the uniform flow is forced to pass. The contour BSB’ is called the half body, because it has only the leading point, it trails to infinity at down stream end. ‘The value of stream function of the stream line passing through stagnation point S and pas 4 the solid boundary (i.e., curve BSB’) is Ws = ‘Thus the composite flow consists of : (1) flow over a plane half-body (i.e, flow over curve BSB’) outside y (2) source flow within the plane half-body. The plane half-body is deseribed by the dividing steam line, y = 4. But the stream function at any point in the combined flow field is given by equation (5.54) as, 4 =U.y+ Le weve oe If we take y= 4 in the above equation, we will get the equation of the dividing stream line. 2 ++ Equation of the dividing stream line (i.e., equation of curve BSB’) will be or (3.57) From te sbove equation, the main dimcasons of he plate aly maybe obaned. rom hs equation, iis ea thay maximum, hen 8 =O Hence A19=0, IRAN ag = the max ode the ordinate above the origin the leading point of the half-body =, y = the ordinate below the origin. ‘The main dimensions are shown in Fig. 5.47. (iii) Resultant velocity at any point ‘The velocity components at any point in the flow field are given by 1-1 dl y.rsino+ Lo] “7 ae de Qn [236 Fluid Mechanics ] [verso =U.c0s 0+ 2 . 2nr Ordinate above the origin Plane halt-body Fig.5.47 ‘The above equation gives the radial velocity at any point in the flow field. This radial velocity is due {© uniform flow and due to souree. Due to source the radial velocity is =4. Hence the velocity due nr fance from the to source diminishes with increase in radial distance from the source. At large di source the contribution of source is negligible and hence free stream uniform flow is not influenced by the presence of source. W.-2ferinee Lo] Oe Oe 2 [U. sin 8 + 0] =-U sin @ Resultant velocity, V= i tug (iv) Location of stagnation point At the stagnation point, the velocity components are zero. Hence equating the ri velocity components to zero, we get 4,20 or Ucos0+—L =0 or Ucos nr But rcos or reos @=- 4 2nu 4q 2a 0 or sin@ rsin 8 y ‘as U cannot be zero Kinematics of Flow and Ideal Flow _237 (0) Pressure at any point in flow field Let pp pressure at infinity where velocity is U ’P= pressure at any point P in the flow field, where velocity is V Now applying the Bemouli's equation at a poin at infinity and ata point P inthe flow field, we get rove gy Vp po p- Pe pe 2e pe 2e 2g 28 PR Pk pe The pressure co-efficient is defined as (5.58) Problem 5.39 A uniform flow with a velocity of 3 m/s is flowing over a plane source of strength 30 m*/s. The uniform flow and source flow are in the same plane. A point P is situated in the flow ‘field. The distance of the point P from the source is 0.5 m and itis at an angle of 30° to the uniform ‘low. Determine : (i) stream function at point P, (ii) resultant velocity of flow at P and (iii) location of stagnation point from the source. Solution, Given : Uniform velocity, U = 3 m/s ; source strength, q = 30 m’/s ; co-ordinates of point P are r= 0.5 m and @ = 30°. (0 Stream function at point P The stream function at any point in the combined flow field is given by equation (5.544) 4 =U. rsinos Le V Qn a 159m = Fig. 548, ‘Stream function at point P, ve anossin a 2x( 23) ain [238 Fluid Mechanics ] = 015425 = .25 mis. Ans. (ii Resultant velocity at P The velocity components anywhere in the flow are given by Low 14 q W148 ly sins Lo 708738 “OR = [u.rcose+]=u.cose + 2. 2nr 30 2exO5 2.14 = 3 xcos 30° + » AUP, r= 05,0 = 30°, q = 30) = 2.598 49.55 Tunes Lol Bean jUrsnos ol Usin®+0=-Usin@ =~ 3xsin 30° =- 1.5 = 124? +(-15)° = 12.24 mvs. Ans. (iii) Location of stagnation point The horizontal distance of the stagnation point S from the source is given by equation (5.55) as aoe aU 2Rxd The stagnation point will be at a distance of 1.59 m to the left side of the source on the x-axis Problem 5.40 A uniform flow with a velocity of 20 m/s is flowing over a source of strength 10 m’/s. The uniform flow and source flow are in the same plane. Obtain the equation of the dividing stream line and skeich the flow pattern Solution. Given : Uniform velocity, U = 20 m/s ; Source strength, q = 10 m’/s (i) Equation of the dividing stream line ‘The stream function at any point in the combined flow field is given by equation (5.544) =U.rsino+ v=U.rsind+ 1 0 and. ug= Resultant velocity, V 1.59 m. Ans. ts =2xrsino+ 29 2x U = 20 mis and q = 10 mss) ‘The value of the stream function for the dividing stream line is y= 4. Hence substituting y = the above equation, we get the equation of the dividing stream line. 4 2 or sin d+ 12.9 2x 10 or = 200 sin 0+ 5 Cr q= 10) Kinematics of Flow and Ideal Flow 239 or 5=20rsin 0+ 12 6= 207+ 26 G rsin@=y) 2n an 10 W0y=5-—O Po Oe 6 6 =o25- 2 Ai or s 2 “a ‘The above relation gives the equation of the dividing stream line. From the above equation, for different values of 8 the value of y is obtained as Value of © | Value of v from (i Remarks 0 025 m ‘Max. half width of body z 0.125 m ‘The +ve ordinate above the origin ® 0 ‘The leading point * = 0.125 m ‘The ~ve ordinate below the origin 2m. = 025 m ‘The max. ~ve ondinate (i) Sketch of fiow pattern For sketching the flow pattern, let us first find the location of the stagnation point, The horizontal distance of the stagnation point S from the source is given by the equation, q __ 0 2nU 2nx20 Hence the stagnation point lies on the x-axis at a distance of 0.0795 m or 79.5 mm from the source towards left of the source. The flow pattern is shown in Fig. 5.49. _————————— ee 0.0795 m 735 mm Fig. 5.49 Problem 5.41 A uniform flow with a velocity of 2 m/s is flowing over a source placed at the origin. The stagnation point occurs at (~ 0.398, 0). Determine : (i) Strength of the source, (ii) Maximum width of Rankine half-body and (iti) Other principal dimensions of the Rankine half-body. Solution, Given : Uniform velocity, U=2mis [240 Fluid Mechanics ] Co-ordinates of stagnation point = (~ 0.398, 0) ‘This means r, = 0.398 and stagnation point lies on x-axis at a distance of 0.398 m towards left of, origin, The source is placed at origin. (O Strength of the source Let q = strength of the source In equation (5.70), r= R} and |x @=-2U sin® (5.73) —ve sign shows the clockwise direction of tangential velocity at that point. The value of ug is maximum, when 8 = 90° and 270°, ‘At 8 = 0° or 180°, the value of up = 0. Hence on the surface of the cylinder, the resultant velocity is zero, when @ = 0° or 180°. These two points on the surface of cylinder [i-e., at © = 0° and 180°] where resultant velocity is zero, are known as stagnation points. They are denoted by 5, and Sy. Stagnation point S, corresponds to @ = 180° and S; corresponds to = 0°. Pressure distribution on the surface of the cylinder Let pg = pressure at a point in the uniform flow far away from the cylinder and towards the left of the cylinder {ie., approaching uniform flow] U = velocity of uniform flow at that point P= pressure at a point on the surface of the cylinder V = resultant velocity at that point on the surface of the cylinder. This velocity is equal to lug as u, is zero on the surface of the cylinder. Kinematics of Flow and Ideal Flow 249 x V= y= 2U sin @ Applying Bemouli's equation atthe above two points, ve 24 [-2Usin ef! or [Ve ug=~2U sin 0] Lip 2 or Sut asin?) 5% ) or foe 3 Pu’ But £—Pe js a dimensionless term and is known as dimensionless pressure co-efficient and is Tou? 3° denoted by C,. fF 1 -4sin?@ Toy? 30" Value of pressure co-efficient for different values of 6 Value of 8 Value of ° 1-4sin?@ =1-0=1 30° 14 sin® 30° 90° 14 sin? 90° = 130° 1-4 sin® 150° 180° 1-4 sin? 180° = 1-0=1 At ©= 0 and 180°, there are stagnation points) and 5; respectively ‘At @= 30° and 150°, the pressure co-efficient is zero. At @= 90°, the pressure co-efficient is ~ 3 (ke. least pressure) ‘The variation of pressure co-efficient along the surface of the cylinder for different values of @ are shown in Fig. 5.54, [250 Fluid Mechanics ‘The positive pressure is acting normal to the surface and towards the surface of the cylinder whereas the negative pressure is acting normal to the surface and away from the surface of the cylinder as shown in Fig. 5.55. pressure pressure Fig. 5.55 Problem 5.43 A uniform flow of 12 més is flowing over a doublet of strength 18 m*/s. The doublet is in the line of the uniform flow. Determine : (i) shape of the Rankine oval (ii) radius of the Rankine circle (iti) value of stream line function at Rankine circle (iv) resultant velocity at a point on the Rankine circle at an angle of 30° from x-axis (v) value of maximum velocity on the Rankine circle and location of the point where velocity is max. Solution. Given : U= 12 mis; w= 18 mils (O Shape of the Rankine oval ‘When a uniform flow is flowing over a doublet and doublet and uniform flow are in line, then the shape of the Rankine oval will be a circle of radiu Kinematics of Flow and Ideal Flow 251 (i Radius of the Rankine circle i 8 2nU ~ \2nx12 (ii) Value of stream line function at the Rankine circle ‘The value of stream line function (¥y) at the Rankine circle is zero i.e., w= 0. (iv) Resultant velocity on the surface of the circle, when @ = 30° On the surface of the eylinder, the radial velocity (u,) is zero, The tangential velocity (ug) is given by equation (5.73) as Rer= 2U sin @ =~ 2x 12 x sin 30° = - 12 mvs. Ans, ve sign shows the clockwise direction of tangential velocity at that point Resultant velocity, V= qu tug = yO? +(-12) = 12 mvs. Ans. (0) Maximum velocity and its location ‘The resultant velocity at any point on the surface of the cylinder is equal to wg. But ug is given by, ug =~ 2U sin @ This velocity will be maximum, when @ = 90°. Max. velocity =-2U=-2*12=-24 mis, Ans. Problem 5.44 A uniform flow of 10 més is flowing over a doublet of strength 15 m’/s. The doublet is inthe line of the uniform flow. The polar co-ordinates of a point P in the flow field are 0.9 m and 30°. Find : (i) stream line function and (i) the resultant velocity at the point. Solution. Given : U = 15 mis; r= 0.9 m and Let us frst find the radius (R) of the Rankine circle. This is given by R= BS} 0.488 m Vand” \2mxi0 ‘The polar co-ordinates of the point P are 0.9 m and 30°. Hence r= 0.9 mand @ = 30°. AS the value of r is more than the radius of the Rankine circle, hence point P lies outside the cylinder. (® Value of stream line function at the point P ‘The stream line function for the composite flow at any point is given by equation (5.69) as P(0.9 m, 20°) 252. Fluid Mechanics v-u(-2)ane as } sin 30°C r= 0.9 m, R = 0.488 and @ = 30°) =10 (09- = 10(0.9 - 0.2646) x ; = 3177 mils, Ans. (ii) Resultant velocity at the point P ‘The radial velocity and tangential velocity at any point in the flow field are given by equations (5.70) and (5.71) respectively. : wnt (i-8) eee +4ve sign shows the radial velocity is outward. R and menu (ie sin fe Sign shows the clockwise direction of tangential velocity. Resultant velocity, era oir? + (647) = 373344086 = 8.89 m/s. Ans. v Kinematics of Flow and Ideal Flow 253 2 % 10, te 2 13. rs 16. 17. ‘Continuity equation in differentiat form, ea = = 0 for three-dimensional flow 0 for two-dimensional flow. a ‘The components of acceleration in x, y and z direction are au, du twee av av ‘The components of velocity in 2, y and z direction in terms of velocity potential (6) are pad 2% snd ay ar ‘Te steam function (y) is defined only for two-dimensional ow. The velocity components ins and 2 ag y= lreetins in temas of team fanetion are u =~ 2% and v= 2Y. . ay ar Angular deformation or shear sin rate is given as strain rate = 2 | 4 2H serena 2-2] Rotational components of a fluid ps oy i Vorticity is two times the value of roution Flow ofa fluid along a curved path is known as vortex flow. Ifthe particles are moving round in curved path with the help of some extemal torque the flow is ealled forced vortex flow. And if no extemal torque is required to rotate the Muid particles, the flow is called ftee-vortex flow. ‘The relation between tangential velocity and radius: for forced vortex, y= Ox, for free vortex, r= constant. ‘The pressre variation along the radial direction for vortex flow along a horizontal plane, 22. ar a ‘and pressure variation in the vertical plane 22 = — pe, a For the forced vortex flow, Z= —— ee 2g 2e le where Z= height of paraboloid formed = angular velocity, 254 Fluid Mechanics EXERCISE (A) THEORETICAL PROBLEMS 4, What are the methods of describing fluid flow ? 2. Explain the terms : (@ Path line, (di) Streak line, iif) Stream line, and (iv) Stream tube, Distinguish between (@ Steady flow and un-steady flow, (i) Uniform and non- (iii) Compressible and incompressible flow, (Gv) Rotational and irrotational flow, (v) Laminar and turbulent flow. 4. Define the following and give one practical example for each 3m flow, (@) Laminar flow, (Gi) Turbulent flow, (iid) Steady flow, and (i) Uniform flow. 5. Define the equation of continuity. Obtain an expression for continuity equation for a three-dimensional flow. (R.G.P.V, S 2002) 16 What do you understand by the terms ; () Total acceleration, (ii) Convective acceleration, and (ii) Local acceleration ? (Delhi University, Dec. 2002) 7. (a) Define the terms = (i) Velocity potential function, and (i) Stream function. (®) What are the conditions for flow to be inrotational ? '8 What do you mean by equipotential line and a line of constant stream function ? 9 (a) Deseribe the use and limitations of the flow nels. (©) Under what conditions can one draw flow net ? Define the terms : (Vortex ow, i) Forced vortex flow, and (ii) Free vortex flow. LL. Differentiate between forced vortex and free vortex flow. Kinematics of Flow and Ideal Flow 255 12, Derive an expression for the depth of paraboloid formed by the surface of a liguid contained in a cylindrical tank which ig rotated at a constant angular velocity «@ about its vertical axis. 13, Derive an expression for the difference of pressure between two points ina free vortex flow. Does the Gifference of pressure satisty Bernoulli's equation ” Can Bernoulli's equation be applied to a forced vortex flow ? 14, Derive, from first principles, the condition for irrotatonal flow. Prove that, for potential flow, both the stream function and velocity potential function satisfy the Laplace equation, 15, Define velocity potential function and stream function 16. Under what conditions can one treat real fluid flow as irotational (as an approximation) 17. Define the following (0 Steady flow, (i) Non-uniform flow, (iid) Laminar flow, and (iv) Two-dimensional flow 18, (@) Distinguish between rotational flow and irrotational flow. Give one example of each (®) Cite two examples of unsteady, non-uniform flow. How can the unsteady flow be transformed to steady flow ? (LNT. University, § 2002) 19. Explain uniform flow with source and sink. Obtain expressions for stream and velocity potential funetions, 20. A point source is a point where an incompressible fluid is imagined to be created and sent out evenly in all dictions. Determine its velocity potential and stream function, 21. (@ Explain doublet and define the strength of the doublet Gi) Distinguish between a source and a sink. 22, Sketch the flow pater of an ideal fluid flow past a eylinder with cireulation. 23. Show that in case of forced vortex flow, the rse of liquid level at the ends is equal to the fall of liquid level atthe axis of rotation. 24, Differentiate between (@ Stream function and velocity potential function (ii) Stream line and streak tine and id) Rotational and irrotational flows, (B) NUMERICAL PROBLEMS 1. The diameters of a pipe atthe sections I and 2 are 15 em and 20 em respectively, Find the discharge trough the pipe if Velocity of water at section I is 4 mvs, Determine also the velocity at section 2. [Ans. 0.07068 m*/s, 2.25 m/s} 2. 40 cm diameter pipe, conveying wate, branches into two pipes of diameters 30 em and 20 em respectively. I the average velocity inthe 40 cm diameter pipe is 3 nus. Find the discharge inthis pipe ‘Also determine the velocity in 20 em pipe ifthe average velocity in 30 em dlameter pipe is 2 m/s. (Ans. 0.3769 m's, 7.5 ms) 3. A.30 cm diameter pipe caries ol of sp. gr 0.8 a a velocity of 2 m/s, AL another section the diameter is 20 ci, Find the velocity at this section and also mass rate of low of oi, (Ans. 4.5 mf, 113 kgA] 4. The velocity vector ina fluid flow is given by V= 2x} — Sy) +41, Find the velocity and acceleration ofa fluid partite at (1, 2, 3) at time, = 1. [Ans 10:95 units, 16.12 units} 5. The following cases represent the 1wo velocity components determine the third component of velocity such that they satisfy the continuity equation [256 Fluid Mechanics ] Te 12. 1. 4 15. 16. 7 Ou = day (i) w= 4x + Bay, wae day — 292. try 4392 4705.2] [Ans.() w=—8x2— 227 +059 Ci) Calculate the unknown velocity components so that they sais the following equations @ ua2e,va2e, wa? Gun2e 42a, weed diet 2;e,0—2 [Ams (w= Lee Z] Prove that itis a case of possible steady incompressible Aid flow. Calculate the velocity and acceleration at the point [1 2, 3} [Ans. 36.7 units, 874.50 units} Find the convective acceleration at the middle of a pipe which converges uniformly from 0.6 m diameter t0 0.3 m diameter over 3 m length. The rate of flow is 40 lis. I the rate of flow changes uniformly from 40 livs to 80 livs in 40 seconds, find the total acceleration at the middle of the pipe at 20th second. [Ans 0499 mus; 11874 mi ‘The velocity potential function, 6, is given by § = .x°— y7. Find the velocity components in x and y direction. Also show that 6 represents a possible case of fluid flow. (Ans. u = 2x and v=~2)) . For the velocity potential function, @= 1° — y* find the velocity components atthe point (4, 5) [Ans. v= 8, =~ 10 units] A stream function is given by = y = 2x ~ Sy. Calculate the velocity components and also magnitude and ireotion of the resultant velocity at any point. [Ans. w= 5, v= 2, Resultant = 5.384 and 8 = 21° 48) If for a two-dimensional potential flow, the velocity potential is given by : = 4x(3y — 4), determine the velocity at the point 2, 3). Determine also the value of stream funetion y at the point (2, 3). [ms 40 ons ‘The stream function for a two-dimensional ow is given by y = Bay, calculate the velocity atthe point (4,5). Find the velocity potential function @. [Ans =—32 units, v= 40 units, = 4y? 4x7] ‘Sketch the stream lines represented by y =.xy. Also find out the velocity and is direction at point (2,3). [Ans. 3.60 units and 6 = 56° 18.6’ or 123° 42’) For the velocity components given as :u = ay sin xy, v= ax sin ay. Obtain an expression for the velocity potential function Ans. ¢ [A fhid flow is given by : V= 102% — By). Find the shear strain rte and state whether the flow is rotational or irvotational. (Ans. ~ 8x, rotational] cos xy] The velocity components in a two-dimensional flow are w=sey-8 Pandye—ary' + 8 x v8 ay s8 Show that these velocity components represent @ possible ease of an irrottional flow. Bu, av Ans. ‘An open circular cylinder of 20 em diameter and 100 em long contains water upto a height of 80 em. It is rotated about its vertical axis. Find the speed of rotation when : ( no water spills, (i axial depth is zero. (Ans. ()) 267.51 rpm. (i) 422.98 rpm] A cylindrical vessel 15 em in diameter and 40 em long is completely filled with water. The vessel is open atthe top. Find the quantity of water left in the vessel, when it is rotated about its vertical axis with a speed of 300 rpm. [Ans. 4566.3 em?) Kinematics of Flow and Ideal Flow _ 257 19. An open circular cylinder of 20 em diameter and 120 cm long contains water upto a height of 80 cu. Tt js rotated about its vertical ais at 400 rpm. Find the difference in total pressure force (atthe bottom, of the eylinder, and (i) atthe sides ofthe eylinder due to rotation. (Ans. (i) 14.52 N, (i) 2465.45 N} 20, A closed cylindrical vessel of diameter 15 cm and length 100 cm contains water upto a height of 80 em, The vessel is rotated ata speed of $00 rpm, about its vertical axis. Find the height of paraboloid formes. TAs. 56.06 em] 21. For the data given in question 20, find the speed of rotation of the vessel, when axial depth is zero. (Ans. 891.7 pm] 22. Ifthe cylindrical vessel of question 20, is rotated at 980 r-pam. about its vertical axis, find the area uncovered atthe base of the tank Ans. 20.4 em] 23. A closed cylindrical vese! of diameter 20 om and height 100 cm contains water upto a height of 70 em. The air above the water surace is ata pressure of 78.48 KN. The vesel is rotated a a speed of 300 rp. about its vertical axis, Find the pressure head atthe bottom of the vessel ; (a) atthe centre, and (D) at the edge [Ams (a) 8.4485 m () 89515 m] 24. A closed cylinder of diameter 30 em and height 20 em is completely filled with water. Caleulate the total pressure fore exerted by water on the top and bottom of the eylindr, if iti rotated about its vertical axis a 300 span, (Ans. Fp= 392.4 N, Fy= 531 NI 25. In free cylindrical vortex flow of water, ata point ata radius of 150 mm the velocity and pressure are ‘S mis and 14.715 Niem*. Find the pressure at a radius of 300 mm. [Ans. 15.65 Neem") 26. Do the following velocity components represent physically possible flows ? wart t5,vayt2,wednz [Ans. No] 27. State ifthe flow represented by u= 3x + 4y and v= 2x ~ 3y is rotational or itrotational, [Ans Rotational] 28. A vessel, eylindrieal in shape and closed at the top and bottom, contains water upto a height of 700 ram, The diameter of the vessel is 200 mm and length of vessel is 1.1 m, Find the speed of rolation ofthe vessel if the axial depth of water is zero 29 Define rotational and irrtational flow. The steam function and velocity potential fora flow are given ty y=20y, b= 27-y Show thatthe conditions of continuity and irrotational flow are satisfied. 30, For the steady incompressible lw, are the following Values of and v possible ? () w= dy ty ve Oxy + Brand Gi u=20 +y?,v=—day. [Ams (No, (ii) Yes} 31. Define two-dimensional stream function and velocity potential. Show tha following stream funtion y =6r-4y +749 represents an irrotational flow. Find its velocity potential. [Ans. §= 4r + 6y-3.5° +3.5y°+ C] 32. Check if 6 = 1° — y* + y represents the velocity potential for 2-limensional irrotational flow. If it does, then determine the stream function . [Ams. Yes, y =~ 2xy +x] 33. If stream function for steady flow is given by y= ()* ~ x"), determine whether the flow is rotational or inrotational, Then determine the velocity potential @, Ans. Irrotational, @=~ 2xy + C) 34. A pipe (1) 450 mm in diameter branches into two pipes (2) and (3) of diameters 300 mm and 200 mm respectively as shown in Fig, 5.57. If the average velocity in 450 mm diameter pipe is 3 m/s, find (@ discharge through 450 mm dia. pipe and (ii) velocity in 200 mm diameter pipe if the average velocity in 300 mm pipe is 2.5 m/s. (NT.U., Hyderabad, § 2002) (Hint. Given 4, =450mm=0.45 m, d;=300mm=0.3m 4, =200 mm =0.2.m, V;=3 mls, Vy=2.5 mis [258 Fluid Mechanics @ w But Also =F (045%) x3 0.477 mi, Fig.5.57 yo 925001760 1s = Q ~ Q)= 0.477 - 0.176 = 0.301 =F 02x", Qs 0.301 ag 0.0314 F023) DYNAMICS OF FLUID \ rahi 1 INTRODUCTION In the previous chapter, we studied the velocity and acceleration at a point in a fluid flow, without taking into consideration the forces causing the flow. This chapter includes the study of forces causing fluid flow. Thus dynamics of fluid flow is the study of fluid motion with the forces causing flow. The dynamic behaviour of the fluid flow is analysed by the Newton's second law of motion, which relates the acceleration with the forces. The fluid is assumed to be incompressible and non-viscous. » 6.2 EQUATIONS OF MOTION According to Newton's second law of motion, the net force F, acting on a fluid element in the dircetion of xis equal 10 mass m of the fluid element multiplied by the acceleration a, inthe x-direction. ‘Thus mathematically, Fema, 6.1) In the fluid flow, the following forces are present : (Fy gravity force. (Gi) Fy. the pressure force. (iii) F,,foree due to viscosity. (iv) Fy force due to turbulence. (») Force due 10 compressibility. “Thus in equation (6.1), the net force F,2 (Ect Fyhet Eat Pct Foe (If the force duc to compressibility, F, is negligible, the resulting net force F,2 (e+ Fyiet Ect Fs and equation of motions are called Reynold’s equations of motion. (ii) For flow, where (F,) is negligible, the resulting equations of motion are known as Navier-Stokes Equation. (iit) Ifthe flow is assumed to be ideal, viscous force (F,) is zer0 and equation of motions are known as Euler’s equation of motion, 259 [260 Fluid Mechanics » 6.3 EULER'S EQUATION OF MOTION ‘This is equation of motion in which the forces due to gravity and pressure are taken into considera- tion. This is derived by considering the motion of a fluid element along a stream-line as : Consider a stream-line in which flow is taking place in s-direction as shown in Fig. 6.1. Consider a cylindrical element of cross-section dA and length ds. The forces acting on the cylindrical element are: 1. Pressure force pdA in the direction of flow. 3. Weight of element pgdAds. Let @ is the angle between the direction of flow and the line of action of the weight of element. ‘The resultant force on the fluid element in the direction of s must be equal to the mass of fluid ‘eration in the direction s. clement : pat-(n 3a) r-patnscoss pdAds x a, 6.2) whee ashe aceleration inte destin of Now a= 2 wee a tune oa ae a vv as ar és ov ite now i seas, % =0 d vay. Lee F Substituting the value of a, in equation (6.2) and simplify- ing the equation, we get paihen a = cu @ C ~ 9g Mah ~ Pa AAAS cos O= pads x 5, Fig. 6.1 Forces on a fluid element a via Dividing by pdsda,~ 2 — g cos = B by psd — SF — 8 a a av or fe teem Oey S a0 at But fom Fig. 6.1 (6), we have cos @ = A By gh MY go Ps gaz s vt pds Sas” as P or Pe ates vivo (6.3) Equation (6.3) is known as Euler's equation of motion. [ Dynamics of Fluid Flow 261 | >» 6.4 BERNOULLI'S EQUATION FROM EULER’S EQUATION equation is obtained by integrating the Buler’s equation of motion (6.3) as dp Ps fede + J vav = constant J > Seaes f If flow is incompressible, p is constant and. 2s ges = constant Pp 2 or Peet = constant pe 2g or 24% 4 z= constant (A) pe 2g Equation (6.4) is a Bernoulli’s equation in which w = pressure energy per unit weight of fluid or pressure head. 8 v?iag = kinetic energy per unit weight or kinetic head. = potential energy per unit weight or potential head, >» 6.5 ASSUMPTIONS The following are the assumptions made in the derivation of Bernoulli's equation (i) The fuid is ideal, ie., viscosity is zero (ii) The flow is steady (iii) The flow is incompressible (iv) The flow is irotational. Problem 6.1 Water is flowing through a pipe of 5 cm diameter under a pressure of 29.43 N/em? (gauge) and with mean velocity of 2.0 ms. Find the total head or total energy per unit weight of the ‘water at a cross-section, which is Sm above the datum line Solution. Given : Diameter of pipe =Sem=05m Pressure, p= 29.43 Niem? = 29.43 x 10* Nim? Velocity, v= 20 mis Datum head, z=5m Total head pressure head + kinetic head + datum head Pressure head = 2 2 BBA W59m p for water = pe 1000x981 Kinetic head 22 22.9 204m 2g 2981 2 Total head Py 522 30+0.204+5 = 35.204m. Ans. “ps 28 Problem 6.2 A pipe, through which water is lowing, is having diameters, 20 cm and 10 cm atthe crosssections I and2 respectively. The velocity of water at section 1 is given 4:0 mis. Find the velocity head at sections 1 and 2 and also rate of discharge [262 Fluid Mechanics ] Solution. Given : D,=20em=0.2m @ Area, Ay= 5 De =F (2)"= 0034? V,=40 mis pas ten D,=0.1m Aye © (a)? 00785 m? a (@ Velocity head at section 1 (ii) Velocity head at section 2 = V;3/2g To find V,, apply continuity equation at 1 and 2 AWW, =4,V, or V,= 4M = B14 40-16. ms “00785 4s Velocity head at section 2 = V2 = 46.0%162 _ 53.947 m, Ans. 2g 2x91 (iii) Rate of discharge = AV, or AV, 0.0314 x 4.0 = 0.1256 ms = 125.6 litres/s. Ans. {+5 Lm? = 1000 litres) Problem 6.3 State Bernoulli's theorem for steady flow of an incompressible fluid. Derive an expression for Bernoulli's equation from first principle and state the assumptions made for such a derivation. Solution. Statement of Bernoulli’s Theorem. It states that in a steady, ideal flow of an incom- pressible fluid, the total energy at any point of the fluid is constant. The total energy consists of pressure energy, kinetic energy and potential energy or datum energy. These energies per unit weight of the fluid are : Pressure energy = 2 pg y Kinetic energy = 2 rays > Datum energy =z ‘Thus mathematically, Bemoulli’s theorem is written as 2424 2= Constant, pg 2g Derivation of Bernoulli's theorem. For derivation of Bemoulli’s theorem, Articles 6.3 and 6.4 should be written, Assumptions are given in Article 6.5. Problem 6.4 The water is flowing through @ px pipe having diameters 20cm and 10.cm at sections 1 and 2 respectively. The rate of flow through pipe is 35 litres/s. The section | is 6 m above datum and section 2 is 4m above datum. If the pressure at sec- tion 1 is 39.24 N/m’, find the intensity of pressure at section 2. Solution. Given : At section 1, D,=20em=0.2 m Roope 2 A= = (2 = 0314 5 im’ py = 39.24 Niem? 39.24 x 10" Nim? ,=6.0m D,=0.10m At section 2, Dynamics of Fluid Flow 263 | 96, 4m DATUM LINE Fig. 6.3 y= Eon? aos? Rate of flow, eu Now Q=A,V, = AV, 0 _ 035 ‘A, 0314 2.08 and “a” Doves = 35 livs= > = 035 ms Q=35livs= 5 114 m/s = —035_= 4.456 mis Applying Bernoulli's equation at sections 1 and 2, we get 39.24 x10" | UIA) 6 (450) 49 1000x981” 2x98i 1000x981” 2x931 or 40 +0063 +.60= 2 + 1.0124.40 3810 or 46.063 = 2 + 5.012 9810 22 = 46.063 - 5.012 = 41.051 9810 . y= 41.051 x 9810 Nim? = £1051 9810 yy 10 sm? = 40.27 Nlem?. Ans. [264 Fluid Mechanics ] Problem 6.5 Water is flowing through a pipe having diameter 300 mm and 200 mm at the bottom ‘and upper end respectively. The intensity of pressure at the bottom end is 24.525 N/cm? and the pressure at the upper end is 9.81 N/em?. Determine the difference in datum head if the rate of flow through pipe is 40 lits. Solution. Given : , = 200 mm Section 1, D, = 300 mm = 0.3m | Py = 24.525 Nlem? = 24.525 x 10* Nim? Section 2, D; = 200 mm = 0.2m (py =9.81 Niem?=9.81x 108 Nim? *? Peer oT 40 _ 3) or = Hop = O04 ms DATUMLNE Now A,V, = AzV3 = rate of flow = 0.04 Fig. 64 yj. O00 95658 ms q £3) qe ) = 0566 mt yates Ot 5 0 sant Ey =e2)p gery F02y Applying Bernoulli's equ: jon at sections (1) and (2), we get 24.525 x 10" pep = BSL (1274)? 1000x981” 2x981 *“'~ 1000x981” 2x98”? or 254 3242, = 10+ 1623+ 2 or 25.32 + 2 = 11.623 + zy 2-1 = 25.32 ~ 11,623 = 13.697 = 13.70 m Difference in datum head = z,~z, = 13.70 m. Ans. Problem 6.6 The water is flowing through a taper pipe of length 100 m having diameters 600 mm at the upper end and 300 mm at the lower end, at the rate of 50 litres/s. The pipe has a slope of I in 30. Find the pressure at the lower end if the pressure at the higher level is 19.62 N/cm’. Solution. Given : Length of pipe, L Dia. atthe upperend, Dy Area, Ay [ Dynamics of Fluid Flow 265] = 19.62 x 10* Nim? Dia, at lower end, D, = 300 mm = 0.3 m Area, x © (3)? = 0.07068 m 7 Q = rate of flow = 50 litres/s = > = 0,05 mis 1000 Let the datum fine passes through the centre of the lower end. en oe Assets nates g2 hx 100 !2 m Also we know Q=A\V, = A> j= B= BS » 6.6 BERNOULLI'S EQUATION FOR REAL FLUID ‘The Bernoulli's equation was derived on the assumption that fluid is inviscid (non-viscous) and therefore frictionless. But all the real fluids are viscous and hence offer resistance to flow. Thus there are always some losses in fluid flows and hence in the application of Bernoulli's equation, these losses hhave to be taken into consideration. Thus the Bernoulli's equation for real fluids between points 1 and. 2s given as Paty a Be pe 2e° "pg 2e where / is loss of energy between points 1 and 2 tnth, (6.5) [266 Fluid Mechanics ] Problem 6.7 A pipe of diameter 400 mm carries water at a velocity of 25 m/s. The pressures at the points A and B are given as 29.43 N/em? and 22.563 N/em® respectively while the datum head at A and B are 28 m and 30 m. Find the loss of head between A and B. Solution. Given > + Dia. of pipe, D=400 mm = 04 m ott Velocity, V=25 mis on gt At potnt A, pg = 2943 Neem? = 20.43 x 10" Nim? eo Total energy at A, Ey= cm = 29d3x10" 25" ag 1000x981” 2x981 = 30 + 31.85 + 28 = 89.85 m At point B, Pa = 22.563 Nlem? = 22.563 x 10* Nim? z= 30m ge v= vy = 25 mis Di Total energy at BB, Eg= 2B Pe = 2563x108 25% 1000x981 " 2x98i Loss of energy = Ey ~ y= 89.85 ~ 84.85 = 5.0m. Ans. Problem 6.8 A conical tube of length 2.0 m is fixed vertically with its smaller end upwards. The velocity of flow at the smaller end is 5 m/s while ai the lower end it is 2 m/s. The pressure head at the 0.35(¥, - v2)? smaller end is 2.5 m of liquid The loss of headin the ube is "2, where vis the velocity at 8 the smaller end and v, at the lower end respectively. Determine the prescure inead at the lower end. Flow takes place in the downward direction. Solution. Let the smaller end is represented by (1) and lower end by (2) +30 = 23 + 31.85 + 30 = 84.85 m Given : e Length of tube, L=20m S mis Pilpg = 2.5 mof liquid v= 2 mis 035(v, - v,) | Loss of head o35tei =v) 28 @ Fig. 67 [ ‘Dynamics of Fluid Flow _ 035[5-27 _ 0.35%9 =0.16m 2g 2x981 Oo Pressure head, ea Ps. Applying Bernoulli's equation at sections (1) and (2), we get ry % Peay e Began, pg 28 pg 28 * Let the datum line passes through section (2). Then z; Oz 4040.1 pe * 2xagr SOF O18 Ps Pa +0.208 + 16 or (25 + 1.27 + 2.0) ~ (203 + 16) = 9.77 ~ 363 = $.407 m of fluid. Ans. 267) Problem 6.9 A pipeline carrying oil of specific gravity 0.87, changes in diameter from 200 mm diameter at a position A to 500 mm diameter at a position B which is 4 metres at a higher level. I the pressures at A and B are 9.81 N/cm? and 5.886 Nem? respectively and the discharge is 200 litres/s determine the loss of head and direction of flow. Solution. Discharge, Q = 200 lit/s = 0.2 m'is + 5 SF Sp. gf of oil = 087 os LA8 p for oil = 87x 1000 = 870 8. 8 Arca, n= = (Oy? A “ a0 9 aroma = 0.0314 m? ee a= 981 Nem? 7 =9.81x10'Nim? gf If datum tne i passing through A, then Fig. 68 0 O02 Ay” D081 At section B, y= 500 mm =0.50m Area, y= E og P= 5.886 Niem? = 5.886 x 10* Nim? [268 Fluid Mechanics Q _ 02 Total energy at A Bus Pha zy _ 98110" ~ 810x981" 2x981 Po Me Total energy at B E_= 84h 25 " ps 28 = 5886x108 | (LOIs) 870x981 2x98I (i) Direction of flow. As E, is more than Ey and hence flow is taking place from A to B. Ans. Gi) Loss of head = in, = Ex ~ Ey= 13.557 ~ 10.948 = 2.609 m. Ans. +0= 11.49 + 2.067 = 13.557 m 6.896 + 0.052 + 4.0 = 10.948 m > 6.7 PRACTICAL APPLICATIONS OF BERNOULLI'S EQUATION Bernoulli's equation is applied in all problems of incompressible fluid flow where energy consid «rations are involved. But we shall consider its application to the following measuring devices : 1. Venturimeter. 2. Orifice meter. 3. Pitot-tube. 6.7.1 Venturimeter. A venturimeter is a device used for measuring the rate ofa flow of a fluid flowing through a pipe. It consists of three parts: (i A short converging part, (ji) Throat, and (ii) Diverging part. It is based on the Principle of Bernoulli’ equation Expression for rate of flow through venturimeter Consider a venturimeter fitted in a horizontal pipe through which a fluid is flowing (say water), as 69. diameter at inlet or at section (1), pressure at section (1) 1, = Velocity of fluid at section (1), area at section (1) = = and Applying Bernoulli's equation at sections (1) and (2), we get 2» Yas ate corresponding values at section (2). ‘ Fig. 6.9 Venturimeter. Bay 2g [ Dynamics of Fluid Flow 269 | h paps Substituting this value of in the above equation, we get (66) Substituting this value of y in equation (6.6) neh (ey 28 28 or v= 2gh ze 5 = v2 = gh" a Discharge, =a (6.7) Equation (6.7) gives the discharge unde ideal conditions andi aed, theoretical discharge. Actual discharge will be less than theoretical discharge. Quer = Cy Ter 2h (6.8) where C,=Co-efficient of venturimeter and its value is less than 1. Value of ‘h’ given by differential U-tube manometer Case I. Let the differential manometer contains a liquid which is heavier than the liquid flowing through the pipe. Let 5S, = Sp. gravity of the heavier liquid Sp. gravity of the liquid flowing through pipe X= Difference of the heavier liquid column in U-tube Then 46.9) Case II. If the differential manometer contains a liquid which is lighter than the liquid flowing through the pipe, the value of h is given by [270 Fluid Mechanics ] nosh where S,= Sp. gr. of lighter liquid in U-tube 5, = Sp. gr. of fluid flowing through pipe x= Difference of the lighter liquid columns in U-tube. Case IIL. Inclined Venturimeter with Differential U-tube manometer. The above two cases are given for a horizontal venturimeter. This case is related to inclined venturimeter having differential U-tube manometer. Let the differential manometer contains heavier liquid then / is given as ees-le) eso] Case IV. Similarly, for inclined venturimeter in which differential manometer contains a liquid which is lighter than the liquid flowing through the pipe, the value of / is given as w(ges)ee)nfi$] Problem 6.10 A horizontal venturimeter with inlet and throat diameters 30 em and 15 cm respec tively is used to measure the flow of water. The reading of differential manometer connected 10 the inlet and the throat is 20 cm of mercury. Determine the rate of flow. Take C, = 0.98. Solution. Given : (6.10) Di aint, Anan, Dis ath, y= Sem * age B1st= 1767 on? cy=098 Reading of differential manometer = x = 20 em of mercury. Difference of pressure head is given by (6.9) where 5, = Sp. gravity of mercury = 13.6, 5, #20 [1881] «20% 126 0m =252.0em of water Sp. gravity of water = 1 ‘The discharge through venturimeter is given by eqn. (6.8) Q=Cy x (eh 706.85 x 176.7 px Bx 252 [ Dynamics of Fluid Flow 271] = 8606759336 _ 8606759336 Jas06369- 312229 «BAA = 125756 ems = 125756 tips = 128.756 tts. Ans. Problem 6.11 An oil of sp. gr 0.8 is flowing through a venturimeter having inlet diameter 20 cm and throat diameter 10 cm. The oil-mercury differential manometer shows a reading of 25 cm. Caleu- late the discharge of oil through the horizontal venturimeter. Take C, = 0.98. Solution. Given : Sp. gt. of oil, 5, = 0.8 Sp. gr. of mercury, S,= 136 Reading of differential manometer, x = 25 em «Dies of ps x= [51] ~ 1] = 400 em of ot Dia. at inlet, y= 0.98 The discharge Q is given by equation (6.8) or O= C, HS x igh Jat - a = 0.98 x [25981400 (14.16) 2142137568 _ 2142137568 4 * [98696 — 6168 = 70465 cm*/s = 70.468 litres/s. Ans. Problem 6.12 A horizontal venturimeter with inlet diameter 20 cm and throat diameter 10 cm is used to measure the flow of oil of sp. gr. 0.8. The discharge of oil through venturimeter is 60 litres/s. Find the reading of the oil-mercury differential manometer. Take C, = 0.98. Solution. Given : dy =20em a, = © 20? = 314.16 cm? 4 4,= 100m (272. Fluid Mechanics 10710687805, 304 Zx98Ixh 304 x 60000 107106878 hn= (1.029)? = 289.98 em of oil mh P. ar. of oil = 0.8 x= Reading of manometer or = 17.029 o 289.98=x| 8 08 x= 78998 «18.19 om. 16 Reading of oil-mercury differential manometer = 18.12 em. Ans. Problem 6.13 A horizontal venturimeter with inlet diameter 20 cm and throat diameter 10 cm is used to measure the flow of water. The pressure at inlet is 17.658 N/cm? and the vacuum pressure at the throat is 30 cm of mercury. Find the discharge of water through venturimeter, Take Cy = 0.98. Solution. Given : Dia. at inlet, d,= 200m a a, = % x (20)'= 314.16 em? Dia, at throat, 4,=106m © x 10? = 78.74 cm? 4 py = 17.658 Nlem? 7.658 x 10" Nim? PL. 17658 x10 pg 981x1000 Pee = 1000 88 an. Pe = = 30 em of mercury Pe =~ 0.30 m of mercury = 030 x 13.6 = ~ 4.08 m of water Differential head Dynamics of Fluid Flow 273 | -Ak PS PE = 18 + 4.08 = 22.08 m of water = 2208 em of water = 18 (- 4.08) ‘The discharge @ is given by equation (6.8) 0 Gy Ta ah = 0.98% aoa x08 (314.16)? -(78.74)" = 5052885721 165555 om'/s = 165.555 lis. Ans. 304 Problem 6.14 The inlet and throat diameters of a horizontal venturimeter are 30 em and 10 cm respectively. The liquid flowing through the meter is water. The pressure intensity at inlet is 13.734 N/em® while the vacuum pressure head at the throat is 37 cm of mercury. Find the rate of flow. Assume that 4% of the differential head is lost between the inlet and throat, Find also the value of Cy {for the venturimeter. Solution. Given : Dia. at inlet, Dia. at throat, Pressure, Pressure head, Differential head, Head lost, 4,=30cm a= A (30)? = 706.85 cm? d= 100m ae A (10)? = 78.54 em? py = 13.134 Nlem? = 13.734 x 10 Nim? Bu 3734 x10" 14 m of water pe 1000x981 22 = _37.¢m of mercury Ps 371x136 = SPS in of water = ~ 5.032 m of water a Her =~ 5.0 te ‘n= pulps - palo = 140 ~ (5.032) = 1400 + 5.032 = 19.032 m of water = 1903.2 em 519.032 = 0.7613 m 100 4% of in hy 19.032 = 7613 G een Th = 0.95 “ he 19.032 098 [274 Fluid Mechanics ] Discharge 19032 7854)" 149692.8 cm/s = 0.14969 mis. Ans. PROBLEMS ON INCLINED VENTURIMETER Problem 6.15 A 30cm x15 cm venturimeter is inserted in a vertical pipe carrying water, flowing in the upward direction. A differential mercury manometer connected to the inlet and throat gives a reading of 20 cm. Find the discharge. Take Cy Solution, Dia. at inlet, Dia. at throat, dy= 15cm Kase 2 a= © 5¢= 176.7 em’ ne FAs) =20 [1884 0 12.6 = 252.0 em of water 10 Discharge, = 0,98 x 106851767 _ px ORT x252 (70685)° - (176.7 8606759336 _ 8606759336 4996363312229 «6844 = 125756 cm*/s = 125.756 lits. Ans. Problem 6.16 A 20cm x/0.m venturimeter is inserted in a vertical pipe carrying oll of sp. gr. 0.8, the flow of oil is in upward direction. The difference of levels between the throat and inlet section is 50 cm. The oil mercury differential manometer gives a reading of 30 cm of mercury. Find the discharge of oil. Neglect losses. Solution. Dia. at inlet, d, = 20m F (any = 314.16 Dia. at throat, d,= 100m a, = © (10) = 78.54 cm? 4 Sp. gr. of oil, 5,208 Sp. gr. of mercury, S,= 13.6 Differential manometer reading, «= 30 om PS Ps =30 [B81] =30 07-1 08 Cy= 10 ‘The discharge, O=G year* Pgh 1031416 x 7854 _ 23932630.7 304 Dynamics of Fluid Flow 275] 2X 981X480 emis ‘= 78725.75 cm*/s = 78.725 litres/s. Ans. Problem 6.17 In a vertical pipe conveying oil of specific gravity 0.8, two pressure gauges have been installed at A and B where the diameters are 16 cm and 8 cm respectively. A is 2 metres above B. The pressure gauge readings have shown that the pressure at B is greater than at A by 0.981 N/cm’. Neglecting all losses, calculate the flow rate. If the gauges at A and B are replaced by tubes filled with the same liquid and connected to a U-tube containing mercury, calculate the difference of level of mercury in the two limbs of the U-tube. Solution. Given : Sp. gr. of oil, 5,08 Density, =08 x 1000 = 800 *S. Dia. a, Dy = I6em=0.16m Area at A, A, = £ (16)? = 0.0201 m* Dia. a B, Dy=8em= 008m ‘Area at B, Ar= fi (.08)? = 0.005026 m? (@ Difference of pressures, py py = 0.981 Nlem? 9810N 0.981 x 10* Nim? Difference of pressure head Pa~Ps 9810 _j 96 pe 800x931 Fig. 69 @) p = 800 kg/m’) [276 Fluid Mechanics Applying Bernoulli's theorem at A and B and taking the reference line passing through section B, we get Yen, 2 vi or a 24 or (2B) «20-00. te Mi pg 2g 2g or =1.25+2.0= %e-Ya 2g 2g ons- BM 2g 2g Now applying continuity equation at A and B, we get Vy XAy = VX Ay Substituting the value of Vz in equation (i), we get 224-125) Ps oO} o7s= OV Va = IVE 2g 2g 2g p= [OBEERT 50 us Pos Rate of ow, 0=¥,x4, Fig. 69 0) = 0.99 x 0.0201 = 0.01989 m*/s. Ans. (@ Difference of level of mercury in the U-tube. Let/r= Difference of mercury level 5 Then here So wiere ba (L+7,) (225) = BEBE 6 2.24 Ps Ps Ps =-125+2.0-0 = 075 (= 252-125) be [ Dynamics of Fluid Flow 277] Problem 6.18 Find the discharge of water flowing through a pipe 30 cm diameter placed in an inclined position where a venturimeter is inserted, having a throat diameter of 15 cm. The difference of pressure between the main and throat is measured by a liquid of sp. gr. 0.6 in an inverted U-tube which ‘gives a reading of 30 om. The loss of head between the main and throat is 0.2 times the kinetic head of, the pipe. Solution. Dia. at inlet, d, = 30cm Dia. at throat, d,= 150m Rasy 2 a, = © (15) = 176.7 em’ n= 7 (15) = 176. Reading of differential manometer, x = 30 em Difference of pressure head, f is given by He Aso rash where $, = 0.6 and S, = 1.0 =20/1 ss =30x4 10 2.0 em of water : Fig. 6.10 Loss of head, fh, = 0.2 x kinetic head of pipe = 0.2 x 5 e Now applying Bernoulli's equation at sections (1) and (2), we get Pega Peneton, pe" 2g pg.” 2g ol) [278 Fluid Mechanics ] Applying continuity equation at sections (1) and (2), we get ayy, = ay Substituting this value of v, in equation (1), we get 08, vi sane 8A) ce wane fat or “ [05-1] =- 12.0 or 2g 095 Discharge ay, = 176.7 x 157.4 em'/s = 27800 cm'/s = 27.8 litres/s. Ans. Problem 6.19 A 30 cm x 15 cm venturimeter is provided in a vertical pipe line carrying oil of specific gravity 0.9, the flow being upwards. The difference in elevation of the throat section and entrance section of the venturimeter is 30 cm. The differential U-tube mercury manometer shows a ‘gauge deflection of 25 cm. Calculate : (i) the discharge of oil, and (ii) the pressure difference between the entrance section and the throat section. Take the co-efficient of discharge as 0.98 and specific gravity of mercury as 13.6. Solution. Given : Dia. at inlet, 30 em Area, 4, = 5 G0)" = 706.85 em? Dia. at throat, d,= 15 em Area, y= 5 (1S) = 176.7 om* Let section (1) represents inlet and section (2) represents throat. Then zz, = 30 em Sp. gr. of oil, 5,209 Sp. gr of mercury, 5, 136 Reading of diff. manometer, x= 25 em ‘The differential head, his given by “Gye 392.77 em of oil [ Dynamics of Fluid Flow 279 | (ote asebage oro = 6, 7 _ 098 x 70685 176.7 _ {(706:85)" 2x 981x352.77 _ 1018322199 oR 148.79 litres/s. Ans. (i Pressure difference between entrance and throat section ha (Bi+e)-(Bra) =asa7 z = 148790.5 em*is ps 30 4 or 4-25 352.77 Pr iE But 27% = 30m I Pe) _39 = 352.77 a) tm Fig. 6.11 Pi _ 22 = 352.77 + 30 = 382.77 cm of oil = 3.8277 m of oil. Ans. Pe pw or (1 — 9) = 3.8277 x pg But density of oil = Sp. gr. of oil x 1000 kg/m? = 0.9 x 1000 = 900 kg/em? (=p) = 3.8277 x 900 x 9.81 Problem 6.20 Crude oil of specific gravity 0.85 flows upwards at a volume rate of flow of 60 litre per second through a vertical venturimeter with an inlet diameter of 200 mm and a throat diameter of 100 mm. The co-efficient of discharge of the venturimeter is 0.98. The vertical distance between the pressure tapings is 300 mm. (i) If two pressure gauges are connected at the tappings such that they are positioned at the levels of their corresponding tapping points, determine the difference of readings in Nem of the two pres- sure gauges. (ii) Ifa mercury differential manometer is connected, in place of pressure gauges, to the tappings such that the connecting tube upto mercury are filled with oil, determine the difference in the level of the mercury column. Solution. Given : Specific gravity of oil, $= 0.85 [280 Fluid Mechanics Density, p= 0.85 x 1000 = 850 kg/m? Discharge, 2 = 60 litres = = 0.06 mis 1000 Inlet dia, 4, = 200 mm = 0.2 m ‘Area, a= F (2) = 0.0318 m? Throat dia. y= 100 mm = 0.1 m ‘Area, 200 0m Value of Cy Let section (1) represents inlet and section (2) represents throat, Then 222 = 300 mm=0.3 m (i Difference of readings in Nlem’ of the two pressure gauges, ‘The discharge Q is given by, Fig. 6.11 (@) x (2X981xh = 2S XOOIDIS 9 J Vix__00600308 agg * 0.98 x 0.00024649 x 4.429 — - ‘A= 1.705? = 2.908 m Bisa) (Boe) PS PS But fora vertical venturimeter, h = ( rane (B+ )-(2+5) PS Ps PKPa 2.008 + 232 = 2.908 + 03 pg (2-2, =03m) = 3.208 m of oil Pi ~P2= PBX 3.208 = 850 9.81 x 3.208 Nim? = SOx OS 3208 Niem* = 2.675 Niem*, Ans. [ Dynamics of Fluid Flow 281] (i Difference in the levels of mercury columns (i.e., x) ‘The value of fis given by, k= Problem 6.21 In a [00 mm diameter horizontal pipe a venturimeter of 0.5 contraction ratio has been fixed. The head of water on the metre when there is no flow is 3 m (gauge). Find the rate of flow {for which the throat pressure will be 2 metres of water absolute. The co-efficient of discharge is 0.97. Take atmospheric pressure head = 10.3 m of water. Solution. Given : Dia. of pipe, d= 100 mm = 10 em E aoe. 5} Area, = © (0)? = 78.54 em GMO = 78.54 Dia. at throat, 15x 10= Area, 19.635 em? Head of water fornoflow = PL. = 3m (gauge) = 3+ 103= 133 m abs) 8 ‘Throat pressure head 2m of water absolute. Difference of pressure head, f= 2 Ps pg 3.3 - 2.0 = 11.3 m= 1130 em Rate of flow, 0 is given by Q = C, = 0.97 x 6.7.2 Orifice Meter or Orifice Plate. It is a device used for measuring the rate of flow of a tn amet a pe nght may vay fon Oto. umes pe amet A differential manometer is connected at section (1), which is at a distance of about 1.5 to 2.0 times [282 Fluid Mechanics ] Let _p, = pressure at section (1), 1, = velocity at section (1), 4, = area of pipe at section (1), and Fig. 6.12. Orifice meter. Pp, V2» dz ate corresponding values at section (2). Applying Bernoulli's equation at sections (1) and Q), we get riya Pug tigg pe 2g Ps Ps 2g 2g m Ese) Differential head eM oe 2gh=vy—v? 2g 28 ue fogh+ v7 oli) Now section (2) is at the vena-contracta and a, represents the area at the vena-contracta. If ay is the area of orifice then, we have cat % where C,= Co-ffcient of contraction 5 0, = 4X C, « By continuity equation, we have asa, of 12% = om Ait) Substituting the value of v, in equation (i), we get v= Pgh + [ Dynamics of Fluid Flow 283 | “ sana) tes [(SP oom + ay = aC, from (iD) wiv) (6.13) ‘o-efficient of discharge for orifice meter. The co-efficient of discharge for orifice meter is much smaller than that for a venturimeter. Problem 6.22 An orifice meter with orifice diameter 10 cm is inserted in a pipe of 20 cm diameter. The pressure gauges fitted upstream and downstream of the orifice meter gives readings of 19.62 N/em? and 9.81 N/cm® respectively. Co-efficient of discharge for the orifice meter is given as 0.6. Find the discharge of water through pipe. [284 Fluid Mechanics ] Solution. Given : Dia. of orifice, dy= 10 em x 2 Area, ag = © (10)? = 78.54 em 5 (oy Dia. of pipe, d,= 20m Area, a, = F QO)" = 314.16 em* py = 19.62 Niem? = 19.62 x 10* Nim? Bu 196210" 90 m of water pe 1000x931 Similarly Ba = SBIXI0" = 19 m of water pg 100098 = PL — 22 = 20,0 - 10.0 = 10 m of water = 1000 em of water Ps pg Cq= 06 ‘The discharge, Q is given by equation (6.13) Q=C, Le x Reh LiSeRETSY x 2 x 981 x 1000 (314.16)° —(78.54)" =06 _ 2073683809 * 304 Problem 6.23 An orifice meter with orifice diameter 15 em is inserted in a pipe of 30.cm diameter. The pressure difference measured by a mercury oil differential manometer on the two sides of the orifice meter gives a reading of 50cm of mercury. Find the rate of flow of oil of sp. gr. 0.9 when the co- efficient of discharge of the orifice meter = 0.64. Solution. Given : 18213.28 cm/s = 68.21 litres/s. Ans. Dia. of orifice, y= 15cm Area, y= F (sh 1187 6m? Dia. of pipe. 4, = 300m Area, 4, = 5 (0)? = 706.85 em? Sp. gr. of oil, 5,=09 Reading of diff. manometer, x= 50 em of mercury Difernial head, he [z - i|- 50 [88-1] em oro 5 08 [ Dynamics of Fluid Flow 285] = 50 x 14.11 = 705.5 em of oil C,= 0.64 -n by equation (6.13) O=C, Jane The rate of the flow, = 0,64 x 176710685 _ RS OBTX TOSS: ((706.85)° _ 94046317.78 Cor] 6.7.3 Pitot-tube. It is a device used for measuring the velocity of flow at any point ina pipe or a channel. Itis based on the principle that if the velocity of flow at a point becomes zero, the pressure there is increased due to the conversion of the kinetic energy into pressure energy. In its simplest form, the pitot-tube consists of a glass tube, bent at right angles as shown in Fig. 6.13. The lower end, which is bent through 90° is directed in the up- stream direction as shown in Fig. 6.13. The liquid rises up in the tube due to the conversion of kinetic energy into pressure energy. The velocity is determined by measuring the rise of liquid in the tube. 7 Consider two points (1) and (2) at the same level in such a way that point (2) is just as the inlet of the pitot-tube and point (1) is far away from the tube. Let intensity of pressure at point (1) velocity of flow at (1) p= pressure at point (2) velocity at point (2), which is zero H = depth of tube in the liquid ‘= rise of liquid in the tube above the free surface. Applying Bernoulli's equation at points (1) and (2), we get au u Bates hs Bey pe 2g" pg” 2g +, 8 points (1) and (2) are on the same line and y, = 0. PL = pressure head at (I) =H Pg P2 = pressure head at (2) = (+ H) Ps Substituting these values, we get neticnem 2 net 2g 0 2g ‘This is theoretical velocity. Actual velocity is given by or v= gh = 137414.25 em’/s = 137.414 litres/s. Ans. Fig. 6.13 Pitottube. [286 Fluid Mechanics ] (Dea = Cy Pah where C, = Co-efficient of pitot-tube Velocity at any point. — v= C, /2ghr (6.14) ‘Velocity of flow in a pipe by pitot-tube. For finding the velocity at any point in a pipe by pitot- tube, the following arrangements are adopted 1, Pitot-tube along with a vertical piezometer tube as shown in Fig. 6.14. 2. Pitot-ube connected with piezometer tube as shown in Fig, 6.15, 3. Pitot-tube and vertical piezometer tube connected with a differential U-tube manometer as shown in Fig. 6.16. PIEZOMETER ..| PIroT-T TUBE BE fe L- Za) Fig. 6.16, Fig. 6.17 4. Pitot-static tube, which consists of two circular concentric tubes one inside the other with some annular space in between as shown in Fig. 6.17. The outlet of these two tubes are connected 0 the differential manometer where the difference of pressure head *h’ is measured by knowing the 5 Aiton ft kl ote manos igi» Then =x [=] S, Problem 6.24 A pitor-static tube placed in the centre of a 300 mm pipe line has one orifice pointing upstream and other perpendicular to it. The mean velocity in the pipe is 0.80 of the central velocity. Find the discharge through the pipe if the pressure difference between the two orifices is 60 mm of water. Take the co-efficient of pitot tube as C, = 0.98. Solution. Given : Dia. of pipe, d= 300 mm = 0.30 m Diff. of pressure head, ‘= 60 mm of water = .06 m of water 0.98 Mean velocity, V =0.80 x Central velocity Central velocity is given by equation (6.14) = C, \2gh = 0.98 x J2X9RIX.06 = .063 mis [ Dynamics of Fluid Flow _ 287] . V = 0.80 x 1.063 = 0.8504 m/s Discharge, = Area of pipe x 7 = fe xv 5 (30)? x 0.8504 = 0.06 m*/s. Ans. Problem 6.25 Find the velocity of the flow of an oil through a pipe, when the difference of mercury level in a differential U-tube manometer connected to the two tappings of the pitot-tube is 100 mm. Take co-efficient of pitot-tube 0.98 and sp. gr. of oil = 0.8. Solution. Given Dit. ofpse ten, hf [38-1] <16moron Velocity of flow = C, J2gh = 0.98 J2X9BIXL6 = 5.49 mis. Ans. Problem 6.26 A pitot-static tube is used fo measure the velocity of water in a pipe. The stagnation pressure head is 6 m and static pressure head is 5 m. Calculate the velocity of flow assuming the co- efficient of tube equal 0 0.98. Solution. Given Stagnation pressure head, Static pressure head, “ Im Velocity of flow, V=C, J2gh = 0.98 JEXORTXI = 4.34 mis. Ans. Problem 6.27 A sub-marine moves horizontally in sea and has its axis 15 m below the surface of water. A pitot-tube properly placed justin front ofthe sub-marine and along its ais is connected tothe two limbs ofa U-tube containing mercury. The difference of mercury level is found 0 be 170 mm. Find the speed ofthe sub-marine knowing thatthe sp. gr. of mercury is 13.6 and that of sea-water is 1.026 with respect of fresh water. Solution. Given > Diff. of mercury level, x= 170 mm=0.17m Sp. gr of mercury, Sp. ef. of seawater, V= igh = J2X9RIXD0834 = 6.393 mis = 6393 x 60 x 60, “1000 Problem 6.28 A pitot-tube is inserted in a pipe of 300 mm diameter. The static pressure in pipe is 100 mm of mercury (vacuum). The stagnation pressure at the centre of the pipe, recorded by the km/hr = 23.01 krn/hr. Ans. [288 Fluid Mechanics ] pitot-tube is 0.981 N/em?. Calculate the rate of flow of water through pipe, if the mean velocity of flow is 0.85 times the central velocity, Take C, = 0.98. Solution. Given : Dia. of pipe, d= 300 mm = 0.30 m Area, $ (3)*= 0.07068 m* Static pressure head = 100 mm of mercury (vacuum) 100. 5613,6 =~ 1.36 m of water Stagnation pressure 981 x 10" Nim? 81x10" __981x10" Pe 1000x981 ‘i= Stagnation pressure head ~ Static pressure head 1.0 ~ © 1.36) = 1.0 + 1.36 = 2.36 m of water Stagnation pressure head = Velocity at centre Pgh = 0.98 x JX 9BTX236 = 6.668 mis Mean velocity, V = 0.85 x 6.668 = 5.6678 mis Rate of flow of water V x area of pipe = 5.6678 x 0.07068 m*/s = 0.4006 m*/s. Ans. > 6.8 THE MOMENTUM EQUATION Tes based onthe law of conservation of momentum or onthe momentum principle, which sates tha the met fore acting on ald mass sequal othe change in omentum of flow por unit ine in that direction. The force acting on a fluid mass ‘m’ is given by the Newton's second law of motion, Pema where sth acceleration acting inthe same dretion a force F. “~ dt tv dt But F=m _ dmv) 7 _ dmv) re (6.15) Equation (6.15) is known as the momentum principle. Equation (6.15) can be written as F.dt = d(m) (6.16) which is known as the impulse-momentum equation and states that the impulse of a force F acting on a fluid of mass m in a short interval of time dr is equal to the change of momentum d(mv) inthe direction of force. {mis constant and can be taken inside the differential} [ Dynamics of Fluid Flow 289 | Force exerted by a flowing fluid on a pipe bend The impulse-momentum equation (6.16) is used to determine the resultant force exerted by @ flowing fluid on a pipe bend. Consider two sections (1) and (2), as shown in Fig. 6.18. Let ¥, = velocity of flow at section (1), 1) = pressure intensity at section (1), A, = area of cross-section of pipe at section (1) and. Vy, Pay Ap = Comresponding values of velocity, pressure and area at section (2). Let F, and F, be the components of the forces exerted by the flowing fluid on the bend in x-and y-direetions respectively. Then the force exerted by the bend on the fluid in the directions of x and y will be equal to F, and F, but in the opposite directions. Hence component of the force exerted by bend on the fluid in the x-direction = ~ F, and in the direction of y = ~ F, The other external forces acting on the fluid are pyA, and pA, on the sections (1) and (2) respectively. Then momentum ‘equation in x-direction is given by @ © Fig. 618 Forceson bend. Net force acting on fluid in the direction of x = Rate of change of momentum in x-direction 1A, ~ PaAp Cos @ ~ F,, = (Mass per sec) (change of velocity) PQ (Final velocity in the direction of x ~ Initial velocity in the direction of x) PQ (V, cos 8- Vi) (6.17) F,= pQ (V, ~ V2 cos 8) + p,Ay ~ ppAp cos 8 (6.18) Similarly the momentum equation in y-direction gives 0—poA, sin 8 F, = p@ (Vasin 0) (6.19) F, = pQ ( Vasin 8) ~ pyAy sin @ (6.20) Now the resultant force (F) acting on the bend +R 6.21) ‘And the angle made by the resultant force with horizontal direction is given by tan O= 5 46.22) none 22 Problem 6.29. A.45° reducing bend is connected in a pipe line, the diameters atthe inlet and outlet ofthe bend being 600 mm and 300 mm respectively, Find the force exerted by water on the bend i the intensity of pressure at inlet to bend is 8.829 N/em? and rate of flow of water is 600 litres/s. [290 Fluid Mechanics Solution. Given : ‘Angle of bend, Dia. at inlet, ‘Area, Dia. at outlet, Area, Pressure at inlet, p, = 8829 Nlem? = 8.829 x 10" Nim? Q = 600 livs = 0.6 mis 2 _ 06 Viera == ype 22122 mis _2_ 06 Vy B= aig = 488 mis Applying Bernoulli's equation at sections (1) and (2), we get, PM ye Beye, pe 28 28 But nen PM pe VE 889.108 | 24227 _ p, pg’ 2g 2¢ 1000x981” 2x981 ~ pg 9+ 2295 = pxlpg + 3.672 22 = 0.2905 ~ 3.672 = 5.5575 m of water Pe py = 5.5575 1000 x 9.81 Nim? = 5.45 x 10" Ni 8.4887 2x981 Forces on te bend in x- and y-drections re given by equations (6.18) and (6.20) as F.= pQ IV, ~ V2.0 6] + piA, ~ pxA, cos 0 = 1000 x 0.6 [2.122 ~ 8.488 cos 45°] + 8.829 x 10* x 2827 ~ 5.45 x 10" x .07068 x cos 45° 24959.6 ~ 5048.2 = 2827.9 + 24959.6 ~ 2720.3 199114.N and F,= pO F Vp sin 0] pyAp sin O == 3601.1 - 2721.1 ve sign means F, is acting in the downward direction Resultant force, Fee R+k 6322.2 N 1000 x 0.6 [~ 8.488 sin 45°] ~ 5.45 x 10" x .07068 x sin 45° [ Dynamics of Fluid Flow 291] = ylis911ay? +(—6322.2)° Besta = 20890.9 N. Ans. ‘The angle made by resultant force with x-axis is given by ‘equation (6.22) or = 69222N Fy tan 0 = 2 = £3222 _ 3195 Fig. 6.20 F, 19914 ‘ @= tan! 3175 = 17° 36’. Ans. Problem 6.30 250 litres/s of water is flowing in a pipe having a diameter of 300 mm. Ifthe pipe is bent by 135° (that is change from initial to final direction is 135°), find the magnitude and direction of the resultant force on the bend. The pressure of water flowing is 39.24 N/m’. Solution. Given : Pressure, 39.24 Niem? = 39.24 x 10* N/m? Discharge, resis = 0.25 m/s Dia. of bend at inlet and outlet, D, 300 mm = 0.3 m Area, Ay= A= © DP = © x3?= 0.07068 m? 4 4 025 Velocity of water at sections (1) and (2), V= V, = Va=— 2— = 729 = 3.537 mis. (07068 a Vgsin 45° UP) pats sin as Vy 48) Pah 008 487 @ Y v x Fig. 6.21 Force along x-axis F, 21 Vaal + Pucds + Pac where, Vj, = initial velocity in the direetion of x = 3.537 m/s Va, = final velocity in the direction of x = ~ V, cos 45° = ~ 3537 x 7071 Pig = pressure at section (1) in x-direction = 30.24 Nlom? = 39.24 x 10" N/m? ax = pressure at section (2) in x-direction = py cos 45° = 39.24 x 104 x 7071 000 x .2513.537 — (3.537 x .7071)] + 39.24 10" x 07068 + 39.24 x 10% x 07068 x 7071 = 1000 x .25[3.537 + 3.537 x .7071] + 39.24 x 10° x 07068 [1 + .7071] [292 Fluid Mechanics ] = 1509.4 + 47346 = 4855.4 N Force along y-axis = Fy= pQIViy— Vay) + (PrAdy + (rAd), where Vj, = initial velocity in y-direction = 0 Voy = final velocity in y-direction = ~ V, sin 43° = 3.537 x .7071 (yA), = pressure force in y-ditection = 0 (p2A2), = pressure force at (2) in y-tirection ~ pzAz Sin 45° = ~ 39,24 x 10 x ,07068 x .7071 F, = 1000 x .25{0 ~ 3.537 x .7071] + 0 + (~ 39.24 x 10* x 07068 x .7071) 625.2 - 19611.1 = - 20236.3 N ve sign means F, is acting in the downward direction Resultant force, Fy= fh +k (4885547 + 202363" = $2880.6 N. Ans. ‘The direction of the resultant force Fg, with the x-axis is given as 202363 _ 4142 Fr, 488554 ig. 6.22 o 0 = 22°30’, Ans. Problem 6.31 4 300 mm diameter pipe carries water under a head of 20 metres with a velocity of 3.5 més, Ifthe axis of the pipe turns through 45°, find the magnitude and direction of the resultant force a the bend. Solution. Given = Dia. of bend, D=D,=D,=300mm=030m aes, A= Ay=Ay= 2 pte Ex 3007068 0? Velocity, Ve V2 ¥9=35 mis o=45° Discharge, Q=Ax V= 0.07068 x 3.5 = 0.2475 m/s Pressure head =20mof water or 2 =20mof water pe p= 20x pg = 20x 1000 x 9.81 N/m? = 196200 Nim? 2 Pressure intensity, P=, = Py = 196200 Nim? Now Vi_= 35 mls, Vay = V9008 45° = 3.5 x.7071 Vj, =0, Vay = Vasin 45° = 35%.7071 (Adc Ay = 196200 x .07068, (4A,), =O (As)c== Pra 608.45° (P2A3), == Poa sn 45° Force along x-axis, F = POW g— Vad + PAD + aADdy = 1000 x 2475)3.5 -3.5 x.7071] + 196200 x.07068 - pr’ cos 45° [ Dynamics of Fluid Flow 293 | = 253.68 + 196200 x .07068 ~ 196200 x .07068 x 0.7071 = 253.68 + 13871.34 ~ 9808.04 = 4316.98 N Force along y-axis, F,= pO Wy ~ Val + (Ady + Paty 1000 x .2475{0 ~ 3.5 x .7071] + 0 + [- pyA, sin 45°) = ~ 612.44 ~ 196200 x .07068 x .7071 = 612.44 ~ 9808 = ~ 10420.44 N Resultant force Fy= [Fe + Fe = ¥(431698) + (1042044)? = 11279 N. Ans. peAgsin as" Vosin a5 fen Fig. 6.23 ‘The angle made by Fy with x-axis 10420.44 4316.98 * @= tan"! 2411 = 67° 28°. Ans. Problem 6.32 In a 45° bend a rectangular air duct of | m? cross-sectional area is gradually reduced 10 0.5 m? area. Find the magnitude and direction of the force required to hold the duct in position if the velocity of flow at the I m? section is 10 m/s, and pressure is 2.943 N/em’. Take density of air as 1.16 kg/m’, Solution. Given : ‘Area at section (1), ‘Area at section (2), tan @ AL Velocity at section (1), V, = 10 mis Pressure at section (I), py = 2.943 N/em? = 2.943 x 10* N/m? = 29430 N/m? Density of air, p= 1.16 kg/m? Applying continuity equation at sections (1) and (2) AV, = AsV AM v,=44 2. x 10=20ms > 05 [294 Fluid Mechanics Discharge Q=A\V,= 1x 10= 10 ms Vosin 45" Fig. 624 Applying Bernoulli's equation at sections (1) and (2) Me Ve BWR we 2 L=2) ps 28 pe 2g x108 107 p, , 20° 116x981" 2x981 pg” 2x981 10 pg 116x981 2x981 2x981 = 2586.2 + 5.0968 - 20.387 = 2570.90 m ’ 2 = 2570.90 x 1.16 x 9.81 = 29255.8 N Force along x-axis, F,= pO Wi~ Vadl + DADs + PAD) where Ay, = 10 mis, Va, = Vz ¢08 45° = 20 x .7071, (ride = PyAy = 29430 % 1 = 29430 N and (psAz), =~ pxAz €08 45° = ~ 29255.8 x 0.5 x.7071 F,= 1.16 x 10[10 ~ 20 x .7071] + 29430 x 1 ~ 29255.8 x 5 x.7071 =~ 48.04 + 29430 ~ 10343.37 = 0 ~ 19038.59 N Similarly force along y-axis, F, = QLV,y ~ Vay] + @rAy)y + W2A2)y = 0, Vay = Vz sin 45° = 20 x 7071 = 14.142 yA Sin 45° = ~ 29255.8 x5 X.7071 1.16 x 10 [0 — 14.142] + 0 — 10343.37 = ~ 164.05 ~ 10343.37 = ~ 10507.42 N Resultant force, F= FB +F> = ¥(190386)* + (1050742)" = The direction of Fy with x-axis is given as F _ 10507.42 where Vy. (Ay), = 0 and (yA), = F, ~~ 10343.37 15519 tan 8 F, 190386 = tan! 5519 = 28° 53’. Ans. Dynamics of Fluid Flow 295] Fp is the force exerted on bend. Hence the force required to hold the duct in position is equal to 21746.6 N but itis acting in the opposite direction of Fp. Ans. Problem 6.33 A pipe of 300 mm diameter conveying 0.30 m/s of water has a right angled bend in @ horizontal plane. Find the resultant force exerted on the bend ifthe pressure at inlet and outlet of the bend are 24.525 Nem? and 23.544 N/em. Solution. Given : Dia. of bend, D= 300 mm = 0.3m Area, $ 63)? = 0.07068 m? Discharge, = 0.30 ms 030 Velocity, = 4.244 mis a 07068 Angle of bend, py = 24.525 Nlom? = 24.525 x 10* Nim? = 245250 Nim? 23.544 Nlem? = 23.544 x 108 Nim? = 235440 Nim? Force on bend along x-axis F, = pQ (Vs, ~ Val + (PrAvs + P2ADde V, = 4.244 mis, Vp, =0 where F,, = 1000 x 0.30 [4.244 ~ 0] + 245250 x .07068 + 0 1273.2 + 173343 = 18607.5 N Js, Fy= 00 Viy~ Va + DAs + PAD, 1.244 m/s. ~ psAz = ~ 235440 x .07068 = ~ 16640.9 F, = 1000 x 0.30[0 ~ 4.244] + 0 ~ 16640.9 — 1273.2 — 16640.9 = - 17914.1 N Resultant force, -Fg= JF? +} = y(186075)" + (17914) FE Force on bend along y- where Vi, = 0. Vag A), = 0, 0A 25829.3 N 179144 186075 @= 43° 54’. Ans. and. tan 9627 F, [296 Fluid Mechanics ] Problem 6.34 A nozzle of diameter 20 mm is fitted to a pipe of diameter 40 mm. Find the force exerted by the nozzle on the water which is flowing through the pipe at the rate of 1.2 m’/minute. Solution. Given : Dia. of pipe, D, = 40 mm = 40 x 10° m= 04 m ‘Area, Ay rae F (0 = 0.001256 m? Dia. of nozzle, D,= 20 mm = 0.02 m Area, Discharge, vs seh) Applying continuity equation at sections (1) and (2), AW, = AV = yx 22221592 ms 4, 001256 @__02 a = 63.69 mis a ‘00318 ps 28 Now = 206.749 - 12.917 pe 2g 2g 2x981 2x981 = 193,83 m of water Peseta eaeorer te : : [ Dynamics of Fluid Flow 297] Let the force exerted by the nozzle on water = Fy Net force in the direction of x= rate of change of momentum in the direction of x PrAy ~ Poa + Fy = POWs ~ Vy where p) = atmospheric pressure = 0 and p - 1901472 x .001256 - 0 + F, = 1000 x 0.02(63.69 ~ 15.92) or 2388.24 + F, = 916.15 : F =~ 2388.24 + 916.15 =~ 1472.09. Ans. —Ve sign indicates that the force exerted by the nozzle on water is acting from right to left Problem 6.35 The diameter of a pipe gradually reduces from I m to 0.7 m as shown in Fig. 6.27. The pressure intensity atthe centre-line of I m section 7.848 kN/m? and rate of flow of water through the pipe i600 lites/s. Find the intensity of pressure at the centre-line of 0.7 m section. Also determine the force exerted by flowing water on transition of the pipe. Solution. Given : Dia. of pipe at section 1, D, = 1m Area, y= E (yt = 0.7854 m* oO @ Dia. of pipe at section 2, D, = 0.7 m Area, Ags F OD? = 0.3848 m? Pressure at section 1, Dy = 7.848 kN/m? = 7848 Nim? 600 3 Discharge, = 600 litres/s = 0 = 9.6 ms re = 600 p00 70° Applying continuity equation, A\V, = AQV2= 0 o_ 06 Sa OS = 0.764 mis A, 0.7854 . = 2 = 98 2155 mis Ay” 3854 Applying Bernoulli's equation at sections (1) and 2), ve “ “pipe is horizontal, 2. 2) = 2 Ce ip nan) or 7848 Pr , (155) 1000x981 *2x981 ~ pg * 2x981 [298 Fluid Mechanics ] Ps ogy (764) (158)? pe “"2x981 2x 981 = 0.8 + 0.0297 ~ 0.122 = 0.7077 m of water ps = 0.7077 x 9.81 x 1000 ‘= 6942.54 N/m? or 6.942 KN/m?. Ans. Let F, = the force exerted by pipe transition on the flowing water in the direction of flow ‘Then net force in the direction of flow = rate of change of momentum in the direction of flow or Dry = Pala + F,= pW ~ V) 748 x 7854 ~ 6942.54 x 3848 + F, or 61638 ~ 2671.5 + F,= 471.56 F,= 411.56 - 61638 + 2671. “The force exerted by water on pipe transition — (— 3020.74) = 3020.74 N. Ans. 1000 x 0.6[1.55 ~ .764] = 3020.74 N > 6.9 MOMENT OF MOMENTUM EQUATION, ‘Moment of momentum equation is derived from moment of momentum principle which states that the resulting torque acting on a rotating fluid is equal to the rate of change of moment of momentum, Let V, = velocity of fluid at section 1, radius of curvature at section 1, Q = rate of flow of fluid, p= density of fluid, and Vq and ry = velocity and radius of curvature at section 2 Momentum of fluid at section I = mass x velocity = pQ x Vi/s Moment of momentum per second at section 1, = pOxV,xr, Similarly moment of momentum per second of fluid at section 2 =pOxV, xr, Rate of change of moment of momentum. = PQVars — PQVirs = POLVara — Vari) ig to moment of momentum principle Resultant torque = rate of change of moment of momentum or T= pOlVor— Vir) (6.23) Equation (6.23) is known as moment of momentum equation. This equation is applied : 1. For analysis of flow problems in turbines and centrifugal pumps. 2. For finding torque exerted by water on sprinkler. Accor Problem 6.36 A lawn sprinkler with nwo nozzles of diameter 4 mm each is connected across a tap of water as shown in Fig. 6.28. The nozzles are at a distance of 30 cm and 20 cm from the centre ofthe tap. The rate of flow of water through tap is 120 cm'/s. The nozzles discharge water in the downward direction. Determine the angular speed at which the sprinkler will rotate free. [ Dynamics of Fluid Flow 299 | Solution. Given : [+20 cm->}—30 om—»} Dia. of nozzles A and B, == 6 D=D,=Dy=4mm=.004m ° Area, A= (004)? = 00001256 m? 4 Fig. 6.28 Discharge Q=120cm's Assuming the discharge to be equally divided between the two nozzles, we have 2, = p= 2 = 2 = 60 emis = 60x 10% mis 272 Velocity of water at the outlet of each nozzle, Vq=Vp= 28 = X10 4.777 mis, ‘A 00001256 ‘The jet of water coming out from nozzles A and B is having velocity 4.777 mis. These jets of water will exert force in the opposite direction, ie, force exerted by the jets will be in the upward direction. ‘The torque exerted will also be in the opposite direction. Hence torque at B will be inthe anti-clockwise ‘itection and at A in the clockwise direction, But torque at B is more than the torque at A and hence sprinkle, if free, will rotate in the anti-clockwise direction as shown in Fig. 6.28, Let = angular velocity of the sprinkler. ‘Then absolute velocity of water at A, V=Vy+ xr, istance of nozzle A from the centre of tap =200m=0.2m (00 X ry = tangential velocity due to rotation} V, = 4.777 + @ x 0.2) mis Here @ Xr, is added to V, as Vy and tangential velocity due to rotation (« X r,) are in the same direction as shown in Fig. 6.28. Similarly, absolute velocity of water at B, V, = Vp tangential velocity due to rotation =4.777-0% rg {where rp = 30m = 0.3 my = (4.777 - @x03) Now applying equation (6.23), we get T= pQWor,~ Vari) Here r)= ra) = 4 = pQuIVare— Viral = Q,= Op = 1000 x 60 x 10° ((4.777 x 0.3 @) x 3 ~ 4.777 + 0.2 @) x2] ‘The moment of momentum of the fluid entering sprinkler is given zero and also there is no external torque applied on the sprinkler. Hence resultant external torque is zero, ie., T= 0 1000 x 60 x 10° (4.777 - 0.3 «) x .3 - (4.777 + 0.2 @) x.21=0 or (4.777 -03 @) X0.3- (4.777 +02 0) x.2=0 or 4.977 x3-.09 ©-4.777 x.2-.04 @=0 or 0.1 4.777 = (09 + 04)0= 13.0 777 ATT § 3.6746 rad/s. Ans. Dye = 36746 rads. Ans [300 Fluid Mechanics ] Problem 6.37 A lawn sprinkler shown in Fig. 6.29 has 0.8 cm diameter nozzle at the end of a rotating arm and discharges water at the rate of 10 m/s velocity. Determine the torque required to hold the rotating arm stationary. Also determine the constant speed of rotation of the arm, if free 10 rotate t Solution. Dia, of each nozzle = 0.8 em =.008 m 2oem 1. 250m "mse" ‘Area of each nozzle = F (008)? = 00005026 m* I* Velocity of flow at each nozzle = 10 mis. 5 ‘ Discharge through each nozzle, = Area x Veloey ro msec = .00005026 x 10 = .0005026 mis TB 629 Torque exerted by water coming through nozzle A on the sprinkler = moment of momentum of water through A = 14 X PX OX Vq = 0.25 x 1000 x .0005026 x 10 clockwise Torque exerted by water coming through nozzle B on the sprinkler = rp X PX OX Vp= 0.20 x 1000 x 0005026 x 10 clockwise ‘Total torque exerted by water on sprinkler 25 x 1000 x .0005026 x 10 + .20 x 1000 x .0005026 x 10 = 1.2565 + 1.0052 = 2.26 Nm ‘Torque required to hold the rotating arm stationary = Torque exerted by water on sprinkler = 2.26 Nm. Ans. Speed of rotation of arm, if free to rotate Let @ = speed of rotation of the sprinkler The absolute velocity of flow of water at the nozzles A and B are V, = 10.0 - 0.25 x @ and V_ = 10.0 ~ 0.20 x @ Torque exerted by water coming out at A, on sprinkler 4 XP XQ V, = 0.25 x 1000 x .0005026 x (10 ~ 0.25 «) 0.12565 (10 - 0.25 «) Torque exerted by water coming out at B, on sprinkler = rp X p XOX V, = 0.20 x 1000 x .0005026 x (10.0 - 0.2 @) 0.10052 (10.0 - 0.2 @) ++ Total torque exerted by water = 0.12565 (10.0 ~ 0.25 @) + 0.10052 (10.0 - 0.2 @) Since moment of momentum of the flow entering is Zero and no external torque is applied on sprinkler, so the resultant torque on the sprinkler must be zero. <- 0.12565 (10.0 - 0.25 «) + 0.10052(10.0 - 0.2 @) = 0 1.2565 - 0.0314 «+ 1.0052 - 0.0201 @=0 1.2565 + 1.0052 = « (0.0314 + 0.0201) 2.2617 = 0.0515 @ = 22617 00515 = 43.9 rad/s, Ans. [ Dynamics of Fluid Flow 301] 60x@ _ 60x439 Son On and, N = 419.2 rpam. Ans. > 6.10 FREE LIQUID JETS Free liquid jet is defined as the jet of water coming out from the nozzle in atmosphere. The path travelled by the free jet is parabolic. Consider a jet coming from the nozzle as shown in Fig. 6.30. Let the jet at A, makes an angle @ with the horizontal direction. If U is the velocity of jet of water, then the horizontal component and vertical component of this velocity at A are U cos @ and U sin 8. Consider another point P(x, y) on the centre line of the jet. The co-ordinates of P from A are x and y. Let the velocity of jet at P in the x- and y-irections are u and v. Let a liquid particle takes time ‘to reach from A to P, Then the horizontal and vertical distances travelled by the liquid particle in time “fate : TRAJECTORY PATH NOZZLE 6.30 Free liquid jet. velocity component in x-direction xr =U cosOxt of) and = (vertical component in e?) irection x time ~ svsmonied gt a From equation (i), the value of ris given as f= Teosd Substituting this value in equation (i x sin @ gx? =Usino - a * Teo cos@ 2U? cos? @ =xtan6- 2-500? @ fo cagewetel 620) U7 wo" Equation (6.24) gives the variation of y with the square of x. Hence this isthe equation of a parabola. ‘Thus the path travelled by the free jet in atmosphere is parabolic. [302 Fluid Mechanics ] () Maximum height attained by the jet. Using the relation V,” — V, case V, = 0 atthe highest point V, = Initial vertical component = Usin® Ve sign on right hand side is taken as g is acting in the downward direction but particles is moving up. . 0- (U sin 0° =~ 2¢x 5 ‘where S is the maximum vertical height attained by the particle. = 2g8, we get in this or =U? sin’e =~ 295 = Pane (6.25) 2 (i Time of flight. 11 is the time taken by the fluid particle in reaching from A to B as shown in Fig. 6.30. Let T'is the time of flight. Using uo weave y= Un 6x 1-2 g@ when peaches f= Dan = 7 :. Above equation becomes as0 = U sin @x T— 3 xP or (Cancelling 7) or (6.26) (ii) Time to reach highest point. The time to reach highest point is half the time of flight. Let T* is the time to reach highest point, then T_ sind _ Usind 2° gx2 g (jv) Horizontal range of the jet. The total horizontal distance travelled by the fluid particle is called horizontal range of the jet, i.e., the horizontal distance AB in Fig. 6.30 is called horizontal range of the jet. Let this range is denoted by x* Then x = velocity component in x-direction time taken by the particle to reach from A to B = U cos 6% Time of flight re 46.27) : 2Usind A g = 2 e080 sin 0 = sin 20 (6.28) @ 8 (®) Value of @ for maximum range. The range x* will be maximum for a given velocity of projection (U), when sin 20 is maximum or when sin 28 = 1 or sin 26 = sin 90° = 1 2 20 = 90° or @= 45° ‘Then maximum range, X%pqq= 2—sin® @= 2 sin 90° = 1) ..(6.29) 8 8 [ Dynamics of Fluid Flow 303 | Problem 6.38 A vertical wall is of & m in height. A jet of water is coming out from a nozzle with a velocity of 20 m/s. The nozzle is situated at a distance of 20 m from the vertical wall. Find the angle of projection of the nozzle t0 the horizontal so that the jet of water just clears the top of the wall Solution. Given : Height of wall Velocity of jet, Distance of jet from wall, Let the required angle Using equation (6.24), we have tan @~ 25 sec? @ ye se ee where y = 8m, x= 20m, U = 20 m/s . = 981x207 > 8=20.an 0- TT sec 8 = 20 tan 8 ~ 4.905 sec? @ = 20 tan 6 ~ 4.905 [1 + tan? 6] = 20 tan @ ~ 4.905 - 4.905 tan? @ + tan? @} or 4.905 tan? 6 ~ 20 tan 6 + 8 + 4.905 = 0 or 4.905 tan? @ — 20 tan @ + 12.905 = 0 tan 9 = 20420? = 412.905 x4.905 _ 20+ J400= 25319 * 2% 4.905 981 = 204VIA6ST _ 20412116 _ 32116 |, 7889 ~98I 981 981 981 = 3.273 or 0.8036 0 = 73° 0.8" or 38° 37. Ans. Problem 6.38 A ire-brigade man is holding a fre sream nozele of 50 mm diameter as shown in Fig. 6.32. The jet issues out with a velocity of 13 m/s and strikes the window. Find the angle or angles of inclination with which the jt issues from the nozzle. What will be the amount of water falling on the window ? Solution. Given : Dia. of nozzle, d=50mm= 05m Area, Velocity of jet, U=13 ms. ‘The jet is coming out from nozzle at A. It strikes the window and let the angle made by the jet at A with horizontal is equal t0 @. ‘The co-ordinates of window, with respect to origin at A. x=Smy=75-15=60m ‘The equation of the jet is given by (6.24) as [304 Fluid Mechanics ] > ye xtand- 2 sec? 2: or 6.0=5x tan 0- 251*5 11 44 Dee or 6.02 5 tn 07256 (1+ tan" 6) 5 tan @~.7256 ~ 1256 tan? 0 or 0.7256 tan? - 5 tan 8+ 6-4 7256 = 0 or 0.7256 tan? @ = 5 tan @ + 6.725 This is a quadratic equation in tan @, Hence solution is 542341 = 14512 14512 es @= tan"! 5.058 or tan! 1.8322 = 78.8° or 61.37". Ans. Amount of water falling on window = Discharge from nozzle Area of nozzle x Velocity of jet at nozzle = 0.001963 x U = 0.001963 x 13.0 = 0.0255 m’/s. Ans. Problem 6.40 A nozzle is situated at a distance of I m above the ground level and is inclined at an angle of 45° to the horizontal. The diameter of the nozzle is 50 mm and the jet of water from the nozzle Strikes the ground at a horizontal distance of 4 m. Find the rate of flow of water. Solution. Given : Distance of nozzle above ground 058 or 1.8322 Angle of inclination, o=4s" Dia. of nozzle, d= 50mm x Area, A= (05) rea, $609) 8 Fig. 6.33 ‘The co-ordinates of the point B, which is on the centre-line of the jet of water and is situated on the ‘The horizontal distance x= 4m ‘ground, with respect to A (origin) are x= 4m and y =~ 1.0 m {From A, point B is vertically down by 1 m} ‘The equation of the jet is given by (6.24) as y = x tan @ ~ wee e Substituting the known values as = 10 = 4 an 45° — 28H sootase aU wai fe [ Dynamics of Fluid Flow 305] -10=4- 0 TBAB? gy TBAB? _ 404 10= v u _ 784820 a) . U = J3139 = 5.60 mis Now the rate of flow of fluid = Area x Velocity of jet AKU = 001963 x 5.6 m'/see 0.01099 = 011 m’ss. Ans. Problem 6.41 A window, in a vertical wall, is ata distance of 30 m above the ground level A jet of water, issuing from a nozzle of diameter 5O mm isto strike the window. The rate of flow of water through the nozzle is 3.5 m'/minute and nozzle is situated at a distance of 1 m above ground level. Find the ‘greatest horigontal distance from the wall of the nozzle so tat jet of water strikes the window. Solution. Given : vw 31.39 Distance of window from ground level = 30 m | puinoow Dia. of nozzle, d= 50-mm = 0.05 m 5 T Area A= F605)" = 0.001963 m? The discharge, Q=3.5 m°/minute 35 = 0.0583 mis 60 Distance of nozzle from ground = 1 m. Let the greatest horizontal distance of the nozzle from the wall If the jet reaches the window, then the point B on the window is on the centre-line of the jet. The co-ordinates of B with respect to A are Fig. 6.34 = x and let angle of inclination = 8. The velocity of jet, v= Discharge 29.69 misee ‘Area A .001963 Using the equation (6.34), which is the equation of jet, y=xtan@~ 2° sec?@ 2? or 29.0 = xtan @- 981 sect 2x (29.69) = xtan 0 ~ 0.0055 sec? 0x x7 =x tan 9 00552 cos" @ tan 8 ~ 0055 x"/c0s°@ - 29 = 0 a) ‘The maximum value of x with respect 8 is obtained, by differentiating the above equation W.r4. © and substituting the value of = 0, Hence differentiating the equation (i) w.rt. 8, we have (2) 1 de EB) ano = var] [sect oto] = 0.0055 ax 8. [306 Fluid Mechanics ] ccna) =sie? ovrmot om {ae AA) 4 1 ds a a 8 \ cos* 6 d8\cos* 6) cos’ 8 dO 2x_de cos70 a0 |=° ax For maximum value of x, wrt. 8, we have “= 0 rr) Substituting this value in the above equation, we have ast oss[ 22380 <0 7 x 9552009 op «oye atl a 7.1 INTRODUCTION Orifice is a small opening of any cross-section (such as circular, triangular, rectangular etc.) on the side or at the bottom of a tank, through which a fluid is flowing. A mouthpiece is a short length of a pipe which is two to three times its diameter in length, fitted in a tank or vessel containing the fluid. Orifices as well as mouthpieces are used for measuring the rate of flow of fluid. > 7.2 CLASSIFICATIONS OF ORIFICES ‘The orifices are classified on the basis of their size, shape, nature of discharge and shape of the upstream edge. The following are the important classifications : 1. The orifices are classified as smalll orifice or large orifice depending upon the size of orifice and head of liquid from the centre of the orifice. Ifthe head of liquid from the centre of orifice is more than five times the depth of orifice, the orifice is called small orifice. And if the head of liquids is less than five times the depth of orifice, it is known as large orifice. 2. The orifices are classified as ()) Circular orifice, (i) Triangular orifice, (if) Rectangular orifice and (iv) Square orifice depending upon their cross-sectional arcas. 3. The orifices are classified as (j) Sharp-edged orifice and (if) Bell-mouthed orifice depending. upon the shape of upstream edge of the orifices. 4, The orifices are classified as (i) Free discharging orifices and (ii) Drowned or sub-merged ori- fices depending upon the nature of discharge. ‘The sub-merged orifices are further classified as (a) Fully sub-merged orifices and (b) Partially sub-merged orifices. >» 7.3 FLOW THROUGH AN ORIFICE Consider a tank fitted with a circular orifice in one of its sides as shown in Fig. 7-1. Let H be the head Of the liquid above the centre of the orifice. The liquid flowing through the orifice forms a jet of liquid whose area of cross-section is less than that of orifice. The area of jet of fluid goes on decreasing and at a section C-C, the area is minimum. This section is approximately at a distance of half of diameter of the orifice. At this section, the streamlines are straight and parallel to each other and perpendicular to the 317 [318 Fluid Mechanics ] plane of the orifice. This section is called Vena-contracta, Beyond this section, the jt diverges and is attracted in the downward direction by the gravity. i Consider two points 1 and 2 as shown in Fig. 7.1. Point Lis ey inside the tank and point 2 at the vena-contracta, Let the flow is 4 | 1 Wa steady and at a constant head H. Applying Bernoull's equation a re poins 1 and 2. © UENtaacta 11 Tank with an orifice. But Now (atmospheric pressure) 48 very small in comparison tov as area of tank is very large as compared to the aren ofthe Je of hig 2e : va= lal 1.) ‘This is theoretical velocity. Actual velocity will be less than this value. H+0=04 > 7.4 HYDRAULIC CO-EFFICIENTS The hydraulic co-efficients are 1. Co-efficient of velocity, C, 2. Covefficient of contraction, C, 3. Covefficient of discharge, Cy. 7.4.1 Co-efficient of Velocity (C,). It is defined as the ratio between the actual velocity of a Jet of liquid at vena-contracta and the theoretical velocity of jet. It is denoted by C, and mathemati cally, C, is given as = Actual velocity of jet at vena -contracta a ‘Theoretical velocity Feat actual velocity, 2H = Theoretical velocity 72) 3 ‘The value of C, varies from 0.95 to 0.99 for different orifices, depending on the shape, size of the orifice and on the head under which flow takes place. Generally the value of C, = 0.98 is taken for sharp-edged orifices. [ Orifices and Mouthpieces 319] 7.4.2 Co-efficient of Contraction (C.). It is defined as the ratio of the area of the jet at vvena-contracta to the area of the orifice. It is denoted by C.. Let a= area of orifice and area of jet at vena-contracta, area of jet at vena-contracta area of orifice Then @ a (73) ‘The value of C, varies from 0.61 to 0.69 depending on shape and size of the orifice and head of liquid under which flow takes place. In general, the value of C, may be taken as 0.64. 7.4.3 Co-efficient of Discharge (C,). It is defined as the ratio of the actual discharge from an orifice to the theoretical discharge from the orifice. It is denoted by C,. If Q is actual discharge and Q,, is the theoretical discharge then mathematically, C, is given as OQ ___ Actual velocity x Actual area ‘, Actual velocity x Actnal area 4 04, Theoretical velocity x Theoretical area Actual velocity Actual area Theoretical velocity” Theoretical arca (y= CXC 5) The value of C, varies from 0.61 to 0.65. For general purpose the value of C, is taken as 0.62. Problem 7.1 The head of water over an orifice of diameter 40 mm is 10 m. Find the actual dis- charge and actual velocity of the jet at vena-contracta. Take C, = 0.6 and C, = 0.98. Solution. Given : Head, H= 100m Dia. of orifice, = 40 mm = 0.04 m Area, a= 08)? = 001256 m? C,=06 Actual discharge _ @ ‘Theoretical discharge But Theoretical discharge = V,, x Area of orifice Vy = Theoretical velocity, where Vy = 2gH = f2x981%10 =14 mis ‘Theoretical discharge = 14 x .001256 = 0.01758 Actual discharge = 0.6 x Theoretical discharge = 0.6 x 01758 = 0.01054 m/s. Ans. [320 Fluid Mechanics ] Actual velocity Theoretical velocity Actual velocity = 0.98 x Theoretical velocity 98 x 14 = 13.72 mvs. Ans. Problem 7.2. The head of water over the centre of an orifice of diameter 20 mm is I m. The actual discharge through the orifice is 0.85 litre/s. Find the co-efficient of discharge. Solution. Given : (i) = C,= 0.98 Dia. of orifice, a= 20 mm = 0.02 m o Area, a= 710.02)" = 0.000314 m* Head, H=1m Actual discharge, Q = 0.85 litres = 0.00085 mls Veh = J2XORTHI= 4.429 mis in X Area of orifice 4.429 x 0.00031. 0.00139 m/s Actual discharge __ 0.00085 charge” 0.00139 ‘Theoretical velocity, Vy s. Theoretical discharge, Q,, Co-efficient of discharge = 0.61. Ans. ‘Theoretical > 7.5 EXPERIMENTAL DETERMINATION OF HYDRAULIC CO-EFFICIENTS 7.5.1 Determination of Co-efficient of Discharge (C4). The water is allowed to flow through an orifice fitted to a tank under a constant head, H as shown in Fig. 7.2. The water is collected in a measuring tank for a known time, t. The height of water in the measuring tank is noted down. Then actual discharge through orifice, q.= Mewof measuring tank_ > Height of water in measuring tank Time () and. theoretical discharge = area of orifice x J2@H SUPPLY WATER i MEASURING TANK —*| Fig.7.2 Value of Cx. 27.5) [ Orifices and Mouthpieces 321] 7.5.2 Determination of Co-efficient of Velocity (C,). Let C-C represents the vena- ccontracta of a jet of water coming out from an orifice under constant head fas shown in Fig. 7.2. ‘Consider a liquid particle which is at vena-contracta at any time and takes the position at P along the jet in time “7. Let x= horizontal distance travelled by the particle in time *t = vetieal distance between P and C-C = actual velocity of jet at vent-contracta, Then horizontal distance, x= Vxt wl) and vertical distance, ys hg? i) From equation (), Pe Vv Substituting this value of *? in (i), we get vee ay vi ext 2y But theoretical velocity, Vy = V2eH v_ fer oo +. Co-efficient of velocity, C,= = J xt = ja “ * MO Voy * Bagel Vayll 7.5.3 Determination of Co-effic is determined from the equation (7.4) as (76) int of Contraction (C,). The co-efficient of contraction ChE CXC, G Cat 7.7) cme aa Problem 7.3 A jet of water, issuing from a sharp-edged vertical orifice under a constant head of 100m, at a certain point, has the horizontal and vertical co-ordinates measured from the vena-contracta 44s 20.0 em and 10.5 om respectively. Find the value of C,, Also find the value of C, if Cy= 0.60. Solution. Given : Head, Horizontal distance, Vertical distance, (322. Fluid Mechanics ] The value of C, is given by equation (7.6) as pee 200 20 Jen Ja0Sx100 20.493 ‘The value of C, is given by equation (7.7) as Co 06 . ns. oo Engrg 7 SIT = ONS, Ans. Problem 7.4 The head of water over an orifice of diameter 100 mm is 10 m. The water coming out {from orifice is collected in a circular tank of diameter 1.5 m. The rise of water level in this tank is 1.0 m in 25 seconds. Also the co-ordinates of a point on the jet, measured from vena-contracta are 4.3 m horizontal and 0.5 m vertical. Find the co-efficients, Cy C, and C- Solution. Given : Head, H Dia. of orifice, 4 0.9759 = 0.976. Ans. 0 m 100 mm = 0.1 m ++ Area of orifice, .007853 m? Dia. of measuring tank, = D= 1.5m + Area, Rise of water, Time, Horizontal distance, Vertical distance, y=0.5m Now theoretical velocity, Vj, = y2gH = y2<9BTXIO = 14.0 mis Theoretical discharge, Qj, = Vy X Area of orifice = 14.0 x 0.007854 = 0.1099 m°/s Axh _ 1.76710 Actual discharge, = AEM MIST «07968 y= 2a SS 968A ‘The value of C, i given by equation (7.6) as G lar a GE, = 096. Ans. Cis given by equation (7.7) as C, = G4 = 968 — 9,669, ans. CG 096 Problem 7.8 Water discharge at the rate of 98.2 litres/s through a 120 mm diameter vertical sharp-edged orifice placed under a constant head of 10 metres. A point, on the jet, measured from the vena-contracta of the jet has co-ordinates 4.5 metres horizontal and 0.54 metres vertical. Find the co-efficient C,, C, and C, of the orifice. [ Orifices and Mouthpieces 323] Solution. Given : Discharge, Dia. of orifice, +: Area of orifice, a= 500.127 = 0.01131 m* Head, H=10m Horizontal distance of a point on the jet from vena-contracta, x = 4.5 m and vertical distance, y = 0.54 m Now theoretical velocity, Vi, Theoretical discharge, Qn, = 2g XH = JEX9TXIO = 14.0 mis iy x Atea of oifice 14.0 x 0.01131 = 0.1583 m? Actual discharge __Q _ 0.0982 The value of C, is given by, Cy= —Actual discharge _ _ 2. _ 0.0982. 9.69, Ans, we value Of Cy is given BY. Ca= Te tical discharge ~ Qy, 0.1583 " The value of C, is given by equation (7.6), x 45 Vat J4x054x10 The value of C, is given by equation (7.7) as Ga 062 °C, 0968 Problem 7.6 4 25 mm diameter nozzle discharges 0.76 m’ of water per minute when the head is 60 m. The diameter of the jet is 22.5 mm. Determine : (i) the values of co-efficients C,, C, and C, and (ii) the loss of head due to fluid resistance. Solution. Given : G 968. Ans. = 0.64. Ans. Dia. of nozzle, D=25 mm = 0.025 m ‘Actual discharge, Qe, = 0.76 mfminute = 79 = 0,01267 m*is Head, NOZZLE JET OF Dia. of jet, WATER (0 Values of co-efficients = Co-efficient of contraction (C,) is given by, Area of jet “Area of nozzle Fig. 7.3 £ 2 4d 00225" 981. Ans. pb DP 002s Co-efficient of discharge (C,) is given by, Actual discharge ‘Theoretical discharge 4 [324 Fluid Mechanics ] 0.01267 ~ Theoretical velocity x Area of nozzle 0.01267 0.01267 eH xt EX 9.B1% 60 x 5 (0.025)* = 0.752. Ans. Co-efficient of velocity (C,) is given by, = Si 9752 «9.098, ans, C081 (i) Loss of head due to fluid resistance : Applying Bernoulli's equation at the outlet of nozzle and to the jet of water, we get, +2 + Loss of head But = Atmospheric pressure head 2 V,= 28H , Vz= Actual velocity of et = C, 28 (28H) (C.2eH) (Pst) (CAREY sot ead 2 28 or H=C2xH + Loss of head -. Loss of head H-C?xH=H(U-C) = 60(1 ~ 0.928%) = 60 x 0.1388 = 8.328 m. Ans. Problem 7.7 4 pipe, 100 mm in diameter, has a nozzle attached to it at the discharge end, the diameter of the nozzle is 50 mm. The rate of discharge of water through the nozzle is 20 litres/s and the pressure at the base of the nozzle is 5.886 N/om*. Calculate the co-efficient of discharge. Assume that the base of the nozzle and outlet of the nozzle are at the same elevation. Solution. Given : Dia. of pipe, Noza.e o Dia. of nozzle, (05)? = .001963 m* BASE OF NOZZLE Actual discharge, Q = 20 livs = 0.02 ms py = 5.886 Niem* Fig. 7.4 a re eerie From continuity equation, A\V, = A,V> or 007854 V, [ Orifices and Mouthpieces 325] 007854 4 where V, and V, are theoretical velocity at sections (1) and (2) Applying Bernoulli's equation at sections (1) and (2), we get By, ed But “y 5886x10" 4 v2 or S8BGXIOT Aa) gy 1000x981” 2¢ 26 2 60+ 2gx16~ 28 or E[i-Z]-60 or ZR 60 2 |! a6 2¢ [16 V,= J60x2x981x-2 = 11.205 misec fy X Ay = 11.205 x .001963 = 0.022 mis :. Theoretical discharge Actual discharge 0.02 charge 0.022 Problem 7.8 A tank has two identical orifices on one of its vertical sides. The upper orifice is 3 m below the water surface and lower one is 5 m below the water surface. If the value of C, for each orifice is 0.96, find the point of intersection of the two jets. Solution. Given : Height of water from orifice (1), From orifice (2), H,=5m , for both = 0.96 Let P is the point of intersection of the two jets coming from orifices (1) and (2), such that x= horizontal distance of P ertical distance of P from orifice (1) Y= vertical distance of P from orifice (2) ya+2m 1.909, Ans. ‘Theoretical 3m ‘Then ‘The value of C, is given by equation (7.6) as [326 Fluid Mechanics ] For orifice (1), on 4y, A, ay, x3.0 ¢,«t—- Yay Hy J4xy, x50 As both the orifices are identical For orifice (2), Cy 26, or —_— or 3y, = Syp fay, x30 ~ Jay, x50 yy ey +20 3(y, + 2.0) = Sy, 2y,= 6.0 2230 From (ii), es rom (i, i 0.96 = ——*___ “" JAx3.0%50 ‘ x= 0.96 x J4XROXSO = 7.436 m. Ans. Problem 7.9 A closed vessel contains water upto a height of 1.5 m and over the water surface there is air having pressure 7.848 N/m? (0.8 keffem") above atmospheric pressure. At the bottom of the vessel there is an orifice of diameter 100 mm. Find the rate of flow of water from orifice. Take Cy = 06. = ‘Solution. Given : Dia. of orifice, d= 100 mm = 0.1 m C,=06 Height of water, H=15m Air pressure, = 7.848 Niem? = 7.848 x 10" Nin? Applying Bernoulli’s equation at sections (1) (water surface) and @), we get we Vee vee, PyM yeh 2g ps 2g PE Taking datum line passing through (2) which is very close to the bottom surface of the tank. Then za =0, z= L5 m Yig.75 Also P2 = 0 (atmospheric pressure) 7848 x 10° and 2818 10- _ 8 m of water (000x981 8+0+15=0+2 40 (V; is negligible} 2g seh 28 [ Orifices and Mouthpieces 327] PXOBIXIS = Rate of flow of water = Cy x a, x V> 3.652 mis 6x F(A)? x 13.652 mls = 0.0643 m's, Ans. Problem 7.10 A closed tank partially filled with water upto a height of 0.9 m having an orifice of diameter 15 mm at the bottom of the tank. The air is pumped into the upper part of the tank. Determine the pressure required for a discharge of 1.5 litres/s through the orifice. Take C, = 0.62. Solution. Given : Height of water above ori Dia. of orifice, 015 m os Area, $ (015)? = 0.001767 m* Discharge, 15 lites/s = 0015 ms 1.62 Let p is intensity of pressure required above water surface in N/em?, Pp __pxl0* _ 0p “Then pressure head of a= -P- = —PX1O™ 10D oy of water ne pg 1000x981 981 tet LF Vs isthe velocity at outlet of ites, then ae(n+2) = fpxssi(ao 22) PS. 981 Discharge Q=C,xax J2e(H* vipa) 0015 = 0.6 x .0001767 x [2 x981(09 + pi pa) v= 0015 __ sig tag top) xt (a9+!22) «4484s 2p, _ HMB XI4IMS 9.9 - 10.202-0.9 = 9.302 981 2x981 >» 7.6 FLOW THROUGH LARGE ORIFICES If the head of liquid is less than 5 times the depth of the orifice, the orifice is called large orifice. In ‘case of small orifice, the velocity in the entire cross-section of the jet is considered to be constant and discharge can be calculated by Q = C, x ax 2gh. But in case of a large orifice, the velocity is not ‘constant over the entire cross-section of the jet and hence Q cannot be calculated by Q= Cx ax J2gh- [328 Fluid Mechanics ] 7.6.1. Discharge Through Large Rectangular Orifice. Consider a large rectangular orifice in one side of the tank discharging freely into atmosphere under a constant head, H as shown in Fig. 7.7. Let H, = height of liquid above top edge of orifice Hz = height of liquid above bottom edge of orifice 6 = breadth of orifice d= depth of orifice = Hy ~ H, C,= co-efficient of discharge Consider an elementary horizontal strip of depth ‘dh’ at a depth of *h’ below the free surface of the liquid in the tank as shown in Fig. 7.7 (b). © Fig. 7.7 Large rectangular orifice. Area of strip = b x dh and theoretical velocity of water through strip = /2gh Discharge through elementary strip is given dQ = C,x Area of stip x Velocity “XD x dh x gh = C,b x gh dh By integrating the above equation between the limits, and FH, the total discharge through the whole orifice is obtained 2» [cy xox aan bx Bil ial = Gye] 2) " 372, Cy xb fe [H3” - H??} (78) Problem 7.11. Find the discharge through a rectangular orifice 2.0 m wide and 1.5 m deep fitted to water tank, The water level in the tank is 3.0 m above the top edge of the orifice. Take Cy = 0.62. Solution. Given : Width of orifice, b=20m Depth of orifice, d=15m Height of water above top edge of the orifice, H, Orifices and Mouthpieces 329] Height of water above bottom edge of the orifice, Hy=Hy+d=3415=45m C,= 0.62 Discharge @ is given by equation (7.8) as Cy xb x fie H32— 371 X 0.62 x 2.0 x /2+98T[4.5'5—3"] miss = 3.66[9.545 — 5.196] m°/s = 15.917 m/s. Ans. Problem 7.12 A rectangular orifice, 1.5 m wide and 1.0 m deep is discharging water from a tank. I the water level in the tank is 3.0 m above the top edge of the orifice, find the discharge through the orifice. Take the co-efficient of discharging for the orifice = 0.6. Solution. Given : Width of orifice, Depth of orifice, 15m 16 Discharge, Q is given by the equation (7.8) as 2 0-2 xc,xbx Jie HHH =2x06x 15x 2937 40'S 3.0'} mis = 2.657 [8.0 ~ 5.196] m°/s = 7.45 m/s. Ans. Problem 7.13 A rectangular orifice 0.9 m wide and 1.2 m deep is discharging water from a vessel. The top edge of the orifice is 0.6 m below the water surface in the vessel. Calculate the discharge through the orifice if Cy = 0.6 and percentage error if the orifice is treated as a small orifice. Solution. Given : Width of orifice, b=0.9m d=12m Depth of ori Discharge Q is given as 2 3 = 2 0.6 x2.9 x J2XORI [1.8% 0.67] m/s 1.5946 [2.4149 ~ 4647] = 3.1097 m’s. Ans. Discharging for a small orifice Q,=C,xax J2eh 12 2manda=6xd=09%1.2 wre b= He =06 [330 Fluid Mechanics 0, = 0.6 x 9x 1.2 x J2xOBIxI2 = 3.1442 mis 0,0 _ 31442-31097 Q 31097 % error = = 0.01109 or 1.109%. Ans. > 7.7 DISCHARGE THROUGH FULLY SUB-MERGED ORIFICE Fully sub-merged orifice is one which has its whole of the outlet side sub-merged under liquid so that it discharges a jet of liquid into the liquid of the same kind. It is also called totally drowned orifice. Fig. 7.8 shows the fully sub-merged orifice. Consider two points (1) and (2), point 1 being in the reservoir on the upstream side of the orifice and point 2 being at the vena-contracta as shown in Fig. 7.8. Let _H, = Height of water above the top of the orifice on the upstream side, Height of water above the bottom of the orifice, Difference in water level, Width of orifice, “o-efficient of discharge. Height of water above the centre of orifice on upstream side Hy-Hy, _ Wy+Hy Fig. 78 Fully sub-merged orifice. =iy+ = =) 2 z Height of water above the centre of orifice on downstream side Atty 0) Applying Bernoulli's equation at (1) and (2), we get a we a+ i yea) ps 2g pg 28 Now Mitte Ps HM _ ipand v, is negligible 2 "pe 2 we 2¢ Area of orifice x (Hy Hy) Discharge through orifice = C, x Area x Velocity = C,X b (Hy ~ Hy) x 2g Q=Cyxb (Hy—Hy)x J2gH 9) [ Orifices and Mouthpieces 331] Problem 7.14 Find the discharge through a fully sub-merged orifice of width 2 m ifthe difference of water levels on both sides of the orifice be 50 cm. The height of water from top and bottom of the orifice are 2.5 m and 2.75 m respectively. Take C, = 0.6 Solution. Given : Width of orifice, Difference of water level, Height of water from top of orifice, Height of water from bottom of orifice, H, Cy Discharge through fully sub-merged orifice is given by equation (7.9) or O= C,x 6x Hy~ Hy) x QQ 0.6 x 2.0 x (2.15 ~ 2.5) x J2X9BIXOS mis = 0.9396 mifs. Ans. Problem 7.15 Find the discharge through a totally drowned orifice 2.0 m wide and I m deep, if the difference of water levels on both the sides of the orifice be 3 m. Take C, = 0.62. ‘Solution. Given : Width of orifice, b=20m Depth of orifice, a=1m. Difference of water level on both the sides H=3m C,= 0.62 Discharge through orifice is Q = C, x Area x J2gH 62x bx dx J2eH = 0.62 x2.0x 1.0 x J2X9RTXS m/s = 9.513 m'ss. Ans. > 7.8 DISCHARGE THROUGH PARTIALLY SUB-MERGED ORIFICE Partially sub-merged orifice is one which has its outlet side partially sub-merged under liquid as shown in Fig. 7.9. It is also known as partially drowned orifice. Thus the partially sub-merged orifice has two portions. The upper portion behaves as an orifice discharging free while the lower portion behaves as sub-merged orifice. Only a large orifice can behave as a partially sub-merged orifice. The total discharge @ through partially sub-merged orifice is equal to the discharges through free and the sub-merged portions. Fig. 7.9 Partially sub-merged Discharge through the sub-merged portion is given by orifice. ‘equation (7.9) Q,= Cy 0 x (Hy — Hx PQ (332. Fluid Mechanics ] Discharge through the free port Q ‘Total discharge o is given by equation (7.8) as 2 cy xox te UH — Hi) Q1+ Q2 “yb x Hy W) x fa +2 Cx 6x J2g LHS? - HP), (7.10) Problem 7.16 A rectangular orifice of 2 m width and 1.2 m deep is fitted in one side of a large tank. The water level on one side of the orifice is 3 m above the top edge of the orifice, while on the other side of the orifice, the water level is 0.5 m below its top edge. Calculate the discharge through the orifice if Cy = 0.64 Solution. Given : Width of orifice, b = 2 m Depth of orifice, d = 1.2 m Height of water from top edge of orifice, Hy = 3 m Difference of water level on both sides, H!= 3 + 0.5 = 3. Height of water from the bottom edge of orifice, H =H, + d=3+1.2=42m “The orifice is partially sub-merged. The discharge through sub-merged portion, “pbx (Hy~ Hx PQ 64 x 2.0 x 4.23.5) x J2KOBTRAS = 7.4249 mls The discharge through free portion is Ca bx Fg UP? — HPP % 0.64 x 2.0 x 2981 [3.5% - 3.07] 1961] = 5.108 m/s. Total discharge through the 0 Q= OQ, + Q,= 7.4249 + 5.108 = 12.5329 m/s. Ans. > 7.9 TIME OF EMPTYING A TANK THROUGH AN ORIFICE AT ITS BOTTOM Consider a tank containing some liquid upto a height of H. Let an orifice is fited at the bottom of the tank. Iti required to find the time forthe liquid surface to fll from the height #, to a height H. Let A= Area ofthe tank a= Area ofthe orifice Initial height ofthe Hiquid fi inal height ofthe liquid t Time in seconds for the qld 0 fal from Hy 0 Hy. Let at anytime, the height of liquid from orifice is and let the liquid surface fall by a small height din me dT. Then Volume of liquid leaving the tank in ime, d7= Ax dh — = ish Fig. 79. (@) fh Also the theoretical velocity through ori [ Orifices and Mouthpieces 333] Discharge through orifice/sec, Q = Cx Area of orifice x Theoretical velocity = Cy..a.. /2gh Discharge through orifice in time interval at =C,.0. f3gh dT {As the volume of liquid leaving the tank is equal to the volume of liquid flowing through orifice in time dT, we have AG di) = Cy.a. (ah aT — ve sign is inserted because with the increase of time, head on orifice decreases. Adh Ath)? Cy-a.¥2gh Cy .a.y28° By integrating the above equation between the limits H, and H1 the total time, Tis obtained as dh ~Adh=Cy.a. gh dT or aT Ah" dh Hy SAN a 1 an Cy. 28 Ca Padi ‘ Te 1 or ee ee age] Tay] ~ Gage] E 2 Ms, 2 dn, : 2a[ Vi - Vi =A m)-2NEB) AY) Gua Pie yt Pe For emptying the tank completely, H, = 0 and hence = 2A VE a2) © Cy.a.52g Problem 7.17 A circular tank of diameter 4 m contains water upto a height of 5 m. The tank is provided with an orifice of diameter 0.5 m at the bottom. Find the time taken by water (i) to fal from 5 m to 2 m (ii) for completely emptying the tank. Take Cy = 0.6. Solution. Given : Dia. of tank, D=4m + Area, Dia. of orifice, d=05m “Arca, a= F (5° = 0.1963 m* Initial height of water, Hy = 5m Final height of water, (3) First Case. When [334 Fluid Mechanics ] 212566 * paves x Bear A592] scons = 20653 _ 39.58 seconds. Ans. 5217 Second Case. When H, = 2A 212.566 x45 Gyea.ye MO * 96% .1963 x f2 987 = 107.7 seconds. Ans. Problem 7.18 A circular tank of diameter 1.25 m contains water upto a height of 5 m. An orifice of 50 mm diameter is provided at its bottom. If Cy = 0.62, find the height of water above the orifice after 15 minutes. Solution. Given : Dia. of tank, D=125m X25)? 2 + Area, A= 50.25)" = 1.227 m Dia. of orifice, 4= 50 mm=.05 m Area, Initial height of water, Time in seconds, Let the height of water after 90 seconds = H, Using equation (7.11), we have T- 2x1227[5-Ji] DarxomIeK 2x9RT __ 90 455215 35.215 [2.236 - J] or = 236 - 0.1977 = 2.0383 .0383 x 2.0383 = 4.154 m. Ans. [ Orifices and Mouthpieces 335) > 7.10 TIME OF EMPTYING A HEMISPHERICAL TANK Consider a hemispherical tank of radius R fitted with an orifice of area ‘a’ at its bottom as shown in Fig. 7.10. The tank contains some liquid whose initial height is 4, and in time T, the height of liquid falls to Hy. It is required to find the time 7. Let at any instant of time, the head of liquid over the orifice ‘rand at this instant let xbe the radius ofthe liquid surface. Then ‘Area of liquid surface, A = mx” and theoretical velocity of liquid = gh. Fig. 740 Hemispherical tank. Let the liquid level falls down by an amount of dh in time dT. Volume of liquid leaving tank in time d7™= A x dh = me xdh ei) Also volume of liquid flowing through orifice = C, x area of orifice x velocity = Cya. Y2gh second ‘Volume of liquid flowing through orifice in time d7™ = Cpa. J2gh x aT id, From equations (f) and (ii), we get na? (dh) = Cya. gh . dT ~ve sign is introduced, because with the increase of 7; lt will decrease . =n dh= Cpa. gh dT iii) But from Fig. 7.10, for AOCD, we have OC = R DO=R-h cD=x= oc? - oD F PR - (Rh) = Substituting x? in equation (ii), we get = RQRh~ h?\dh = Cya. f2gh . aT IR? —(R-h) (+ HP = Rh) = 2Rh — HP aebee—)ae n=") ah “ Cah” Gyo ag RH) -—- 12 _ pan § ag Rw Ha ‘The total time T required to bring the liquid level from H, to H, is obtained by integrating the above ‘equation between the limits H, and H. [336 Fluid Mechanics pi 9p gry aR wee Gxax aft ori 2h, = fa x2 ap 3 acre [i ef (HP = mp? * Gxaxdig 3 = [4 erat _ pan orca Ei For completely emptying the tank, H, = 0 and hence 4 aye T a, e(7.14) Problem 7.19 A hemispherical tank of diameter 4 m contains water upto a height of 1.5 m. An orifice of diameter 50 mm is provided at the bottom. Find the time required by water (i) to fall from 06. 1.5 m to 1.0 m (ii) for completely emptying the tank. Tank C, Solution. Given : Dia, of hemispherical tank, D = 4 m Radius, R=20m Dia. of orifice, d= 50 mm = 0.05 m es Area, $05)? = 0.001963 m* Initial height of water, First Case. Hy = 1.0 Time T is given by equation (7.13) ou] 32 gx) 2 (552 z x[$xao(ist 10") -2 (se 0:6x.001963x J2x981 602.189 [2.2323 ~ 0.7022} = 921.4 second 1S min 21.4 sec. Ans. Second Case. H, = 0 and hence time T is given by equation (7.14) a] Tem rom G7 21s? Fos] 5 RE? C, a. 2g 13 * ex oones EXOH o] [ Orifices and Mouthpieces 337] = 602.189 [4.8989 ~ 1.1022] sec = 2286.33 sec 8 min 6.33 see. Ans. Problem 7.20 A hemispherical cistern of 6 m radius is full of water. It is flted with a 75 mm diameter sharp edged orifice at the bottom. Calculate the time required to lower the level in the cistern by 2 metres, Assume co-efficient of discharge for the orifice is 0.6. Solution. Given : Radius of hemispherical cistern, R = 6m Initial height of water, m Dia. of orifice, ‘5 mm = 0.075 m c Area, a= £075)? = 004418 m? Fall of height of water =2m Final height of water, H, = 6-2=4m C,=06 ‘The time T is given by equation (7.31) Tr 4 4p axa x * 0.6 x.004418 x f2x9.81 x [$xo(ao" -40)-2 (60% -40%)] 67.56 [8(14.6969 - 8.0) - 0.4 (88.18 - 32.0)] = 267.56 [53.575 - 22.472] sec 8321.9 see Problem 7.21 A cylindrical tank is having a hemispherical base. The height of cylindrical portion is 5 m and diameter is 4 m. At the bottom of this tank an orifice of diameter 200 mm is fitted. Find the time required to completely emptying the tank. Take C, = 0. hrs 18 min 42 sec. Ans. Solution. Given : Height of eylindrical portion (I) Dia, of tank 40m sm Area, Az = aay " 4 j= 4.0 m ——+f Dia. of ariftce, 4=200 mm = 0.2m ORIFICE Fig. 7.11 (338. Fluid Mechanics ] ‘The tank is splitted in two portions. First portion is a hemispherical tank and second portion is cylindrical tank. Let T, = time for emptying hemispherical portion 1 T,, = time for emptying cylindrical portion II. Then total time T= T, + Ty For Portion I. H, = 2.0 m, H,= 0. Then 7; is given by equation (7.14) as 2x20| 5 x 4 7.646 (7.5424 ~ 2.262] sec = 198.78 sec. 202.087 = For Portion IL. #1, = 2.0 + 5.0 = 7.0 m, Hy = 2.0. Then T; is given by equation (7.11) as 2A[ JH = J] | 2125667120) 55.99 se Ty = NIE) ON Cy xax2g 06x.0314x J2x981 Total time, T=, + T= 198.78 + 370.92 = 569.7 see =9min 29 sec. Ans. > 7.11 TIME OF EMPTYING A CIRCULAR HORIZONTAL TANK Consider a circular horizontal tank of length L and radius R, containing liquid upto a height of H, Let an orifice of area ‘a’ is fitted at the bottom of the tank. Then the time required to bring the liquid level from H, to H, is obtained as = Let at any time, the height of liquid over orifice is ‘hand in time d7, let the height falls by an height of ‘dh’. Let at this time, the width of liquid surface = AC as shown in Fig. 7.12. b+ 1 —_. Surface area of liquid = L x AC But ac=2%an=2[[a0% = 08" = a [a =(R= A) | = yh? = (R20? —2Rh) = 2Y2RH— [ Orifices and Mouthpieces 339 | Surface area, A=Lx 22Rh= WP ‘Volume of liquid leaving tank in time dT = Axdh=2L J2Rh—W? xdh od) Also the volume of liquid flowing through orifice in time dT” = C,x Area of orifice x Velocity x dT But the velocity of liquid at the time considered = J2gh ‘Volume of liquid flowing through orifice in time dT = Cyxax Fgh xaT Aid) Equating (i) and (i), we get 2L Y2Rh—h? x (~ dh) = C, xa x 2ghxaT ~ ve sign is introduced as with the increase of T, the height hr decreases, -2L{2Rh—W dh -2L f@R—M dh ax 2gh , xax oe [Taking Vi common] =2L(2R-h)'” dh Total ime, T= f Cxex le -2L pth rs =n) dh Gxaxfg PR" 1, oR— ni" One x1) Tat 2 In 2 2 sia Cxan ig 3* (eR hn 4 a va = 4 (or- 4," -(0R- #, 7.15 saxon fal 2? -(2R- 4,7] vas) For completely emptying the tank, H = 0 and hence aL rt 2 a -(er- #4)". 7. Tr aang (OR) (2R- H,) } (7.16) Problem 7.22 An orifice of diameter 100 mm is fitted at the bottom of a boiler drum of length 5 m and of diameter 2 m. The drum is horizontal and half full of water. Find the time required to empty the boiler, given the value of Cq = [340 Fluid Mechanics ] Solution. Given : Dia. of orifice, Area, Length, of drum, Radius, Initial height of water, Final height of water, 4x50 * 3x06 x 007854 x 2981 = 319.39 [2.8284 ~ 1.0] = 583.98 sce = 9 min 44 sec. Ans. Problem 7.23 An orifice of diameter 150 mm is fitted at the bottom of a boiler drum of length 8 m and of diameter 3 metres. The drum is horizontal and contains water upto a height of 2.4 m. Find the (exp? -@x1-)") time required to empty the boiler. Take Cy = 0.6. Solution. Given : Dia. of orifice, d= 150 mm=0.15 m Area, a= E(15)? = 0.01767 m? Length, L=80m Dia. of boiler, D=30m +: Radius, R=15m Initial height of water, Hy = 2.4m Find height of water, Hy 0 C= 06. For completely emptying the tank, T is given by equation (7.16) as, ory"? OR Hy" . 4x80 3x 6X.01767 x2 XOBT = 227.14 [5.196 ~ 0.4647] = 1074.66 see = 17 min 54.66 sec. Ans. [2x 15?- 2x 15-24)" [ Orifices and Mouthpieces 341] > 7.12 CLASSIFICATION OF MOUTHPIECES 1. The mouthpieces are classified as () Extemal mouthpiece or (ii) Internal mouthpiece depend- ing upon their position with respect to the tank or vessel to which they are fitted. 2. The mouthpiece ate classified as (i) Cylindrical mouthpiece of (ii) Convergent mouthpiece or (iii) Convergent-divergent mouthpiece depending upon their shapes. 3. The mouthpieces are classified as (?) Mouthpieces running full or (i) Mouthpieces running free, depending upon the nature of discharge at the outlet of the mouthpiece. This classification is only for internal mouthpieces which are known Borda’s or Re-entrant mouthpieces. A mouthpiece is said to be running free if the jet of liquid after contraction does not touch the sides of the mouthpiece. But if the jet after contraction expands and fills the whole mouthpiece it is known as running full > 7.13 FLOW THROUGH AN EXTERNAL CYLINDRICAL MOUTHPIECE ‘A mouthpiece is a short length of a pipe which is two or three times its diameter in length. If this pipe is fitted externally to the orifice, the mouthpiece is called external cylindrical mouthpiece and. the discharge through orifice increases. Consider a tank having an external cylindrical mouthpiece of rp cross-sectional area aj, attached to one of its sides as shown in = Fig. 7.13. The jet of liquid entering the mouthpiece contracts to form a 4 vena-contracta ata section C-C. Beyond this section, the jet again is expands and fll the mouthpiece completely. Let H=Height of liquid above the centre of mouthpiece Figs 743 External cylindrical v¢= Velocity of liquid at C-C section mouthpieces. a, = Area of flow at vena-contracia ¥, = Velocity of liquid at outlet 4, = Area of mouthpiece at outlet C, = Co-efficient of contraction. ‘Applying continuity equation at C-C and (1)-(1), we get But * Please refer Art. 11.4.1 for loss of head due to sudden enlargement. (342. Fluid Mechanics But v= ence hy = 02 -f les | _ 0375 2g 2g [0.62 28 where <4=21.¥4 is negligible, PL = atmospheric pressure = 0.855 JH ‘Theoretical velocity of liquid at outlet is vy, = J2gH Co-efficient of velocity for mouthpiece Actual velocity ‘Theoretical velocity 0.855 2a SSO Ve8H = 0.855. Pet C, for mouthpiece = 1 as the area of jet of liquid at outlet is equal tothe area of mouthpiece at outlet. Thus y= CX C, = 10x 855 = 0.855 Thus the value of C, for mouthpiece is more than the value of C, for orifice, and so discharge through mouthpiece will be more. Problem 7.24 Find the discharge from a 100 mm diameter external mouthpiece, fitted to a side of a large vessel ifthe head over the mouthpiece is 4 metres. Solution. Given : Dia. of mouthpiece = 100 m = 0.1 m * Area, (0.1)? = 0.007854 m? Om = 0.855 C,x Area x Velocity = 0.855 x a xJ2gH = 855 x 007854 x J2X9RTX40 = 05948 m*/s. Ans. Problem 7.25 An external cylindrical mouthpiece of diameter 150 mm is discharging water under a constant head of 6 m. Determine the discharge and absolute pressure head of water at vena-contracta, Take C, = 0.855 and C,’ for vena-contracta = 0,62. Atmospheric pressure head = 10.3 m of water. [ Orifices and Mouthpieces 343 | Solution. Given : Dia. of mouthpiece, d= 150 mm = 0.15 em Area, a= 515)? = 0.01767 m* Head, H=60m Cy= 0.855 , at vena-contracta = 0.62 Atmospheric pressure head, H,, = 10.3 m . Discharge =C,xax J2gH = 0.895 x 01767 x 2X 9BTX60 = 0.1639 ms. Ans. Pressure Head at Vena-contracta Applying Bernoulli’s equation at A and C-C, we get Bu Fig. 714 Bu 1 fA xdenen- 2s 2g” (62) But H=1375 2h 26 2 “4 lonnu 2g * 1375 + H-rrHx, (62) 89H =H, +H-189 H=H, 10.3 ~ 89 x 6.0 4 = 4.96 m (Absolute). Ans. vv H,= 103 and H [344 Fluid Mechanics > 7.14 FLOW THROUGH A CONVERGENT-DIVERGENT MOUTHPIECE If a mouthpiece converges upto vena-contracta and then diverges as shown in Fig. 7.15 then that type of mouthpiece is called Convergent-Divergent Mouthpiece. As in this mouthpiece there is no sudden enlargement of the jet, the loss of energy due to sudden enlargement is eliminated. The co- efficient of discharge for this mouthpiece is unity. Let H is the head of liquid over the mouthpiece. Applying Bernoulli's equation to the free surface of water in tank and section C-C, we have py as pg 2g : Ps H,+0+H= x (i) ps enetet o| 1 es Ae ii cv 2g a divergent mouthpiece. norte 2g Also from (i), H, + v22g=H+H, ‘ H, + {Rg =H +H, v, = 2a Now by continuity equation, a ¥,= ¥) x a (1.7) The discharge, Q is given as Q =a, x Y2gH (7.18) where a, = area at vena-contracta, Orifices and Mouthpieces 345) Problem 7.26 A convergent-divergent mouthpiece having throat diameter of 4.0 cm is discharging water under a constant head of 2.0 m, determine the maximum outer diameter for maximum discharge. Find maximum discharge also. Take H,, = 10.3 m of water and Hg = 2.5 m of water (absolute) Solution. Given : Dia. of throat, Oem . Area, (4) = 12.566 em? Constant head, H=2.0m Find max. dia, at outlet, d; and Qyax H, = 10.3 m 2.5 m (absolute) ‘The discharge, Q in convergent-divergent mouthpiece depends on the area at throat, = 12.566 x J2X9BIX200 = 7871.5 em'ss. Ans. Qua = 4% 29H Now ratio of areas at outlet and throat is given by equation (7.17) as = i+ = t+ 08525 Us Hee Hag = H 2.0 * = 2.2135, Bap [az =2.2135 ot AY 222135 a4la , 4s Jaa = 14877 d, = 14877 x d, = 14877 x 4.0 = 5.95 em. Ans. Problem 7.27 The throat and exit diameters of convergent-divergent mouthpiece are 5 cm and 10.cm respectively. It is fitted to the vertical side of a tank, containing water. Find the maximum head of a water for steady flow. The maximum vacuum pressure is 8 m of water and take atmospheric pressure = 10.3 m water. Solution. Given : Dia, at throat, Dia. at exit, Atmospheric pressure head, H, = 10.3 m ‘The maximum vacuum pressure will be at a throat only Pressure head at throat = 8 m (vacuum) or H,= H, ~ 8.0 (absolute) = 103 - 8.0 = 23 m (abs.) 1m of water. Let maximum head of water over mouthpiece ‘The ratio of areas at outlet and throat of a convergent-divergent mouthpiece is given by equation (7.17). [346 Fluid Mechanics ] x (ay Ss of, mai «4 fj, S23 al) 3 8 8 he or = 14 or 15= 7 3333 m of water Maximum head of water = 0.533 m. Ans. Problem 7.28, A comvergentadivergent moutpiece i fted 19 the se ofa tank The discharge through mouthpiece under a constant head of 1.5 m is 5 litres/s. The head loss in the divergent portion is 0.10 times the kinetic head at outlet. Find the throat and exit diameters, if separation pressure is 2.5.m and atmospheric pressure head = 10.3 m of water. Solution. Given : Constant head, Discharge, 1h, or Head loss in divergent = 0.1 x kinetic head at outlet HOF Higy = 2.5 (abs.) (0.3 m of water Find (i) Dia. at throat, d, (ii) Dia. at outlet, d, ( Dia. at throat (d,). Applying Bernoulli's equation to the free water surface and throat section, wwe get (See Fig. 7.15). Pah ee ety og 2g **" pg” 2¢** ‘Taking the centre line of mouthpiece as datum, we get H,+0+H=H.+ 2 28 “8 2 H,4H-H,= 103415 ~2.5=9.3 mof water [2x9B1X93 xy, oF 005 = F d.?x 13.508 W5xa O00AT = 0217 m= ‘x 13.508 ” (i Dia. at outlet (d,). Applying Bernoulli’s equation to the free water surface and outlet of mouth- piece (See Fig. 7.15), we get 3.508 mis Now 17 em. Ans. [ Orifices and Mouthpieces 347) } Pr vi By cosa fa 0+0. 36 z netieaeteut 2g 2g 2g . vee PE «PESOS. 5 sng Ara Now =Ain of 005-742 xy, 4 a, = [=O _ [005 1S Vaexy, ” Yexsit24 > 7.15 FLOW THROUGH INTERNAL OR RE-ENTRANT ON BORDA’S MOUTHPIECE 1.035 m = Sem. Ans. ‘A short cylindrical tube attached to an orifice in such a way that the tube projects inwardly to a tank, is called an internal mouthpiece. It is also called Re-entrant or Borda’s mouthpiece. If the length of the tube is equal to its diameter, the jet of liquid comes out from mouthpiece without touching the sides of the tube as shown in Fig. 7.16. The mouthpiece is known as running free. But if the length of the tube is about 3 times its diameter, the jet comes out with its diameter equal to the diameter of ‘mouthpiece at outlet as shown in Fig. 7.17. The mouthpiece is said to be running full, (® Borda's Mouthpiece Running Free. Fig. 7.16 shows the Borda’s mouthpiece running free. Let height of liquid above the mouthpiece, area of mouthpiece, 4. = area of contracted jet in the mouthpiece, velocity through mouthpiece. RUNNING FREE RUNNING FULL ig. 716 Fig. 7.17 ‘The flow of fluid through mouthpiece is taking place due to the pressure force exerted by the fluid ‘on the entrance section ofthe mouthpiece. As the area of the mouthpiece is “a’ hence total pressure force on entrance = pe ah where h= distance of C.G. of area ‘a’ from free surface = H. =pe.a.H ol) [348 Fluid Mechanics ] According to Newton's second law of motion, the net force is equal to the rate of change of momen- tum. Now mass of liquid flowing/sec = p x a, x ¥, ‘The liquid is initially at rest and hence initial velocity is zero but final velocity of fluid is v,. Rate of change of momentum = mass of liquid flowing/sec x [final velocity ~ initial velocity] = pa. x vey. 0] = pa. v2 did) Equating (i) and (i), we get pg .a.H. = pa,. Applying Bernoulli’s equation to free surface of liquid and section (1)-(1) of Fig. 7.16 eliti) Petey ze Pe 2g” pg 28 Taking the centre line of mouthpiece as datum, we have v= 2g Substituting the value of v, in (ii), we get pg.a.H.=p.a..2g.H 1 2 fe +. Co-ticient of conuaction, C,= 2 0s Since there is no loss of head, co-efficient of velocity 10 Co-efficient of discharge, C,= C, x C,= 0.5 x 1.0= 0.5 Discharge Q=C,a\2gH (7.19) = 05x a2eH (i) Borda’s Mouthpiece Running Full. Fig. 7.17 shows Borda's mouthpiece running full. Let H = height of liquid above the mouthpiece, vy, = velocity at outlet or at (1)-(1) of mouthpiece, a= area of mouthpiece, 4, = area of the flow at C-C, v, = velocity of liquid at vena-contracta or at C-C. ‘The jet of liquid after passing through C-C, suddenly enlarges at section (1)-(1). Thus there will be a loss of head due to sudden enlargemer (= hy 0) [ Orifices and Mouthpieces 349) Now from continuity, we have a, ¥, = ay X or Substituting this value of v, in (), we get 2g 28 ‘Applying Bemoull's equation o fice surface of water in tank and section (1), we ast Lelece De teneh, pg 2g pg 2 Taking datum tine passing through the centre line of mouthpiece 0+0+H=04+ 1 404 *L 2 v= VeH Here v, is actual velocity as losses have been taken into consideration, But theoretical velocity, vj, = 2g vet :. Co-efficient of velocity, C,= “> = ¥S— == 0.707 va Patt” JE As the area of the jet at outlet is equal to the area of the mouthpiece, hence co-efficient of ‘contraction = 1 1X C, = LO .707 = 0.707 Discharge, $: C,xax J2gH = 0.107 xax 2H (7.20) Problem 7.29 An internal mouthpiece of 80 mm diameter is discharging water under a constant head of 8 metres. Find the discharge through mouthpiece, when (i) The mouthpiece is running free, and (ii) The mouthpiece is running full. Solution. Given : Dia, of mouthpiece, d = 80 mm = 0.08 m Area, a= £08)" = 005026 Constant head, H=4m. ( Mouthpiece running free. The discharge, Q is given by equation (7.19) as Q=05xax Jig = 05 x.005026 x J2xORIKAD 0.02226 m/s = 22.26 litres/s. Ans. (Gi) Mouthpiece running full. The discharge, Q is given by equation (7.20) as 350 Fluid Mechanics 2 1707 x ax J2gH 0.707 x .005026 x J2x9B1x4.0 = 0.03147 m/s = 31.47 litre/s. Ans. HIGHLIGHTS |= Orifice is a small opening on the side or atthe bottom of a tank while mouthpiece is a short length of pipe which is two or three times its diameter in length. (Otifices as well as mouthpieces are used for measuring the rate of flow of liquid. ‘Theoretical velocity of jet of water from orifice is given by V = Y2aH, where H = Height of water from the centre of orifice ‘There are three hydraulic co-efficients namely Actual velocity at vena -contraeta | __ ‘Theoretical velocity Jon (a) Co-eficient of velocity, C, Area of jet at vena ~contracta ‘Area of orifice ‘Actual discharge ‘Theoretical discharge Where + and y are the co-ordinates of any’ point of jet of water from Vena-contract, A large orifice is one, where the head of liquid above the centre of orifice is less than 5 times the depth of, fice. The discharge through a large rectangular o 0-2 cyxbx feito? — 1771 (&) Co-efficient of contraction, C, (©) Coefficient of discharge, where b= Widih of orifice, C,= Co-eficient of discharge for orifice, Height of liquid above top edge of orifice, and Height of liquid above bottom edge of orifice. ‘The discharge through fully sub-merged orifice, Q = C, x bx (H, ~ H,) x [29H where _b = Width of orifice, Cy = Co-etficient of discharge for orifice, 11, = Height of liquid above bottom edge of orifice on upstream side, H, = Height of liquid above top edge of orifice on upstream side, H = Difference of liquid levels on both sides ofthe orifice. Discharge through partially sub-merged orifice, O=0,+ = Cyb (Hy H) x J2gH +213 Cb x JBg (Hh? where b = Width of orifice Gj My, Hy and H are having their usual meaning. ‘Time of emptying a tank through an orifice at its bottom is given by, 2alsmi— Gye inital height of liquid in tank, Final height of liquid in tank, r where Hy Orifices and Mouthpieces 351 10, u 2 13. 4 A= Area of tank, a= Area of orifice, C,= Co-efficient of discharge. IF'the tank isto be completely emptied, then time 7, 2aVv Gy.4.2e Time of emptying a hemispherical tank by an orifice fitted at its bottom, a #°)] wale ‘and for completely emptying the tank, T= ——™ —_| rs cia emt T= ‘where R= Radius of the hemispherical tank, Hi, = Initial height of liquid, ial height of liquid, rea of orifice, and ‘efficient of discharge. ‘Time of emptying a circular horizontal tank by an orifice atthe bottom of the tank, og 3C,.4. 2g aL and for completely empying the tank, T= 5 (2A — OR Hy" 3G, a. Pe r. (OR Hy? R-H,)"") ‘where L = Length of horizontal tank. Coefficient of discharge for, (@ External mouthpiece, 855 i, Internal mouthpiece, running full, 07 (ii) Internal mouthpiece, running free, C, =0.50 (iv) Convergent or convergent-livergent, Foran external mouthpiece, absolute pressure head at vena-contracta H =H, 089 H where H, = atmospheric pressure head = 10.3 m of water Hf = head of liquid above the mouthpiece For a convergentdivergent mouthpiece, the ratio of areas at outlet and at vena-contracta is Where a, = Area of mouthpiece at outlet 44, = Area of mouthpiece at vena-contracta H, = Atmospheric pressure head LH, = Absolute pressure head at vena-contracta H = Height of liquid above mouthpiece. In case of internal mouthpieces, ifthe jet of liquid comes out from mouthpiece without touching its sides itis known as running free. But if the jet touches the sides of the mouthpiece, itis known as running full (352 Fluid Mechanics ] 2 13. 4 EXERCISE (A) THEORETICAL PROBLEMS Define an orifice and a mouthpiece. What is the difference between the two ? Explain the classification of orifices and mouthpieces based on their shape, size and sharpness ? What are hydraulic co-efficients ? Name them, Define the following co-efficients = (i) Co-effici (iii) Co-elficient of discharge. Derive the expression Cy = C, x Ce Define vena-contracta, Differentiate between a large and a small orifice. Obtain an expression for discharge through a large rectangular orifice What do you understand by the terms wholly sub-merged orifice and partially sub-merged orifice ? Prove that the expression for discharge through an external mouthpiece is given by = .855xaxv where a = Area of mouthpiece at outlet and ¥y = Velocity of jet of water at outlet Distinguish between : (i) External mouthpiece and internal mouthpiece, (if) Mouthpiece running free and ‘mouthpiece running full, Obtain an expression for absolute pressure head at vena-contracta for an external mouthpiece. What is a convergent-divergent mouthpiece ? Obtain an expression for the ratio of diameters at outlet and at Vena-contracta for a convergent-divergent “mouthpiece’ in terms of absolute pressure head at vena- contract, head of liquid above mouthpiece and atmospheric pressure head ‘The length of the divergent outlet part in a Ventutimeter is usually made longer compared with that of the converging inlet part. Why ? Justify the statement, “In a convergent-divergent mouthpiece the loss of head is practically eliminated” nt of velocity, (ii) Co-efficient of contraction and (B) NUMERICAL PROBLEMS ‘The head of water over an orifice of diameter SO mmm is 12 m. Find the actual discharge and actual velocity of jet at vena-contracta. Take C,=0.6 and C, = 0.98, {[Ans. 018 m'/s ; 15.04 mvs} ‘The head of water over the centre of an orifice of diameter 30 mm is 1.5 m The actual discharge theough the orifice is 2.35 litres/see. Find the co-efficient of discharge. [Ans. 0.613] }. A jet of water, issuing from a sharp edged vertical orifice under a constant head of 60 em, has the Norizon- tal and vertical co-ordinates measured from the vena-contracta at a certain point as 10.0 cm and 0.45 em respectively. Find the value of C,. Also find the value of C, if C,= 0.60. [Ans, 0,962, 0.623] J. The head of water over an orifice of diameter 100 min is 5 m. The water coming out from orifice is collected in a circular tank of diameter 2 m. The rise of water level in circular tank is 45 m in 30 seconds. Also the co-ordinates of a certain point on the jet, measured from vena-contracta are 100 em horizontal and 5.2 em vertical. Find the hydraulic co-efficients C,, C, and C.. TAns. 0.605, 0.98, 0.617] ‘A tank has two identical orifices in one of its vertical sides. The upper orifice is 4 m below the water surface and lower one 6 m below the water surface. If the value of C, foreach orifice is 0.98, find the point of intersection of the two jets. Ans. At a horizontal distance of 9.60 em] A closed vessel contains water upto a height of 2.0 m and over the water surface there is air having pressure 8.829 N/em? above atmospheric pressure, At the bottom of the vessel there isan orifice of diam cer 15 cm. Find the rate of flow of water Irom orifice. Take C, = 0.6, Ans. 0.15575 ms) Orifices and Mouthpieces 353 | 8, 10. u. 12. 1B. 4. 15. 16. 1 1. 20. a. 2. 2. A closed tank partially filled with water upto a height of | m, having an orifice of diameter 20 mm atthe bottom of the tank, Determine the pressure required fora discharge of 3.0 litres/s through the orifice. Take C= 0:82. [Ans. 10.88 Nlem*] Find the discharge through a rectangular orifice 3.0 m wide and 2 m deep fitted to a water tank. The water evel in the tank is 4 m above the top edge ofthe orifice. Take C, = 0.62 [Ans 36.77 m/s] A rectangular orifice, 2.0 m wide and 1.5 m deep is discharging water from a tank. Ifthe water level inthe tank is 3.0 m above the top edge of the orifice, find the discharge through the orifice. Take C,= 0.6. [Ans. 15.40 m/s} A rectangular orifice, 1.0 m wide and 1.5 m deep is discharging water from a vessel. The top edge of the orifice is 0.8 m below the water surface in the vessel. Calculate the discharge through the orifice if C= 0.6. Also caleulate the percentage error ifthe orifice is treated as a small orifice. [Ams. 1.058%] Find the discharge through a fully sub-merged orifice of width 2 m ifthe difference of water levels on both the sides ofthe orifice be 800 mm. The height of water from top and bottom of the orifice are 2.5 m and 3 m respectively. Take C, = 0.6. (Ans. 2377 ms} Find the discharge through a totally drowned orifice 1.5 m wide and I m deep, ifthe difference of water levels on both the sides of the orifice be 2.5 m. Take C, = 0.62. [Ans 6.513 ms] A rectangular orifice of 1.5 m wide and 1,2 m deep is fitted in one side ofa large tank. The water level on one side of the orifice is 2 m above the top edge of the orifice, while on the other side of the orifice, the ‘water level is 0.4 m below its top edge. Calculate the discharge through the orifice if C, = 0.62. Ans. 7.549 mss} A circular tank of diameter 3 m contains water upto a height of 4 m, The tank is provided with an orifice of diameter 0.4 m at the bottom. Find the time taken by water :(?) 0 fall from 4 m to 2 m and (ii) for completely emptying the tank, Take C, = 0.6. Ans. (3) 248 s, (i) 84.7] A circular tank of diameter 1.5 m contains water upto a height of 4 m, An orifice of 40 mm diameter is provided at its bottom. If C,= 0.62, find the height of water above the orifice after 10 minutes. (Ans. 2m] A hemispherical tank of diameter 4 m contains water upto a height of 2.0 m. An orifice of diameter 50 mm is provided atthe bottom. Find the time required by water () to fall from 2.0 m to 1.0m (i) for completely emptying the tank. Take C,= 0.6 [Ans. () 30 min 14.34 s, (a) 52 min 59 5] ‘A hemispherical cistern of 4m radius is full of water It is fitted with a 60 mm diameter sharp edged orifice atthe bottom. Calculate the time required to lower the level in the cistern by 2 metres. Take C= 0.6. Ans. Ir $8 min 45.9 s} ‘A cylindrical tank is having a hemispherical base. The height of cylindrical portion is 4 m and diameter is 3m. At the bottom of this tank an orifice of diameter 300 mm is fitted. Find the time required to completely emptying the tank. Take C, = 0.6 Ans. 2 min 7.37 s] ‘An otifice of diameter 200 mm is fitted atthe bottom of a boiler drum of length 6 m and of diameter 2m, ‘The drum is horizontal and half full of water. Find the time required to empty the boiler, given the value of C,=06 [Ans. 2 min 55.20 s} ‘An otifice of diameter 150 mm is fitted at the bottom of a boiler drum of length 6 m and of diameter 2m, ‘The drum is horizontal and contains water upto a height of 1.8 m, Find the time required to empty the boiler. Take C,= 06. (Ans. 7 min 46.64 s) Find the discharge from a 80 mm diameter external mouthpiece, ited oa side ofa large vessel ifthe head cover the mouthpiece is 6 Ans. 0.0466 m's] ‘An external cylindrical mouthpiece of diameter 100 mm is discharging water under a constant head of 8. Determine the discharge and absolute pressure head of water at vena-contracta, Take C, = 0.855 and C, for vena-contracta = 0.62. Take atmospheric pressure head = 10.3 m of water. [Ans. 0.084 m'fs; 3.18 ml A convergent-divergent mouthpiece having throat diameter of 60 mm is discharging water under a con- stant head of 30 m, Determine the maximum outlet diameter for maximum discharge. Find maximum discharge also. Take atmospheric pressure head = 10.3 m of water and separation pressure head = 2.5 m of water absolute Ans. 6.88 em, Qpay = 0.01506 m5] [354 Fluid Mechanics ] 24. ‘The throat and exit diameter of a convergent-divergent mouthpiece are 40 mm and 80 mm respectively. Its fited to the vertical side of a tank, containing water. Find the maximum head of water for steady flow. The maximum vacuum pressure is 8 m of water. Take atmospheric pressure head = 10.3 m of water. [Ans 0.533 m] 25, The discharge through a convergent-divergent mouthpiece fitted to the side of a tank under a constant head of 2 mis 7litess. The head loss in the divergent portion is 0.10 times the kinetic head at outlet. Find the throat and exit diameters, if separation pressure head = 2.5 m and atmospheric pressure head = 10.3 m of water. Ans, 25.3 mm ; 38.6 mm) 26. An internal mouthpiece of 100 mm diameter is discharging water under a constant head of $m. Find the Gischarge through mouthpiece, when (O the mouthpiece is running free, and (i) the mouthpiece is running ful. [Ans. () 38.8 litress, (i) $4.86 lites/s} NOZGHES AND Ny osc > 8.1 INTRODUCTION ‘A notch is a device used for measuring the rate of flow of a liquid through a small channel or a tank. It may be defined as an opening in the side of a tank or a small channel in such a way that the liquid surface in the tank or channel is below the top edge of the opening. ‘A weir isa concrete or masonary structure, placed in an open channel over which the flow occurs. Itis generally in the form of vertical wall, witha sharp edge atthe top, running all the way across the ‘open channel. The notch is of small size while the weir is of a bigger size. The notch is generally made ‘of metallic plate while weir is made of concrete or masonary structure. 1. Nappe or Vein. The sheet of water flowing through a notch or over a weir is called Nappe or Vein. 2. Crest or Sill. The bottom edge of a notch of a top of a weir over which the water flows, is known, as the sill or crest. > 8.2 CLASSIFICATION OF NOTCHES AND WEIRS ‘The notches are classified as : 1. According to the shape of the opening : (a) Rectangular notch, (b) Triangular notch, (c) Trapezoidal notch, and (4) Stepped notch. 2. According to the effect of the sides on the nappe : (@) Notch with end contraction. (b) Notch without end contraction or suppressed notch. Weirs are classified according to the shape of the opening, the shape of the crest, the effect of the sides on the nappe and nature of discharge. The following are important classifications. (@) According to the shape of the opening : (® Rectangular weir, (i Triangular weir, and (ii) Trapezoidal weir (Cipolletti weir) (b) According to the shape of the crest: (®) Sharp-crested weir, (i) Broad-crested weir, (ii) Narrow-crested weir, and (iv) Ogee-shaped weir. 355 [356 Fluid Mechanics ] (©) According to the effect of sides on the emerging nappe : (Weir with end contraction, and (ii. Weir without end contraction. > 8.3 DISCHARGE OVER A RECTANGULAR NOTCH OR WEIR ‘The expression for discharge over a rectangular notch or weir is the same. Orsi (esectionar — REST CREST (2) RECTANGULAR NOTCH (©) RECTANGULAR WEIR Fig. 8:1 Rectangular notch and weir. Consider a rectangular notch or weir provided ina channel carrying water as shown in Fig. 8.1 Let H= Head of water over the rest L= Length ofthe notch ot weir For finding the discharge of water flowing over the weir or notch, consider an elementary horizontal strip of water of thickness dh and length L at a depth from the free surface of water as shown in Fig. 8.1(0). The area of stip =Lxdh and theoretical velocity of water flowing through stip = 2gi The discharge dQ, through strip is dQ "4 Area of sttip x Theoretical velocity = C,x Lx dh x Pah A) where C, = Co-efficient of discharge. The total discharge, Q , for the whole notch or weir is determined by integrating equation (i) between the limits 0 and H. O= fl Cyt. Pah dh Cox tx fe fH? ah yee peat aX Lx fg | Cyxtx 2g | Ae 312], o Cy bx fe CH. wp 3 Problem 8.1 Find the discharge of water flowing over a rectangular notch of 2 m length when the constant head over the notch is 300 mm. Take C, Solution. Given : Length of the noteh, L=20m = 0.60. [ Notches and Weirs 357) Head over notch, H= 300 m= 0.0m 60 cx Lx [1] Discharge, 0.6 x 2.0 x JEXSRT x [0.30]"$ mis = 3.5435 x 0.1643 = 0.582 m’ss. Ans. Problem 8.2 Determine the height of a rectangular weir of length 6 m to be built across a rectan- ‘gular channel. The maximum depth of water on the upstream side of the weir is 1.8 m and discharge is 2000 litres/s. Take C, = 0.6 and neglect end contractions. Solution. Given : Length of weir, Depth of water, Discharge, 16 Let H is height of water above the crest of weir, and H, = height of weir (Fig. 8.2) ‘The discharge over the weir is given by the equation (8.1) as =F cyxbxyig Hw or Height of weir, Hy =H, pth of water on upstream side ~ H .8—.328= 1.472 m. Ans. Problem 8.3 The head of water over a rectangular notch is 900 mm. The discharge is 300 litres’ Find the length of the notch, when C, = 0.62. Solution. Given : Head over notch, H=90em=09m Discharge, Q =300 livs = 0.3 mis Let length of notch Using equation (8.1), we have [358 Fluid Mechanics ] or 0.3 = % x 062 x Lx JIKDRT «(099% = 1.83 x L x 0.8538 03 3 __ joan [83 x 8538 192 mm. Ans. > 8.4 DISCHARGE OVER A TRIANGULAR NOTCH OR WEIR ‘The expression for the discharge over a triangular notch or weir is the same. It is derived as : Let H = head of water above the V-notch © = angle of notch Consider a horizontal strip of water of thickness “dh’ at a depth of h from the free surface of water as shown in Fig. 8.3. From Fig. 8.3 (b), we have 2 ae \d A 6 he AC= (=H) tan 5 7, . Width of strip =AB=2AC=2(H—M) tan > Fig. 83 The triangular notch. ‘The theoretical velocity of water through strip = J2gh Discharge, through the strip, dQ = C, x Area of strip x. Velocity (theoretical) 2x2 Wan thx =2C, (HI) tan & [Pah x dh + Toval discharge, =f" 2¢, (Hy tan 9 Bah > ah = 2, x tan Sx 3g fH Wy! a = 26, x tan Sx (3g [ (Hh! =H") dh etme? 327 =2 0 2x i ao xCaxtans ue ay [ Notches and Weirs 359) " = 2 Cyxtan Sx (oe [2 -2 | 22x c,xtan & val en LHe. excynt Ba fa] Cextan $x fe x He (82) 15 For a right-angled V-notch, if C, = 0.6 =90, . tan Sat 2 x 0.6 x 1x y2X9RT x H? (8.3) 417 H, Problem 8.4 Find the discharge over a triangular notch of angle 60° when the head over the V-notch is 0.3 m. Assume Cy = 0.6. Solution. Given : Angle of V-notch, 6 Head over notch, Discharge, Discharge, Q over a V-notch is given by equation (8.2) 8 8 0 = Sc xtan Sx (ae x1? S x06 an Sx 2x98T x 0.3) = 0.8182 x 0.0493 = 0.040 m’/s. Ans. Problem 8.5 Water lows over a rectangular weir I m wide at a depth of 150 mm and afterwards passes through a triangular right-angled weir. Taking C. for the rectangular and triangular weir as 0.62 and 0.59 respectively, find the depth over the triangular wer. Solution. Given : For rectangular weir, length, Depth of water, For triangular weir, Let depth over triangular weir =H, ‘The discharge over the rectangular weir is given by equation (8.1) as [360 Fluid Mechanics ] @ 2 x cqx tx fie x? 2 0.62 « 1.0 x (35987 x (15)"2 ms = 0.10635 ms 3 ‘The same discharge passes through the triangular right-angled weir. But discharge, Q, is given by ‘equation (8.2) for a triangular weir as = 3 xcyxtan x [2g x HE? 8 90" 2 0.10635 = 5x59 tan Bg x A, (-5 8 = 90° and H= Hy) = x59 14409 18 = 1.3996 1,0 sa _ 0.10635 = = 0.07631 13936 4H, = (.07631)°* = 0.3572 m. Ans. Problem 8.5A. Water flows through a triangular right-angled weir first and then over a rectangu- lar weir of I m width. The discharge co-effcients of the triangular and rectangular weirs are 0.6 and 0.7 respectively. Ifthe depth of water over the triangular weir is 360 mm, find the depth of water over the rectangular weir. Solution. Given : For triangular weir = 90°, C,= 0.6, H = 360 mm = For rectangular weir: L=1m,C,=0.7,H=? ‘The discharge for a triangular weit is given by equation (8.2) as 36m 8 8 8 = 5 x cyxtan Ox f2e xH Oe ex Caxton x PE x = § xoxtan ( )x (25981 x 036)" ‘The same discharge is passing through the rectangular weir. But discharge for a rectangular weir is. given by equation (8.1) as. 1102 m/s or = (0.0533)? = 0.1415 m= 141.5 mm. Ans. [ Notches and Weirs 361] Problem 8.6 A rectangular channel 2.0 m wide has a discharge of 250 litres per second, which is measured by a right-angled V-notch weir. Find the position of the apex of the notch from the bed of the channel if maximum depth of water is not to exceed 1.3 m. Take Cy = 0.62. Solution. Given : Width of rectangular channel, L = 2.0 m Discharge, Q Depth of water in channel Let the height of water over V-notch = HT 25 m/s ‘The rate of flow through V-notch is given by equation (8.2) as 8 a= Be cyx fF en? xi where C= 0.62, 0 = 90° 8 90° ge = 15 62x [PROT x tan HP 8 sa or 25 = 3 x 624.429 x 1H 0.25 = FE x62 4429 x po = 25x15 3X02 x4829 a H= (1707) = (1707) Position of apex of the notch from the bed of channel = depth of water in channel-height of water over V-notch = 1.3 ~ 493 = 0.807 m. Ans. or 1707 0.493 m > 8.5 ADVANTAGES OF TRIANGULAR NOTCH OR WEIR OVER RECTANGULAR NOTCH OR WEIR A triangular notch ot weir is preferred to a rectangular weir or notch due to following reasons : 1. The expression for discharge for a right-angled V-notch or weir is very simple. 2. For measuring low discharge, a triangular notch gives more accurate results than a rectangular notch. 3. In case of triangular notch, only one reading, ie., H is required for the computation of discharge. 4, Ventilation of a triangular notch is not necessary. >» 8.6 DISCHARGE OVER A TRAPEZOIDAL NOTCH OR WEIR As shown in Fig. 8.4, a trapezoidal notch or weir is a ‘combination of a rectangular and triangular notch or weir. Thus the total discharge will be equal to the sum of discharge through a rectangular weir or notch and discharge through a triangular notch or wei Let H = Height of water over the notch L = Length of the crest of the notch Fig. 8.4 The trapezoidal notch. [362 Fluid Mechanics ‘o-efficient of discharge for rectangular portion ABCD of Fig. 8.4. Cy, = Co-efficient of discharge for triangular portion [FAD and BCE] ‘The discharge through rectangular portion ABCD is given by (8.1) or 2 Cy, Lx fle x? ‘The discharge through two triangular notches FDA and BCE is equal to the discharge through a single triangular notch of angle @ and it is given by equation (8.2) as 8 sa x Cy, xtan 2 x 2g x HO? tan > x 2B Discharge through trapezoidal notch or weit FDCEF = 2 1+ Or CLR Ie Cun 02 LR KI, A8A Problem 8.7 Find the discharge through a trapezoidal notch which is 1 m wide at the top and (0.40 m at the bottom and is 30 cm in height. The head of water on the notch is 20 cm. Assume C, for rectangular portion = 0.62 while for triangular portion = 0.60. Solution. Given : Top width, AE=1m Base width, CD =L=04m Head of water, H= 020m 62 0.60 For rectangular portion, Cy, For tiangular portion, Cy, From AABC, we have tan & = AB_(AE~CD)/2 2° BC H (10-04)/2 _ 0.6/2 _ 03 03 03-03 Discharge through trapezoidal notch is given by equation (8.4) = 3 x 060 x08 x JOROBT x (02! + Fx 60x 1 x YPXOBT x (02)? = 0,06549 + 0.02535 = 0.09084 ms = 90:84 litres/s. Ans. > 8.7 DISCHARGE OVER A STEPPED NOTCH ‘A stepped notch is a combination of rectangular notches. The discharge through stepped notch is ‘equal to the sum of the discharges through the different rectangular notches. [ Notches and Weirs 363) Consider a stepped notch as shown in Fig. 8.6. <= Let H, = Height of water above the rest of notch 1, 3 la 1, = Length of notch 1, ae ca te Ha, La and Ha, Ly are corresponding values for notches 5 ie 2 and 3 respectively. C= Corefficient of discharge for all notches ae ‘Total discharge Q = Q, + Q2 + Os ns or O= 3 x 0,x 1, x fg (Hs? - #34) Fig. 8.6 The stepped notch. +2 Gyxtax Reh B14 2 cyxtyx Pg xn. (8.5) Problem 8.8 Fig. 8.7 shows a stepped notch. Find the discharge through the notch if C, for all section = 0.62. Solution. Given : 40 em, Ly = 80 cm, 20 em “@® H, = 50+ 30+ 15=95 em, ee @ H, = 80.em, Hy = 50 em, Fo ai ° 80m C= 0.62 = 120m ——» Total discharge, Q = Q, + 0+ Qs Fig. 8.7 where O1= Ex Cyt x Pe UH) = z % 0.62 x40 x 2X ORT x [95° — 80°] = 732.261925.94 — 715.54] = 154067 em’Js = 154.067 livs 2 xcpxlax Pg x HHS) = % 0.62 x 80 x J2X9BT x (80°? = 1464.52[715.54 ~ 353.55] cm/s = 530141 cm'/s = 530.144 livs. and 5 = x Cy yx fe x HS" = 3 5062 120. J2X98I % 50° = 776771 ens = 796.771 tvs = 0, +0; + y= 154.067 + 530.144 + 776.771 = 1460.98 lit/s. Ans. [364 Fluid Mechanics ] > 8.8 EFFECT ON DISCHARGE OVER A NOTCH OR WEIR DUE TO ERROR IN THE MEASUREMENT OF HEAD For an accurate value of the discharge over a weir or notch, an accurate measurement of head over the weir or notch is very essential as the discharge over a triangular notch is proportional to H*” and in ‘case of rectangular notch it is proportional to H*, A small error in the measurement of head, will affect the discharge considerably. The following cases of error in the measurement of head will be considered : (i) For Rectangular Weir or Notch. (id) For Triangular Weir or Notch. 8.8.1 For Rectangular Weir or Notch. The discharge for a rectangular weir or notch is given by equation (8.1) as ° 2x cyxbx Pg xn? = KH? “ where K = 2 Differentiating the above equation, we get 3 yin aQ=Kx > H'? dH ti) Dividing (ii) by (), (8.6) oO KE Equation (8.6) shows that an error of 1% in measuring H will produce 1.5% error in discharge over a rectangular weir or notch. 8.8.2 For Triangular Wei by equation (8.2) as or Notch. The discharge over a triangular weir or notch is given Cp tan 4 fog x H He soit) 8 8 where K= © ¢, tan 9 oe 5 Co tam 578 Differentiating equation (ii), we get 4Q=K 5 Hx att «) 3 pa KW? a ag_*> 5 dH Dividing (iv) by (ii), we get 42-2 3 8 (i) by (iid, we get T= Ean — = 9 ap Equation (8.7) shows that an error of 1% in measuring H will produce 2.5% error in discharge over a triangular wei or notch. (8.7) [ Notches and Weirs 365) Problem 8.9 A rectangular notch 40 cm long is used for measuring a discharge of 30 litres per second. An error of 1.5 mm was made, while measuring the head over the notch. Calculate the percentage error in the discharge. Take C, = 0.60. Solution. Given : Length of notch, Discharge, Error in head, Let the height of water over rectangular notch = H ‘The discharge through a rectangular notch is given by (8.1) or CXL fg xB? or 30000 = 2 x 0.60 « 40x JERBT x 4? or sew 2.33 2x60 x 40x J2x 981 H = 42.33)"% = 12.16 em Using equation (6), we get 40 34H 3,0 072 82" iDI6 Problom 8.10 A ightangled Vnochis used for measuring discharge of 30 lies An errorof ZeSnm was made wile measuring the head over the notch, Colelate the percentage crtor In he discharge Tate Gy= 02 Solution. Given: Angle of Vac, Discharge, Er in head, 10185 = 1.85%. Ans. 30000 em’Vs 5 mm = 0.15 em 1.62 Let the head over the V-notch = H ‘The discharge Q through a triangular notch is given by equation (8.2) @ = 8 Cyan x87 x1 2 is = 3 x62 1x 44.29 x H 1s 30000 x15 8x.62 x44.29 H = (2048.44)? = 21.11 em 048.44 [366 Fluid Mechanics ] Using equation (8.7), we get AO SMH 45, OS Q 2H” *2ur Problem 8.11 The head of water over a triangular notch of angle 60° is 50 cm and co-efficient of discharge is 0.62. The flow measured by itis to be within an accuracy of 1.5% up or down. Find the limiting values of the head. Solution. Given : Angle of V-notch, o= Head of water, H = 0.01776 = 1.77%. Ans. @ 215% = 20015 Q The discharge Q over a triangular notch is ° =F Ca y2e tan 5 0.62 2RBBT x tan 9P 5 (50) = 14.64 x 0.5773 x 17677.67 = 149405.86 cm*/s, Now applying equation (8.7), we get LSM os ois-25 Ht o Q 24H HH O15 ‘ 3B xs0=203 50.3 em, 49.7 em = 50.3 cm and 49.7 cm. Ans. > 8.9. (a) TIME REQUIRED TO EMPTY A RESERVOIR OR A TANK WITH A. RECTANGULAR WEIR OR NOTCH Consider a reservoir or tank of uniform cross-sectional area A. A rectangular weir or notch is provided in one of its sides. Let = Length of crest of the weir or notch Co-efficient of discharge Initial height of liquid above the crest of notch inal height of liquid above the crest of notch T= Time required in seconds to lower the height of liquid from H, to H,. Let at any instant, the height of liquid surface above the crest of weir or notch be /t and in a small time dT, let the liquid surface falls by ‘dh’. Then, ~Adh = Q x dT ve sign is taken, as with the increase of T, fr decreases. [ Notches and Weirs 367) - a= 2 cpetx oR x0" adh ~Adh= 2 C,xLx fig Wx aP ot aT oS 3 5 Cex Lx 2g xn®? ‘The total time T is obtained by integrating the above equation between the limits H, and Hf fr ae" -Adh In, 2 a "£0, x Lx fie xh 5 g Gs Te hi ae | TG xixta 2G xT | a1 =— A ee sete (3) 20,xLxfig| 1 | ~2¢,xix fie 1 2 Ia, 3A 1 1 (b) TIME REQUIRED TO EMPTY A RESERVOIR OR A TANK WITH A TRIANGULAR, WEIR OR NOTCH (88) Consider a reservoir or tank of uniform cross-sectional area A, having a triangular weir or notch in ‘one of its sides Let @= Angle of the notch C,= Co-efficient of discharge H, = Initial height of liquid above the apex of notch H, = Final height of liquid above the apex of notch, ‘Time required in seconds, to lower the height from H, to H, above the apex of the notch. Let at any instant, the height of liquid surface above the apex of weir or notch be and in a small time dT, let the liquid surface falls by ‘dh’. Then ~Adh = Qx dT ve sign is taken, as with the increase of T, h decreases. And Q for a triangular notch is o B xcyxtan oe xn ~Adh = © x Cy xctan 2x fg x 18 aT 15 2 [368 Fluid Mechanics ] are a Bye xun?s sn Sx, xtane ® eX Cex ta x 8x ‘The total time T is obtained by integrating the above equation between the limits H, and H, r are f ; ~Adh so, tan & a8? or 4x C,xtan 9 Pe Problem 8.12 Find the time required to lower the water level from 3 m to 2 m in a reservoir of dimension 80 m x80 m, by a rectangular notch of length 1.5 m. Take C, = 0.62. Solution. Given : Initial height of water, H, (8.9) Final height of water, Hy =2m Dimension of reservoir = 80 m x 80 m or Area, A= 80 x 80 = 6400 m? Length of notch, L= 15m, C,=0.62 Using the relation given by the equation (8.8) ae ie GxLx ie | Jt _ 36400 Low * Dexisx Ixom [VT 1661.35 (0.7071 ~ 0.5773] seconds = 605.04 seconds = 10 min 5 sec. Ans. Problem 8.13. Ifin problem 8.12, instead of a rectangular notch, a right-angled V-notch is used, find the time required. Take all other data same. [ Notches and Weirs 369) Solution. Given : Angle of notch, @=90° Initial height of water, Hy =3m Final height of water, H,=2m Area of reservoir, A= 80x 80 = 6400 m? C= 0.62 Using the relation given by equation (8.9) Tr SA [ 4x, xtan 9 xg Hy 56400 1 4x.62x tan ox foxdaI X.62% tan) x Y2X9BT sonax( ata 28284 51961 = 2913.34 (0.3535 - 0.1924] seconds 169.33 seconds = 7 min 49.33 sec. Ans. Problem 8.14 A right-angled V-notch is inserted in the side of a tank of length 4 m and width 2.5 m. Initial height of water above the apex of the notch is 30 cm. Find the height of water above the apex if the time required to lower the head in tank from 30 cm to final height is 3 minutes. Take Cy = 0.60. Solution. Given : Angle of notch, 8-90" Area of tank, A= Length x width = 4 x 2.5 = 10.0 m? Initial height of water, H, = 30cm=0.3 m Time, T = 3 min = 3 x 60 = 180 seconds C,=0.60 Let the final height of water above the apex of notch = H, Using the relation given by equation (8.9) 7-4, | 4x, xtan 9x Y2e Hy 180 = Si BE? Gay? ee 4x.60x1x4.429| 5? (03) 180 x4 x 0.60 x 4.429 50. or 38.266. [370 Fluid Mechanics ] or Ty ~ 6.0858 = 38.266 8.266 + 6.0858 = 44.35 or Hy! = = 0.0225 was Hy = (0.0225)""* = (0.0225)" = 0.0822 m = 8.22 em. Ans. > 8.10 VELOCITY OF APPROACH Velocity of approach is defined as the velocity with which the water approaches or reaches the weir or notch before it flows over it. Thus if V, is the velocity of approach, then an additional head fi, y equal {0 2 due to velocity of approach, i acting on the water flowing over the notch. Then inital height of water over the notch becomes (H+ h,) and final height becomes equal to /,. Then all the formulae are changed taking into consideration of velocity of approach. The velocity of approach, V, is determined by finding the discharge over the notch or weir neglecting velocity of approach. Then dividing the discharge by the cross-sectional area of the channel oon the upstream side of the weir or notch, the velocity of approach is obtained. Mathematically, v= ‘Area of channel : calculated and above process is repeated for more accurate discharge. Discharge over a rectangular weir, with velocity of approach = ixcyxtx Wie [Ht + hy)? 43%) (8.10) Problem 8.15 Water is flowing in a rectangular channel of I m wide and 0.75 m deep. Find the discharge over a rectangular weir of crest length 60 cm, ifthe head of water over the crest of weir is 20cm and water from channel flows over the weir. Take C, = 0.62. Neglect end contractions. Take velocity of approach into consideration. Solution, Given : Area of channel, A= Width x depth = 1.0 07 Length of weir, T= 60cm =06 m Head of water, 1H, =20em=02.m 175m? [ Notches and Weirs 371] = 1.098 x 0.0894 = 0.0982 m/s 0 _ 0982 Velocity of approach, 2982 = 0.1309 mis y of appr FoR = 0.1309 mi Additional head, YE. = (1309) x 9.81 = 0008733 m eo Oe ‘Then discharge with velocity of approach is given by equation (8.10) XCyXLX 2g (Hy +h)? = 7) X 0.62 x 0.6 x J2X9BT [(0.2 + .00087)*? - (.00087)*?] 1.098 [0.09002- 00002566} = 1,098 x 0.09017 = .09881 m’/s. Ans. Problem 8.16 Find the discharge over a rectangular weir of length 100 m. The head of water over the weir is 1.5 m. The velocity of approach is given as 0.5 m/s. Take Cy = 0.60. Solution. Given : Length of weir, L= 100m Head of water, Hy=15m Velocity of approach, —_V, = 0.5 mis Cy .. Additional head, b= ‘The discharge, Q over a rectangular weir duc to velocity of approach is given by equation (8.10) =F xcyx Lx Vie UH, + ny” 2 0.6 x 100 x J2X9BT [(1.5 + .0127)"* ~ 01277] 3 = 177.16 [1.5127 ~ .0127%7] 77.16 [1.8605 ~ 00143] = 329.35 m'/s, Ans. Problem 8.17 A rectangular weir of crest length 50 cm is used to measure the rate of flow of water in a rectangular channel of 80 cm wide and 70cm deep. Determine the discharge in the channel if the water level is 80 mm above the crest of weir. Take velocity of approach into consideration and value of C4 = 0.62. Solution. Given : Length of weir, L=S0cm=0.5m ‘Area of channel, A= Width x depth = 80 cm x 70 em = 0.80 x 0.70 = Head over weir, H= 80mm = 0.08 m C= 0.62 ‘The discharge over a rectangular weir without velocity of approach is given by equation (8.1) (372. Fluid Mechanics ] Velocity of approach, Head due to Vj. Discharge with velocity of approach is FX CaX Lx V2 (CH, + hl? ed 2 0.62 x05 x YTXDRT [(.08 + 0000697)" - 0000697") 1.9153 x [.0800697'* ~ .0000697'*] 153 [.02265 ~ 000000582] = 0.2073 m°/s. Ans. Problem 8.18 A suppressed rectangular weir is constructed across a channel of 0.77 m width with @ head of 0.39 m and the crest 0.6 m above the bed of the channel. Estimate the discharge over it. Consider velocity of approach and assume C, = 0.623. Solution. Given : Width of channel, b=0.17m Head over weir, H=039m Height of crest from bed of channel = 0.6 m Depth of channel 6 +0.39 = 0.99 Value of C, = 0.623 Suppressed weir means that the width of channel is equal to width of weir ie., there is no end ‘contraction. 2 Width of channel = Width of weir = 0.77 m Now area of channel, A= Width of channel x Depth of channel = 0.77 x 0.99 ‘The discharge over a rectangular weir without velocity of approach is given by equation (8.1). x Cyxb x 2g x HE? (ys Here b= L) = 2 0.623 «0.77 x 2x9BT x 0.39* = 0.345 mis Q 0345, Now velocity of approach, V, = <= ‘Area of channel 0.77099 = 0.4526 mis [ Notches and Weirs 373) Head due to velocity of approach, ae 0.0104 m 2g 2x981 Now the discharge with velocity of approach is given by, x Cyx bx J2g [H+ hy? Ao] % 0.623 x 0.77 x J2* 981 [(0.39 + 0.0104)" - (0.0104)"] 3 2 3 2 3 % 0.623 x 0.77 x 4.43 [0.2533 - 0.00106] 13873 m'Vs. Ans. Problem 8.19 A sharp crested rectangular weir of I m height extends across a rectangular channel of 3 m width. If the head of water over the weir is 0.45 m, calculate the discharge. Consider velocity of approach and assume Cy = 0.623. Solution. Given : Width of channel, b=3m Height of weir Im Head of water over weir, H = 0.45 m Depth of channel Height of weir + Head of water over weir +045 = 145 m Value of Cy = 0.623 ‘The discharge over a rectangular weir without velocity of approach is given by equation (8.1) as, Q==xC,xbx J2g x? 1665 ms 0,623 x 3x J2X9RI x 0.45 Now velocity of approach is given by o ‘Area of channel 1.665 1.665 idth of channel x Depth of channel 31.45 Head due to velocity of approach is given by, 0.382 mis, 0074 m 2g 2x 981 Now the discharge with velocity of approach is given by, x Cyx bx V2g [H+ hr? ~ (h,)7] [374 Fluid Mechanics ] > 8.11 EMPIRICAL FORMULAE FOR DISCHARGE OVER RECTANGULAR WEIR ‘The discharge over a rectangular weir is given by Cy \2@ x Lx (H°?] without velocity of approach wi) = 26, YP Lx (E+ hy) =n wit velocity of approach iD) Equations (i) and (ii) are applicable to the weir or notch for which the crest length is equal to the ‘width of the channel. This type of weir is called Suppressed weir. But if the weir is not suppressed, the effect of end contraction will be taken into account. (a) Franels’s Formula. Francis on the basis of his experiments estab- lished that end contraction decreases the effective length of the crest of weir and hence decreases the discharge. Each end contraction reduces the crest length by 0.1 x H, where His the head over the weir. For a rectangu- lar weir there are two end contractions only and hence effective length L=(L-02H) 04 and 22x C,x(L-02x Hx J2g H? If Cy = 0.623, g = 9.81 mis®, then => «623 x J2X9BI x (L-0.2 x Hx H? 84 [L— 0.2 x HIM? 811) If end contractions are suppressed, then H= 184.4? (8.12) If velocity of approach is considered, then Q= 184 L (H+ hy? A) 48.13) () Bazin's Formula. On the basis of results of a series of experiments, Bazin’s proposed the following formula for the discharge over a rectangular weir as Q=mxLx fog x1? (8.14) 2 003 where m = 2 x, 0.405 + 03 = height of water over the weir If velocity of approach is considered, then Q=m, xLx \2g (H +h)" (8.15) where m, = 0.405 + . 1 = 0405 + TT Problem 8.20 The head of water over a rectangular weir is 40 cm. The length of the crest of the weir with end contraction suppressed is 1.5 m. Find the discharge using the following formulae : (i) Francis’s Formula and (ii) Bazin’s Formula. [ Notches and Weirs 375) Solution, Given : Head of water, Length of weir, (i Francis’s Formula for end contraction suppressed is given by equation (8.12). Q= 1.84 L x HY = 1.84 x 15 x (40)? 1.6982 m*/s (ii) Bazin’s Formula is given by equation (8.14) Q=mxLx Jig x #? 003 _ 0.405 + 7 = 0.4125 Q= 4125 x 1.5 x J2KOBI x (4)? 1.6932 m/s. Ans. Problem 8.21. A weir 36 metres long is divided into 12 equal bays by vertical posts, each 60 cm wide. Determine the discharge over the weir if the head over the crest is 1.20 m and velocity of approach is 2 metres per second. Solution. Given : Length of weir, 1,=36m Number of bays, 12 For 12 bays, no, of vertical post = 11 Width of each post = 60cm = 0.6 m Effective length, L=L,~ 11X06 = 36-66 = 294 m Head on weir, H= 120m Velocity of approach, V,=2 mis Head due to V,, hy 0.2038 m 2g 2981 Number of end contraction, n= 2x 12 {Each bay has two end contractions} =24 Discharge by Francis Formula with end contraction and velocity of approach is Q = 1.84 [L— 0.1 x nH + h MH + 1)? = hy) 1.84[29.4 — 0.1 x 24(1.20 + .2038)] x [(1.2 + .2038)'* - .2038'*] = 1,84[29.4 — 3,369]11.663 ~ .092] = 75.246 m’s. Ans. Problem 8.22 A discharge of 2000 m'ss is to pass over a rectangular weir. The weir is divided into a number of openings each of span 10 m. If the velocity of approach is 4 m/s, find the number of openings needed in order the head of water over the crest is not to exceed 2 m. Solution. Given : Total discharge, = 2000 ms 10 Length of each opening, L = [376 Fluid Mechanics ] Velocity of approach, -V,= 4 mis. Head over weir, Let number of openings Head due to velocity of approae v2 4x4 ‘2g 2x981 For each opening, number of end contractions are two. Hence discharge for each opening considering velocity of approach is given by Francis Formula ie, Q= LB4[L - 0.1 x 2x (H+ hMCH + A)? = bh, $4[10.0 = 0.2 x (2 + .8155)][2.8155"* - 8155'9] 7.36314.7242 — 0.7364] = 69.24 mis Total discharge 2000 Discharge for one opening 69.24 = 28.88 (say 29) = 29. Ans. > 8.12 CIPOLLETTI WEIR OR NOTCH = 0.8155 m e- Number of opening caer a Stic in a a’ i °F 8 ay be gene Satan P= 142 q 4 By giving this slope tothe sides, an inerease in discharge through the © ° triangular portions ABC and DEF of the weir is obtained. If this slope is not provided the weir would be a rectangular one, and due to end Fig. 8.9. The cipollett weir. contraction, the discharge would decrease. Thus in case of cipollett Weir, the factor of end contraction is not required whieh is shown below. “The discharge through a rectangular weir with two end contractions is x Cyx (L= 0.2 H) 2g x HY? ho XC,x Lx fg 2 x cx og x Ho? Thus ue en conan, dhe dees by 2: CYR Tis dese a chase cn be compen by ig sch» ope oh hate drs ough times pron qa 2% Cy x BF 1. Lathe pe is hen hy 2 he shane trough Vn of ag Shen by = Ax cyx (ie xtan © 1s 2 [ Notches and Weirs 377) Thus J x cyx Yq xtan 2 HY 22 x0, x fg x He 5 2 15 6 ay . tan & or 62 =tan! = 47, 27s 8.13 DISCHARGE OVER A BROAD-CRESTED WEIR A weir having a wide crest is known as broad-crested weir. Let H = height of water above the crest length of the crest eo) Fig. 810 Broadcrested weir. If 2L > H, the weir is called broad-crested weir If 2L < H, the weir is called a narrow-crested weir Fig. 8.10 shows a broad-crested weir. Let fr= head of water at the middle of weir which is constant v= velocity of flow over the weir Applying Bemoulli’s equation to the still water surface on the upstream side and running water at the end of weir, Hoh v= 2¢(H-A) The discharge over weir Q = CX Atea of flow x Velocity = C,xLxhx 42g (Hh) xx alii? —) (6.18) [ Notches and Weirs 379] ‘The discharge will be maximum, if (Hh? — f°) is maximum or © (Hi? = 18) = 0 0r 2h x H ~ 31? = 0 or 2H = 3h ah 2 nei Sn Qn Will be obtained by substituting this value of fin equation (8.18) as 22) _(2,) yx LX ps[ (3a) -G)] Tp Sp i bx Bg HS HP SH aX Lx fie fo nas tp 3p xix ia jw - SH a $9" oF 1px Lx fg x 0.3849 x 1? q g XLX 2g ht 3849 x JIXOT x Cy x Lx H*? = 1.7087 x Cy x Lx A? = 1.105 x Cx Lx #?, (8.19) > 8.14 DISCHARGE OVER A NARROW-CRESTED WEIR For a narrow-crested weir, 2L < H. Itis similar to a rectangular weir or notch hence, Q is given by 2 == xC,xLx \2g x H” (8.20) 3 > 8.15 DISCHARGE OVER AN OGEE WEIR Fig. 8.11 shows an Ogee weir, in which the crest of the weir rises upto maximum height of 0.115 x H (where H is the height of water above inlet ofthe weit) and then falls as shown in Fig. 8.11, SHAS? ‘The discharge for an Ogee weir is the same as that of a rectangular weir, and itis given by = 2 xcyx bx fae x” (8.21) Fig. 8.11 An Ogee weir. > 8.16 DISCHARGE OVER SUB-MERGED OR DROWNED WEIR ‘When the water level on the downstream side of a weir is above the crest of the weir, then the weir is called to be a sub-merged or drowned weir. Fig. 8.12 shows a sub-merged weir. The total discharge, over the weit is obtained by dividing the weir into two parts. The portion between upstream and downstream water surface may be treated as free weir and portion between downstream water surface and crest of weir as a drowned weit [380 Fluid Mechanics ] Fig.8.12 Sub-merged weir. Let H eight of water on the upstream side of the weir height of water on the downstream side of the weir Then = discharge over upper portion XC, XL 2g [H- np? discharge through drowned portion “s, X Area of flow x Velocity of flow = Cy, xLx hx J2g(H— +. Total discharge, Q=01+0, 2 = 504, XLX 2g [H~ AP + Cy, x Lx x f2Q( Hh) «.(8.22) Problem 8.25 (a) A broad-crested weir of 50 m length, has 50 cm height of water above its crest. Find the maximum discharge. Take C, = 0.60. Neglect velocity of approach. (b) If the velocity of approach is to be taken into consideration, find the maximum discharge when the channel has a cross- sectional area of 50 m* on the upstream side. Solution. Given : Length of weir, Head of water, (0 Neglecting velocity of approach. Maximum discharge is given by equation (8.19) as, 105 x Cx Lx HP 705 x 0.60 x 50 x (.5)** = 18.084 m*/s. Ans. (ii) Taking velocity of approach into consideration ‘Area of channel, Velocity of approach, Head due toV,, Maximum discharge, Ogu is given by ‘Oman = 1.705 X Cy x LX ((H +h) 1.705 x 0.6 x 50 x {(-50 + 0066)! ~ (.0066)'%] 1.15[0.3605 ~ 000536] = 18.412 mvs. Ans. [ Notches and Weirs 381] Problem 8.26 An Ogee weir 5 metres long has a head of 40 cm of water. If Cy = 0.6, find the discharge over the weir. Solution. Given : Length of weir, L Head of water, H G Discharge over Ogee weir is given by equation (8.21) as. Q= 5x C,xLx 2g XH? % 0.60 x 5.0 x y2x98I x (0.4)°* = 2.2409 m/s. Ans. Problem 8.27 The heights of water on the upstream and downstream side of a sub-merged weir of 3 m length are 20 cm and 10 cm respectively. If Cy for free and drowned portions are 0.6 and 0.8 respectively, find the discharge over the weir. Solution. Height of water on upstream side, H = 20 em = Height of water on downstream side, f= 10 em Length of weir, Total discharge Q is the sum of discharge through free portion and discharge through the drowned portion. This is given by equation (8.22) as 2 pn = 3% Ci, xLx 2g (Hh +O, xLx hx 2g (H=h) 2 x06%3% EXOBT [.20 -.10]"5+0.8x3x10x J2x981(2—1) = 0.168 + 0.336 = 0.504 m/s. Ans. HIGHLIGHTS A. A notch is a device used for measuring the rate of flow of a liquid through a small channel. A weir is ‘concrete or masonary structure placed in the open channel over which the flow occurs. 2. The discharge through a rectangular notch or weir is given by on? qxuxi™ ent of discharge, nat of notch of weit, lead of water over the notch or weir 4, The discharge over a angular notch or wir is piven by Boy ® = © cyan 8 x ie x ® 1s Coma ae where @ = total angle of triangular noteh. 382_ Fluid Mechanics 4. The discharge through a trapezoidal notch or weir is equal 10 the sum of discharge through a rectangular notch and the discharge through a triangular notch. It is given as ex 2 86, m9, 0-2 G, xt xsi x74 Sy, xtan where C,, = co-efficient of discharge for rectangular notch, Cy, = co-etfcient of discharge for tlangular notch, 6/2 = slope ofthe side of trapezoidal notch ' The error in discharge due to the error inthe measurement of head over a rectangular an triangular notch ‘or weir is given by Fora rectangular weir or notch For a tangular weir or notch where Q = discharge through rectangular of triangular notch of weir FH = head over the notch or weit 6. The time required to empty a reservoir ora tank by a rectangular ora triangular notch is given by sa fa 4 - By a recingular notch aaa Lr an oe 5A 14 [te gh a septa x gE ross-sectional area ofa tank oF @ reservoir ial eight of lguid above the rest or apex of notch £1, = final height of liquid above the eestor apex of notch 7. The velocity with which the water approaches the wei or notch is ealled the velocity of approach I is ‘denoted by V,, and is given by ischarge over the notch or weir yep acear ee over inesnatel AWN ‘Cross-sectional area of channel 8. The head due to velocity of approach is given by hy = 2 2g 9. Discharge over a rectangular wer with vlociy of approach, 0-2 cg fe WH, +h OMe cet eee ae Q=1.84[L-0.2 HH? For two end contractions: = 18h? end contractions are suppressed 84 LH +h)? If velocity of approach is considered where = length of weir, H = height of water above the crest of the weir, fh, = head due to velocity of approach, 1, Bazin’s Formula for discharge over a rectangular weir, Notches and Weirs 383 Q=m fogei*? without velocity of approach mL \2g (H+ hy) with velocity of approach 2 03 where m= 2 ¢,=0.405 +208 without velocity of approal = ¢,=0405-+ hout velocity of approach 03 Rise with velocity of approach (H+h,) ila 12, A trapezoidal weie, with side slope or | horizontal to 4 vertical, is called Cipolletti weir. The discharge {through Cipolletti weir is given by =? xix fim without velocity of approach yx bx PB (H+ nS? HO" with velocity of approach 13. The discharge over a broad-crested weir is given by, = C4 y2e(H—) whore H'= height of water above the ces ‘h-= head of water at the middle of the weie which is constant = length of the weir. Pe ene een eset eeeeer eT) and maximum discharge is given by Quay = 1.708 Cy L 2: 15. The discharge over an Ogee weir is given by Q: z Gib x ig x HE. 16. ‘The discharge over sub-merged or drowned weir is given by Q =discharge over upper portion + discharge through downed portion 2 x2 Ca, Lx Be (A= 1)" + Cy, Lh x f2e(H= A) 304 EX V2e HWY + Cy, Lh x f29(H A) where = height of water onthe upstream side of the wei, ‘= height of water on the downstream side of the weir. EXERCISE (A) THEORETICAL PROBLEMS 1. Define the terms : notch, weit, nappe and erest. 2. How are the weirs and notches classified ? 3. Find an expression for the discharge over a rectangular weir in terms of head of water over the crest of, the weir, 4. Prove that the discharge through triangular notch or weir is given by 8 @ pa o= S cyxtan Sx oe H where H = head of water over the notch or weir © = angle of notch or weir (384 Fluid Mechanics ] 5. 6 om 13. 4. 18. 16. What are the advantages of triangular notch or weir over rectangular notch ? Prove that the error in discharge due to the error inthe measurement of head over a rectangular notch is given by do _3dH Q 2H where Q = discharge through rectangular notch and H =head over the rectangular notch, Find an expression for the time required to empty a tank of area of cross-section A, with a rectangular notch. What do you understand by “Velocity of Approach’ ? Find an expression forthe discharge over a rectan- gular weir with velocity of approach, Define ‘end contraction’ of a weir. What is the effect of end contraction on the discharge through a weir ? What is a Cipolletti Weir ? Prove that the discharge through Cipolleti weir is given by on Fc few where L = length of weir, and H = head of water over weir. Differentiate between Broad-crested weir and Narrow crested weit. Find the condition for maximum dis- charge over a Broad-crested weir and hence derive an expression for maximum discharge over a broad= crested weir What do you mean by a drowned weir ? How will you determine the discharge for the downed weir ? Discuss ‘end contraction’ of a weir State the different devices that ean be used to measure the discharge through a pipe also through an open channel. Describe one of such devices with a neat sketch and explain how one can obtain the actual Gischarge with its help. What isthe difference between a notch and a weir ? Define velocity of approach. How does the velocity of approach affect the discharge over a weir ? (B) NUMERICAL PROBLEMS |. Find the discharge of water flowing over rectangular notch of 3 m length when the constant head of water lover the notch is 40 em. Take C, = 0.6. [Ans. 1.344 m/s] Determine the height of @ rectangular weir of length 5 m to be built across a rectangular channel. The ‘maximum depth of water on the upstream side of the weir is 1.5 m and discharge is 1.5 m? per second. Take C,= 0.6 and neglect end contractions. [Ans. 1.194 m) }. Find the discharge over a triangular notch of angle 60° when the head over the triangular notch is 0.20 m. Take C,= 06, [Ans. 0.0164 m's} |. A rectangular channel 1.5 m wide has a discharge of 200 litres per second, which is measured by a right- angled V-notch weit. Find the position of the apex of the notch from the bed of the channel if maximum {depth of water is not be exceed | m. Take C= 0.62. [Ans. 549 m) Find the discharge through a trapezoidal notch which is 1.2 m wide atthe top and 0.50 m at the bottom and is 40 em in height. The head of water on the notch is 30 em. Assume C, for rectangular portion as 0.62 while for triangular portion = 0.60, [Ans. 0.22 m/s} ‘A rectangular notch 50 cm long is used for measuring a discharge of 40 lites per second. An error of 2mm ‘was made in measuring the head over the notch. Calculate the percentage error in the diseharge. Take C= 086, (Ans. 2.37%] [A tight-angled V-noteh is used for measuring a discharge of 30 litres/s. An error of 2-mm was made in ‘measuring the head over the notch, Calculate the percentage error in the discharge, Take C, = 0.62, (Ans. 2.37%) Notches and Weirs 385) 2 13. 4 18, 16. 7. Find the time required to lower the water level from 3 m to 1.5 m ina reservoir of dimension 70 m x 70m, by a rectangular notch of length 2.0 m. Take C= 0.60. [Ans. 11 min I's} I in the problem 8, instead of a rectangular notch, a right angled V-notch is used, find the time required. Take all other data same. Ans. 13 min 31 s] ‘Water is flowing in a rectangular channel of 1.2 m wide and 0.8 m deep. Find the discharge over a rectan- gular weir of erest length 70 cm if the head of water over the erest of weir is 25 em and water from channel flows over the weir. Take C= 0.60. Neglect end contractions but consider velocity of approach. [Ans, 0.1557 m/s} Find the discharge over a rectangular weir of length 80 m. The head of water over the weir is 1.2 m. The velocity of approach is given as 1.5 m/s, Take C, = 3.6. Ans. 208.11 m/s} ‘The head of water over a rectangular weir is SO cm. The length ofthe crest of the weit with end contraction suppressed is 1.4 m, Find the discharge using following formulae : () Francis's Formula and (i) Bazin's Formula [Ans. (9 0.91 m°s, (if) 901 m°/5] A discharge of 1500 m°ss is to pass over a rectangular weir. The weir is divided into a number of openings cach of span 7.5 m. Ifthe velocity of approach is 3 mis, find the number of openings needed in order the hhead of water over the crest isnot to exceed 1.8 [Ans. 37.5 say 38] Find the discharge over a cipolletti weir of length 1.8 m when the head over the weir is 1.2 m. Take y= 0.62 Ans. 4.331 ms) (@) A broad-crested weir of length 40 m, has 400 mm height of Water above its crest. Find the maximum discharge. Take C, = 0.6. Neglect velocity of approach. [Ans. 10,352 m/s} (b) If the velocity of approach is to be taken into consideration, find the maximum discharge when the ‘channel has a cross-sectional area of 40 m? on the upstream side. ans. 10.475 m/s} ‘An Ogee weir 4 m long has a head of 500 mm of water. If C= 0.5, find the discharge over the wei. Ans. 2.505 m/s) ‘The heights of water on the upstream and downstream side of a sub-merged weir of length 3.5 m are 300 mm and 150 mm respectively. If C, for free and drowned portion are 0.6 and 0.8 respectively, find the discharge over the wei. Ans. 1.0807 m/s} VISCOUS FLOW > 9.1 INTRODUCTION This chapter deals with the flow of fluids which are viscous and flowing at very low velocity. At low velocity the fluid moves in layers. Each layer of fluid slides over the adjacent layer. Due to relative velocity between two layers the velocity gradient “ exists and hence a shear stress t=“ acts on the layers. 4 4 ‘The following cases will be considered in this chapter : 1. Flow of viscous fluid through circular pipe. 2. Flow of viscous fluid between two parallel plates. 3. Kinetic energy correction and momentum correction factors. 4. Power absorbed in viscous flow through (@) Journal bearings, (6) Foot-step bearings, and _(c) Collar bearings. » 9.2 FLOW OF VISCOUS FLUID THROUGH CIRCULAR PIPE For the flow of viscous fluid through circular pipe, the velocity distribution across a section, the ratio of maximum velocity to average velocity, the shear stress distribution and drop of pressure for a ‘given length is to be determined. The flow through the circular pipe will be viscous or laminar, if the Reynolds number (R,*) is less than 2000. The expression for Reynold number is given by pvD R= 2 # Density of fluid flowing through pipe Average velocity of fluid iameter of pipe and iscosity of fluid. DIRECTION OF FLOW where reap e @ o Fig. 9.1 Viscous flow through a pipe. * For derivation, please refer to Ad 12.8.1. 387 [388 Fluid Mechanics ] Consider a horizontal pipe of radius R. The viscous fluid is flowing from left to right in the pipe as shown in Fig. 9.1 (a). Consider a fluid element of radius +, sliding in a cylindrical fluid element of radius (r + dr). Let the length of fluid clement be Ax. If ‘p” is the intensity of pressure on the face AB, 1. The pressure force, p x mr? on face AB. 2p ) 2 posto (2 3. The shear force, t x 2nrAx on the Surface of fluid element. As there is no acceleration, hence the summation of all forces in the direction of flow must be zero ie., ret (peBbas) ne reaereas cs or ~ 2 aun? ex 2m An =0 0.1) ap ‘The shear stress ¢ across a section varies with‘ as 22 across a section is constant, Hence shear ox stress distribution across «section i linear as shown in Fig. 92 (@)- DISTRIBUTION DISTRIBUTION @) ® Fig. 9.2 Shear stress and velocity distribution across a section. (0 Velocity Distribution. To obtain the velocity distribution across a section, the value of shear sess t= 4-4 is substituted in equation (2.1). But in the relation = 4 Sy is measured from the pipe wal, Hence y yeR-r and du ar ar ‘Substituting this value in (9.1), we get [ Viscous Flow 389) - feu « Integrating this above equation wet. “7, we gst La 1B pyc 92 “au ax a where C is the constant of integration and its value is obtained from the boundary condition that at r=Ryu=0. 1® RL minoe) au ar Substituting this value of C in equation (9.2), we get 1 ge -P (9.3) In equation (9.3), values of 1, 2 and R are constant, which means the velocity, u varies with the square of r. Thus equation (9.3) is a equation of parabola. This shows that the velocity distribution across the section of a pipe is parabolic. This velocity distribution is shown in Fig. 9.2 (b). (ii) Ratio of Maximum Velocity to Average Velocity. The velocity is maximum, when r= 0 in equation (9.3). Thus maximum velocity, Ua, is obtained as 1 ® ax * ma (94) ‘The average velocity, i, is obtained by dividing the discharge of the fluid across the section by the area of the pipe (R°). The discharge (Q) across the section is obtained by considering the flow through ‘circular ring element of radius r and thickness dr as shown in Fig. 9.1 (b). The fluid flowing pet second through this elementary ring dQ = velocity ata radius r x area of ring element Bux nr dr - 1 Pie x nr dr ane [R= P] x Qn di o- fiao-f'-2 2 a? Py x 2K dr = 2p Ld 2) rd = E(R) x ff Py rd L (2) can ff oer oe [390 Fluid Mechanics Average velocity, it or 49.5) Dividing equation (9.4) by equation (9.5), 1 a po 1 R 4 oe 1 (_a j 1 (_a ge al ox, Ratio of maximum velocity to average velocity = 2.0, (iii) Drop of Pressure for a given Length (L) of a pipe From equation (9.5), we have (2-8 ax) Ree 1 au i = SEE bx) or 9 = SEE bs x ~ x, = L from Fig. 9.3} fm ul, a (o.=pp= EEE, erp, 7 the rp of res, Loss of pressure head Pi- Pr 32quL ps psD® Equation (9.6) is called Hagen Poiseuille Formula, (9.6) [ Viscous Flow 391) Problem 9.1 A crude oil of viscosity 0.97 poise and relative density 0.9 is flowing through a horizontal circular pipe of diameter 100 mm and of length 10 m. Calculate the difference of pressure at the two ends of the pipe, if 100 kg of the oil is collected in a tank in 30 seconds. Solution. Given : H = 0.97 poise = 7 = 0.097 Nsim? Relative density 9 Po, or density, = 0.9 x 1000 = 900 kg/m? Dia. of pipe, D= 100 mm = 0.1 m L=10m Mass of oil collected, -M-= 100 kg Time, 1= 30 seconds Calculate difference of pressure of (p, ~ p2) ‘The difference of pressure (p, ~ p;) for viscous or laminar flow is given by ae = py = 22H here i = average velocity = pine ge velocity = 3S Now, mass of oilsee 10 ais 30 = p)x=900x0 (2 pp= 900) 100 100 = 699 30 a For laminar or viscous flow, the Reynolds number (R,) is less than 2000. Let us calculate the Reynolds number for this problem. Reynolds umber, Rt = OV. n 0471, D=01 m= 0.097 900 x A71x oA 436.91 ‘As Reynolds number is less than 2000, the flow is laminar. 320097 x 47110 py? «ay 1462.28 x 10-4 Niem? = 0.1462 N/em*, Ans. * For derivation, please refer to Art. 12.8.1 and (392. Fluid Mechanics Problem 9.2. An oil of viscosity 0.1 Ns/m* and relative density 0.9 is flowing through a circular Pipe of diameter 50 mm and of length 300 m. The rate of flow of fluid through the pipe is 3.5 litres/s. Find the pressure drop in a length of 300 m and also the shear stress at the pipe wall. Solution. Given : Viscosity, = 0.1 Ns/m? Relative density 09 0.9 x 1000 = 900 kg/m* D=50:mm=.05m L= 300m 9 oF density of oi Density of water = 1000 kg/m") Q=35 litrsss= 35 0035 ms Find (i) Pressure drop, Ps Gi) Shear stress at pipe wall, t 32uuL (Pressure drop (py-p2) = 22H 0035 __.0035 oe = 1.782 mis Fo F(0s) ‘The Reynolds number (R,) is given by, where p = 900 kg/m’, V = average velocity = u = 1.782 m/s. 4900 x £782.05 On As Reynolds number is less than 2000, the flow is viscous or laminar pupae 3201 1.782 «3000 1 Pa (05)? 184288 Nim” (i) Shear Stress at the pipe wall (to) 8428 x 104 Nie ‘The shear stress at any radius r is given by the equation (9.1) 68.43 N/em’. Ans. el “aed Shear stress at pipe wall, where r= R is given by Now _ 684288 N/m? 300 m Dos ty = 2280.96 x 5B [ Viscous Flow 393) Problem 9.3 A laminar flow is taking place in a pipe of diameter 200 mm. The maximum velocity is 15 m/s, Find the mean velocity and the radius at which this occurs. Also calculate the velocity at 4 cm from the wall of the pipe. Solution. Given : Dia. of pipe, D = 200 mm = 0.20 m Upax = 15 m/s Find (i) Mean velocity, « (ii) Radius at which & occurs: (iii) Velocity at 4 em from the wall ( Mean velocity, i Ratio of (ii) Radius at which i occurs ‘The velocity, u, at any radius ‘is given by (9.3) 1a ge. 1 Re aor! But from equation (9.4) Ups is given by o() Now, the radius r at which w= i = 0.75 mis 0.15 = 15 [+] 0.1 x V5 = 0.1 x .707 = .0707 m 70.7 mm. Ans. (il) Velocity at 4 em from the wall = 4.0 10 - 4 = 6.0 em = 0.06 m [394 Fluid Mechanics ‘The velocity at a radius = 0.06 m ‘or 4-cm from pipe wall is given by equation (1) vaali(sh pot er 1.5{L.0 ~ 36] = 1.5 x64 = 0.96 mis. Ans. Fig. 9.4 Problem 9.4 Crude oil of = 1.5 poise and relative density 0.9 flows through a 20 mm diameter vertical pipe. The pressure gauges fixed 20 m apart read 58.86 N/cm? and 19.62 N/em? as shown in Fig. 9.5. Find the direction and rate of flow through the pipe. 15 Solution. Given HW 15 poise = 7> = 0.15 Nim* Relative density 9 Density of oil .9 x 1000 = 900 ke/m* Dia. of pipe, 58.86 x 10* N/m? 19.62 x 10" Nim’, 58.86 Nem? 19.62 Nlem? Find (i) Direction of flow (i Rate of flow. (0 Direction of flow. To find the direction of flow, the otal energy (£ e. 7) atthe lower end Pg 2g ‘and at the upper end Bist be eased. The drcton of flow wil Be giver rom th highs nergy tothe lower eeray: As hee the ameter he pps noome and hence Enc ene at and Bl be same. Hence to find the direction of flow, calculate (E+ 2) aA and B. 8 Paez) at a2, Pg stags me mean 158.86 Nicm* = 6667 m P , FE 20mm the vane of (2+ z)at B= 2242, w fe Fig. 95 = 210981 59-9999 4 202 42.22 m 900x981 As the value of (2+ 2 is higher at A and hence flow takes place from A 10 B. Ans. 08 (i) Rate of flow. The loss of pressure head for viscous flow through circular pipe is given by [ Viscous Flow 395) For a vertical pipe Loss of peizometric head = (Be+2,)- (Be 29] = 6667 4220 = 2445 m pe Ps a4.as = 32015 xix 200 ‘900 x 9.81 x (02) or 7 = 2485 9009.81 %.0004 _ 9 555 0.9 mis 32.x015x200 ‘The Reynolds number should be calculated. If Reynolds number is less than 2000, the flow will be Jaminar and the above expression for loss of pressure head for laminar flow can be used. Now Reynolds number ei # where p = 900 kg/m? and Reynolds number = 900 x oe 108 015 ‘As Reynolds number is less than 2000, the flow is laminar. Rate of flow = average velocity x area = WE D209 x F x (02)? ms = 2.827 x 10-4 ms = 0.2827 litres/s. Ans. (10% m?= 1 litre) Problem 9.5 A fluid of viscosity 0.7 Ns/m’ and specific gravity 1.3 is flowing through a circular pipe of diameter 100 mm. The maximum shear stress at the pipe wall is given as 196.2 N/m, find (i) the pressure gradient, (i) the average velocity, and (iii) Reynolds number of the flow. Solution. Given : M Sp. gr. = 1. Density = 1.3 x 1000 = 1300 kg/m? Dia. of pipe, D=100 mm=0.1 m Shear stress, % Find (Pressure gradient, 2 (ii) Average velocity, iii) Reynolds number, R, ap Pressure gradient, “2 @ a a The maximum shear stress (%) is given by WR or 1962 2 ax 4 Ox a [396 Fluid Mechanics ] 196.2%4 _ 2% =~ 7848 Nim? per m ax On Pressure Gradient = — 7848 N/m? per m. Ans. (ii) Average velocity, AY_ 1 2 pe m2 | ay ae (2 ee marl ze X (7848) x (.05)° { .50 mis (iii) Reynolds number, R, GxD_pxaxD Hip 350 x01 80.00. Ans. = 1300 Problem 9.6 Whar power is required per kilometre of a line 10 overcome the viscous resistance 10 the flow of glycerine through a horizontal pipe of diameter 100 mm at the rate of 10 litres/s ? Take l= 8 poise and kinematic viscosity (v) = 6.0 stokes. Solution. Given : Length of pipe, L=1 km = 1000 m Dia. of pipe, D= 100mm =0.1m 10 9) . Discharge, = 10 iees’s= 19 ms =01 m i 2 1000 2 Viscosity, H=8 poise NS =08 Nm? Kinematic Viscosity, = 6.0 stokes (pose = ns/m?) = 6.0 cm/s = 6.0 x 10-* m/s, Loss of pressure head is given by equation (9.6) as y= 22MM pad Power required = W x hy watts oy cigh of oil flowing per see = pg x Q Substituting the values of W and fn equation (D, (32 nue) x32 pal Power required = (sx watts = 2°32 nul (ancelin r req (8x 0) pr wat > « 18 Ps) But a=—2. a ors 1.273 mis xx(? [ Viscous Flow 397) = 132 x08 x 1.273 x 1000 ay = 32588.8 W = 32.588 kW. Ans. “. Power required » 9.3 FLOW OF VISCOUS FLUID BETWEEN TWO PARALLEL PLATES In this case also, the shear stress distribution, the velocity distribution across a section ; the ratio of ‘maximum velocity to average velocity and difference of pressure head for a given length of parallel plates, are to be calculated, PARALLEL PLATE DIRECTION OF FLOW t fe Eay)axxt a roves | PARALLEL PLATE an Fig. 9.6 Viscous flow between two parallel plates. Consider two parallel fixed plates kept at a distance ‘r’ apart as shown in Fig. 9.6. A viscous fluid is flowing between these two plates from left to right. Consider a fluid element of length Ax and thick- ness Ay at a distance y from the lower fixed plate. If p is the intensity of pressure on the face AB of the Oy the pressure force, p x Ay x I on face AB. 3. The shear force, t x Axx 1 on face BC. For steady and uniform flow, there is no acceleration and hence the resultant force inthe direction of flow is zero. pivxt-(r+ae)ancr rarer (eo) aoe s0 ar a» a ae or ® sray + F syar=o ap. at ap _at Dividing by Ardy, we got — 22.42% jeer 97 bY ArAy, weet —3, +5, =O OF ae By a [398 Fluid Mechanics ] (® Velocity Distribution. To obtain the velocity distribution across a section, the value of shear from Newton’s law of viscosity for laminar flow is substituted in equal n (9.7), af, de) oe ax ay! ay) a? ou 1a a? wa Integrating the above equation wars y, we get au _ 1 ap { a } ou 1 2 is constant ay war’ ar Lap»? Integrating again u miaiae + Cyt (9.8) where C, and C; are constants of integration, Their values are obtained from the two boundary condi- tions that is (?) at y = 0, u = 0 (ii) aty = 1, u= 0. ‘The substation of y= 0,40 in equation (2.8) ves 0=04C,x04 Cor C,=0 ‘The substiution of y=, =O in equation (©8) gives 02122 exiso par? 9.9) equation (9.9) 4 equation of a parabola. Hence velocity distribution across a section of the parallel plate is parabolic. This velocity distribution és shown in Pig. 9.7 a). ie — Fig. 9.7 Velocity distribution and shear stress distribution across a section of parallel plates. Viscous Flow 399) (i Ratio of Maximum Velocity to Average Velocity. The velocity is maximum, when y = 1/2. Substituting this value in equation (9.9), we get mel] Lae Lae tele tee (0.10 tule 2y ax 4 a ‘The average velocity, 1, is obtained by dividing the discharge (Q) across the section by the area of the section (1 1). And the discharge Q is obtained by considering the rate of flow of fluid through the strip of thickness dy and integrating it. The rate of flow through strip is dQ = Velocity at a distance y x Area of strip LP hry 9 2y ar [ty y']xdy x1 E o=f do=f- 22% wy yu Ox -- Leo vy] tale e 3), 1 23 2 2y ax 2y ax tape 2p ax (9.11) (9.12) (ii) Drop of Pressure head for a given Length. From equation (9.11), we have ap __ in [400 Fluid Mechanics ] or Pi-pa= L eects If his the drop of pressure head, then Le a Par ed (jv) Shear Stress Distribution. Itis obtained by substituting "7b the value of u from equation (9.9) into eee au Fig. 98 meee) (9.14) a Inequaton 14) 2 an ae cons Hen ves ntl wih. The sear es dttaton " is shown in Fig, 9.7 (6). Shear stress is maximum, when y= 0 orf atthe walls ofthe plates. Shear stress is zero, when y = 12 that i at the centre line between the two plates. Max. shear stress (t,) is given by 1, 2 ax" Problem 9.7 Calculate : (i) the pressure gradient along flow, (ii) the average velocity, and (ii) the discharge for an oil of viscosity 0.02 Non? flowing between two stationary parallel plates 1 m wide maintained 10 mm apart. The velocity midway between the plates is 2 m/s. Solution. Given : % (9.15) Viscosity, Width, Distance between plates, = 10 mm Velocity midway between the plates, Up (@ Pressure gradient (2) Lope 1 (2) 2 sing equation 9.10), Ugg =-—- 2? or 20=-— (2) cor Using equation (8:10), Uma =~ gh 0=- (2) con’ tb. _ 20X82? __ 3209 N/m? perm. Ans. dv O1xo1 (ii) Average velocity (i) Using equation (9.12), v= Mom 222 1.39 mis, Ans (id) Discharge (Q) = Area of flow x= bx 1x i= 1.01% 1.3 [ Viscous Flow 401) Problem 9.8 Determine (i) the pressure gradient, (i) the shear stress at the two horizontal parallel plates and (iti) the discharge per metre width for the laminar flow of oil with a maximum velocity of 2 m/s between two horizontal parallel fixed plates which are 100 mm apart. Given w= 2.4525 N sim’. Solution. Given : Unyay = 2 ts, t= 100 mm = 0.1 m, w= 2.4525 Nim? Pint) Psa sate, (i) Shear stress at the wall, t (iii) Discharge per metre width, Q. (0 Pressure gradient, & ae Maximum velocity, Ua is given by equation (9.10) toe Bu de Substituting the values 2 xc or 2.0= ~ Bx 2.4525 dp __20x8%24525 == 3924 Nim? per m. Ans. x 1x Ls (i Shear stress at the wall, t) 1 dp. 2 dx so ie by euton 15) 25 r=} (e309 x01 ©1962 Nw. Ams (iii) Discharge per metre width, Q = Mean velocity x Area Ans. 22 Ugg xix = 22004 10888 Problem 9.9 An oil of viscosity 10 poise flows between two parallel fixed plates which are kept at «a distance of $0 mm apart. Find the rate of flow of oil between the plates if the drop of pressure in a length of 1.2 m be 0.3 Niem’. The width of the plates is 200 mm. Solution. Given H= 10 poise =10 22 2 = 7g Nim? = 1 N si 0.05 m 0.3 x 104 N/m? Width, Find Q, rate of flow ‘The difference of pressure is given by equation (9.13) [402 Fluid Mechanics ] 12puL pins F Substituting the values, we get . ax120 03 x 10= 12 10 x * 05 x.05 4 = O3%10" X10%.08%.05 559 ay, 72120 +. Rate of flow ix Area = 0.52 «(BX 0 = 0.52 x 0.20 x .05 m°/s = .0052 m*/s .0052 x 10° litre/s = 5.2 litres. Ans. Problem 9.10 Water at 15°C flows between two large parallel plates at a distance of 1.6 mm apart. Determine (i) the maximum velocity (i) the pressure drop per unit length and (ii) the shear stress at the walls of the plates if the average velocity is 0.2 m/s. The viscosity of water at 15°C is given as 0.01 poise. Solution. Given : 1.6 mm = 1.6 10° m .0016 m 2 m/sec, [= 01 poise = a = 0.001 N sim? ( Maximum velocity, Ujyax is given by equation (9.12) he, Upay = (ii) The pressure drop, (p, ~ 3) is given by equation (9.13) Qual Pi-m= PH : or pss di per aig = 384 a 02s oxr4a nh 10” (,0016)" (uy Shear sree at the wats given by equation 215) La od 2ax “'2 Problem 9.11 There is a horizontal crack 40 mm wide and 2.5 mm deep in a wall of thickness 100 mm. Water leaks through the crack. Find the rate of leakage of water through the crack ifthe difference of pressure between the two ends of the crack is 0.02943 N/em?. Take the viscosity of water equal to 0.01 poise Solution. Given Width of rack, b= 40: mm=0.04m Depth of erack, 2.5 mm = 0025 m Length of crack, L= 100 mm = 0.1 m 937.44 x 0016 = 0.749 Nim? Ans. Viscous Flow 403) Py ~ Pz = 0.02943 N/em? = 0.02943 x 10* Nim? = 294.3 Nim? ONS = .01 poise Find rate of leakage (Q) (1 ~ 2) is given by equation (9.13) as pecs BBR oy 2063 = 1a Mb AON 10 * (0025 x.0025) 294.3 x 10 x.0025 x.0025 12x.01x04 5328 m/s Rate of leakage = ii x area of cross-section of crack 1,538 x (b x1) 1,538 x .04 x ,0025 m/s = 1.538 x 10°“ m/s = 1.538 x 10 x 10° litre/s = 0.1538 litre/s. Ans. Problem 9.12 The radial clearance between a hydraulic plunger and the cylinder walls is Ol mm : the length of the plunger is 300 mm and diameter 100 mm. Find the velocity of leakage and rate of leakage past the plunger at an instant when the difference of the pressure between the two ends of the plunger is 9 m of water. Take = 0.0127 poise. Solution. Given : ‘The flow through the clearance area will be the same as the flow between two parallel surfaces, 1= 0.1 mm = 0.0001 m L= 300 mm = 0.3m Diameter, D= 100mm =0.1 m Difference of pressure = PLO Pt <9 m of water Ps i= P2= 9% 1000 x 9.81 N/m? = 88290 Nim™ Viscosiy, 11 =.0127 poise = 27.85 10m Find (Velocity of leakage, ie, mean velocity (ii) Rate of leakage, 0 (0 Velocity of leakage (7). The average velocity (if) is given by equation (9.11) ge-laee Tay ax -_! Pry = onT L (0001) x (.0001) 10, 138290 — .0001) x (0001 xo 03 (,0001) x (,0001) «193 m/s = 19.3 ems. Ans. [404 Fluid Mechanics ] (Gi) Rate of leakage, Q Q =u xarea of flow = 0.193 x RD x ¢m°/s = 0.193 x xx .1 x.0001 ms .06 x 10-6 mss = 6.06 x 10 x 10° titre/s 6.06 x 10° litre/s. Ans. > 9.4 KINETIC ENERGY CORRECTION AND MOMENTUM CORRECTION FACTORS Kinetic energy correction factor is defined as the ratio of the kinetic energy of the flow per second based on actual velocity across a section to the kinetic energy of the flow per second based on average velocity across the same section. It is denoted by «. Hence mathematically, K.EJsec based on actual velocity ee (9.16) KB/sec based on average velocity Momentum Correction Factor. It is defined as the ratio of momentum of the flow per second based on actual velocity t the momentum of the flow per second based on average velocity across a section, It is denoted by f, Hence mathematically, B Problem 9.13 Show that the momentum correction factor and energy correction factor for laminar Slow through a circular pipe are 4/3 and 2.0 respectively Solution. (i) Momentum Correetion Factor or B ‘The velocity distribution through a circular pipe for laminar flow at any radius r is given by ‘Momentum per second based on actual velocity rn Momentum per second based on average velocity equation (9.3) 1 x) qe or u=t{-2) @_7) 1 2 ae ) (1) Consider an elementary area dA in the form of a ring at a radius r and of width dr, then dA = 2nr dr fH A= 2nd Fig.9.9 Rate of fluid flowing through the ring = dQ = velocity x area of ring element = ux 2nr dr ‘Momentum of the fluid through ring per second = mass x velocity = pxdQxu=p x 2nrdrxux us 2np wr dr ++ Total actual momentum of the fluid per second across the section = [fo atear [ Viscous Flow 405] Substituting the value of u from (1) =a [ [t (S)e - “iy rr - anf (2 [weer ir ; (2) Jf te 2R°) rar wont 6 4 wy GR +2R° —6R® Ox. 12 > aS gp wy an)’, RE map)" ps 0 ) 6 we o Momentum of the fluid per second based on average velocity mass of fluid = MASS OF TINE | average velocity where A= Area of cross-section = rR”, = average velocity 12) 9p sath) -1(-2)e CH) ‘Momentum/see based on average velocity pxars (22) -pae mal ax na +) ‘Momentum / sec based on actual velocity ‘Momentum / sec based on average velocity [406 Fluid Mechanics ] (ii) Energy Correction Factor, a. Kinetic energy of the fluid flowing through the elementary ring of radius *r’ and of width ‘dr’ per sec x mass x 1 X px (ux 2nr dr) xu = +e x 2nnw dr= mpru dr Total actual kinetic energy of flow per second ig -_™ (2) R [22] Bau ae 4 = (_) et ae 3) 8 aed Substituting the value of and [ Viscous Flow 407] Kinetic energy of the flow/see wdcpxnntx | ®)e] atep mex] Xf 2) xpxnr?x—1__(_ 2) , xn aml 3x) = PR (a) gs mar (de) ** “8 K.EJ/sec based on actual velocity _ Equation (4) K-E./see based on average velocity Equation (5) 7p ( zy = MW ar) 81288 20, Ans. . -2) Re NS 128 x 8p? ax » 9.5 POWER ABSORBED IN VISCOUS FLOW For the lubrication of the machine parts, an oil is used, Flow of oil in bearings is an example of viscous flow. Ifa highly viscous oil is used for lubrication of bearings, it will offer great resistance and thus a greater power loss will take place. But if alight oil is used, a required film between the rotating part and stationary metal surface will not be possible. Hence, the wear of the two surface will take place. Hence an oil of correct viscosity should be used for lubrication. The power required to overcome the viscous resistance in the following cases will be determined 1. Viscous resistance of Joumal Bearings, 2. Viscous resistance of Foot-step Bearings, 3. Viscous resistance of Collar Bearings. 9.5.1 Viscous Resistance of Journal Bearings. Consider a shaft of diameter D rotating in a journal bearing. The clearance between the shaft and journal bearing is filled with a viscous oil. The oil film in contact with the shaft rotates atthe same speed as that of shaft while the ol film in contact with journal bearing is stationary, Thus the viscous resistance will be offered by the oil to the rotating shat Let N = speed of shaft in rpm. iekness of oil film ngth of oil film 2nN ‘Angular sped of the shaft, o = 2%. yur 0 Tangential speed of the shaft= 0 R or V= 22, D_ EDN 6 <2" 60 du The shear stress inthe ois given by.t= y [408 Fluid Mechanics ] 1 Fi t Fig. 9.10 Journal bearing. As the thickness of oil film is very small, the velocity distribution in the oil film can be assumed as linear. Hence a V-0Y xDH ocr aor ow TSE G0xr Shear fre or vcous resistance» Ae ofa shat = BRON y ppp = HE DNL oor or Torque required to overcome the viscous resistance, T= Viscous resistance x 2 z _MREDENL D_ x? D?NL or <2 120r Power absorbed in overcoming the viscous resistance ape DENT _2AN yx? DNL “0 6” 1207 xD N'L 60x60xt Problem 9.14 A shaft having a diameter of 50 mm rotates centrally in a journal bearing having a diameter of 50.15 mm and length 100 mm. The angular space between the shaft and the bearing is filled with oil having viscosity of 0.9 poise. Determine the power absorbed in the bearing when the speed of rotation is 60 rp.m. watts. Ans. (9.18) Solution. Given : Dia. of shaft, D= 50mm or 05 m Dia. of bearing, D, = 50.15 mm or 0.05015 m Length, L = 100 mm or 0.1 m ‘Power, P=Tx oo: [ Viscous Flow 409] 09 Ns of oil = 09 poise = 22 NS HOFOI= 09 poise = 9 a? N= 600 rpm. Power =? ‘Thickness of oil film, r= 2—P 5015-50 z z 5. 0.075 mm = 0075 10 m RDN _ %x 0.05% 600 Tangential speed of shaft, V = BON. 2X005%600 _ 9.5 xm mis ema pee 0 0 Shear stress ay" 110 * 0075x10 1 Area = 1883.52 x RD XL = 1883.52 x x x .05 x 0.1 = 29.586 N dt 2 V 208, _O5%R_ _ 1583.52 wim? Shear force (F) 2nNT _ 21x 600% 0.7387 “OO 60 Problem 9.15 A shaft of 100 mm, diameter rotates at 60 rp.m. in a 200 mm long bearing. Taking that the two surfaces are uniformly separated by a distance of 0.5 mm and taking linear velocity distribution in the lubricating oil having dynamic viscosity of 4 centipoises, find the power absorbed in the bearing. Solution. Power = 46.41 W. Ans. Dia. of shaft, D=100mm=0.1m Length of bearing, L= 200 mm = 0.2m 1=0.5 mm=.5x 10% m = 4 cemtipoise = .04 poise = 9. N 0 rpm. Find power absorbed pate DNL Using equation (9.18), sing equation (9.18), eoenet . 04 1? x(A)* x (60)? x02 “10 * 60x 60%05x10" Problem 9.16 A shaft of diameter 0.35 m rotates at 200 r-p.m. inside a sleeve 100 mm long. The dynamic viscosity of lubricating oil in the 2 mm gap between sleeve and shaft is 8 poises. Calculate the power lost in the bearing. Solution. Given : Dia. of shat, D=035m 1.961 x 10 W. Ans. [410 Fluid Mechanics ] ecient aren eae Sie ee Distance between sleeve and shaft, f= 2mm = 2x 10° m a w= 8 pabe = NE De erat eset re ate ux’D'N*L 60x 60 xr aus 8 — x x(35) x(200)? x01 10 60x 60x 2x 10° Problem 9.17 A sleeve, in which a shaft of diameter 75 mm, is running at 1200 rp.m., is having a radial clearance of 0.1 mm. Calculate the torque resistance if the length of sleeve is 100 mm and the space is filled with oil of dynamic viscosity 0.96 poise. .59 kW. Ans. 5908 W= ‘Sonar, Give: paveeaaed pe eeeeones pees 1= 0.1 mm =0.1 x 103m Perna = oe sores 096 poise = 058.8 H poi 10 m? EDN _ X75 x1200 Tangential vloty of tan, v= 22". -A7t mi i y pueeas) a sep Yn 28 472 45095 Nim? 70 xt ieee es porn aes Problem 9.18 A shaft of 100 mm diameter runs in a bearing of length 200 mm with a radial clear ‘ance of 0.025 mm at 30 r.p.m. Find the velocity of the oil, if the power required to overcome the viscous resistance is 183.94 watts. Solution. 100 mm 025 mm = 0.025 x 10° m N= 30 rpm. ; HP. = 0.25 Find viscosity of oil, ‘The h.p. is given by equation (9.18) as [ Viscous Flow 411] 3p) 2 (a)? (30)? pe BR DNL 5 sg394 = HE X(A) x(30)? x02 60x 60x (60 x 60 x 0.025 X10 183.94 x60 x 60 x.025 x 10> Ns x.001X900x02 — m? = 2.96 96 x 10 = 29.6 poise. Ans. 9.5.2 Viscous Resistance of Foot-Step Bearing. Fig. 9.11 shows the foot-step bearing, in which a vertical shaft is rotating. An oil film between the bottom surface of the shaft and bearing is provided, to reduce the wear and tear. The viscous resistance is offered by the oil to the shaft. In this case the radius of the surface of the shaft in contact with oil is not constant as in the case of the journal bearing. Hence, viscous resistance in foot-step bearing is calculated by considering an elementary ular ring of radius r and thickness dr as shown in Fig. 9.11. Let N = speed of the shaft 1 = thickness of oil film R= radius of the shaft in| ‘Area ofthe elementary ring = 2nrdr av srry) Now shears given by r= Mt = where Vis the tangential velocity of shaft at radius r and is equal tot: oxr=2® x, ° 60, Shear force on the ting = dF = + x area of elementary ting 2nW or wen ax NE some dr= OT 3 dr ‘Torque required to overcome the viscous resistance, Fig. 9.11 Footstep bearing. dT = dF xr = Py drxr= EH eNP ar (9.19) 1st ist Total torque required to overcome the viscous resistance, reff ar= [fh ener fe iss ww [Pare win] ath gn is is aL sg HR! =H ene’ 9.194) oor ™ i ) Power absorbed, P an watts: = ZN BM gtypt WON RY (9.20) 60° 600 60 x30r (412. Fluid Mechanics Problem 9.19. Find the torque required to rotate a vertical shaft of diameter 100 mm at 750 r-p.m. The lower end of the shaft rests in a foot-step bearing. The end of the shaft and surface ofthe bearing are both flat and are separated by an oil film of thickness 0.5 mm. The viscosity of the oil is given 1.5 poise. Solution. Given : Dia, of shat, D= 100 mm =0.1m ra 2 =O 005m 272 N= 750 rpm Thickness of oi film, = 0.5 mm = 0.0005 m 15 Ns = 1.5 poise = 15. Ns He 15 poise = ToS The torque required is given by equation (9.19) as HO oeyRt r= ene Nm or AS? x 750%(.05)* = 0.2305 Nm. Ans. (060.0005 Problem 9.20 Find the power required to rotate a circular disc of diameter 200 mm at 1000 r-p.m. The circular disc has a clearance of 0.4 mm from the bottom flat plate and the clearance contains oil of viscosity 1.05 poise. Solution. Given : Dia. of dise, D=200mm=0.2m N= 1000 rpm. ‘Thickness of oil film, ‘The power required to rotate the disc is given by equation (9.20) as aytpe ENR ants 60 x30%F 105 |? x 1000? x (0.1)* 10 60%30%.0004 9.5.3 Viscous Resistance of Collar Bearing. Fig. 9.12 shows the collar bearing, where the face of the collar is separated from bearing surface by an oil film of uniform thickness. Let peed of the shaft in p.m. Internal radius of the collar 152.1 W. Ans. [ Viscous Flow 413] Ry = External radius of the collar COLLAR Thickness of oil film, Fig.9.12 Collar bearing. Consider an elementary circular ring of radius *r’ and width dr of the bearing surface. Then the torque (dT) required to overcome the viscous resistance on the elementary circular ring is the same as given by equation (9.194) or at=* NP dr 1st Total torque, required to overcome the viscous resistance, on the whole collar is Rs Rs nye ref" ars fh ewe n| 4d, R, In, 151 = wir —Ry 920 1stx4 i . y Power absorbed in overcoming viscous resistance pe 2ENT 2h May iR, 60 60 60r wen? 4g og BENT i yt watts, 022 60x30 [Ry vd 4 ) Problem 9.21 A collar bearing having external and internal diameters 150 mm and 100 mm respectively is used to take the thrust of a shaft. An oil film of thickness 0.25 mm is maintained between the collar surface and the bearing. Find the power lost in overcoming the viscous resistance When the shaft rotates at 300 rp.m. Take 4 = 0.91 poise. Solution. Given : External Dia. of collar, —_D, = 150 mm = 0.15 m Internal Dia, of collar, 100 mm = 0.1 m D,_ od R= et 005m = =< 005 ‘Thickness of oil film, = 0.25 mm = 0.00025 m N= 300 pam, = 0.91 poise = 291 Ns 10 [414 Fluid Mechanics ] ‘The power required is given by equation (9.22) or = tren 4 ps * Goxa0xr m1 agi _ x? «300 x [075+ - 0s] 10 (60 x30 x.00025 164314 [.00003164 ~ .00000625] 564314 x 00002539 = 14.327 W. Ans. Problem 9.22 The external and internal diameters of a collar bearing are 200 mm and 150 mm respectively. Between the collar surface and the bearing, an oil film of thickness 0.25 mm and of viscosity 0.9 poise, is maintained. Find the torque and the power lost in overcoming the viscous resistance of the oil when the shaft is running at 250 r.p.m. Solution. Given : D,=200 mm = 0.2 m Pe 02 Loim 2 D, = 150 mm = 0.15 m Torque required is given by equation (9.21) x’ x250[0.1* —.075* re! ewe og. x250[0*— 075'] oor 10 * 60 x0.0025 = 14804.4 [ .0001 ~ 00003164] = 1.0114 Nm. Ans. Power fost in viscous resitanee Nm = 26.48 W. Ans. 0 oO ™ » 9.6 LOSS OF HEAD DUE TO FRICTION IN VISCOUS FLOW The loss of pressure head, ina pipe of diameter D, in which a viscous fluid of viscosity fis flowing with a velocity w is given by Hagen Poiseuille formula i.e., by equation (9.6) as 24k “@ where = length of pipe ‘The loss of head due to friction® is given by Se i) (+: velocity in pipe is always average velocity Dx2g "For derivation, please refer to Art 103.1 [ Viscous Flow 415] where f= co-efficient of frict n between the pipe and fluid, 32qaL_4.f.L.ae pgD? Dx2g Equating (0) and (ii), we get 32M x Dx 2g __ 164 fo i8, (9.23) Problem 9.23 Water is flowing through a 200 mm diameter pipe with coefficient of friction {= 0.04. The shear stress at a point 40 mm from the pipe axis is 0.00981 N/em?. Calculate the shear stress at the pipe wall. Solution. Given : Dia. of pipe, Co-efficient of friction, Shear stress at r= 40 mm, Let the shear stress at pipe wal First find whether the flow is viscous or not. The flow will be viscous if Reynolds number R, is less than 2000. Using equation (9.23), we get/: 16 R= 18 =a te = oy = 400 This means flow is viscous. The shear stress in case of viscous flow through a pipe is given by the ‘equation (9.1) as pe ax 2 But 2 js constant aross a section, Across a section, there is no variation of x and there is no variation of p. : coe {At the pipe wall, radius = 100 mm and shear stress i tp tt 5, 000981_ 7-100“ ~ 40 ~ 100 «= 1000.00581 : = 0.0245 N/cm?. Ans. 40 [416 Fluid Mechanics ] Problem 9.24 A pipe of diameter 20 cm and length 10° m is laid at a slope of 1 in 200. An oil of sp. gr. 0.9 and viscosity 1.5 poise is pumped up at the rate of 20 litres per second. Find the head lost due t0 friction. Also calculate the power required to pump the oil. Solution. Given : Din ofp, D= 200m =250m tenga pie, 1 1000 1 lope fp, tetin2000 Slope of pipe 100 200 Sp. gr. of oil, s=09 Density of oil, p= 0.9 x 1000 = 900 kg/m? ont oro, w= 1 pie = 1S Discharge, Q = 20 litrels = 0.02 ms 1m} Velotyofow, = 2 = RO O02 96356 Wea Ep Rap go 502) 10 x0.66%.2 3 10 900 6366x210 ots 15 = 763.89 As the Reynolds number is less than 2000, the flow is viscous. The co-efficient of friction for viscous flow is given by equation (9.23) as 02094 af Lie Dx2g *. Head lost due to friction, jy 4.02094 10000 x (.6366)* 022x981 Due to slope of pipe 1 in 200, the height through which oil is to be raised by pump = Slope x Length of pipe 1 m = 86.50 m. Ans. op * 10000 = 50m Total head against which pump is to work, H = hy+ 1x L= 86.50 + 50 = 136.50 m Power required to pump the oil [ Viscous Flow 417] H _ 900%9810.20%13650 _ 944 kw Ans, 1000 > 9.7 MOVEMENT OF PISTON IN DASH-POT. Consider a piston moving in a vertical dash-pot containing oil as shown in Fig. 9.13. Let D = Diameter of piston, L = Length of piston, W-= Weight of piston, Viscosity of oil, Velocity of piston, t Average velocity of oil in the clearance, Clearance between the dash-pot and piston, Difference of pressure intensities between the two ends of the piston, The flow of oil through clearance is similar to the viscous flow between two parallel plates. The difference of pressure for parallel plates Eisicx for length “L’is given by Fig. 9153 1p. \p = = w [Also the difference of pressure atthe two ends of piston is given by, Weight of w _4aw Ap = Neightof piston __W _ AW ‘Area of piston Fp RD: 4 Equating (9 and (7, we get 2HEL . AW 2 owe 12HL 3RuLD™ Vis the velocity of piston or the velocity of oll in dash-pot in contact with piston. The rate of flow of oil in dash-pot = velocity x area of dash-pot = Vx FD? Rate of flow through clearance = velocity through clearance x area of clearance = x RD Xt Due to continuity equation, rate of flow through clearance must be equal to rate of flow through dash-pot. TxmDxt=Vvx= DP 4 w=vx% px_-2 w) aD ar quuting the value of F from (i) and (i), we get we vb 3muLD? 4 [418 Fluid Mechanics ] __4ew awe BS SRLD'V 3RLD'V" Problem 9.25 An oil dash-pot consists of a piston moving in a cylinder having oil. This arrange- iment is used to damp out the vibrations. The piston falls with uniform speed and covers 5 cm in 100 seconds. If an additional weight of 1.36 N is placed on the top of the piston, it falls through 5 cm in 86 seconds with uniform speed. The diameter of the piston is 7.5 cm and its length is 10 cm. The clearance between the piston and the cylinder is 0.12 cm which is uniform throughout. Find the viscosity of oil. Solution. Given : Distance covered by piston due to self weight, (9.24) Time taken, = 100 see Additional weight, 1.36 N ‘Time taken to cover 5 cm due to additional weight, = 86 sec Dia. of piston, 7.5 em = 0.075 m Length of piston, 10 cm = 0.1 m Clearance, 0.12 em = 0.0012 m Let the viscosity of oil H ‘Weight of piston, ‘Velocity of piston without additional weight, = Velocity of piston with additional weight. awe _4[W+136]° BS 3RD'LV 3RD'LV* W _ W+136 awe a Cancelling —— . out (cassie v w or ae od ve” W+136 io But V= Velocity of piston duc to self weight of piston = Distance covered 5 ig * Time taken 100 cerns + = Distance covered due to self weight + additional weight Time taken cm/s ii) Equating (i) and (ii), we get W136 or W=0.86 W + 86 x 1.36 or W~ 0.86 W=0.14 W = 86 x 1.36 [ Viscous Flow 419 | 086 x1 086136 _ 9.354 0.14 aaa , awe Using equation 2.24), we gt = = 2b noibota} 075)" x. 31x (0.075) x 0% (3% ato) = 0.29 N sim? = 0.29 x 10 poise = 2.9 polse. Ans. > 9.8 METHODS OF DETERMINATION OF CO-EFFICIENT OF VISCOSITY ‘The following are the experimental methods of determining the co-efficient of viscosity of a liquid: 1. Capillary tube method, 2. Falling sphere resistance method, 3. By rotating cylinder method, and 4. Orifice type viscometer. ‘The apparatus used for determining the viscosity of a liquid is called viscometer. 9.8.1 Capillary Tube Method. In capillary tube method, the viscosity of a liquid is calculated bby measuring the pressure difference for a given length of the capillary tube. The Hagen Poiseuille law is used for calculating viscosity CONSTANT HEAD TANK. MEASURING TANK —| Fig. 9.14 Capillary tube viscometer. Fig. 9.14 shows the capillary tube viscometer. The liquid whose viscosity is to be determined is filled in a constant head tank. The liquid is maintained at constant temperature and is allowed to pass through the capillary tube from the constant head tank. Then, the liquid is collected in a measuring tank for a given time. Then the rate of liquid collected in the tank per second is determined. The pressure head *h’ is measured at a point far away from the tank as shown in Fig. 9.14. ‘Then h = Difference of pressure head for length L. ‘The pressure at outlet is atmospheric. Let D = Diameter of capillary tube, L= Length of tube for which difference of pressure head is known, = Density of fluid, [420 Fluid Mechanics ] and. ne ‘o-efficient of viscosity. Using Hagen Poiseuille’s Formula, ft But ‘where Q is rate of liquid flowing through tube. 32u x fe xL he a> 128na.k = gd mpgD* pghD* or (9.25) 280. Measurement of D should be done very accurately. 9.8.2 Falling Sphere Resistance Method. ‘Theory. This method is based on Stoke’s law, according to which the drag force, F on a small sphere moving with a constant velocity, U through a viscous fluid of viscosity, it for viscous conditions is given by Fe 3mud @ ae es where d= diameter of sphere U = velocity of sphere, consrat ihe ao eloiy Us the dag PERAURE = force isthe difference between the weight of sphere and tin) buoyant foree acting oni sPaene Let stance traveled by_ sphere in viscous lid, ime taken by sphere to cover distance i i 1 and F Fig. 918 Falling sphere resistance method. rp volume of sphere = © 4 { a 6 } and buoyant force, Fy = weight of fluid displaced = volume of liquid displaced x density of fluid x g Px px g (volume of liquid displaced = volume of sphere} [ Viscous Flow 421] For equilibrium, Drag force = Weight of sphere ~ buoyant force or F=W-Fy Substituting the values of F, W and Fy, we get Smid = Fa xp,xe Kp ® ZB xpxge = exe lps- Eaxpxe=F@ xs lp.-pA n@xsfp,-Ps]_ ga? 6 3d 187 or # (0, - pd (9.26) where p= Density of liquid Hence in equation (9.26). the values of d, U, p, and py are known and hence the viscosity of liquid can be determined. Method. Thus this method consists of a tall vertical transparent cylindrical tank, which is filled with the liquid whose viscosity is to be determined. This tank is surrounded by another transparent tank to keep the temperature of the liquid in the cylindrical tank to be constant. ‘A spherical ball of small diameter ‘a’ is placed on the surface of liquid. Provision is made to release this ball. After a short distance of travel, the ball attains a constant velocity. The time to travel a known vertical distance between two fixed marks on the cylindrical tank is noted to calculate the constant velocity U of the ball. Then with the known values of d, P,P; the viscosity Ht of the fluid is ‘calculated by using equation (9.26). 9.8.3 Rotating Cylinder Method. This method TORSIONAL consists of two concentric cylinders of radii R, and R, as SPRING shown in Fig. 9.16. The narrow space between the two ow power Dexa cylinders is filled with the liquid whose viscosity is to be determined. The inner cylinder is held stationary by means of a torsional spring while outer cylinder is rotated at con- \isgoys k= stant angular speed «. The torque T acting on the inner LIQUID (uy = cylinder is measured by the torsional spring. The torque on the inner cylinder must be equal and opposite to the torque ourer_.| applied on the outer cylinder. ROTATING ‘The torque applied on the outer cylinder is due to CYLINDER {scous resistance provided by liquid in the annular space and at the bottom of the inner cylinder. Let = angular speed of outer cylinder. ‘Tangential (peripheral) speed of outer cylinder Fig. 946 Rotating cylinder viscometer. =OxR, ‘Tangential velocity of liquid layer in contact with outer cylinder will be equal to the tangential velocity of outer cylinder. ‘Velocity of liquid layer with outer cylinder = @ x Ry ‘Velocity of liquid layer with inner cylinder = 0 {+> Inner cylinder is stationary} Velocity gradient over the radial distance (R, ~ Rj) du ++ Shear stress (7) (422. Fluid Mechanics ] ¢+ Shear force (F) = shear stress area of surface x 2m R\H shear stress is acting on surface area = 2 R, x H} =u x 2n RH (R,-R)) ‘The torque T, on the inner cylinder due to shearing action of the liquid in the annular space is T, = shear force X radius OR coe RHR, (=a) EN “@ If the gap between the bottom of the two cylinders is “h (7) is given by equation (9.194) as then the torque applied on inner cylinder But here NR? Tr, 200 Rta MO Rs Ai) con Qn 2h ‘Total torque 7 acting on the inner eylinder is T=1,+T; 2( Ry RAT ‘o[sHnR, + R (R,— R)] xrque measured by the strain of the torsional spring, radii of inner and outer cylinder, Jearance at the bottom of cylinders, weight of liquid in annular space, ‘o-efficient of viscosity to be determined. Hence, the value of it can be calculated from equation (9.27). 9.8.4 Orifice Type Viscometer. In this method, the time taken by a certain quantity of the liquid whose viscosity is to be determined, to flow through a short capillary tube is noted down. The co-efficient of viscosity is then obtained by comparing with the co-efficient of viscosity of a liquid whose viscosity is known or by the use conversion factors. Viscometers such as Saybolt, Redwood or Engler are usually used. The principle for all the three viscometer is same, In the United Kingdom, Redwood viscometer is used while in U.S.A., Saybolt viscometer is commonly used, 49.27) where [ Viscous Flow 423 | Fig. 9.17 shows that Saybolt viscometer, which consists of a tank at the bottom of which a short capillary tube is fitted, In this tank the liquid whose viscosity is to be determined is filled. This tank is sur- rounded by another tank, called constant temperature bath. The lig- uid is allowed to flow through capillary tube at a. standard temperature. The time taken by 60 c.¢. of the liquid to flow through the capillary tube is noted down, The initial height of liquid in the tank is previously adjusted to a standard height. From the time measure- -ONSTANT IEASURING ‘ment, the kinematic viscosity of liquid is known from the relation, Cs B ve At~— Fig. 9.17 Saybolt viscometer. where A = 0.24, B = 190, 1 = time noted in seconds, v= kinematic viscosity in stokes. Problem 9.26 The viscosity of an oil of sp. gr. 0.9 is measured by a capillary tube of diameter 50 mm. The difference of pressure head between two points 2 m apart is 0.5 m of water. The mass of oil collected in a measuring tank is 60 kg in 100 seconds. Find the viscosity of oil Solution. Given : Sp. gt of oil 9 Dia. of capillary tube, D= 50 mm = 5 em = 0.05 m Length of ube, L=2m Difference of pressure head, = 0.5 m Mass of oil M= ook Time, += 1005, Mass of oil per second = © Lo kes 100° Density of oil, P= sp. gr. of oil x 1000 = 0.9 x 1000 = 900 kg/m? Discharge, = Massofoll/s _ 06 9345 - 0.000667 mss Density 900 Using equation (9.25), we get viscosity, ee there h 1280.0 = FX 900% 9.81 0.5 x (05) 5075 (SI Units) N sim? 128% 0.000667 «2.0 = 0.5075 x 10 poise = $.075 poise. Ans. Problem 9.27 A capillary tube of diameter 2 mm and length 100 mm is used for measuring viscosity of aliquid. The difference of pressure between the two ends ofthe tube is 0.6867 N/em? and the viscosity of liquid is 0.25 poise. Find the rate of flow of liquid through the tube. Solution. Given : Dia. of capillary tube, D=2m=2x10%m Length of tube, L= 100 mm = 10.em=0.1 m Difference of pressure, ‘Ap = 0.6867 Niem? = 0.6867 x 10" Nim? Ap _ 0.686710" Ps Difference of pressure head, ht [424 Fluid Mechanics Viscosity, b= 0.25 poise = 225. Not? 10 Let the rate of flow of liquid = Q 16867210" 9 19>) pghD* Pe. Using equation (9.25), we get j= SOSMO™ = og x B equation (2.25), we BELM= 199 9 77 HPS 128xOx01 as _ x 0686710" x(2x10°)" o os eae 10 128% 0X0. oe p 6710" 2" 10"? 10 9, 128% 010.25 107.86 x 10° m/s = 107.86 x 10° x 10° emis (07.86 x 10 cm/s = 1.078 em’/s. Ans. Problem 9.28 A sphere of diameter 2 mm falls 150 mm in 20 seconds in a viscous liquid. The density of the sphere is 7500 kg/m’ and of liquid is 900 kg/m’. Find the co-efficient of viscosity of the liquid Solution. Given = of sphere, d=2mm=2x 103m Distance travelled by sphere = 150 mm = 0.15 m Time taken, 1= 20 seconds Velocity of sphere, pista SO 0075 m/s Time 20 Density of sphere, p, = 7500 kg/m? Density of liquid, y= 900 ke/m? Using tation 026, we sete Ip, p= E*P*7T 50900 Ising relation (9.26), we get w= $4 fp, p,q = ——- —__I._7500- i Ct 18 Ps PA 18 x 0.0075, = 281A 10-6600 gy NS 18x 0.0075 m = 1.917 X 10 = 19.17 poise. Ans. Problem 9.29 Find the viscosity of a liquid of sp. gr. 0.8, when a gas bubble of diameter 10 mm rises steadily through the liquid at a velocity of 1.2 cm/s. Neglect the weight of the bubble. Solution. Given : Sp. gr. of liquid =08 s: Density of liquid, p= 0.8 x 1000 = 800 kg/m? Dia. of gas bubble, D=10mm=1em=0.01m Velocity of bubble, U= L.2emis=.012 mis As weight of bubble is neglected and density of bubble [ Viscous Flow Now using the relation, Ip, ~ pl which is for a falling sphere. For a rising bubble, the relation will become as Bt ip = Sly Pd Substituting the values, we get w= 28L (0.06% 10°) 1000%9.81 Cancelling (mx 0.06 x 10) throughout, we get 3x0 = 1 0.06? x 10° 2650 «9.81 ~ 2 x 0.06 10-1000 9.81 x 0.06? x 10° x 9.81 (2650 ~ 1000) 6 =! coco 09 16 <0 v 100971273 .00323 misee. Ans. HIGHLIGHTS 4. A flow is said to be viscous if the Reynolds number is less than 2000, or the fluid flows in layers. 2. For the viscous flow through circular pipes, (i Veocty wate 2 a4 {Shear tess ° ay ax (iid) Ratio of velocities (jv) Loss of pressure head, a where 22 = pressure gradient, dius at any point, By 7 pressure a YP R= radius of the pipe, Unga, = maximum velocity oF velocity at wverage velocity = 2, = co-efficient of viscosity, 6 ar # v D = diameter of the pipe. 3. For the viscous flow between two parallel plates, Lap w= 1 Gy _y) ... Velocity distribution we y Ratio of maximum and average velocity “ Loss of pressure head Per Lap 2-2 yay). Shear stress distribution 7 yt where 1 thickness or distance between two plates, y= distance in the vertical direction from the lower plate, ‘T= shear stress at any point in flow. 428 Fluid Mechanics 4. The kinetic energy correction factor ais given as K.E per second based on actual velocity KE per second based on average velocity 2.0 .. for a circular pipe. ‘5. Momentum correction factor, Bis given by Momentum per second based on actual velocity ‘Momentum per second based on average velocity 4 = .. tora circular pipe Fi ips 6, For the viscous resistance of Journal Bearing DN du _V_xDN oo "dy Or c= BRAN shear force = oo po BEDINE Ror Where L= length of bearing, N= sped of shat Jearance between the shaft and bearing. 7. For the Foot Step Bearing, the shear fore, trae and hp. absrbed are given as Tomue, Shear foree, Feast wars rorgue, net Torque, ne sy Pome pian ox30x7 here Re radius of the shaft, N= speed of the shat 8. For the collar bearing the torque and power absorbed are given as LL pages — en? T= ay PN IRS Ri) P Where R= intemal radius of the collar, 1 = thickness of oil film, ‘external radius of the collar, power in watts 9. For the viscous flow the co-efficient of fiton is given by = 7 where R, =the Reynolds number = YD VP. uy awe 10, ‘The co-efficient of viscosity is determined by dash-pot arrangement as = 2" y yy dash-pot arrange DE nV] where W = weight of the piston, 1 = clearance between dash-pot and piston, L= length of the piston, D = diameter of the piston, V= velocity ofthe piston [ Viscous Flow 429] 11. The co-efficient of viscosity of aliquid is also determined experimentally by the following method , _ eghp* (Copia abe metho, y= SPREE a” |p, - Cerner eee | 180 2( Ry ~ RAT fi) Rotating cylinder method, = ce oe wo = a [SHAR + RF (RR) where w = specific weight of ud, = length of the tubo, D =diameer ofthe capillary whe, Q = ate of flow of ud through capillary tube, 4 diameter ofthe sphere, ,= density of sphere, y= density of fd, U = velocity of sphere, Fy mrdius of outer rotting cylinder, R= dius ofiner stationary cylinder, T = torque. EXERCISE (A) THEORETICAL PROBLEMS |. Define the terms : Viscosity, kinematic viscosity, velocity gradient and pressure gradient. ‘What do you mean by "Viscous Flow"? Derive an expression for the velocity distribution for viscous flow through a circular pipe. Also sketch the velocity distribution and shear stress distribution across a section of the pipe. 4. Prove that the maximum velocity in a circular pipe for viscous flow is equal to two times the average velocity of the flow. (Dethi University, December 2002) 5. Find an expression for the loss of head of a viscous fluid flowing through a circular pipe. What is Hagen Poiseville’s Formula ? Derive an expression for Hagen Poiseuille’s Formuls. Prove that the velocity distribution for viscous flow between two parallel plates when both plates are fixed across a section is parabolic in nature. Also prove that maximum veloeity is equal to one and a half times the average velocity. '8. Show thatthe difference of pressure head for a given length ofthe two parallel plates which are fixed and through which viscous fluid is flowing is given by 12 ui Pe. where 1 = Viscosity of fluid, Average velocity, P= Distance between the two parallel plates, ___L = Length ofthe plates. 9. Define the terms : Kinetic energy correction factor and momentum correction factor. 10. Prove that for viscous flow through a circular pipe the kinetic energy correction factor is equal to 2 while 4 momentum correction factor = 4 11. A shaft is rotating in a journal Bearing. The clearance between the shaft and the bearing is filled with a viscous oil Find an expression for the power absorbed in overcoming Viscous resistance. 12, Prove that power absorbed in overcoming viscous resistance in foot-step bearing is given by pa BENT (0x 300 where R'= Rais of the shat, Speed ofthe shat, 1 = Clearance between shaft and fot-step bearing, y= Viscosity of fu [430 Fluid Mechanics ] 13. 4. 18. 16. 1 18. 19. ‘Show that the value of the co-efficient of frietion for viscous flow through a circular pipe is given by, $= 7S, where R,= Reynolds number. Prove thatthe co-efficient of viscosity by the dash-pot arrangement is given by, awe 3RLDV where W = Weight of the piston, f= Clearance between dash-pot and piston, L = Length of piston, D = Diameter of piston, V = Velocity of piston. What are the different methods of determining the co-cfficient of viscosity of aliquid ? Describe any two ‘method in details Prove thatthe loss of pressure head forthe viscous flow through a circular pipe is given by fy SL pad” where i= Average velocity, w = Specific weight For a laminar steady flow, prove thatthe pressure gradient in a direction of motion is equal to the shear {gradient normal to the direction of motion. Describe Reynolds experiments to demonstrate the two types of flow. For the laminar flow through a circular pipe, prove that (2) the shear stress variation across the section ofthe pipe is linear and Gi) the velocity variation is parabolic, (B) NUMERICAL PROBLEMS A crude oil of viscosity 0.9 poise and sp. gt. 0.8 is flowing through a horizontal circular pipe of diameter 80 mm and of fength 15 m. Calculate the difference of pressure atthe two ends ofthe pipe, if SO kg of the oil is collected in a tank in 15 seconds. Ans. 0.559 Nim’) - A viscous flow is taking place in a pipe of diameter 100 mm. The maximum velocity is 2 m/s. Find the ‘mean velocity and the radius at which this occurs. Also calculate the velocity at 30 mm from the wall of the pipe (Ans. 1 nvs, r= 35.35 mm, «= 1.68 mvs} - A fluid of viscosity 0.5 poise and specific gravity 1.20 is lowing through a circular pipe of diameter 100 mm, The maximum shear stress at the pipe wall is given as 147.15 Nin’, find : (a) the pressure gradient, (b) the average velocity, and (c) the Reynolds number of the flow. Ans. (a) — 64746 Nim? per m, (b) 3.678 ms, (¢) 882.72] |. Determine (a) the pressure gradient, (b) the shear stress at the two horizontal parallel plates and (c) the discharge per metre width for the laminar flow of oil with a maximum velocity of 1.5 mvs between two horizontal parallel fixed plates which are 80 mam apart. Take viscosity of elas 1°62, NS m= {[Ans. (a) ~ 3678.7 Nim? per m, (b) 147.15 Nin’, (c).08 m/s} 5. Water is flowing between two large parallel plates which are 2.0 mm apart, Determine : (a) maximurn velocity, (b) the pressure drop per unit length and (c) the shear stress at walls ofthe plate if the average velocity is 0.4 mvs. Take viscosity of water as 0.01 poise. Ans, (a) 0.6 mis, (b) 1199.7 Nim? per m, (¢) 1.199 N/m?) There is @ horizontal erack SO mm wide and 3 mm deep in a wall of thickness 150 mm. Water leaks through the crack. Find the rate of leakage of water through the crack ifthe difference of pressure between the two ends ofthe ctack is 245.25 Nim*. Take the viscosity of water as 0.01 poise. (Ans. 183.9 cm/s] Viscous Flow 431] 9%, 10. 2 Be. 4. 18. 16. WW. 1. {A shalt having a diameter of 10 em rotates centrally ina journal bearing having a diameter of 10,02 em and length 20 cm. The annular space between the shaft andthe bearing is filled with oil having viscosity of 08 poise. Determine the power absorbed inthe bearing. when the speed of rotation is $00 rpm Tans. 343.6 W] {A shaft 150 mm diameter runs in @ bearing of length 300 mm, with a radial clearance of 0:04 mm at 40 rpm. Find the viscosity ofthe oll if the power required to overcome the viscous resistance is 220.725 W. [Ans. 6.32 poise] Find the torque required to rotate a vertical shaft of diameter 8 em at 800 rpm. The lower end ofthe shaft rests ina foot-step bearing. The end of the shaft and surface ofthe bearing are both fla and are separated by a ol film of thickness 0.075 em. The viscosity of the ols given as 1.2 poise. [Ams 0.0538 Nan] ‘A collar bearing having external and intemal diameters 20 em and 10 em respectively is used to take the thrust ofa shaft An oil film of thickness 0.03 em is maintained between the collar surface and the bearing. Find the power lst in overcoming the viscous resistance when the shaft rotates at 250 rpm. Take = 0.9 poise [Ans. 30.165 W) Waters flowing through a 150 mm diameter pipe witha co-efficient of frietion/=.0S. The shear stress at 4 point 40 mon from the pipe wall is 0.01962 Nfem*. Caleulate the shear stres atthe pipe wal Ans. 0.04198 Nem") {An oil dash-pot consists ofa piston moving in a cylinder having il. The piston falls with uniform speed and covers 45 em in 80 seconds. If an addtional weight of 1.5 N is placed on the top of the piston i falls through 4.5 em in 70 seconds with uniform speed. The diameter ofthe piston is 10 em and its length is 15 em. The clearance between the piston and the cylinder is 0.15 em, which is uniform throughout. Find the viscosity of ol [Ans. 0.177 poise] ‘The viscosity of ol of sp. gr. 0.8 is measured by a capillary tube of diameter 40 mm. The difference of pressure head between two points IS m apart is 0.3 m of water. The mass of oll collected in a measuring tank is 40 kg in 120 seconds. Find the viscosity ofthe oil [Ans. 2.36 poise] A capillary tube of diameter 4 ram and length 150 mm is used for measuring viscosity ofa liquid. The difference of pressure between the two ends ofthe tube is 0.7848 Nem? and the viscosity of the liquid is (0.2 poise. Find the rate of flow of liquid through the tube TAs. 1643 em) [A sphere of diameter 3 mm falls 100 mm in 1.5 seconds ina viscous liquid. The density ofthe sphere is 7000 kgf’ and of liquid is 800 kg/m’. Find the co-efficient of viscosity ofthe liquid. (Ans. 45.61 poise] ‘The viscosity of aliquid is determined by rotating eylinder method, in which case the inner cylinder of diameter 25 em is stationary. The outer eylinder of diameter 25.5 em contain the liquid upto a height of 40 em. The clearance at the bottom of the two eylinders is 0.6 em. The outer cylinder is rotated at 300 pam. The torgue registered on the torsion metre attached to the inner eylinder is 4.905 Nm. Find the viscosity of liquid. {Ans..77 poise) Calculate: (a) the pressute gradient along the flow, (b) the average velocity, and (c) the discharge for an cil of viscosity 0.02 Nsim® lowing between two stationary parallel plates 1 m wide maintained 10 mm apart. The velocity midway between the plates is 2.5 m/s. {As (a) — 4000 Nim* perm, (b) 1.667 mis, () 01667 ms) Calculate (d the pressure gradient along the flow, i) the average velocity, and (it) the discharge for an oil of viscosity 0.03 N s/t? flowing between two stationary plates which ate parallel and are at 10 mim apart, Width of plats is 2m. The velocity midway between the plates is 2.0 ms A cylinder of 100 mm diameter, 0.15 m length and weighing 10 N slides axially in a vertical pipe of 104 mm dia. I the space between cylinder surface and pipe wall is filled with liquid of viscosity y and the cylinder slides downwards at a velocity of 0.45 mis, determine 4. (Hint. D = 100 mm = 0.1, L= 0.15 m, W= 10N, D, = 1.4 mm = 0.104 m, V= 0.45 mis. Hence t= (0.104 —0.1)2 = 0.002 m. (432. Fluid Mechanics ] awe 4x10%0.0025 BRDLV 3RxO.P xO1Sx.A5 20, A liquid is pumped through a 15 cm diameter and 300 m long pipe atthe rate of 20 tonnes per hour. The density of liquid is 910 kg/m? and kinematic viscosity = 0.002 m/s. Determine the power required and show that the flow is viscous. He = 803 «10° N sim] (Hint, D = 15 em= 0.15 m, L=300 m, W= 20 tonneshe = 20% 1000 kl x 60sec = 5.555 kgtsee = 5.555 x 9.81 Nis. 5555x981 061 910% 981 2 at) Now hich is less than 2000, Hence flow is viscous. y= 32 LVpgD?, where v= 01002910 182 329 189300200348 (p10 081015") P = pe.0.h/1000=910 x 9.81 x 0,0061 x 30/1000 = 1.633 kW. | 21, Anil of specific gravity 0.9 and viscosity 10 poise is flowing through a pipe of diameter 110 mm. The velocity at the centre is mv, find :() pressure gradient inthe direction of flow, (i) shear stress atthe pipe wall; (if) Reynolds number, and (iv) velocity ata distance of 30 mm from the wall, [Hint p= 900 kg/m® ; = 10 poise = I N sim? ; D= 110 mm =0.11 m, ; 1 (=a mis 57 = 1 ms Ugg = ae ( - o Un wlan) ® Hence, AMX gag _ 412 oR 005s )e pxtxD _ 900x1x011 = 2644.6 Nim? ; =99 5 and L Fa 2544) 0. 22, Determine (i) the pressure gradient, (i) the shear stess atthe two horizontal plates, (ii) the discharge per metre width for laminar flow of oll with a maximum velocity of 2 mvs between two plates which are 150 mm apart. Given : 1 =2.5 N sim (Dethi University, December 2002) 586 mis.] (Hint. U,,., = 2 mvs, = 150 mm =0.15 m, w=2.5 N sim? (0 Ugg he 5, Wa B as 88292 pry Nt a TS tag) «tx )=(e2) x81 TURBULENT FLOW ~\ > 10.1 INTRODUCTION ‘The laminar flow has been discussed in chapter 9. In laminar flow the fluid particles move along straight parallel path in layers or laminae, such that the paths of individual fluid particles do not cross those of neighbouring particles. Laminar flow is possible only at low velocities and when the fluid is highly viscous. But when the velocity is increased or fluid is less viscous, the fluid particles do not ‘move in straight paths. The fluid particles move in random manner resulting in general mixing of the Particles. This type of flow is called turbulent flow. ‘A laminar flow changes to turbulent flow when (i) velocity is increased or (i) diameter ofa pipe is Increased or (ii) the viscosity of fluid is decreased. O. Reynold was first to demonstrate that the iso fom ania tale pendant on een ely ao he uty PE, This quantty 2% tu dimensionless quany and called Reynoks mb (R) Incase of ella . pipe if R, < 2000 the flow is said to be laminar and if R, > 4000, the flow is said to be turbulent. If By tes berween 2000 to 400, the flow changes tom laminar ole > 10.2 REYNOLDS EXPERIMENT ‘The type of flow is determined from the Reynolds number ie,, PY *4 . This was demonstrated by ©. Reynold in 1883. His apparatus is shown in Fig. 10.1. DYE CONTAINER ove ; waren _|94g8° —\guawenror Fig. 10.1 Reynold apparatus. 433 [434 Fluid Mechanics ] ‘The apparatus consists of : (A tank containing water at constant head, (ii) A small tank containing some dye, (ii) ‘8 plas tube having a bell-mouthed entrance at one end and a regulating value at other ends. The water from the tank was allowed to flow through the glass tube. The velocity of flow was varied by the regulating valve. A liquid dye having same specific welght as water was inttoduced Into the glass tube as shown in Fig. 10.1 The following observations were made by Reynold : OYE (@_ When the velocity of flow was low, the dye fila- ment in the glass tube was inthe form ofa straight line ‘This straight line of dye filament was parallel to the @Lammarfow ~~ Wavy glass tube, which was the ease of laminar flow as shown, FILAMENT in Fig. 10.2(a). (ii) With the increase of velocity of flow, the dye- filament was no longer a straight-line but it became a —_(®) Transition PUAMENT wavy one as shown in Fig. 10.2 (6). This shows that flow is no longer laminar. (iii) With further increase of velocity of flow, the (@)Tubulntfow wavy dye-filament broke-up and finally diffused in water as shown in Fig. 10.2 (c). This means that the fTuid particles of the dye at this higher velocity are moving in random fashion, which shows the case Of turbulent flow. Thus in case of turbulent flow the mixing of dye-filament and water is intense and flow is irregular, random and disorderly. In case of laminar flow, the loss of pressure head was found to be proportional tothe velocity but in case of turbulent flow, Reynold observed that loss of head is approximately proportional to the square of velocity. More exactly the loss of head, lire: V", where m varies from 1.75 to 2.0 Fig. 10.2 Different stages of filament. > 10.3. FRICTIONAL LOSS IN PIPE FLOW When a liquid is flowing through a pipe, the velocity ofthe liquid layer adjacent to the pipe wal is, zet0. The velocity of liquid goes on inereasing from the wall and thus velocity gradient and hence shear stresses are produced in the whole liquid duc to viscosity. This viscous action causes loss of energy which is usually known as frictional loss. On the basis of his experiments, William Froude gave the following laws of fluid fraction for turbulent flow. ‘The frictional resistance for turbulent flow is (i, proportional to V", where n varies from 1.5 to 2.0, (ii) proportional to the density of fluid, (iii) proportional to the area of surface in contact, (iv) independent of pressure, (©) dependent on the nature ofthe surface in contact. 10.3.1 Expression for Loss of Head Due to Fri nin Pipes. Consider a uniform hori- zontal pipe, having steady flow as shown in Fig. 10.3. Let 1-1 and 2-2 are two sections of pipe. Let p, = pressure intensity at section 1-1, V, = velocity of flow at section 1-1, [ Turbulent Flow 435) L = length of the pipe between sections 1-1 and 2-2, Hiameter of pipe, frictional resistance per unit wetted area per unit velocity, loss of head due to friction, we values of pressure intensity and velocity at section 2-2. Fig. 103 Uniform horizontal pipe. Applying Bernoulli's equations between sections 1-1 and 2-2, Total head at 1-1 = Total head at 2-2 + loss of head due to friction between 1-1 and 2-2 PW or t= eB neh pg 2g But 24 = 2,8 pipe is horizontal V, = V2 as dia, of pipe is same at 1-1 and 2.2 Pe Pe ator y= hh o Ps pe Ps pe But fis the head lost due to friction and hence intensity of pressure will be reduced in the direction of flow by frictional resistance. Now frictional resistance = frictional resistance per unit wetted area per unit velocity x wetted area x velocity” or Fr "x nd x V? [+ wetted area = nd x L, velocity v= Val =f'XPXLXV [is nd = Perimeter = P) i) The Forces acting on the fluid between sections 1-1 and 2-2 are : 1. pressure force at section I-1 = p, x A where A = Area of pipe 2, pressure force at section 2-2 = px A 3. frictional force Fas shown in Fig. 10.3. Resolving all forces inthe horizontal direction, we have PrA=PA~F, or @-ppA 10.1) + From (ii), Fy =f/PLY?] or Pi-Pa= But from equation (i), p, ~ p2= psy [436 Fluid Mechanics ] Equating the value of (p, ~ p,), we get XP REX? yx LPR? paghy= LP or lt) In equation (in, © equ aA (iv) Putting =F, where fis known as co-efficient of friction, p72 af LW _4f.L.v? Equation (i), Becomes as y= P= Se 40.2) Equation (10.2) is known as Darcy-Weisbach equation. This equation is commonly used for finding loss of head due to friction in pipes. ‘Sometimes equation (10.2) is written as fib? 10.24) dx2g see) ‘Then fis known as friction factor. 10.3.2 Expression for Co-efficient of Friction in Terms of Shear Stress. The equation (10.1) ‘gives the forces acting on a fluid between sections 1-1 and 2-2 of Fig. 10.3 in horizontal direction as PA~ pod F,=0 o (1— POA = Fy = force duc to shear sess t ear stress x surface area Bayxnd KL or Oye co X Rd x L. tA Cancelling id fom both sides, we have O1-p) 4 =mxL 4 or @-Pd= (10.3) Equation (10.2) can be written as fy [ Turbulent Flow 437) ~py= SEW or Or P= “Tg * PE (10.4) Equating the value of (p, ~ p3) in equations (10.3) and (10.4), AtyXL or we or west (10.5) (10.6) > 10.4 SHEAR STRESS IN TURBULENT FLOW The shear stress in viscous flow is given by Newton's law of viscosity as ene hy Similar to the expression for viscous shear, J. Boussinesq expressed the turbulent shear in math- ‘ematical form as 7 where 1, = shear stress due to viscosity. 10.7) where 1, = shear stress due to turbulence 11= eddy viscosity W = average velocity at a distance y from boundary. ‘The ratio of n| (eddy viscosity) and p (mass density) is known as kinematic eddy viscosity and is denoted by & (epsilon). Mathematically it is written as a ext 10.8) ° If the shear stress due to viscous flow is also considered, then the total shear stress becomes as, du ae centtep Hay 0.9) Wate The value of n = 0 for laminar flow. For other cases the value of 1 may be several thousand times the value of j. To find shear stress in turbulent flow, equation (10.7) given by Boussinesq is used. But as the value of 1) (eddy viscosity) cannot be predicted, this equation is having limited use, 10.4.1 Reynolds Expression for Turbulent Shear Stress. Reynolds in 1886 developed an expression for turbulent shear stress between two layers of a fluid at a small distance apart, which is given as, t= pu'y’ (10.10) fluctuating component of velocity in the direction of x and y due to turbulence. ‘As .u and v’ are varying and hence t will also vary. Hence to find the shear stress, the time average ‘on both the sides of the equation (10.10) is taken. Then equation (10.10) becomes as where u’, [438 Fluid Mechanics ] (10.11) ‘The turbulent shear stress given by equation (10.11) is known as Reynold stress. 10.4.2 Prandtl Mixing Length Theory for Turbulent Shear Stress. In equation (10.11), the turbulent shear stress can only be calculated if the value of u’ v’ is known. But its very difficult to measure wv”. To overcome this difficulty, L. Prandtl in 1925, presented a mixing length hypothesis which can be used to express turbulent shear stress in terms of measurable quantities. According to Prandtl, the mixing length 1, is that distance between two layers in the transverse direction such that the lumps of fluid particles from one layer could reach the other layer and the particles are mixed in the other layer in such a way that the momentum of the particles in the direction of x is same. He also assumed that the velocity fluctuation in the x-direction u” is related to the mixing length fas, du dy and v” , the fluctuation component of velocity in y and hence rection is of the same order of magnitude as u” du dy Now wv becomes as av? =(12"] (124) =P { ay)" (ay Substituting the value of #7” in equation (10.11), we get the expression for shear stress in turbulent flow due to Prandtl as dy ‘Thus the total shear stress at any point in turbulent flow is the sum of shear stress due to viscous shear and turbulent shear and can be written as du. o(du) 44 on( ae 0.9 a? (@) f But the viscous shear stress is negligible except near the boundary. Equation (10.13) is used for ‘most of turbulent fluid flow problems for determining shear stress in turbulent flow. t=pr (*) 10.12) > 10.5 VELOCITY DISTRIBUTION IN TURBULENT FLOW IN PIPES In case of turbulent flow, the total shear stress at any point is the sum of viscous shear stress and turbulent shear stress. Also the viscous shear stress is negligible except near the boundary. Hence it ‘can be assumed that the shear stress in turbulent flow is given by equation (10.12). From this equation, the velocity distribution can be obtained if the relation between (, the mixing length and y is known, Prandtl assumed that the mixing length, isa linear function of the distance y from the pipe wall ie., 1 = ky, where k is a constant, known as Karman constant and = 0.4, Substituting the value of 1 in equation (10.12), we get [ Turbulent Flow 439) - a (du) tarcepxcorx(A) a2 (4H) oe (4H) = apy? we (Shoal ome B “a, Pane 10.14) dy Ypk*y? ky Vp For small values of y that is very close to the boundary of the pipe, Prandtl assumed shear stress t to be constant and approximately equal to t) which presents the turbulent shear stress at the pipe boundary. Substituting t= tin equation (10.14), we get du _ 1 fe dy Vp pt [e has the dimensions, MET = |E = © pur £ is velocity and hence ML Vr rT has the dimension of velocity, which is known as shear velocity and is denoted by w.. P FE, du_1 Thus ,|*2 = w,, then equation (10.15) becomes =~ u,, Ve a ky (10.15) In equation (10.15), For a given case of turbulent flow, «. is constant, Hence integrating above equation, we get us" boy (10.16) where C = constant of integration Equation (10.16) shows that in turbulent flow, the velocity varies directly with the logarithm of the distance from the boundary or in other words the velocity distribution in turbulent flow is logarithmic in nature. To determine the constant of integration, C the boundary condition that at y = R (radius of pipe), = gay is Substituted in equation (10.16). Hence log, R+ C Substituting the value of C in equation (10.16), we get, 1 Jog, R= tax + “ (log, y~ log, R) F lOBe R= Hnas +E (oR ¥~ log, R) log. (IR) [vy k= 04 = Karman constant] oO = thgay + 2.5 te log, (9/R) (10.17) Equation (10.17) is called *Prandit's universal velocity distribution equation for turbulent flow in pipes. This equation is applicable to smooth as well as rough pipe boundaries. Equation (10.17) is also written as [440 Fluid Mechanics ] Dividing by uw, we get = 2.5 ue log, (/R) = 2.5 us log, (Rly) 25 log, (Rly) = 2.5 2.3 logy (Ris) [ log, (Rly) = 2.3 ogo (R4)] or = 5.75 logia (Rly) (10.18) In equation (10.18), the difference between the maximum velocity jy. and local velocity « at any point ic, (Wax ~ 4) is known as ‘velocity defect’. 10.5.1 Hydrodynamically Smooth and Rough Boundaries. Let kis the average height of the irregularities projecting from the surface of a boundary as shown in Fig. 10.4. If the value of k is large for a boundary then the boundary is called rough boundary and if the value of k is less, then boundary is known as smooth boundary, in general. This is the classification of rough and smooth boundary based on boundary characteristics. But for proper classification, the flow and fluid charac- teristics are also to be considered. LAMINAR SUBLAYER. LAMINAR SUBLAYER t ® 4 - App ---- | Meter Lill (@) Smooth boundary (b) Rough boundary Fig. 104 Smooth and rough boundaries. For turbulent flow analysis along a boundary, the flow is divided in two portions. The first portion ‘consists ofa thin layer of fluid in the immediate neighbourhood of the boundary, where viscous sheat stress predominates while the shear stress due to turbulence is negligible. This portion is known as laminar sub-layer. The height upto which the effect of viscosity predominates in this zone is denoted by 8°. The second portion of flow, where shear stress due 10 turbulence are large as compared to viscous stress is known as turbulent zone If the average height k ofthe irregularities, projecting from the surface of a boundary is much less than 8, the thickness of laminar sub-layer as shown in Fig. 10.4 (a), the boundary is called smooth boundary. This is because, outside the laminar sub-layer the flow is turbulent and eddies of various size present in turbulent flow try to penetrate the laminar sub-layer and reach the surface of the boundary. But due to great thickness of laminar sub-layer the eddies are unable to reach the surface inmegularities and hence the boundary behaves as a smooth boundary. This type of boundary is called hydrodynamically smooth boundary. Now ifthe Reynolds number ofthe flow is increased then the thickness of laminar sub-layer will decrease. If the thickness of laminar sub-layer becomes much smaller than the average height k of inregularities of the surface as shown in Fig. 10.4 (6), the boundary will act as rough boundary. This is because the irregularities of the surface are above the laminar sub-layer and the eddies present in turbulent zone will come in contact with the irregularities ofthe surface and lot of energy will be lost. Such a boundary is called hydrodynamically rough boundary From Nikuradse’s experiment 1 toast 025 0 025, oan eld snot today TurbulentFlow 441] 2. IF is greater than 6.0, the boundary is rough, 3. 1£025 <() < 60, the boundary isin wansion In terms of roughness Reynolds number “= <4, boundary is considered smooth, 2. lies between 4 and 100, boundary is in transition stage, and tok 3. If 5 > 100, the boundary is rough. 10.5.2. Velocity Distribution for Turbulent Flow in Smooth Pipes. The velocity distri- bution for turbulent flow in smooth or rough pipe is given by equation (10.16) as u= topy+C lope y + may be seen that at y= 0, the velocity w at wall is ~s, This means that velocity u is positive at some distance far away from the wall and ~ » (minus infinity) atthe wall. Hence at some finite distance from wall, the velocity will be equal to zero. Let this distance from pipe wall is - Now the constant Cis determined from the boundary condition ie, at y = y’, = 0. Hence above equation becomes as = tog, y+ CorC=~“ tog, y” 7 Bey 7 Bey Substituting the value of C in the above equation, we get w= tog, y- = tog, y’ =“ log, Oly’ 8.9 log. y= Hos. Or) k Substituting the value of k = 0.4, we get A loge (vy) = 25 nog, (vy) Go 2S 23 login Oh") [rs log, (y’) = 2.3 logyo (Hy) or + 25.75 ogy (ly) (10.19) For the smooth boundary, there exists a laminar sub-layer as shown in Fig. 10.4 (a). The velocity distribution in the laminar sub-layer is parabolic in nature, Thus in the laminar sub-layer, logarithmic velocity distribution does not hold good. Thus it can be assumed that y’ is proportional to 8°, where 3° is the thickness of laminar sub-layer. From Nikuradse’s experiment the value of y’ is given as (442. Fluid Mechanics ] 7 107 oa lov where 5” . where v= kinematic viscosity of fluid lev, 1 _ 0.08v 107 Substituting this value of y’in equation (10.19), we obtain . : 575 bee (222) 52800 o—eeery "0408 = 5.75 logy “ + 5.55 (10.20) 10.5.3 Velocity Distribution for Turbulent Flow in Rough Pipes. In case of rough boundaries, the thickness of laminar sub-layer is very small as shown in Fig. 10.4 (6). The surface irregularities are above the laminar sub-layer and hence the laminar sub-layer is completely destroyed. ‘Thus y’ can be considered proportional to the height of protrusions k. Nikuradse’s experiment shows the value of y" for pipes coated with uniform sand (rough pipes) as y” 30" Substituting this value of y’ in equation (10.19), we get 130. 25250815 (zs) 5.75 [logy 8) x30] = 5.75 logy (91K) + 5.75 logy0 (30.0) = 5.75 log (9k) + 8.5. --(10-21) Problem 10.1 A pipe-line carrying water has average height of irregularities projecting from the surface of the boundary of the pipe as 0.15 mm. What type of boundary is it ? The shear stress developed is 4.9 N/m. The kinematic viscosity of water is .01 stokes. Solution. Given : Average height of irregularities, k = 0.15 mm =0.15 x 10m Shear stress developed, 1%) =4.9 Nim? Kinematic viscosity, V=0,01 stokes = 01 em*/s = 01 x 104 m/s Density of water, p= 1000 kg/m? Shear velocity, JioTP = | 80, = JOO039 = 0.07 mis [ Turbulent Flow 443) ok _ 0.07 x 015 10 v 01x10 Roughness Reynold number = tak Since “* ties between 4 and 100 and hence pipe surface behaves as in transition. Problem 10.2 A rough pipe is of diameter 8.0 cm. The velocity at a point 3.0 cm from wall is 30% ‘more than the velocity at a point I cm from pipe wall. Determine the average height of the roughness. Solution. Given : Dia. of rough pipe, D=8cm=.08m Let velocity of flow at 1 em from pipe wall =u ‘Then velocity of flow at 3 cm from pipe wall = 1.3 u ‘The velocity distribution for rough pipe is given by equation (10.21) as 175 logyo (y/K) + 8.5, where k = height of roughness. For a point, 1 em from pipe wall, we have 5.75 logy (LOUK) + 8.5 ~O. For a point, 3 cm from pipe wall, velocity is 1.3 u and hence 13u 5.15 logyy (3.0/8) + 8.5 i) 5.75 1ogyo(3.0/ k) +85 Dividing (i) by (D, we get 1. BOON Owe Be SS og (1B) +85 or 1,3(8.75 logyg (I/k) + 8.5] = 5.75 logyy (3.0/8) + 8.5 or ATS logy (IK) + 11.05 = 5.75 logy G.0/8) + 8.5 or ATS Logg (Ik) ~ 5.75 logo (3k) = 8.5 ~ 11.05 = ~ 2.55 or 7.475 logo 1.0 ~ log KI ~ 5.75 [log 3.0 ~ logyo kl = ~ 2.55 or 7475 [0 ~ logyg k] ~ 5.75 [4771 — logy k] = ~ 2.55 or = TAT5 logy k~ 2.7433 + 5.75 logis k= ~ 2.55 or = 1.725 logyo k 0.1933 or 04933 _ _ 9.1120 = 7888 =1725 = .7726 em. Ans. Problem 10.3. A smooth pipe of diameter 80 mm and 800 m long carries water at the rate of 0.480 m’/minute. Calculate the loss of head, wall shearing stress, centre line velocity, velocity and shear stress at 30 mm from pipe wall. Also calculate the thickness of laminar sub-layer. Take kin- ematie viscosity of water as 0.015 stokes. Take the value of co-efficient of friction ‘f from the relation given as 2781 wnere Re = Reynolds number. (R,) [444 Fluid Mechanics ] Solution. Given: Dia. of smooth pipe, 80 mm =.08 m Length of pipe, L=800m Discharge, Q = 0.048 m*/minute = 108 m/s Kinematic viscosity, V=.015 stokes = 015 x 104 m/s. [Stokes = cm/s] Density of water, p= 1000 kg/m? Mean velocity, = 2-008 _ 1591 mis Area” Tog)? * (08) Reynolds number, an 8.485 x 10* ‘015 x10 As the Reynolds number is more than 4000, the flow is turbulent. 10791 RI (g495x10°)" Now the value of ‘fis given by (= = .004636 (0 Head lost is given by equation (10.2) as fL.V? _ 4.004636 x 800 x 15917 tye BLL? Sx HES 0021S 90.4, Ane u dx2g 08x 2x 981 me Ane (i) Wal shar ses, ssn by equation 105) soe PE «00635 10 515018 586 Nim An Gi Contin elo yg fo ip i ne by eton 1020 a 4 =5,75 logy“ + 5.55 i) where 1. is shear velocity and = [2 = |>86° - 9.0765 mis P 1000 40s The log wil be maxmam when y= 48 204m y rid Hen: 817 = 44H Sitges ven (we oss. 575 ogy SSDS « 535 10 015x107 5.15 logy 2040 + 5.55 75 x 3.309 + 5.55 = 19.03 + 5.55 = 24.58 s Upax = 0765 X 24.58 = 1.88 m/s. Ans. (iv) The shear stress, + at any point is given by ofA) [ TurbulentFlow 445) where r= distance from centre of pipe and hence shear stress at pipe wall where r = R is DR we - 2S (B) Dividing equation (A) by equation (B), we get aor %) R <. Shear stress ue R ‘A point 30 mm from pipe wall is having r= =3=1em=01m 4% 01 _ 5866 _ 14665 Nim? Ans. oo 4 ‘Velocity at a point 3 cm from pipe wall means y= 3 em = .03 m tat (r= 01 m)= and is en by equation (10.20) as = 5.75 logyy “* + 5.55, where w. = 0765, y =.03 0765 x.03 "015 x10 75 logy 1530 + 5.5 0.0765 x 23.86 = 1.825 mls. Ans. (©) Thickness of laminar sub-layer is given by 6xv_116x.015x10 ue 0765 .274 x 107 em = 02274 em. Ans. Problem 10.4 Determine the wall shearing stress in a pipe of diameter 100 mm which carries water. The velocities at the pipe centre and 30 mm from the pipe centre are 2 m/s and 1.5 m/s respectively. The flow in pipe is given as turbulent. Solution. Given : 274 x 104m Dia. of pipe, D=100 mm = 0.10m Radius, p= M9 005m 2 Velocity at centte, ty = 2 m/s Velocity at 30 mm or 0.03 m from centre = 1.5 m/s Velocity (at r = 0.03 m), w= 1.5 mis Let the wall shearing stress For turbulent flow, the velocity distribution in terms of centre line Velocity (tga) is given by equa- tion (10.18) as ssa where w= 1.5 mis at y = (R ~ 1) = 0.05 ~ 0.03 = .02 m [446 Fluid Mechanics ] 05 = 5.75 logy > = 2.288 oF os 2185 mis Using the relation us = 7p. where p for water = 1000 kg/m? Top to 2 = [2 or 477 0.2185 = 7550" apg = 02185" = 0.0 or ‘t= 0.0477 x 1000 = 47.676 Nim?. Ans. 10.5.4 Velocity Distribution for Turbulent Flow in Terms of Average Velocity. The average velocity U”, through the pipe is obtained by first finding the total discharge Q and then dividing the total discharge by the area of the pipe. ELEMENTARY CIRCULAR RING ad es eel or Fig. 10.5 Average velocity for turbulent flow. Consider an elementary circular ring of radius *r’ and thickness dr as shown in Fig. 10.5. The distance of the ring from pipe wall is y = (R ~ r), where R = radius of pipe. ‘Then the discharge, dQ, through the ring is given by dQ = area of ring x velocity = 2nrdr x w= ux 2nrdr Total discharge, a0= [f wane 002 (a) For smooth pipes. For smooth pipes, the velocity distribution is given by equation (10.20) as Gr 25-75 log = +55, But yeR-1 w= [5156 HA 655] 0 Substituting the value of w in equation (10.22), we get (R=), v o-[' [575 toei0 | tux 2nrdr

You might also like